You are on page 1of 333

APTITUDE

Numbers H.C.F and L.C.M Decimal Fractions Simplification Square and Cube roots Average Problems on Numbers Problems on Ages Surds and Indices Percentage Profit and Loss Ratio And Proportions Partnership Chain Rule Time and Work Pipes and Cisterns Time and Distance Trains Boats and Streams Alligation or Mixture Simple Interest Compound Interest Logorithms Areas Volume and Surface area Races and Games of Skill Calendar Clocks Stocks ans Shares True Discount Bankers Discount Oddmanout and Series Data Interpretation probability Permutations and Combinations Puzzles

ALLIGATION AND MIXTURES


CONCEPT SIMPLE PROBLEMS MEDIUM PROBLEMS COMPLEX PROBLEMS

file:///E|/work/books/placement/09_Aptitude/alligation.html[1/28/2012 12:44:45 AM]

APTITUDE
Numbers H.C.F and L.C.M Decimal Fractions Simplification Square and Cube roots Average Problems on Numbers Problems on Ages Surds and Indices Percentage Profit and Loss Ratio And Proportions Partnership Chain Rule Time and Work Pipes and Cisterns Time and Distance Trains Boats and Streams Alligation or Mixture Simple Interest Compound Interest Logorithms Areas Volume and Surface area Races and Games of Skill Calendar Clocks Stocks ans Shares True Discount Bankers Discount Oddmanout and Series Data Interpretation probability Permutations and Combinations Puzzles BACK

ALLIGATION OR MIXTURES SOLVED PROBLEMS


Complex Problems: 1.Tea worth Rs 126 per Kg are mixed with a third variety in the ratio 1:1:2. If the mixture is worth Rs 153 per Kg , the price of the third variety per Kg will be? Solution: Since First and second varieties are mixed in equal proportions so their average price =Rs (126+135)/2 = 130.50. So the mixture is formed by mixing two varieties ,one at Rs 130.50 per Kg and the other at say Rs x per Kg in the ratio 2:2 i e,1:1 we have to find x.
Costof 1Kg tea of 1st kind RS 130.50 Costof 1Kg tea of 2n d kind Rs x.

Mean Price Rs 153 x-153 22.50

(x=153)/22.5 = 1 =>x-153 = 22.5 x = 175.50. Price of the third variety =Rs 175.50 per Kg.

2.The milk and water in two vessels A and B are in the ratio 4:3 and 2:3 respectively. In what ratio the liquids in both the vessels be mixed to obtain a new mixture in vessel c consisting half milk and half water? Solution:Let the C.P of milk be Re 1 per liter. Milk in 1 liter mixture of A = 4/7 liter. Milk in 1 liter mixture of B = 2/5 liter. Milk in 1 liter mixture of C = 1/2 liter. C.P of 1 liter mixture in A=Re 4/7 C.P of 1 liter mixture in B=Re 2/5. Mean Price = Re 1/2. By rule of allegation we have:
C.P of 1 liter mixture in A 4/7 C.P of 1 liter mixture in B 2/5

Mean Price 1/10 1/14

file:///E|/work/books/placement/09_Aptitude/alligationcomplex.html[1/28/2012 12:44:47 AM]

Required ratio = 1/10 : 1/14 = 7:5. 3.How many Kg s of wheat costing him Rs 1.20,Rs 1.44 and Rs 1.74 per Kg so that the mixture may be worth Rs 1.41 per Kg? Solution: Step1:Mix wheat of first and third kind to get a mixture worth Rs 1.41 per Kg.
C.P of 1 Kg wheat of 1st kind 120p C.P of 1 Kg wheat of 3rd kind 174p

Mean Price 141p 33 21

They must be mixed in the ratio =33:21 = 11:7 Step2:Mix wheats of 1st and 2n d kind to obtain a mixture worth of 1.41.per Kg.

C.P of 1 Kg wheat of 1st

kind 120p

C.P of 1 Kg wheat of 2n d

kind 144p

Mean Price 141p 3 21

They must be mixed in the ratio = 3:21=1:7. Thus,Quantity of 2n d kind of wheat / Quantity of 3rd kind of wheat = 7/1*11/7= 11/1 Quantities of wheat of 1st :2n d:3rd = 11:77:7.

4.Two vessels A and B contain spirit and water mixed in the ratio 5:2 and 7:6 respectively. Find the ratio n which these mixture be mixed to obtain a new mixture in vessel c containing spirit and water in the ratio 8:5? Solution:Let the C.P of spirit be Re 1 per liter. Spirit in 1 liter mix of A = 5/7 liter. C.P of 1 liter mix in A =5/7. Spirit in 1 liter mix of B = 7/13 liter. C.P of 1 liter mix in B =7/13. Spirit in 1 liter mix of C = 8/13 liter. C.P of 1 liter mix in C =8/13.
C.P of 1 liter mixture in A 5/7 C.P of 1 liter mixture in B 7/13

Mean Price 8/13 1/13 9/91

Therefore required ratio = 1/13 : 9/91 = 7:9.

file:///E|/work/books/placement/09_Aptitude/alligationcomplex.html[1/28/2012 12:44:47 AM]

5.A milk vendor has 2 cans of milk .The first contains 5% water and the rest milk. The second contains 50% water. How much milk should he mix from each of the container so as to get 12 liters of milk such that the ratio of water to milk is 3:5? Solution:Let cost of 1 liter milk be Re 1. Milk in 1 liter mixture in 1st can = 3/4 lit. C.P of 1 liter mixture in 1st can =Re 3/4 Milk in 1 liter mixture in 2n d can = 1/2 lit. C.P of 1 liter mixture in 2n d can =Re 1/2 Milk in 1 liter final mixture = 5/8 lit. Mean Price = Re 5/8.
C.P of 1 lt mix in 1st Re3/4 C.P of 1 lt mix in 2nd Re1/2

Mean Price 5/8 1/8 1/8

There ratio of two mixtures =1/8 :1/8 = 1:1. So,quantity of mixture taken from each can=1/2*12 = 6 liters.

6.One quantity of wheat at Rs 9.30 p er Kg are mixed with another quality at a certain rate in the ratio 8:7. If the mixture so formed be worth Rs 10 per Kg ,what is the rate per Kg of the second quality of wheat? Solution:Let the rate of second quality be Rs x per Kg.
C.P of 1Kg wheat of 1st 980p C.P of 1 Kg wheat of 2nd 100x p

Mean Price 1000p 100x-1000p 70 p

(100x-1000) / 70 = 8/7 700x -7000 = 560 700x = 7560 =>x = Rs 10.80. Therefore the rate of second quality is Rs10.80

7.8lit are drawn from a wine and is then filled with water. This operation is performed three more times.The ratio of the quantity of wine now left in cask to that of the water is 16:81. How much wine did the cask hold originally? Solution: Let the quantity of the wine in the cask originally be x liters. Then quantity of wine left in cask after 4 operations = x(1- 8/x)4lit. Therefore x((1-(8/x))4)/x = 16/81. (1- 8/x)4=(2/3) 4 (x- 8)/x=2/3 3x-24 =2x

file:///E|/work/books/placement/09_Aptitude/alligationcomplex.html[1/28/2012 12:44:47 AM]

x=24.

8.A can contains a mixture of two liquids A and B in the ratio 7:5 when 9 liters of mixture are drawn off and the can is filled with B,the ratio of A and B becomes 7:9. How many liters of liquid A was contained by the can initially? Solution: Suppose the can initially contains 7x and 5x liters of mixtures A and B respectively . Quantity of A in mixture left = (7x- (7/12)*9 )lit = 7x - (21/4) liters. Quantity of B in mixture left = 5x - 5/12*9 = 5x - (15/4) liters Therefore (7x 21/4)/ (5x 15/4+9)=7/9 (28x-21)/(20x +21)= 7/9 (252x -189)= 140x +147 112x = 336 => x=3. So the can contains 21 liters of A.

9.A vessel is filled with liquid,3 parts of which are water and 5 parts syrup. How much of the mixture must be drawn off and replaced with water so that the mixture may be half water and half syrup? Solution: Suppose the vessal initially contains 8 liters of liquid. Let x liters of this liquid be replaced with water then quantity of water in new mixture = 3-(3x/8)+x liters. Quantity of syrup in new mixture = 5 - 5x/8 liters. Therefore 3 - 3x/8 +x = 5 - 5x/8 5x+24 = 40-5x 10x = 16. x= 8/5. So part of the mixture replaced = 8/5*1/8 =1/5. BACK

file:///E|/work/books/placement/09_Aptitude/alligationcomplex.html[1/28/2012 12:44:47 AM]

APTITUDE
Numbers H.C.F and L.C.M Decimal Fractions Simplification Square and Cube roots Average Problems on Numbers Problems on Ages Surds and Indices Percentage Profit and Loss Ratio And Proportions Partnership Chain Rule Time and Work Pipes and Cisterns Time and Distance Trains Boats and Streams Alligation or Mixture Simple Interest Compound Interest Logorithms Areas Volume and Surface area Races and Games of Skill Calendar Clocks Stocks ans Shares True Discount Bankers Discount Oddmanout and Series Data Interpretation probability Permutations and Combinations Puzzles BACK

ALLIGATION OR MIXTURES

Important Facts and Formula: 1.Allegation:It is the rule that enables us to find the ratio in which two of more ingredients at the given price must be mixed to produce a mixture of a desired price. 2.Mean Price:The cost price of a unit quantity of the mixture is called the mean price. 3.Rule of Allegation:If two ingredients are mixed then Quantity of Cheaper / Quantity of Dearer = (C.P of Dearer Mean Price) /(Mean PriceC.P of Cheaper).
C.P of a unit quantity of cheaper(c) C.P of unit quantity of dearer(d)

Mean Price(m) (d-m) (m-c)

Cheaper quantity:Dearer quantity = (d-m):(m-c) 4.Suppose a container contains x units of liquid from which y units are taken out and replaced by water. After n operations the quantity of pure liquid = x (1 y/x)n units. BACK

file:///E|/work/books/placement/09_Aptitude/alligationconcept.html[1/28/2012 12:44:47 AM]

APTITUDE
Numbers H.C.F and L.C.M Decimal Fractions Simplification Square and Cube roots Average Problems on Numbers Problems on Ages Surds and Indices Percentage Profit and Loss Ratio And Proportions Partnership Chain Rule Time and Work Pipes and Cisterns Time and Distance Trains Boats and Streams Alligation or Mixture Simple Interest Compound Interest Logorithms Areas Volume and Surface area Races and Games of Skill Calendar Clocks Stocks ans Shares True Discount Bankers Discount Oddmanout and Series Data Interpretation probability Permutations and Combinations Puzzles BACK

ALLIGATION OR MIXTURES SOLVED PROBLEMS


Medium Problems: 1.A butler stole wine from a butt of sherry which contained 40% of spirit and he replaced,what he had stolen by wine containing only 16% spirit. The butt was then of 24% strength only. How much of the butt did he steal? Solution:
Wine containing 40%spirit Wine containing 16% spirit

Wine containing 24% spirit 8 16

They must be mixed in the ratio of =1:2. Thus 1/3 of the butt of sherry was left and hence the butler drew out 2/3 of the butt. 2.The average weekly salary per head of the entire staff of a factory consisting of supervisors and the laborers is Rs 60.The average salary per head of the supervisors is Rs 400 and that of the laborers is Rs 56.Given that the number of supervisors is 12.Find the number of laborers in the factory. Solution:
Average salary of laborer Rs 56 Average salary of supervisors Rs 400

Average salary of entire staff Rs 60 340 4

Number of laborer / Number of Supervisors = 340 / 4=85/1 Thus,if the number of supervisors is 1,number of laborers =85. Therefore if the number of supervisors is 12 number of laborers 85*12=1020.

3.The cost of type 1 rice is Rs 15 per Kg and type 2 rice is Rs 20 per Kg. If both type1 and type 2 are mixed in the ratio of 2:3,then the price per Kg of the mixed variety of rice is? Solution:Let the price of the mixed variety be Rs x per Kg.

file:///E|/work/books/placement/09_Aptitude/alligationmedium.html[1/28/2012 12:44:48 AM]

Cost of 1 Kg of type 1 rice Rs 15

Cost of 1 Kg of type 2 rice Rs 20

Mean Price Rs x 20-x x-15

(20-x) /( x-15) = 2/3 => 60-3x = 2x-30 5x = 90=>x=18. 4.In what ratio must a grocer mix two varieties of tea worth Rs 60 a Kg and Rs 65 a Kg so that by selling the mixture at Rs 68.20 a Kg he may gain 10%? Solution:S.P of 1 Kg of the mixture = Rs 68.20,gain =10% S.P of 1 Kg of the mixture = Rs (100/110*68.20)=Rs 62.
Cost of 1 Kg tea of 1st kind 3 60 Cost of 1 Kg tea of 2nd kind 65 2

Mean Price Rs 62

Required ratio =3:2. 5.A dishonest milkman professes to sell his milk at cost price but he mixes t with water and there by gains 25% .The percentage of water in the mixture is? Solution:Let C. P of 1 liter milk be Re 1. Then S.P of 1 liter mixture=Re 1. Gain=25% C.P of 1 liter mixture =Re(100/125*1) = Re 4/5.
C.P of 1 liter milk Re 1 C.P of 1 liter of water 0

Mean Price 4/5 4/5 1/5

Ratio of milk to water =4/5 : 1/5 = 4:1 Hence percentage of water n the mixture=1/5*100=20%.

12.A merchant has 1000Kg of sugar,part of which he sells at 8% profit and the rest at 18% profit. He gains 14% on the whole. The quantity sold at 18% profit is? Solution:
Profit on 1st part 8% Profit on 2nd part 18%

Mean Profit 14% 4 6

Ratio of 1st and 2nd parts =4:6 =2:3. Quantity of 2nd ind =3/5*1000Kg =600 Kg.

file:///E|/work/books/placement/09_Aptitude/alligationmedium.html[1/28/2012 12:44:48 AM]

6.A jar full of whiskey contains 40% alcohol. A part of this whiskey is replaced by another containing 19% alcohol and now the percentage of alcohol was found to be 26%. The quantity of whiskey replaced is? Solution:
Strength of first jar 40% Strength of 2nd jar 19%

Mean Strength 26% 7 14

So,ratio of 1st and 2nd quantities =7:14 =1:2 Therefore required quantity replaced =2/3. 7.A container contains 40lit of milk. From this container 4 lit of milk was taken out and replaced by water. This process was repeated further two times. How much milk is now contained by the container? Solution:Amount of milk left after 3 operations = 40(1-4/40)3lit =(40*9/10*9/10*9/10) = 29.16 lit BACK

file:///E|/work/books/placement/09_Aptitude/alligationmedium.html[1/28/2012 12:44:48 AM]

Alligation or Mixtures
Important Facts and Formulae: 1.Allegation:It is the rule that enables us to find the ratio in which two of more ingredients at the given price must be mixed to produce a mixture of a desired price. 2.Mean Price:The cost price of a unit quantity of the mixture is called the mean price. 3.Rule of Allegation:If two ingredients are mixed then Quantity of Cheaper / Quantity of Dearer = (C.P of Dearer Mean Price) /(Mean PriceC.P of Cheaper). C.P of a unit quantity of cheaper(c) C.P of unit quantity of dearer(d) Mean Price(m) (d-m) (m-c)

Cheaper quantity:Dearer quantity = (d-m):(m-c) 4.Suppose a container contains x units of liquid from which y units are taken out and replaced by water. After n operations the quantity of pure liquid = x (1 y/x)n units. Simple Problems 1.In what ratio must rice at Rs 9.30 per Kg be mixed with rice at Rs 10.80 per Kg so that the mixture be worth Rs 10 per Kg? Solution: C.P of 1 Kg rice of 1st kind 930 p C.P of 1 Kg rice of 2n d kind 1080p Mean Price 1000p 80 70 Required ratio=80:70 = 8:7 2.How much water must be added to 60 liters of milk at 11/2 liters for Rs 20 so as to have a mixture worth Rs 10 2/3 a liter? Solution: C.P of 1 lit of milk = 20*2/3 = 40/3 C.P of 1 lit of water 0 C.P of 1 lit of milk 40/3 Mean Price 32/3 8/3 32/3 Ratio of water and milk =8/3 : 32/3 = 1:4 Quantity of water to be added to 60 lit of milk =1/4*60=15 liters. 3.In what ratio must water to be mixed with milk to gain 20% by selling the mixture at cost price? Solution:Let the C.P of milk be Re 1 per liter Then S.P of 1 liter of mixture = Re.1 Gain obtained =20%. Therefore C.P of 1 liter mixture = Rs(100/120*1) =5/6 C.P of 1 liter of water 0 C.P of 1 liter of milk1 Mean Price 5/6 1/6 5/6

Ratio of water and milk =1/6 : 5/6 = 1:5. 4.In what ratio must a grocer mix two varieties of pulses costing Rs 15 and Rs 20 per Kg respectively so as to get a mixture worth Rs 16.50 per Kg? Solution: Cost of 1 Kg pulses of 1 kind 15 Cost of 1 Kg pulses of 2nd kind 20 Mean Price Rs 16.50 3.50 Required ratio =3.50 : 1.50 = 35:15 = 7:3. 1.50

5. 4Kg s of rice at Rs 5 per Kg is mixed with 8 Kg of rice at Rs 6 per Kg .Find the average price of the mixture? Solution: rice of 5 Rs per Kg rice of 6 Rs per Kg Average price Aw

file:///E|/work/books/placement/09_Aptitude/alligationormixtures.html[1/28/2012 12:44:48 AM]

6-Aw (6-Aw)/(Aw-5) = 4/8 =1/2 12-2Aw = Aw-5 3Aw = 17 Aw = 5.66 per Kg.

Aw-5

Top
6.5Kg of rice at Rs 6 per Kg is mixed with 4 Kg of rice to get a mixture costing Rs 7 per Kg. Find the price of the costlier rice? Solution: Using the cross method:

rice at Rs 6 per Kg rice at Rs x per Kg Mean price Rs 7 per Kg 5 4 x-7:1=5:4 4x-28 = 5 4x=33=>x=Rs 8.25. Therefore price of costlier rice is Rs 8.25 per Kg Medium Problems 1.A butler stole wine from a butt of sherry which contained 40% of spirit and he replaced,what he had stolen by wine containing only 16% spirit. The butt was then of 24% strength only. How much of the butt did he steal? Solution: Wine containing 40%spirit Wine containing 16% spirit Wine containing 24% spirit 8 16 They must be mixed in the ratio of =1:2. Thus 1/3 of the butt of sherry was left and hence the butler drew out 2/3 of the butt. 2.The average weekly salary per head of the entire staff of a factory consisting of supervisors and the laborers is Rs 60.The average salary per head of the supervisors is Rs 400 and that of the laborers is Rs 56.Given that the number of supervisors is 12.Find the number of laborers in the factory. Solution: Average salary of laborer Rs 56 Average salary of supervisors Rs 400 Average salary of entire staff Rs 60 340 4 Number of laborer / Number of Supervisors = 340 / 4=85/1 Thus,if the number of supervisors is 1,number of laborers =85. Therefore if the number of supervisors is 12 number of laborers 85*12=1020. 3.The cost of type 1 rice is Rs 15 per Kg and type 2 rice is Rs 20 per Kg. If both type1 and type 2 are mixed in the ratio of 2:3,then the price per Kg of the mixed variety of rice is? Solution:Let the price of the mixed variety be Rs x per Kg. Cost of 1 Kg of type 1 rice Rs 15 Cost of 1 Kg of type 2 rice Rs 20 Mean Price Rs x 20-x x-15 (20-x) /( x-15) = 2/3 => 60-3x = 2x-30 5x = 90=>x=18. 4.In what ratio must a grocer mix two varieties of tea worth Rs 60 a Kg and Rs 65 a Kg so that by selling the mixture at Rs 68.20 a Kg he may gain 10%? Solution:S.P of 1 Kg of the mixture = Rs 68.20,gain =10% S.P of 1 Kg of the mixture = Rs (100/110*68.20)=Rs 62. Cost of 1 Kg tea of 1st kind 60 Cost of 1 Kg tea of 2nd kind 65 Mean Price Rs 62 3 2 Required ratio =3:2. 5.A dishonest milkman professes to sell his milk at cost price but he mixes t with water and there by gains 25% .The percentage of water in the mixture is? Solution:Let C. P of 1 liter milk be Re 1. Then S.P of 1 liter mixture=Re 1. Gain=25% C.P of 1 liter mixture =Re(100/125*1) = Re 4/5. C.P of 1 liter milk Re 1 C.P of 1 liter of water 0 Mean Price 4/5 4/5 1/5 Ratio of milk to water =4/5 : 1/5 = 4:1 Hence percentage of water n the mixture=1/5*100=20%. 12.A merchant has 1000Kg of sugar,part of which he sells at 8% profit and the rest at 18% profit. He gains 14% on the whole. The quantity sold at 18% profit is? Solution: Profit on 1st part 8% Profit Mean Profit 14% 4 Ratio of 1st and 2nd parts =4:6 Quantity of 2nd ind =3/5*1000Kg on 2nd part 18% 6 =2:3. =600 Kg.

6.A jar full of whiskey contains 40% alcohol. A part of this whiskey is replaced by another containing 19% alcohol and now the percentage of alcohol was found to be 26%. The quantity of whiskey replaced is?

file:///E|/work/books/placement/09_Aptitude/alligationormixtures.html[1/28/2012 12:44:48 AM]

Solution: Strength of first jar 40% Strength of 2nd jar 19% Mean Strength 26% 7 14 So,ratio of 1st and 2nd quantities =7:14 =1:2 Therefore required quantity replaced =2/3. 7.A container contains 40lit of milk. From this container 4 lit of milk was taken out and replaced by water. This process was repeated further two times. How much milk is now contained by the container? Solution:Amount of milk left after 3 operations = 40(1-4/40)3lit =(40*9/10*9/10*9/10) = 29.16 lit

Top
Complex Problems 1.Tea worth Rs 126 per Kg are mixed with a third variety in the ratio 1:1:2. If the mixture is worth Rs 153 per Kg , the price of the third variety per Kg will be? Solution: Since First and second varieties are mixed in equal proportions so their average price =Rs (126+135)/2 = 130.50. So the mixture is formed by mixing two varieties ,one at Rs 130.50 per Kg and the other at say Rs x per Kg in the ratio 2:2 i e,1:1 we have to find x. Costof 1Kg tea of 1st kind RS 130.50 Costof 1Kg tea of 2n d kind Rs x. Mean Price Rs 153 x-153 22.50 (x=153)/22.5 = 1 =>x-153 = 22.5 x = 175.50. Price of the third variety =Rs 175.50 per Kg. 2.The milk and water in two vessels A and B are in the ratio 4:3 and 2:3 respectively. In what ratio the liquids in both the vessels be mixed to obtain a new mixture in vessel c consisting half milk and half water? Solution:Let the C.P of milk be Re 1 per liter. Milk in 1 liter mixture of A = 4/7 liter. Milk in 1 liter mixture of B = 2/5 liter. Milk in 1 liter mixture of C = 1/2 liter. C.P of 1 liter mixture in A=Re 4/7 C.P of 1 liter mixture in B=Re 2/5. Mean Price = Re 1/2. By rule of allegation we have: C.P of 1 liter mixture in A C.P of 1 liter mixture in B 4/7 2/5 Mean Price 1/10 1/14 Required ratio = 1/10 : 1/14 = 7:5. 3.How many Kg s of wheat costing him Rs 1.20,Rs 1.44 and Rs 1.74 per Kg so that the mixture may be worth Rs 1.41 per Kg? Solution: Step1:Mix wheat of first and third kind to get a mixture worth Rs 1.41 per Kg. C.P of 1 Kg wheat of 1st kind 120p C.P of 1 Kg wheat of 3rd kind 174p Mean Price 141p 33 21 They must be mixed in the ratio =33:21 = 11:7 Step2:Mix wheats of 1st and 2n d kind to obtain a mixture worth of 1.41.per Kg. C.P of 1 Kg wheat of 1st kind 120p C.P of 1 Kg wheat of 2n d kind 144p Mean Price 141p 3 21 They must be mixed in the ratio = 3:21=1:7. Thus,Quantity of 2n d kind of wheat / Quantity of 3rd kind of wheat = 7/1*11/7= 11/1 Quantities of wheat of 1st :2n d:3rd = 11:77:7. 4.Two vessels A and B contain spirit and water mixed in the ratio 5:2 and 7:6 respectively. Find the ratio n which these mixture be mixed to obtain a new mixture in vessel c containing spirit and water in the ratio 8:5? Solution:Let the C.P of spirit be Re 1 per liter. Spirit in 1 liter mix of A = 5/7 liter. C.P of 1 liter mix in A =5/7. Spirit in 1 liter mix of B = 7/13 liter. C.P of 1 liter mix in B =7/13. Spirit in 1 liter mix of C = 8/13 liter. C.P of 1 liter mix in C =8/13. C.P of 1 liter mixture in A 5/7 C.P of 1 liter mixture in B 7/13 Mean Price 8/13 1/13 9/91 Therefore required ratio = 1/13 : 9/91 = 7:9.

Top
5.A milk vendor has 2 cans of milk .The first contains 5% water and the rest milk. The second contains 50% water. How much milk should he mix from each of the container so as to get 12 liters of milk such that the ratio of water to milk is 3:5? Solution:Let cost of 1 liter milk be Re 1. Milk in 1 liter mixture in 1st can = 3/4 lit. C.P of 1 liter mixture in 1st can =Re 3/4

file:///E|/work/books/placement/09_Aptitude/alligationormixtures.html[1/28/2012 12:44:48 AM]

Milk in 1 liter mixture in 2n d can = 1/2 lit. C.P of 1 liter mixture in 2n d can =Re 1/2 Milk in 1 liter final mixture = 5/8 lit. Mean Price = Re 5/8. C.P of 1 lt mix in 1st Re3/4 C.P of 1 lt mix in 2nd Re1/2 Mean Price 5/8 1/8 1/8 There ratio of two mixtures =1/8 :1/8 = 1:1. So,quantity of mixture taken from each can=1/2*12 = 6 liters. 6.One quantity of wheat at Rs 9.30 p er Kg are mixed with another quality at a certain rate in the ratio 8:7. If the mixture so formed be worth Rs 10 per Kg ,what is the rate per Kg of the second quality of wheat? Solution:Let the rate of second quality be Rs x per Kg. C.P of 1Kg wheat of 1st 980p C.P of 1 Kg wheat of 2nd 100x p Mean Price 1000p 100x-1000p 70 p (100x-1000) / 70 = 8/7 700x -7000 = 560 700x = 7560 =>x = Rs 10.80. Therefore the rate of second quality is Rs10.80 7.8lit are drawn from a wine and is then filled with water. This operation is performed three more times.The ratio of the quantity of wine now left in cask to that of the water is 16:81. How much wine did the cask hold originally? Solution: Let the quantity of the wine in the cask originally be x liters. Then quantity of wine left in cask after 4 operations = x(1- 8/x)4lit. Therefore x((1-(8/x))4)/x = 16/81. (1- 8/x)4=(2/3) 4 (x- 8)/x=2/3 3x-24 =2x x=24. 8.A can contains a mixture of two liquids A and B in the ratio 7:5 when 9 liters of mixture are drawn off and the can is filled with B,the ratio of A and B becomes 7:9. How many liters of liquid A was contained by the can initially? Solution: Suppose the can initially contains 7x and 5x liters of mixtures A and B respectively . Quantity of A in mixture left = (7x- (7/12)*9 )lit = 7x - (21/4) liters. Quantity of B in mixture left = 5x - 5/12*9 = 5x - (15/4) liters Therefore (7x 21/4)/ (5x 15/4+9)=7/9 (28x-21)/(20x +21)= 7/9 (252x -189)= 140x +147 112x = 336 => x=3. So the can contains 21 liters of A. 9.A vessel is filled with liquid,3 parts of which are water and 5 parts syrup. How much of the mixture must be drawn off and replaced with water so that the mixture may be half water and half syrup? Solution: Suppose the vessal initially contains 8 liters of liquid. Let x liters of this liquid be replaced with water then quantity of water in new mixture = 3-(3x/8)+x liters. Quantity of syrup in new mixture = 5 - 5x/8 liters. Therefore 3 - 3x/8 +x = 5 - 5x/8 5x+24 = 40-5x 10x = 16. x= 8/5. So part of the mixture replaced = 8/5*1/8 =1/5.

Back Back To Main

Top

Contact: 040-23000700

file:///E|/work/books/placement/09_Aptitude/alligationormixtures.html[1/28/2012 12:44:48 AM]

APTITUDE
Numbers H.C.F and L.C.M Decimal Fractions Simplification Square and Cube roots Average Problems on Numbers Problems on Ages Surds and Indices Percentage Profit and Loss Ratio And Proportions Partnership Chain Rule Time and Work Pipes and Cisterns Time and Distance Trains Boats and Streams Alligation or Mixture Simple Interest Compound Interest Logorithms Areas Volume and Surface area Races and Games of Skill Calendar Clocks Stocks ans Shares True Discount Bankers Discount Oddmanout and Series Data Interpretation probability Permutations and Combinations Puzzles BACK

ALLIGATION OR MIXTURES

SOLVED PROBLEMS

Simple problems: 1.In what ratio must rice at Rs 9.30 per Kg be mixed with rice at Rs 10.80 per Kg so that the mixture be worth Rs 10 per Kg? Solution: C.P of 1 Kg rice of 1st kind 930 p C.P of 1 Kg rice of 2n d kind 1080p Mean Price 1000p 80 70 Required ratio=80:70 = 8:7 2.How much water must be added to 60 liters of milk at 11/2 liters for Rs 20 so as to have a mixture worth Rs 10 2/3 a liter? Solution:C.P of 1 lit of milk = 20*2/3 = 40/3 C.P of 1 lit of water 0 C.P of 1 lit of milk 40/3 Mean Price 32/3 8/3 32/3 Ratio of water and milk =8/3 : 32/3 = 1:4 Quantity of water to be added to 60 lit of milk =1/4*60=15 liters. 3.In what ratio must water to be mixed with milk to gain 20% by selling the mixture at cost price? Solution:Let the C.P of milk be Re 1 per liter Then S.P of 1 liter of mixture = Re.1 Gain obtained =20%. Therefore C.P of 1 liter mixture = Rs(100/120*1) =5/6 C.P of 1 liter of water 0 C.P of 1 liter of milk1 Mean Price 5/6 1/6 5/6 Ratio of water and milk =1/6 : 5/6 = 1:5. 4.In what ratio must a grocer mix two varieties of pulses costing Rs 15 and Rs 20 per Kg respectively so as to get a mixture worth Rs 16.50 per Kg? Solution: Cost of 1 Kg pulses of 1 kind 15 Cost of 1 Kg pulses of 2nd kind 20 Mean Price Rs 16.50 3.50 1.50 Required ratio =3.50 : 1.50 = 35:15 = 7:3. 5. 4Kg s of rice at Rs 5 per Kg is mixed with 8 Kg of rice at Rs 6 per Kg .Find the average price of the mixture? Solution: rice of 5 Rs per Kg rice of 6 Rs per Kg Average price Aw 6-Aw Aw-5 (6-Aw)/(Aw-5) = 4/8 =1/2 12-2Aw =Aw-5 3Aw = 17 Aw = 5.66 per Kg. 6.5Kg of rice at Rs 6 per Kg is mixed with 4 Kg of rice to get a mixture costing Rs 7 per Kg. Find the price of the costlier rice? Solution:Using the cross method: rice at Rs 6 per Kg rice at Rs x per Kg Mean price Rs 7 per Kg 5 4 x-7 : 1 = 5 : 4 4x-28 = 5 4x=33=>x=Rs 8.25. Therefore price of costlier rice is Rs 8.25 per Kg Medium Problems: 7.A butler stole wine from a butt of sherry which contained 40% of spirit and he replaced,what he had stolen by wine containing only 16% spirit. The butt was then of 24% strength only. How much of the butt did he steal? Solution: Wine containing 40%spirit Wine containing 16% spirit Wine containing 24% spirit 8 16 They must be mixed in the ratio of =1:2. Thus 1/3 of the butt of sherry was left and hence the butler drew out 2/3 of the butt. 8.The average weekly salary per head of the entire staff of a factory consisting of supervisors and the laborers is Rs 60.The average salary per head of the supervisors is Rs 400 and that of the laborers is Rs 56.Given that the number of supervisors is 12.Find the number of laborers in the factory. Solution: Average salary of laborer Rs 56 Average salary of supervisors Rs 400 Average salary of entire staff Rs 60 340 4 Number of laborer / Number of Supervisors = 340 / 4=85/1 Thus,if the number of supervisors is 1,number of laborers =85. Therefore if the number of supervisors is 12 number of laborers 85*12=1020. 9.The cost of type 1 rice is Rs 15 per Kg and type 2 rice is Rs 20 per Kg. If both type1 and type 2 are mixed in the ratio of 2:3,then the price per Kg of the mixed variety of rice is? Solution:Let the price of the mixed variety be Rs x per Kg. Cost of 1 Kg of type 1 rice Rs 15 Cost of 1 Kg of type 2 rice Rs 20 Mean Price Rs x 20-x x-15 (20x) /( x-15) = 2/3 => 60-3x = 2x-30 5x = 90=>x=18. 10.In what ratio must a grocer mix two varieties of tea worth Rs 60 a Kg and Rs 65 a Kg so that by selling the mixture at Rs 68.20 a Kg he may gain 10%? Solution:S.P of 1 Kg of the mixture = Rs 68.20,gain =10% S.P of 1 Kg of the mixture = Rs (100/110*68.20)=Rs 62. Cost of 1 Kg tea of 1st kind 60 Cost of 1 Kg tea of 2nd kind 65 Mean Price Rs 62 3 2 Required ratio =3:2. 11.A dishonest milkman professes to sell his milk at cost price but he mixes t with water and there by gains 25% .The percentage of water in the mixture is? Solution:Let C. P of 1 liter milk be Re 1. Then S.P of 1 liter mixture=Re 1. Gain=25% C.P of 1 liter mixture =Re(100/125*1) = Re 4/5. C.P of 1 liter milk Re 1 C.P of 1 liter of water 0 Mean Price 4/5 4/5 1/5 Ratio of milk to water =4/5 : 1/5 = 4:1 Hence percentage of water n the mixture=1/5*100=20%. 12.A merchant has 1000Kg of sugar,part of which he sells at 8% profit and the rest at 18% profit. He gains 14% on the whole .The quantity sold at 18% profit is? Solution: Profit on 1st part 8% Profit on 2nd part 18% Mean Profit 14% 4 6 Ratio of 1st and 2nd parts =4:6 =2:3. Quantity of 2nd ind =3/5*1000Kg =600 Kg. 13.A jar full of whiskey contains 40% alcohol. A part of this whiskey is replaced by another containing 19% alcohol and now the percentage of alcohol was found to be 26%.The quantity of whiskey replaced is? Solution:Strength of first jar 40% Strength of 2nd jar 19% Mean Strength 26% 7 14 So,ratio of 1st and 2nd quantities =7:14 =1:2 Therefore required quantity replaced =2/3. 14.A container contains 40lit of milk. From this container 4 lit of milk was taken out and replaced by water. This process was repeated further two times. How much milk is now contained by the container? Solution:Amount of milk left after 3 operations = 40(1-4/40)3lit =(40*9/10*9/10*9/10) = 29.16 lit Complex Problems: 15.Tea worth Rs 126 per Kg are mixed with a third variety in the ratio 1:1:2. If the mixture is worth Rs 153 per Kg ,the price of the third variety per Kg will be? Solution:Since First and second varieties are mixed in equal proportions so their average price =Rs (126+135)/2 = 130.50. So the mixture is formed by mixing two varieties ,one at Rs 130.50 per Kg and the other at say Rs x per Kg in the ratio 2:2 i e,1:1 we have to find x. Cost of 1 Kg tea of 1st kind RS 130.50 Cost of 1 Kg tea of 2n d kind Rs x. Mean Price Rs 153 x-153 22.50 (x=153)/22.5 = 1 =>x-153 = 22.5 x = 175.50. Price of the third variety =Rs 175.50 per Kg. 16.The milk and water in two vessels A and B are in the ratio 4:3 and 2:3 respectively. In what ratio the liquids in both the vessels be mixed to obtain a new mixture in vessel c consisting half milk and half water? Solution:Let the C.P of milk be Re 1 per liter. Milk in 1 liter mixture of A = 4/7 liter. Milk in 1 liter mixture of B = 2/5 liter. Milk in 1 liter mixture of C = 1/2 liter. C.P of 1 liter mixture in A=Re 4/7 C.P of 1 liter mixture in B=Re 2/5. Mean Price = Re . By rule of allegation we have: C.P of 1 liter mixture in A C.P of 1 liter mixture

file:///E|/work/books/placement/09_Aptitude/alligationproblems.html[1/28/2012 12:44:49 AM]

in B 4/7 2/5 Mean Price 1/10 1/14 Required ratio = 1/10 : 1/14 = 7:5. 17.How many Kg s of wheat costing him Rs 1.20,Rs 1.44 and Rs 1.74 per Kg so that the mixture may be worth Rs 1.41 per Kg? Solution: Step1:Mix wheat of first and third kind to get a mixture worth Rs 1.41 per Kg. C.P of 1 Kg wheat of 1st kind 120p C.P of 1 Kg wheat of 3rd kind 174p Mean Price 141p 33 21 They must be mixed in the ratio =33:21 = 11:7 Step2:Mix wheats of 1st and 2n d kind to obtain a mixture worth of 1.41.per Kg. C.P of 1 Kg wheat of 1st kind 120p C.P of 1 Kg wheat of 2n d kind 144p Mean Price 141p 3 21 They must be mixed in the ratio = 3:21=1:7. Thus,Quantity of 2n d kind of wheat / Quantity of 3rd kind of wheat = 7/1*11/7= 11/1 Quantities of wheat of 1st :2n d:3rd = 11:77:7. 18.Two vessels A and B contain spirit and water mixed in the ratio 5:2 and 7:6 respectively. Find the ratio n which these mixture be mixed to obtain a new mixture in vessel c containing spirit and water in the ratio 8:5? Solution:Let the C.P of spirit be Re 1 per liter. Spirit in 1 liter mix of A = 5/7 liter. C.P of 1 liter mix in A =5/7. Spirit in 1 liter mix of B = 7/13 liter. C.P of 1 liter mix in B =7/13. Spirit in 1 liter mix of C = 8/13 liter. C.P of 1 liter mix in C =8/13. C.P of 1 liter mixture in A 5/7 C.P of 1 liter mixture in B 7/13 Mean Price 8/13 1/13 9/91 Therefore required ratio = 1/13 : 9/91 = 7:9. 19.A milk vendor has 2 cans of milk .The first contains 5% water and the rest milk. The second contains 50% water. How much milk should he mix from each of the container so as to get 12 liters of milk such that the ratio of water to milk is 3:5? Solution:Let cost of 1 liter milk be Re 1. Milk in 1 liter mixture in 1st can = lit. C.P of 1 liter mixture in 1st can =Re Milk in 1 liter mixture in 2n d can = 1/2 lit. C.P of 1 liter mixture in 2n d can =Re Milk in 1 liter final mixture = 5/8 lit. Mean Price = Re 5/8. C.P of 1 liter mixture in 1st can Re C.P of 1 liter mixture in 2n d can Re Mean Price 5/8 1/8 1/8 There ratio of two mixtures =1/8 :1/8 = 1:1. So,quantity of mixture taken from each can =1/2*12 = 6 liters. 20.One quantity of wheat at Rs 9.30 per Kg are mixed with another quality at a certain rate in the ratio 8:7. If the mixture so formed be worth Rs 10 per Kg ,what is the rate per Kg of the second quality of wheat? Solution:Let the rate of second quality be Rs x per Kg. C.P of 1 Kg wheat of 1st kind 980p C.P of 1 Kg wheat of 2n d kind 100x p Mean Price 1000p 100x-1000 p 70 p (100x-1000) / 70 = 8/7 700x -7000 = 560 700x = 7560 =>x = Rs 10.80. Therefore the rate of second quality is Rs10.80 21.8lit are drawn from a wine and is then filled with water.This operation is performed three more times.The ratio of the quantity of wine now left in cask to that of the water is 16:81. How much wine did the cask hold originally? Solution:Let the quantity of the wine in the cask originally be x liters. Then quantity of wine left in cask after 4 operations = x(1- 8/x)4lit. Therefore x(1- 8/x)4 /x = 16/81. (1- 8/x)4 =(2/3) 4 (x- 8)/x =2/3 3x-24 =2x x=24. 22.A can contains a mixture of two liquids A and B in the ratio 7:5 when 9 liters of mixture are drawn off and the can is filled with B,the ratio of A and B becomes 7:9. How many liters of liquid A was contained by the can initially? Solution:Suppose the can initially contains 7x and 5x liters of mixtures A and B respectively . Quantity of A in mixture left = (7x 7/12*9)lit = 7x 21/4 liters. Quantity of B in mixture left = 5x 5/12*9 = 5x 15/4 liters Therefore (7x 21/4) / (5x 15/4+9) = 7/9 (28x-21) / (20x +21) 7/9 252x -189 = 140x +147 112x = 336 => x=3. So the can contains 21 liters of A. 23.A vessel is filled with liquid,3 parts of which are water and 5 parts syrup. How much of the mixture must be drawn off and replaced with water so that the mixture may be half water and half syrup? Solution:Suppose the vessal initially contains 8 liters of liquid. Let x liters of this liquid be replaced with water then quantity of water in new mixture = 3 - 3x/8 +x liters. Quantity of syrup in new mixture = 5 5x/8 liters. Therefore 3 - 3x/8 +x = 5 - 5x/8 5x+24 = 40-5x 10x = 16. x= 8/5. So part of the mixture replaced = 8/5*1/8 =1/5.

file:///E|/work/books/placement/09_Aptitude/alligationproblems.html[1/28/2012 12:44:49 AM]

APTITUDE
Numbers H.C.F and L.C.M Decimal Fractions Simplification Square and Cube roots Average Problems on Numbers Problems on Ages Surds and Indices Percentage Profit and Loss Ratio And Proportions Partnership Chain Rule Time and Work Pipes and Cisterns Time and Distance Trains Boats and Streams Alligation or Mixture Simple Interest Compound Interest Logorithms Areas Volume and Surface area Races and Games of Skill Calendar Clocks Stocks ans Shares True Discount Bankers Discount Oddmanout and Series Data Interpretation probability Permutations and Combinations Puzzles BACK

ALLIGATION OR MIXTURES SOLVED PROBLEMS


Simple problems: 1.In what ratio must rice at Rs 9.30 per Kg be mixed with rice at Rs 10.80 per Kg so that the mixture be worth Rs 10 per Kg? Solution:
C.P of 1 Kg rice of 1st kind 930 p C.P of 1 Kg rice of 2n d kind 1080p

Mean Price 1000p 80 70

Required ratio=80:70 = 8:7 2.How much water must be added to 60 liters of milk at 11/2 liters for Rs 20 so as to have a mixture worth Rs 10 2/3 a liter? Solution:C.P of 1 lit of milk = 20*2/3 = 40/3
C.P of 1 lit of water 0 C.P of 1 lit of milk 40/3

Mean Price 32/3 8/3 32/3

Ratio of water and milk =8/3 : 32/3 = 1:4 Quantity of water to be added to 60 lit of milk =1/4*60=15 liters.

3.In what ratio must water to be mixed with milk to gain 20% by selling the mixture at cost price? Solution:Let the C.P of milk be Re 1 per liter Then S.P of 1 liter of mixture = Re.1 Gain obtained =20%. Therefore C.P of 1 liter mixture = Rs(100/120*1) =5/6
C.P of 1 liter of water 0 C.P of 1 liter of milk1

Mean Price 5/6 1/6 5/6

Ratio of water and milk =1/6 : 5/6 = 1:5.

file:///E|/work/books/placement/09_Aptitude/alligationsimple.html[1/28/2012 12:44:49 AM]

4.In what ratio must a grocer mix two varieties of pulses costing Rs 15 and Rs 20 per Kg respectively so as to get a mixture worth Rs 16.50 per Kg? Solution:
Cost of 1 Kg pulses of 1 kind 15 Cost of 1 Kg pulses of 2nd kind 20

Mean Price Rs 16.50 3.50 1.50

Required ratio =3.50 : 1.50 = 35:15 = 7:3.

5. 4Kg s of rice at Rs 5 per Kg is mixed with 8 Kg of rice at Rs 6 per Kg .Find the average price of the mixture? Solution:
rice of 5 Rs per Kg rice of 6 Rs per Kg

Average price Aw 6-Aw Aw-5

(6-Aw)/(Aw-5) = 4/8 =1/2 12-2Aw = Aw-5 3Aw = 17 Aw = 5.66 per Kg.

6.5Kg of rice at Rs 6 per Kg is mixed with 4 Kg of rice to get a mixture costing Rs 7 per Kg. Find the price of the costlier rice? Solution:Using the cross method:
rice at Rs 6 per Kg rice at Rs x per Kg

Mean price Rs 7 per Kg 5 4

x-7:1=5:4 4x-28 = 5 4x=33=>x=Rs 8.25. Therefore price of costlier rice is Rs 8.25 per Kg BACK

file:///E|/work/books/placement/09_Aptitude/alligationsimple.html[1/28/2012 12:44:49 AM]

APTITUDE
Numbers H.C.F and L.C.M Decimal Fractions Simplification Square and Cube roots Average Problems on Numbers Problems on Ages Surds and Indices Percentage Profit and Loss Ratio And Proportions Partnership Chain Rule Time and Work Pipes and Cisterns Time and Distance Trains Boats and Streams Alligation or Mixture Simple Interest Compound Interest Logorithms Areas Volume and Surface area Races and Games of Skill Calendar Clocks Stocks ans Shares True Discount Bankers Discount Oddmanout and Series Data Interpretation probability Permutations and Combinations Puzzles

BACK Complex Probems: 1.If the area of a square with side a s equal to the area of a triangle with base a, then the altitude of the triangle is sol: area of a square with side a = a sq unts area of a triangle with base a = * a*h sq unts a =1/2 *a *h => h = 2a altitude of the triangle is 2a 2.An equilateral triangle is described on the diagonal of a square. What is the ratio of the area of the triangle to that of the square? Sol: area of a square = a sq cm length of the diagonal = 2a cm area of equilateral triangle with side 2a = 3/4 * (2a) required ratio = 3a : a = 3 : 2 3.The ratio of bases of two triangles is x:y and that of their areas is a:b. Then the ratio of their corresponding altitudes wll be sol: a/b =( * x*H) /(1/2 * y * h) bxH = ayh =>H/h =ay/bx Hence H:h = ay:bx 4 .A parallelogram has sides 30m and 14m and one of its diagonals is 40m long. Then its area is sol: let ABCD be the given parallelogram area of parallelogram ABCD = 2* (area of triangle ABC) now a = 30m, b = 14m and c = 40m s = (30+14+40) = 42m Area of triangle ABC = [ s(s-a)(s-b)(s-c) = (42*12*28*2 = 168sq m area of parallelogram ABCD = 2 *168 =336 sq m 5.If a parallelogram with area p, a triangle with area R and a triangle with area T are all constructed on the same base and all have the same altitude, then which of the following statements is false? Sol: let each have base = b and height = h then p = b*h, R = b*h and T = * b*h so P = R, P = 2T and T = R are all correct statements 6.If the diagonals of a rhombus are 24cm and 10cm the area and the perimeter of the rhombus are respectively. Sol: area = *diagonal 1 *diagonal 2= * 24 * 10= 120 sq cm * diagonal 1 = * 24 = 12cm * diagonal 2 = *10 =5 cm side of a rhombus = (12) + (5) = 169 => AB = 13cm

file:///E|/work/books/placement/09_Aptitude/areacomplex.html[1/28/2012 12:44:50 AM]

7.If a square and a rhombus stand on the same base, then the ratio of the areas of the square and the rhombus is: sol: A square and a rhombus on the same base are equal in area 8.The area of a field in the shape of a trapezium measures 1440sq m. The perpendicular distance between its parallel sides is 24cm. If the ratio of the sides is 5:3, the length of the longer parallel side is: sol: area of field =1/2 *(5x+3x) *24 = 96x sq m 96x = 1440 => x = 1440 /96 = 15 hence, the length of longer parallel side = 5x = 75m 9.The area of a circle of radius 5 is numerically what percent its circumference? Sol: required percentage = (5)/(2 *5) *100 = 250% 10.A man runs round a circular field of radius 50m at the speed of 12m/hr. What is the time taken by the man to take twenty rounds of the field? Sol: speed = 12 k/h = 12 * 5/18 = 10/3 m/s distance covered = 20 * 2*22/7*50 = 44000/7m time taken = distance /speed = 44000/7 * 3/10 = 220/7min 11.A cow s tethered in the middle of a field with a 14feet long rope.If the cow grazes 100 sq feet per day, then approximately what time will be taken by the cow to graze the whole field? Sol: area of the field grazed = 22/7 * 14 * 14 = 616 sq feet 12.A wire can be bent in the form of a circle of radius 56cm. If it is bent in the form of a square, then its area will be sol: length of wire = 2 r = 2 *22/7 *56 = 352 cm side of the square = 352/4 = 88cm area of the square = 88*88 = 7744sq cm 13.The no of revolutions a wheel of diameter 40cm makes in traveling a distance of 176m is sol: distance covered in 1 revolution = 2 r = 2 *22/7 *20 = 880/7 cm required no of revolutions = 17600 *7/880 = 140 14.The wheel of a motorcycle 70cm in diameter makes 40 revolutions in every 10sec. What is the speed of motorcycle n km/hr? Sol: distance covered in 10sec = 2 *22/7 *35/100 *40 =88m distance covered in 1 sec =88/10m = 8.8m speed =8.8m/s = 8.8 * 18/5 *k/h = 31.68 k/h 15.Wheels of diameters 7cm and 14cm start rolling simultaneously from x & y which are 1980 cm apart towards each other in opposite directions. Both of them make the same number of revolutions per second. If both of them meet after 10seconds. The speed of the smaller wheel is sol: let each wheel make x revolutions per sec. Then (2 *7/2 *x)+(2 * 7*x)*10 = 1980 (22/7 *7 * x) + (2 * 22/7 *7 *x) = 198

file:///E|/work/books/placement/09_Aptitude/areacomplex.html[1/28/2012 12:44:50 AM]

66x = 198 => x = 3 distance moved by smaller wheel in 3 revolutions = 2 *22/7 *7/2 *3 = 66cm speed of smaller wheel = 66/3 m/s = 22m/s 16.A circular swimming pool is surrounded by a concrete wall 4ft wide. If the area of the concrete wall surrounding the pool is 11/25 that of the pool, then the radius of the pool is? Sol: let the radius of the pool be R ft radius of the pool including the wall = (R+4)ft area of the concrete wall = [(R+4)2 - R2 ] => = [R+4+R][R+4-R] = 8 (R+2) sq feet 8 (R+2) = 11/25 R2 => 11 R2 = 200 (R+2) Radius of the pool R = 20ft 17.A semicircular shaped window has diameter of 63cm. Its perimeter equals sol: perimeter of window = r +2r = [22/7 * 63/2 +63] = 99+63 = 162 cm 63.The area of the largest triangle that can be inscribed in a semicircle of radius is sol: required area = * base * height = * 2r * r = r 2 18.Three circles of radius 3.5cm are placed in such a way that each circle touches the other two. The area of the portion enclosed by the circles is sol: required area = (area of an equilateral triangle of side 7 cm) - (3 * area of sector with = 6o degrees and r = 3.5cm) = ( * 7 * 7) (3* 22/7 *3.5 *3.5*60/360 ) sq cm = 493/4 11*0.5*3.5 sq cm = 1.967 sq cm 19. Four circular cardboard pieces, each of radius 7cm are placed in such a way that each piece touches two other pieces. The area of the space encosed by the four pieces is sol: required area = 14*14 (4 * * 22/7 * 7 *7) sq cm = 196 154 = 42 sq cm

file:///E|/work/books/placement/09_Aptitude/areacomplex.html[1/28/2012 12:44:50 AM]

APTITUDE
Numbers H.C.F and L.C.M Decimal Fractions Simplification Square and Cube roots Average Problems on Numbers Problems on Ages Surds and Indices Percentage Profit and Loss Ratio And Proportions Partnership Chain Rule Time and Work Pipes and Cisterns Time and Distance Trains Boats and Streams Alligation or Mixture Simple Interest Compound Interest Logorithms Areas Volume and Surface area Races and Games of Skill Calendar Clocks Stocks ans Shares True Discount Bankers Discount Oddmanout and Series Data Interpretation probability Permutations and Combinations Puzzles

BACK Medium Problems: 11.Find the ratio of the areas of the incircle and circumcircle of a square. Sol: let the side of the square be x, then its diagonal = 2 x radius of incircle = x/2 and radius of circmcircle =2 x /2 = x/2 required ratio = x/4 : x/2 = : = 1:2 12.If the radius of a circle is decreased by 50% , find the percentage decrease in its area. Sol: let original radius = r and new radius = 50/100 r = r/2 original area = r and new area = (r/2) decrease in area = 3 r/4 * 1/ r * 100 = 75% 13.Two concentric circles form a ring. The inner and outer circumference of the ring are 352/7 m and 528/7m respectively. Find the width of the ring. sol: let the inner and outer radii be r and R meters then, 2 r = 352/7 => r = 352/7 * 7/22 * = 8m 2 R = 528/7 => R= 528/7 * 7/22 * = 12m width of the ring = R-r = 12-8 = 4m 14.If the diagonal of a rectangle is 17cm long and its perimeter is 46 cm. Find the area of the rectangle. sol: let length = x and breadth = y then 2(x+y) = 46 => x+y = 23 x+y = 17 = 289 now (x+y) = 23 =>x+y+2xy= 529 289+ 2xy = 529 => xy = 120 area =xy=120 sq. cm 15.A rectangular grassy plot 110m by 65cm has a gravel path .5cm wide all round it on the inside. Find the cost of gravelling the path at 80 paise per sq.mt sol: area of theplot = 110 * 65 = 7150 sq m area of the plot excluding the path = (110-5)* (65-5) = 6300 sq m area of the path = 7150- 6300 =850 sq m cost of gravelling the path = 850 * 80/100 = 680 Rs 16. The perimeters of ttwo squares are 40cm and 32 cm. Find the perimeter of a third square whose area is equal to the difference of the areas of the two squares. sol: side of first square = 40/4 =10cm side of second square = 32/4 = 8cm area of third squre = 10 8 = 36 sq cm side of third square = 36 = 6 cm required perimeter = 6*4 = 24cm

file:///E|/work/books/placement/09_Aptitude/areamedium.html[1/28/2012 12:44:50 AM]

17. A room 5m 44cm long and 3m 74cm broad is to be paved with squre tiles. Find the least number of squre tiles required to cover the floor. sol: area of the room = 544 * 374 sq cm size of largest square tile = H.C.F of 544cm and 374cm= 34cm area of 1 tile = 34*34 sq cm no. of tiles required = (544*374) / (34 * 34) = 176 18. The diagonals of two squares are in the ratio of 2:5. Find the ratio of their areas. sol: let the diagonals of the squares be 2x and 5x respectively ratio of their areas = * (2x) : *(5x) = 4:25 19.If each side of a square is increased by 25%. Find the percentage change in its area. sol: let each side of the square be a then area = a new side = 125a/100 = 5a/4 new area = (5a/4) = 25/16 a increase in area = (25/16)a - a = (9/16)a increase % = (9/16)a * (1/a) * 100 = 56.25% 20.The base of triangular field os three times its altitude. If the cost of cultivating the field at Rs. 24.68 per hectare be Rs. 333.18. Find its base and height. sol: area of the field = total cost/ rate = 333.18 /24.68 = 13.5 hectares => = 13.5 * 10000 = 135000 sq m let the altitude = x mt and base = 3x mt then *3x * x = 135000 => x = 90000 => x = 300 base= 900m and altitude = 300m 21.In two triangles the ratio of the areas is 4:3 and the ratio of their heights is 3:4. Find the ratio of their bases? Sol: let the bases of the two triangles be x &y and their heights be 3h and 4h respectively. (1/2*x*3h)/(1/2*y*4h) =4/3 => x/y = 4/3 *4/3 = 16/9 22.Find the length of a rope by which a cow must be tethered in order that it may be able to graze an area of 9856 sq meters. Sol: clearly the cow will graze a circular field of area 9856 sq m and radius equal to the length of the rope. Let the length of the rope be r mts then r=9856 => r=9856*7/22 = 3136 => r=56m 23.The diameter of the driving wheel of a bus is 140cm. How many revolutions per minute must the wheel make inorder to keep a speed of 66 kmph? Sol: Distance to be covered in 1min = (66*1000)/60 m =1100m diameter = 140cm => radius = r =0.7m circumference of the wheel = 2*22/7*0.7 = 4.4m no of revolutions per minute = 1100/4.4 = 250 24.The inner circumference of a circular race track, 14m wide is 440m. Find the radius of the outer circle. Sol: let inner radius be r meters.

file:///E|/work/books/placement/09_Aptitude/areamedium.html[1/28/2012 12:44:50 AM]

Then 2 r =440 => r=440*7/22*1/2 = 70m radius of outer circle = 70+4 =84m 25.A sector of 120 degrees, cut out from a circle, has an area of 66/7 sq cm. Find the radius of the circle. Sol: let the radius of the circle be r cm. Then r/360 =66/7=> 22/7*r*120/360 = 66/7 =>r = 66/7 *7/22*3 =9 radius = 3cm 26.The length of the room is 5.5m and width is 3.75m. Find the cost of paving the floor by slabs at the rate of Rs.800 per sq meter. Sol: l=5.5m w=3.75m area of the floor = 5.5 * 3.75 = 20.625 sq m cost of paving = 800 *20.625 =Rs. 16500 27.A rectangular plot measuring 90 meters by 50 meters is to be enclosed by wire fencing. If the poles of the fence are kept 5 meters apart. How many poles will be needed? Sol: perimeter of the plot = 2(90+50) = 280m no of poles =280/5 =56m 28.The length of a rectangular plot is 20 meters more than its breadth. If the cost of fencing the plot @ 26.50 per meter is Rs. 5300. What is the length of the plot in meter? Sol: let breadth =x then length = x+20 perimeter = 5300/26.50 =200m 2(x+20+x) =200 => 4x+40 =200 x = 40 and length = 40+20 = 60m 29.A rectangular field is to be fenced on three sides leaving a side of 20 feet uncovered. If the area of the field is 680 sq feet, how many feet of fencing will be required? Sol: l=20feet and l*b=680 => b= 680/20 = 34feet length of fencing = l+2b = 20+68 =88 feet 30.A rectangular paper when folded into two congruent parts had a perimeter of 34cm foer each part folded along one set of sides and the same is 38cm. When folded along the other set of sides. What is the area of the paper? Sol: when folded along the breadth we have 2(l/2 +b) = 34 or l+2b = 34...........(1) when folded along the length, we have 2(l+b/2)=38 or 2l+b =38............(2) from 1 &2 we get l=14 and b=10 Area of the paper = 14*10 = 140 sq cm 31.A took 15 seconds to cross a rectangular field diagonally walking at the rate of 52 m/min and B took the same time to cross the same field along its sides walking at the rate of 68m/min. The area of the field is? Sol: length of the diagonal = 52*15/60 =13m sum of length and breadth = 68*15/60 = 17m (l+b)=13 or l+b = 17 area =lb = (2lb) = [(l+b) (l+b)] = [17 -169]

file:///E|/work/books/placement/09_Aptitude/areamedium.html[1/28/2012 12:44:50 AM]

=1/2*120 = 60 sq meter 32 . A rectangular lawn 55m by 35m has two roads each 4m wide running in the middle of it. One parallel to the length and the other parallel to breadth. The cost of graveling the roads at 75 paise per sq meter is sol: area of cross roads = 55*4 +35*4-4*4 = 344sq m cost of graveling = 344 *75/100 =Rs. 258 33.The cost of fencing a square field @ Rs. 20 per metre is Rs.10.080.How much will it cost to lay a three meter wide pavement along the fencing inside the field @ Rs. 50 per sq m sol: perimeter = total cost / cost per m = 10080 /20 = 504m side of the square = 504/4 = 126m breadth of the pavement = 3m side of inner square = 126-6 = 120m area of the pavement = (126*126)-(120*120)=246*6 sq m cost of pavement = 246*6*50 = Rs. 73800 34.Amanwalked diagonally across a square plot. Approximately what was the percent saved by not walking along the edges? Sol: let the side of the square be x meters length of two sides = 2x meters diagonal = 2 x = 1.414x m saving on 2x meters = .59x m saving % = 0.59x /2x *100% = 30% (approx) 36.A man walking at the speed of 4 kmph crosses a square field diagonally in 3 meters. The area of the field is sol: speed of the man = 4*5/18 m/sec = 10/9 m/sec time taken = 3*60 sec = 180 sec length of diagonal = speed * time = 10/9 * 180 = 200m Area of the field = *(dioagonal) = * 200*200 sq m = 20000sq m 37.A square and rectangle have equal areas. If their perimeters are p and q respectively. Then sol: A square and a rectangle with equal areas will satisfy the relation p < q 38.If the perimeters of a square and a rectangle are the same, then the area a & b enclosed by them would satisfy the condition: sol: Take a square of side 4cm and a rectangle having l=6cm and b=2cm then perimeter of square = perimeter of rectangle area of square = 16 sq cm area of rectangle = 12 sq cm Hence a >b 39.An error of 2% in excess is made while measuring the side of a square. The percentage of error in the calculated area of the square is sol: 100cm is read as 102 cm a = 100*100 sq cm and b = 102 *102 sq cm then a-b = 404 sq cm

file:///E|/work/books/placement/09_Aptitude/areamedium.html[1/28/2012 12:44:50 AM]

percentage error = 404/(100*100) = 4.04% 40.A tank is 25m long 12m wide and 6m deep. The cost of plastering its walls and bottom at 75 paise per sq m is sol: area to be plastered = [2(l+b)*h]+(l*b) = 2(25+12)*6 + (25*12)= 744 sq m cost of plastering = Rs . 744*75/100 = Rs. 5581 41.The dimensions of a room are 10m*7m*5m. There are 2 doors and 3 windows in the room. The dimensions of the doors are 1m*3m. One window is of size 2m*1.5m and the other 2 windows are of size 1m*1.5m. The cost of painting the walls at Rs. 3 per sq m is sol: Area of 4 walls = 2(l+b)*h =2(10+7)*5 = 170 sq m Area of 2 doors and 3 windows = 2(1*3)+(2*1.5)+2(1*1.5) = 12 sq m area to be planted = 170 -12 = 158 sq m cost of painting = Rs. 158 *3 = Rs. 474 42.The base of a triangle of 15cm and height is 12cm. The height of another triangle of double the area having the base 20cm is sol: a = *15*12 = 90 sq cm b = 2a = 2 * 90 = * 20 *h => h= 18cm 43.The sides of a triangle are in the ratio of :1/3:1/4. If the perimeter is 52cm, then the length of the smallest side is sol: ratio of sides = :1/3 :1/4 = 6:4:3 perimeter = 52 cm, so sides are 52*6/13 =24cm 52*4/13 = 16cm 52 *3/13 = 12cm length of smallest side = 12cm 44.The height of an equilateral triangle is 10cm. Its area is sol: a = (a/2) +(10) a a/4 = 100 =>3a = 100*4 area = 3/4 *a = 3/4*400/3 = 100/3 sq cm 45.From a point in the interior of an equilateral triangle, the perpendicular distance of the sides are 3 cm, 23cm and 53cm. The perimeter of the triangle is sol: let each side of the triangle be a cm then area(AOB) +area(BOC)+area(AOC) = area(ABC) * a *3 +1/2 *a *23 +1/2 * a*53 = 3/4 a a/23(1+2+5) = 3/4 a => a=16 perimeter = 3*16 = 48cm

file:///E|/work/books/placement/09_Aptitude/areamedium.html[1/28/2012 12:44:50 AM]

Areas
Important Facts and Formulae: Results On Triangle 1.Sum of the angles of a triangle is 180 degrees. 2.The sum of any two sides of a triangle is greater than third side. 3.Pythagoras Theorem: In a right angled triangle (Hypotenuse)2 = (Base)2 +(Height)2 4.The line joining the mid point of a side of a triangle to the opposite vertex is called the MEDIAN. 5.The point where the three medians of a triangle meet, is called CENTROID. The centroid divides each of the medians in the ratio 2:1 6.In an isosceles triangle, the altitude from the vertex bisects the base 7.The median of a triangle divides it into two triangles of the same area. 8.The area of the triangle formed by joining the mid points of the sides of a given triangle is one - fourth of the area of the given triangle. Results On Quadrilaterals 1.The diagonals of a Parallelogram bisect each other. 2.Each diagonal of a Parallelogram divides it into two triangles of the same area. 3.The diagonals of a Rectangle are equal and bisect each other 4.The diagonals of a Square are equal and bisect each other at right angles. 5.The diagonals of a Rhombus are unequal and bisect each other at right angles. 6.A Parallelogram and a Rectangle on the same base and between the same parallels are equal in area. 7.Of all he parallelogram of given sides the parallelogram which is a rectangle has the greatest area.

Top
Formulae 1.Area of a RECTANGLE = length * breadth Length = (Area/Breadth) and Breadth = (Area/Length) 2.Perimeter of a RECTANGLE = 2(Length + Breadth) 3.Area of a SQUARE = (side)2 = ( Diagonal)2 4.Area of four walls of a room = 2(length + breadth) * height 5.Area of a TRIANGLE = * base * height 6.Area of a TRIANGLE = [s * (s - a) * (s - b) * (s - c)], where a,b,c are the sides of the triangle and s = 1/2(a+b+c) 7.Area of EQUILATERAL TRIANGLE = (3/4)* (side)2 8.Radius of in circle of an EQUILATERAL TRIANGLE of side a = r / 23 9.Radius of circumcircle of an EQUILATERAL TRIANGLE of side a = r / 3 10.Radius of incircle of a triangle of area and semi perimeter S = / s 11.Area of a PARALLELOGRAM = (base * height) 12.Area of RHOMBUS = 1/2 (product of diagonals) 13.Area of TRAPEZIUM = =1/2 * (sum of parallel sides)* (distance between them)

file:///E|/work/books/placement/09_Aptitude/areas.html[1/28/2012 12:44:51 AM]

14.Area of a CIRCLE = r2 where r is the radius 15.Circumference of a CIRCLE = 2r 16.Length of an arc = 2 r / 360, where is central angle 17.Area of a SECTOR = (arc * r) = r2 / 360 18.Area of a SEMICIRCLE = r2 / 2 19.Circumference of a SEMICIRCLE = r

Top
Simple Problems 1.One side of a rectangular field is 15m and one of its diagonal is 17m. Find the area of field ? Sol: Other side = [(17*17) (15*15)] = (289 - 225) = 8m Area = 15 * 8 =120 sq. m 2.A lawn is in the form of a rectangle having its sides in the ratio 2:3 The area of the lawn is 1/6 hectares. Find the length and breadth of the lawn. Sol: let length = 2x meters and breadth = 3x mt Now area = (1/6 * 1000)sq m = 5000/3 sq m 2x * 3x = 5000/3 =>x * x =2500 / 9 x = 50/3 length = 2x = 100/3 m and breadth = 3x = 3*(50/3) = 50m

3.Find the cost of carpeting a room 13m long and 9m broad with a carpet 75cm wide at the rate of Rs 12.40 per sq meter Sol: Area of the carpet = Area of the room = 13* 9 =117 sq m length of the carpet = (Area/width) = 117 * (4/3) = 156 m Cost of carpeting = Rs (156 * 12.40) = Rs 1934.40 4.The length of a rectangle is twice its breadth if its length is decreased by 5cm and breadth is increased by 5cm, the area of the rectangle is increased by 75 sq cm. Find the length of the rectangle. Sol: let length = 2x and breadth = x then (2x - 5) (x+5) (2x*x)=75 5x - 25 = 75 => x=20 length of the rectangle = 40 cm

5.In measuring the sides of a rectangle, one side is taken 5% in excess and the other 4% in deficit. Find the error percent in the area, calculate from the those measurements. Sol: let x and y be the sides of the rectangle then correct area = (105/100 * x) * (96 / 100 *y) =(504/500 xy) xy = 4/500 xy Error% = 4/500 xy*(1/xy)*100 % = 4/5% = 0.8%

6.A room is half as long again as it is broad. The cost of carpeting the room at Rs 5 per sq m is Rs 2.70 and the cost of papering the four walls at Rs 10 per sq m is Rs 1720. If a door and 2 windows occupy 8 sq cm. Find the dimensions of the room ? Sol: let breadth=x mt ,length= 3x/2 mt and height=h mt Area of the floor = (total cost of carpeting /rate) = 270/5 sq m = 54 sq m x * 3x/2=54 => x*x= 54*(2/3)=36 => x = 6m so breadth = 6m and length=3/2*6 = 9m now papered area = 1720 /10 = 172 sq m Area of one door and 2 windows =8 sq m total area of 4 walls = 172+8 = 180 sq m 2(9+6)*h = 180 => h=180/30 = 6m 7.The altitude drawn to the base of an isosceles triangle is 8cm and the perimeter is 32cm. Find the area of the triangle ? Sol: let ABC be the isosceles triangle, the AD be the altitude let AB = AC=x then BC= 32 - 2x since in an isoceles triange the altitude bisects the base so BD=DC=16 - x in ADC,(AC) 2 = (AD) 2 + (DC) 2 x*x=(8*8) + (16 - x)*(16 - x) 32x =320 => x = 10 BC = 32-2x = 32-20 = 12 cm Hence, required area = * BC * AD = * 12 * 10 = 60 sq cm 8.If each side of a square is increased by 25%, find the percentage change in its area ? Sol: let each side of the square be a , then area = a * a New side = 125a / 100 = 5a / 4 New area =(5a * 5a)/(4*4) = (25a/16) a = 9a/16 Increase %= 9a/16 * 1/a * 100% = 56.25% 9.Find the area of a Rhombus one side of which measures 20cm and one diagonal 24cm. Sol: Let other diagonal = 2x cm since diagonals of a rhombus bisect each other at right angles, we have 20 = 12 + x => x = [20 - 12]= 256 = 16cm so the diagonal = 32 cm Area of rhombus = * product of diagonals = * 24 * 32 = 384 sq cm

file:///E|/work/books/placement/09_Aptitude/areas.html[1/28/2012 12:44:51 AM]

10. The area of a circular field is 13.86 hectares. Find the cost of fencing it at the rate of Rs. 4.40 per meter. Sol: Area = 13.86 * 10000 sq m = 138600 sq m r= 138600 => r = 138600 * 7/22 => 210 m circumference = 2r = 2 * 22/7 * 210m = 1320 m cost of fencing = Rs 1320 * 4.40 = Rs. 5808

Top
Medium Problems: 11.Find the ratio of the areas of the incircle and circumcircle of a square. Sol: let the side of the square be x, then its diagonal = 2 x radius of incircle = x/2 and radius of circmcircle =2 x /2 = x/2 required ratio = x/4 : x/2 = : = 1:2 12.If the radius of a circle is decreased by 50% , find the percentage decrease in its area. Sol: let original radius = r and new radius = 50/100 r = r/2 original area = r and new area = (r/2) decrease in area = 3 r/4 * 1/ r * 100 = 75% 13.Two concentric circles form a ring. The inner and outer circumference of the ring are 352/7 m and 528/7m respectively. Find the width of the ring. sol: let the inner and outer radii be r and R meters then, 2r = 352/7 => r = 352/7 * 7/22 * = 8m 2R = 528/7 => R= 528/7 * 7/22 * = 12m width of the ring = R-r = 12-8 = 4m 14.If the diagonal of a rectangle is 17cm long and its perimeter is 46 cm. Find the area of the rectangle. sol: let length = x and breadth = y then 2(x+y) = 46 => x+y = 23 x+y = 17 = 289 now (x+y) = 23 =>x+y+2xy= 529 289+ 2xy = 529 => xy = 120 area =xy=120 sq. cm 15.A rectangular grassy plot 110m by 65cm has a gravel path .5cm wide all round it on the inside. Find the cost of gravelling the path at 80 paise per sq.mt sol: area area cost area of theplot = 110 * 65 = 7150 sq m of the plot excluding the path = (110 - 5)* (65 - 5) = 6300 sq m of the path = 7150 - 6300 =850 sq m of gravelling the path = 850 * 80/100 = 680 Rs

16. The perimeters of ttwo squares are 40cm and 32 cm. Find the perimeter of a third square whose area is equal to the difference of the areas of the two squares. sol: side of first square = 40/4 =10cm side of second square = 32/4 = 8cm area of third squre = 10 8 = 36 sq cm side of third square = 36 = 6 cm required perimeter = 6*4 = 24cm

Top
17. A room 5m 44cm long and 3m 74cm broad is to be paved with squre tiles. Find the least number of squre tiles required to cover the floor. sol: area of the room = 544 * 374 sq cm size of largest square tile = H.C.F of 544cm and 374cm= 34cm area of 1 tile = 34*34 sq cm no. of tiles required = (544*374) / (34 * 34) = 176 18. The diagonals of two squares are in the ratio of 2:5. Find the ratio of their areas. sol: let the diagonals of the squares be 2x and 5x respectively ratio of their areas = * (2x) : *(5x) = 4:25 19.If each side of a square is increased by 25%. Find the percentage change in its area. sol: let new side new area increase increase each side of the square be a then area = a = 125a/100 = 5a/4 = (5a/4) = 25/16 a in area = (25/16)a - a = (9/16)a % = (9/16)a * (1/a) * 100 = 56.25%

20.The base of triangular field os three times its altitude. If the cost of cultivating the field at Rs. 24.68 per hectare be Rs. 333.18. Find its base and height. sol: area of the field = total cost/ rate = 333.18 /24.68 = 13.5 hectares => = 13.5 * 10000 = 135000 sq m let the altitude = x mt and base = 3x mt then *3x * x = 135000 => x = 90000 => x = 300 base= 900m and altitude = 300m

file:///E|/work/books/placement/09_Aptitude/areas.html[1/28/2012 12:44:51 AM]

21.In two triangles the ratio of the areas is 4:3 and the ratio of their heights is 3:4. Find the ratio of their bases ? Sol: let the bases of the two triangles be x &y and their heights be 3h and 4h respectively. (1/2*x*3h)/(1/2*y*4h) =4/3 => x/y = 4/3 *4/3 = 16/9 22.Find the length of a rope by which a cow must be tethered in order that it may be able to graze an area of 9856 sq meters. Sol: clearly the cow will graze a circular field of area 9856 sq m and radius equal to the length of the rope. Let the length of the rope be r mts then r=9856 => r=9856*7/22 = 3136 => r=56m 23.The diameter of the driving wheel of a bus is 140cm. How many revolutions per minute must the wheel make inorder to keep a speed of 66 kmph? Sol: Distance to be covered in 1min = (66*1000)/60 m =1100m diameter = 140cm => radius = r =0.7m circumference of the wheel = 2*22/7*0.7 = 4.4m no of revolutions per minute = 1100/4.4 = 250 24.The inner circumference of a circular race track, 14m wide is 440m. Find the radius of the outer circle. Sol: let inner radius be r meters. Then 2r =440 => r=440*7/22*1/2 = 70m radius of outer circle = 70+4 =84m 25.A sector of 120 degrees, cut out from a circle, has an area of 66/7 sq cm. Find the radius of the circle. Sol: let the radius of the circle be r cm. Then r/360 =66/7=> 22/7*r*120/360 = 66/7 =>r = 66/7 *7/22*3 =9 radius = 3cm 26.The length of the room is 5.5m and width is 3.75m. Find the cost of paving the floor by slabs at the rate of Rs.800 per sq meter. Sol: l=5.5m w=3.75m area of the floor = 5.5 * 3.75 = 20.625 sq m cost of paving = 800 *20.625 =Rs. 16500

Top
27.A rectangular plot measuring 90 meters by 50 meters is to be enclosed by wire fencing. If the poles of the fence are kept 5 meters apart. How many poles will be needed ? Sol: perimeter of the plot = 2(90+50) = 280m no of poles =280/5 =56m 28.The length of a rectangular plot is 20 meters more than its breadth. If the cost of fencing the plot @ 26.50 per meter is Rs. 5300. What is the length of the plot in meter ? Sol: let breadth =x then length = x+20 perimeter = 5300/26.50 =200m 2(x+20+x) =200 => 4x+40 =200 x = 40 and length = 40+20 = 60m 29.A rectangular field is to be fenced on three sides leaving a side of 20 feet uncovered. If the area of the field is 680 sq feet, how many feet of fencing will be required ? Sol: l=20feet and l*b=680 => b= 680/20 = 34feet length of fencing = l+2b = 20+68 =88 feet 30.A rectangular paper when folded into two congruent parts had a perimeter of 34cm foer each part folded along one set of sides and the same is 38cm. When folded along the other set of sides. What is the area of the paper ? Sol: when folded along the breadth we have 2(l/2 +b) = 34 or l+2b = 34...........(1) when folded along the length, we have 2(l+b/2)=38 or 2l+b =38.....(2) from 1 &2 we get l=14 and b=10 Area of the paper = 14*10 = 140 sq cm 31.A took 15 seconds to cross a rectangular field diagonally walking at the rate of 52 m/min and B took the same time to cross the same field along its sides walking at the rate of 68m/min. The area of the field is ? Sol: length of the diagonal = 52*15/60 =13m sum of length and breadth = 68*15/60 = 17m (l+b)=13 or l+b = 17 area =lb = (2lb) = [(l+b) (l+b)] = [17 - 169] =1/2*120 = 60 sq meter 32 . A rectangular lawn 55m by 35m has two roads each 4m wide running in the middle of it. One parallel to the length and the other parallel to breadth. The cost of graveling the roads at 75 paise per sq meter is sol: area of cross roads = 55*4 +35*4 - 4*4 = 344sq m cost of graveling = 344 *75/100 =Rs. 258

file:///E|/work/books/placement/09_Aptitude/areas.html[1/28/2012 12:44:51 AM]

33.The cost of fencing a square field @ Rs. 20 per metre is Rs.10.080. How much will it cost to lay a three meter wide pavement along the fencing inside the field @ Rs. 50 per sq m sol: perimeter = total cost / cost per m = 10080 /20 = 504m side of the square = 504/4 = 126m breadth of the pavement = 3m side of inner square = 126 - 6 = 120m area of the pavement = (126*126) - (120*120)=246*6 sq m cost of pavement = 246*6*50 = Rs. 73800 34.Amanwalked diagonally across a square plot. Approximately what was the percent saved by not walking along the edges ? Sol: let the side of the square be x meters length of two sides = 2x meters diagonal = 2 x = 1.414x m saving on 2x meters = .59x m saving % = 0.59x /2x *100% = 30% (approx) 36.A man walking at the speed of 4 kmph crosses a square field diagonally in 3 meters.The area of the field is sol: speed of the man = 4*5/18 m/sec = 10/9 m/sec time taken = 3*60 sec = 180 sec length of diagonal = speed * time = 10/9 * 180 = 200m Area of the field = *(dioagonal) = * 200*200 sq m = 20000sq m 37.A square and rectangle have equal areas. If their perimeters are p and q respectively. Then sol: A square and a rectangle with equal areas will satisfy the relation p < q

Top
38.If the perimeters of a square and a rectangle are the same, then the area a & b enclosed by them would satisfy the condition: sol: Take a square of side 4cm and a rectangle having l=6cm and b=2cm then perimeter of square = perimeter of rectangle area of square = 16 sq cm area of rectangle = 12 sq cm Hence a >b 39.An error of 2% in excess is made while measuring the side of a square. The percentage of error in the calculated area of the square is sol: 100cm is read as 102 cm a = 100*100 sq cm and b = 102 *102 sq cm then a-b = 404 sq cm percentage error = 404/(100*100) = 4.04% 40.A tank is 25m long 12m wide and 6m deep. The cost of plastering its walls and bottom at 75 paise per sq m is sol: area to be plastered = [2(l+b)*h]+(l*b) = 2(25+12)*6 + (25*12)= 744 sq m cost of plastering = Rs . 744*75/100 = Rs. 5581 41.The dimensions of a room are 10m*7m*5m. There are 2 doors and 3 windows in the room. The dimensions of the doors are 1m*3m. One window is of size 2m*1.5m and the other 2 windows are of size 1m*1.5m. The cost of painting the walls at Rs. 3 per sq m is sol: Area of 4 walls = 2(l+b)*h =2(10+7)*5 = 170 sq m Area of 2 doors and 3 windows = 2(1*3)+(2*1.5)+2(1*1.5) = 12 sq m area to be planted = 170 - 12 = 158 sq m cost of painting = Rs. 158 *3 = Rs. 474 42.The base of a triangle of 15cm and height is 12cm. The height of another triangle of double the area having the base 20cm is sol: a = *15*12 = 90 sq cm b = 2a = 2 * 90 = * 20 *h => h= 18cm 43.The sides of a triangle are in the ratio of :1/3:1/4. If the perimeter is 52cm, then the length of the smallest side is sol: ratio of sides = :1/3 :1/4 = 6:4:3 perimeter = 52 cm, so sides are 52*6/13 =24cm 52*4/13 = 16cm 52 *3/13 = 12cm length of smallest side = 12cm 44.The height of an equilateral triangle is 10cm. Its area is sol: a = (a/2) +(10) a a/4 = 100 =>3a = 100*4 area = 3/4 *a = 3/4*400/3 = 100/3 sq cm 45.From a point in the interior of an equilateral triangle, the perpendicular distance of the sides are 3 cm, 23cm and 53cm. The perimeter of the triangle is sol: let each side of the triangle be a cm then area(AOB) +area(BOC)+area(AOC) = area(ABC) * a *3 +1/2 *a *23 +1/2 * a*53 = 3/4 a

file:///E|/work/books/placement/09_Aptitude/areas.html[1/28/2012 12:44:51 AM]

a/23(1+2+5) = 3/4 a => a=16 perimeter = 3*16 = 48cm

Top
Complex Probems: 1.If the area of a square with side a s equal to the area of a triangle with base a, then the altitude of the triangle is sol: area of a square with side a = a sq unts area of a triangle with base a = * a*h sq unts a =1/2 *a *h => h = 2a altitude of the triangle is 2a 2.An equilateral triangle is described on the diagonal of a square. What is the ratio of the area of the triangle to that of the square ? Sol: area of a square = a sq cm length of the diagonal = 2a cm area of equilateral triangle with side 2a = 3/4 * (2a) required ratio = 3a : a = 3 : 2 3.The ratio of bases of two triangles is x:y and that of their areas is a:b. Then the ratio of their corresponding altitudes wll be sol: a/b =( * x*H) /(1/2 * y * h) bxH = ayh =>H/h =ay/bx Hence H:h = ay:bx 4 .A parallelogram has sides 30m and 14m and one of its diagonals is 40m long. Then its area is sol: let ABCD be the given parallelogram area of parallelogram ABCD = 2* (area of triangle ABC) now a = 30m, b = 14m and c = 40m s = (30+14+40) = 42m Area of triangle ABC = [ s(s - a)(s - b)(s - c) = (42*12*28*2 = 168sq m area of parallelogram ABCD = 2 *168 =336 sq m 5.If a parallelogram with area p, a triangle with area R and a triangle with area T are all constructed on the same base and all have the same altitude, then which of the following statements is false ? Sol: let each have base = b and height = h then p = b*h, R = b*h and T = * b*h so P = R, P = 2T and T = R are all correct statements 6.If the diagonals of a rhombus are 24cm and 10cm the area and the perimeter of the rhombus are respectively. Sol: area = *diagonal 1 *diagonal 2= * 24 * 10= 120 sq cm * diagonal 1 = * 24 = 12cm * diagonal 2 = *10 =5 cm side of a rhombus = (12) + (5) = 169 => AB = 13cm

Top
7.If a square and a rhombus stand on the same base, then the ratio of the areas of the square and the rhombus is: sol: A square and a rhombus on the same base are equal in area 8.The area of a field in the shape of a trapezium measures 1440sq m. The perpendicular distance between its parallel sides is 24cm. If the ratio of the sides is 5:3, the length of the longer parallel side is: sol: area of field =1/2 *(5x+3x) *24 = 96x sq m 96x = 1440 => x = 1440 /96 = 15 hence, the length of longer parallel side = 5x = 75m 9.The area of a circle of radius 5 is numerically what percent its circumference ? Sol: required percentage = (5)/(2*5) *100 = 250% 10.A man runs round a circular field of radius 50m at the speed of 12m/hr. What is the time taken by the man to take twenty rounds of the field ? Sol: speed = 12 k/h = 12 * 5/18 = 10/3 m/s distance covered = 20 * 2*22/7*50 = 44000/7m time taken = distance /speed = 44000/7 * 3/10 = 220/7min 11.A cow s tethered in the middle of a field with a 14feet long rope.If the cow grazes 100 sq feet per day, then approximately what time will be taken by the cow to graze the whole field ? Sol: area of the field grazed = 22/7 * 14 * 14 = 616 sq feet 12.A wire can be bent in the If it is bent in the form of sol: length of wire = 2 r side of the square = 352/4 = area of the square = 88*88 = form of a circle of radius 56cm. a square, then its area will be = 2 *22/7 *56 = 352 cm 88cm 7744sq cm

13.The no of revolutions a wheel of diameter 40cm makes in traveling a distance of 176m is sol:

file:///E|/work/books/placement/09_Aptitude/areas.html[1/28/2012 12:44:51 AM]

distance covered in 1 revolution = 2 r = 2 *22/7 *20 = 880/7 cm required no of revolutions = 17600 *7/880 = 140

Top
14.The wheel of a motorcycle 70cm in diameter makes 40 revolutions in every 10sec.What is the speed of motorcycle n km/hr? Sol: distance covered in 10sec = 2 *22/7 *35/100 *40 =88m distance covered in 1 sec =88/10m = 8.8m speed =8.8m/s = 8.8 * 18/5 *k/h = 31.68 k/h 15.Wheels of diameters 7cm and 14cm start rolling simultaneously from x & y which are 1980 cm apart towards each other in opposite directions. Both of them make the same number of revolutions per second. If both of them meet after 10seconds.The speed of the smaller wheel is sol: let each wheel make x revolutions per sec. Then (2 *7/2 *x)+(2 * 7*x)*10 = 1980 (22/7 *7 * x) + (2 * 22/7 *7 *x) = 198 66x = 198 => x = 3 distance moved by smaller wheel in 3 revolutions = 2 *22/7 *7/2 *3 = 66cm speed of smaller wheel = 66/3 m/s = 22m/s 16.A circular swimming pool is surrounded by a concrete wall 4ft wide. If the area of the concrete wall surrounding the pool is 11/25 that of the pool, then the radius of the pool is ? Sol: let the radius of the pool be R ft radius of the pool including the wall = (R+4)ft area of the concrete wall = [(R+4)2 - R2 ] => = [R+4+R][R+4 - R] = 8(R+2) sq feet 8(R+2) = 11/25 R2 => 11 R2 = 200 (R+2) Radius of the pool R = 20ft 17.A semicircular shaped window has diameter of 63cm. Its perimeter equals sol: perimeter of window = r +2r = [22/7 * 63/2 +63] = 99+63 = 162 cm 18.Three circles of radius 3.5cm are placed in such a way that each circle touches the other two. The area of the portion enclosed by the circles is sol: required area - (3 * area of = ( * 7 = 493/4 = (area of an equilateral triangle of side 7 cm) sector with = 6o degrees and r = 3.5cm) * 7) (3* 22/7 *3.5 *3.5*60/360 ) sq cm 11*0.5*3.5 sq cm = 1.967 sq cm

19. Four circular cardboard pieces, each of radius 7cm are placed in such a way that each piece touches two other pieces. The area of the space encosed by the four pieces is sol: required area = 14*14 (4 * * 22/7 * 7 *7) sq cm = 196 154 = 42 sq cm

Back Back To Main

Top

Contact: 040-23000700

file:///E|/work/books/placement/09_Aptitude/areas.html[1/28/2012 12:44:51 AM]

APTITUDE
Numbers H.C.F and L.C.M Decimal Fractions Simplification Square and Cube roots Average Problems on Numbers Problems on Ages Surds and Indices Percentage Profit and Loss Ratio And Proportions Partnership Chain Rule Time and Work Pipes and Cisterns Time and Distance Trains Boats and Streams Alligation or Mixture Simple Interest Compound Interest Logorithms Areas Volume and Surface area Races and Games of Skill Calendar Clocks Stocks ans Shares True Discount Bankers Discount Oddmanout and Series Data Interpretation probability Permutations and Combinations Puzzles BACK

AREAS

Important Facts and Formula: RESULTS ON TRIANGLE 1.Sum of the angles of a triangle is 180 degrees. 2.The sum of any two sides of a triangle is greater than third side. 3.PYTHAGORAS Theorem: In a right angled triangle (Hypotenuse)2 = (Base)2 +(Height)2 4.The line joining the mid point of a side of a triangle to the opposite vertex is called the MEDIAN. 5.The point where the three medians of a triangle meet, is called CENTROID. The centroid divides each of the medians in the ratio 2:1 6.In an isosceles triangle, the altitude from the vertex bisects the base 7.The median of a triangle divides it into two triangles of the same area. 8.The area of the triangle formed by joining the mid points of the sides of a given triangle is one-fourth of the area of the given triangle. RESULTS ON QUADRILATERALS 1.The diagonals of a Parallelogram bisect each other. 2.Each diagonal of a Parallelogram divides it into two triangles of the same area. 3.The diagonals of a Rectangle are equal and bisect each other 4.The diagonals of a Square are equal and bisect each other at right angles. 5.The diagonals of a Rhombus are unequal and bisect each other at right angles. 6.A Parallelogram and a Rectangle on the same base and between the same parallels are equal in area.

file:///E|/work/books/placement/09_Aptitude/areasconcept.html[1/28/2012 12:44:52 AM]

7.Of all he parallelogram of given sides the parallelogram which is a rectangle has the greatest area. FORMULAE 1.Area of a RECTANGLE = length * breadth Length = (Area/Breadth) and Breadth = (Area/Length) 2.Perimeter of a RECTANGLE = 2(Length + Breadth) 3.Area of a SQUARE = (side)2 = ( Diagonal)2 4.Area of four walls of a room = 2(length + breadth) * height 5.Area of a TRIANGLE = * base * height 6.Area of a TRIANGLE = [s * (s-a) * (s-b) * (s-c)], where a,b,c are the sides of the triangle and s = 1/2(a+b+c) 7.Area of EQUILATERAL TRIANGLE = (3/4)* (side)2 8.Radius of in circle of an EQUILATERAL TRIANGLE of side a = r / 23 9.Radius of circumcircle of an EQUILATERAL TRIANGLE of side a = r / 3 10.Radius of incircle of a triangle of area and semi perimeter S = / s 11.Area of a PARALLELOGRAM = (base * height) 12.Area of RHOMBUS = 1/2 (product of diagonals) 13.Area of TRAPEZIUM = =1/2 * (sum of parallel sides)* (distance between them) 14.Area of a CIRCLE = r2 where r is the radius

15.Circumference of a CIRCLE = 2 r 16.Length of an arc = 2 r / 360, where is central angle r2 / 360

17.Area of a SECTOR = (arc * r) = 18.Area of a SEMICIRCLE = r2 / 2 r

19.Circumference of a SEMICIRCLE = BACK

file:///E|/work/books/placement/09_Aptitude/areasconcept.html[1/28/2012 12:44:52 AM]

APTITUDE
Numbers H.C.F and L.C.M Decimal Fractions Simplification Square and Cube roots Average Problems on Numbers Problems on Ages Surds and Indices Percentage Profit and Loss Ratio And Proportions Partnership Chain Rule Time and Work Pipes and Cisterns Time and Distance Trains Boats and Streams Alligation or Mixture Simple Interest Compound Interest Logorithms Areas Volume and Surface area Races and Games of Skill Calendar Clocks Stocks ans Shares True Discount Bankers Discount Oddmanout and Series Data Interpretation probability Permutations and Combinations Puzzles BACK Simple Problems:

1.One side of a rectangular field is 15m and one of its diagonal is 17m. Find the area of field? Sol:Other side = [(17*17) (15*15)] = (289-225) = 8m Area = 15 * 8 =120 sq. m

2.A lawn is in the form of a rectangle having its sides in the ratio 2:3 The area of the lawn is 1/6 hectares. Find the length and breadth of the lawn. Sol: let length = 2x meters and breadth = 3x mt Now area = (1/6 * 1000)sq m = 5000/3 sq m 2x * 3x = 5000/3 =>x * x =2500 / 9 x = 50/3 length = 2x = 100/3 m and breadth = 3x = 3*(50/3) = 50m

3.Find the cost of carpeting a room 13m long and 9m broad with a carpet 75cm wide at the rate of Rs 12.40 per sq meter Sol: Area of the carpet = Area of the room = 13* 9 =117 sq m length of the carpet = (Area/width) = 117 * (4/3) = 156 m Cost of carpeting = Rs (156 * 12.40) = Rs 1934.40

4.The length of a rectangle is twice its breadth if its length is decreased by 5cm and breadth is increased by 5cm, the area of the rectangle is increased by 75 sq cm. Find the length of the rectangle. Sol: let length = 2x and breadth = x then

(2x-5) (x+5) (2x*x)=75 5x-25 = 75 => x=20 length of the rectangle = 40 cm

5.In measuring the sides of a rectangle, one side is taken 5% in excess and the other 4% in deficit. Find the error percent in the area, calculate from the those measurements. Sol: let x and y be the sides of the rectangle then

correct area = (105/100 * x) * (96 / 100 *y) =(504/500 xy) xy = 4/500 xy

file:///E|/work/books/placement/09_Aptitude/areasimple.html[1/28/2012 12:44:52 AM]

Error% = 4/500 xy*(1/xy)*100 % = 4/5% = 0.8%

6.A room is half as long again as it is broad. The cost of carpeting the room at Rs 5 per sq m is Rs 2.70 and the cost of papering the four walls at Rs 10 per sq m is Rs 1720. If a door and 2 windows occupy 8 sq cm. Find the dimensions of the room? Sol: let breadth=x mt ,length= 3x/2 mt and height=h mt

Area of the floor = (total cost of carpeting /rate) = 270/5 sq m = 54 sq m x * 3x/2=54 => x*x= 54*(2/3)=36 => x = 6m

so breadth = 6m and length=3/2*6 = 9m now papered area = 1720 /10 = 172 sq m Area of one door and 2 windows =8 sq m total area of 4 walls = 172+8 = 180 sq m 2(9+6)*h = 180 => h=180/30 = 6m

7.The altitude drawn to the base of an isosceles triangle is 8cm and the perimeter is 32cm. Find the area of the triangle? Sol: let ABC be the isosceles triangle, the AD be the altitude let AB = AC=x then BC= 32-2x since in an isoceles triange the altitude bisects the base so BD=DC=16-x in ADC,(AC) 2 = (AD) 2 + (DC) 2 x*x=(8*8) + (16-x)*(16-x) 32x =320 => x = 10

BC = 32-2x = 32-20 = 12 cm Hence, required area = * BC * AD = * 12 * 10 = 60 sq cm

8.If each side of a square is increased by 25%, find the percentage change in its area? Sol: let each side of the square be a , then area = a * a New side = 125a / 100 = 5a / 4 New area =(5a * 5a)/(4*4) = (25a/16) a = 9a/16 Increase %= 9a/16 * 1/a * 100%

= 56.25%

9.Find the area of a Rhombus one side of which measures 20cm and one diagonal 24cm. Sol: Let other diagonal = 2x cm since diagonals of a rhombus bisect each other at right angles, we have 20 = 12 + x => x = [20 -12]= 256 = 16cm

file:///E|/work/books/placement/09_Aptitude/areasimple.html[1/28/2012 12:44:52 AM]

so the diagonal = 32 cm Area of rhombus = * product of diagonals = * 24 * 32 = 384 sq cm

10. The area of a circular field is 13.86 hectares. Find the cost of fencing it at the rate of Rs. 4.40 per meter. Sol: Area = 13.86 * 10000 sq m = 138600 sq m * 7/22 => 210 m

r= 138600 => r = 138600

circumference = 2 r = 2 * 22/7 * 210m = 1320 m cost of fencing = Rs 1320 * 4.40 = Rs. 5808

file:///E|/work/books/placement/09_Aptitude/areasimple.html[1/28/2012 12:44:52 AM]

APTITUDE
Numbers H.C.F and L.C.M Decimal Fractions Simplification Square and Cube roots Average Problems on Numbers Problems on Ages Surds and Indices Percentage Profit and Loss Ratio And Proportions Partnership Chain Rule Time and Work Pipes and Cisterns Time and Distance Trains Boats and Streams Alligation or Mixture Simple Interest Compound Interest Logorithms Areas Volume and Surface area Races and Games of Skill Calendar Clocks Stocks ans Shares True Discount Bankers Discount Oddmanout and Series Data Interpretation probability Permutations and Combinations Puzzles BACK

AREAS

PROBLEMS
1.One side of a rectangular field is 15m and one of its diagonal is 17m. Find the area of field? Sol:Other side = [(17*17) (15*15)] = (289-225) = 8m Area = 15 * 8 =120 sq. m 2.A lawn is in the form of a rectangle having its sides in the ratio 2:3 The area of the lawn is 1/6 hectares. Find the length and breadth of the lawn. Sol: let length = 2x meters and breadth = 3x mt Now area = (1/6 * 1000)sq m = 5000/3 sq m 2x * 3x = 5000/3 =>x * x =2500 / 9 x = 50/3 length = 2x = 100/3 m and breadth = 3x = 3*(50/3) = 50m 3.Find the cost of carpeting a room 13m long and 9m broad with a carpet 75cm wide at the rate of Rs 12.40 per sq meter Sol: Area of the carpet = Area of the room = 13* 9 =117 sq m length of the carpet = (Area/width) = 117 * (4/3) = 156 m Cost of carpeting = Rs (156 * 12.40) = Rs 1934.40 4.The length of a rectangle is twice its breadth if its length is decreased by 5cm and breadth is increased by 5cm, the area of the rectangle is increased by 75 sq cm. Find the length of the rectangle. Sol: let length = 2x and breadth = x then (2x-5) (x+5) (2x*x)=75 5x-25 = 75 => x=20 length of the rectangle = 40 cm 5.In measuring the sides of a rectangle, one side is taken 5% in excess and the other 4% in deficit. Find the error percent in the area, calculate from the those measurements. Sol: let x and y be the sides of the rectangle then correct area = (105/100 * x) * (96 / 100 *y) =(504/500 xy) xy = 4/500 xy Error% = 4/500 xy*(1/xy)*100 % = 4/5% = 0.8% 6.A room is half as long again as it is broad. The cost of carpeting the room at Rs 5 per sq m is Rs 2.70 and the cost of papering the four walls at Rs 10 per sq m is Rs 1720.If a door and 2 windows occupy 8 sq cm. Find the dimensions of the room? Sol: let breadth=x mt ,length= 3x/2 mt and height=h mt Area of the floor = (total cost of carpeting /rate) = 270/5 sq m = 54 sq m x * 3x/2=54 => x*x= 54*(2/3)=36 => x = 6m so breadth = 6m and length=3/2*6 = 9m now papered area = 1720 /10 = 172 sq m Area of one door and 2 windows =8 sq m total area of 4 walls = 172+8 = 180 sq m 2(9+6)*h = 180 => h=180/30 = 6m 7.The altitude drawn to the base of an isosceles triangle is 8cm and the perimeter is 32cm. Find the area of the triangle? Sol: let ABC be the isosceles triangle, the AD be the altitude let AB = AC=x then BC= 32-2x since in an isoceles triange the altitude bisects the base so BD=DC=16-x in ADC,(AC) 2 = (AD) 2 + (DC) 2 x*x=(8*8) + (16-x)*(16-x) 32x =320 => x = 10 BC = 32-2x = 32-20 = 12 cm Hence, required area = * BC * AD = * 12 * 10 = 60 sq cm 8.If each side of a square is increased by 25%, find the percentage change in its area? Sol: let each side of the square be a , then area = a * a New side = 125a / 100 = 5a / 4 New area =(5a * 5a)/(4*4) = (25a/16) a = 9a/16 Increase %= 9a/16 * 1/a * 100% = 56.25% 9.Find the area of a Rhombus one side of which measures 20cm and one diagonal 24cm. Sol: Let other diagonal = 2x cm since diagonals of a rhombus bisect each other at right angles, we have 20 = 12 + x => x = [20 -12]= 256 = 16cm so the diagonal = 32 cm Area of rhombus = * product of diagonals = * 24 * 32 = 384 sq cm

file:///E|/work/books/placement/09_Aptitude/areassimple.html[1/28/2012 12:44:53 AM]

10. The area of a circular field is 13.86 hectares. Find the cost of fencing it at the rate of Rs. 4.40 per meter. Sol: Area = 13.86 * 10000 sq m = 138600 sq m r= 138600 => r = 138600 * 7/22 => 210 m circumference = 2 r = 2 * 22/7 * 210m = 1320 m cost of fencing = Rs 1320 * 4.40 = Rs. 5808 11.Find the ratio of the areas of the incircle and circumcircle of a square. Sol: let the side of the square be x, then its diagonal = 2 x radius of incircle = x/2 and radius of circmcircle =2 x /2 = x/2 required ratio = x/4 : x/2 = : = 1:2 12.If the radius of a circle is decreased by 50% , find the percentage decrease in its area. Sol: let original radius = r and new radius = 50/100 r = r/2 original area = r and new area = (r/2) decrease in area = 3 r/4 * 1/ r * 100 = 75% 13.Two concentric circles form a ring. The inner and outer circumference of the ring are 352/7 m and 528/7m respectively. Find the width of the ring. sol: let the inner and outer radii be r and R meters then, 2 r = 352/7 => r = 352/7 * 7/22 * = 8m 2 R = 528/7 => R= 528/7 * 7/22 * = 12m width of the ring = R-r = 12-8 = 4m 14.If the diagonal of a rectangle is 17cm long and its perimeter is 46 cm. Find the area of the rectangle. sol: let length = x and breadth = y then 2(x+y) = 46 => x+y = 23 x+y = 17 = 289 now (x+y) = 23 =>x+y+2xy= 529 289+ 2xy = 529 => xy = 120 area =xy=120 sq. cm 15.A rectangular grassy plot 110m by 65cm has a gravel path .5cm wide all round it on the inside. Find the cost of gravelling the path at 80 paise per sq.mt sol: area of theplot = 110 * 65 = 7150 sq m area of the plot excluding the path = (110-5)* (65-5) = 6300 sq m area of the path = 7150- 6300 =850 sq m cost of gravelling the path = 850 * 80/100 = 680 Rs 16. The perimeters of ttwo squares are 40cm and 32 cm. Find the perimeter of a third square whose area is equal to the difference of the areas of the two squares. sol: side of first square = 40/4 =10cm side of second square = 32/4 = 8cm area of third squre = 10 8 = 36 sq cm side of third square = 36 = 6 cm required perimeter = 6*4 = 24cm 17. sol: area of the room = 544 * 374 sq cm size of largest square tile = H.C.F of 544cm and 374cm= 34cm area of 1 tile = 34*34 sq cm no. of tiles required = (544*374) / (34 * 34) = 176 18. sol: The diagonals of two squares are in the ratio of 2:5. Find the ratio of their areas. let the diagonals of the squares be 2x and 5x respectively ratio of their areas = * (2x) : *(5x) = 4:25 19.If each side of a square is increased by 25%. Find the percentage change in its area. sol: let each side of the square be a then area = a new side = 125a/100 = 5a/4 new area = (5a/4) = 25/16 a increase in area = (25/16)a - a = (9/16)a increase % = (9/16)a * (1/a) * 100 = 56.25% 20.The base of triangular field os three times its altitude. If the cost of cultivating the field at Rs. 24.68 per hectare be Rs. 333.18. Find its base and height. sol: area of the field = total cost/ rate = 333.18 /24.68 = 13.5 hectares => = 13.5 * 10000 = 135000 sq m let the altitude = x mt and base = 3x mt then *3x * x = 135000 => x = 90000 => x = 300 base= 900m and altitude = 300m BACK A room 5m 44cm long and 3m 74cm broad is to be paved with squre tiles. Find the least number of squre tiles required to cover the floor.

file:///E|/work/books/placement/09_Aptitude/areassimple.html[1/28/2012 12:44:53 AM]

Averages
Formula: 1.Average=Sum of quantities/Number of quantities. 2.Suppose a man covers a certain distance at x kmph and an equal distance at y kmph ,then the average speed during the whole journey is (2xy/x+y) kmph. Examples: 1.Find the average of all these numbers.142,147,153,165,157. Solution: 142 147 153 165 157 Here consider the least number i.e, 142 comparing with others, 142 147 153 165 157 +5 +11 +23 +15 Now add 5+11+23+15 = 52/5 = 10.8 Now add 10.8 to 142 we get 152.8 (Average of all these numbers). Answer is 152.8 2.Find the average of all these numbers.4,10,16,22,28 Solution: 4,10,16,22,28 As the difference of number is 6 Then the average of these numbers is central one i.e, 16. Answer is 16. 3.Find the average of all these numbers.4,10,16,22,28,34. Solution: Here also difference is 6. Then middle numbers 16,22 take average of these two numbers 16+22/2=19 Therefore the average of these numbers is 19. Answer is 19. 4.The average marks of a marks of a student in 4 Examination is 40.If he got 80 marks in 5th Exam then what is his new average. Solution: 4*40+80=240 Then average means 240/5=48. Answer is 48. 5.In a group the average income of 6 men is 500 and that of 5 women is 280, then what is average income of the group. Solution: 6*500+5*280=4400 then average is 4400/11=400. Another Method: here consider for 6 men 6 men each 500. so 5th women is 280. then 500-280=220. then 220*6/11=120. therefore 120+280=400. Answer is 400. 6.The average weight of a class of 30 students is 40 kgs if the teacher weight is included then average increases by 2 kgs then find the weight of the teacher? Solution: 30 students average weight is 40 kgs. So,when teacher weight is added it increases by 2 kgs so total 31 persons ,therefore 31*2=62. Now add the average weight of all student to it we get teachers weight i.e, 62+40=102 kgs. Answer is 102 kgs. 7.The average age of Mr and Mrs Sharma 4 years ago is 28 years . If the present average age of Mr and Mrs Sharma and their son is 22 years. What is the age of their son. Solution: 4 years ago their average age is 28 years. So their present average age is 32 years. 32 years for Mr and Mrs Sharma then 32*2=64 years. Then present age including their son is 22 years. So 22*3 =66 years. Therefore son age will be 66-64 = 2 years. Answer is 2 years. 8.The average price of 10 books is increased by 17 Rupees when

file:///E|/work/books/placement/09_Aptitude/average.html[1/28/2012 12:44:53 AM]

one of them whose value is Rs.400 is replaced by a new book. What is the price of new book? Solution: 10 books Average increases by 17 Rupees so 10*17= 170. so the new book cost is more and by adding its cost average increase,therefore the cost of new book is 400+170=570Rs. Answer is 570 Rs. 9.The average marks of girls in a class is 62.5. The average marks of 4 girls among them is 60.The average marks of remaining girls is 63,then what is the number of girls in the class? Solution: Total number of girls be x+4. Average marks of 4 girls is 60. therefore 62.5-60=2.5 then 4*2.5 =10. the average of remaining girls is 63 here 0.5 difference therefore 0.5*x=10(since we got from 4 girls) (this is taken becoz both should be equal) x=10/0.5 x=20. This clear says that remaining are 20 girls therefore total is x+4=20+4=24 girls Answer is 24 girls.

Top
10.Find the average of first 50 natural numbers. Solution: Sum of the Natural Numbers is n(n+1)/2 therefore for 50 Natural numbers 50*51/2=775. the average is 775/50=15.5 Answer is 15.5 . 11.The average of the first nine prime number is? Solution: Prime numbers are 2,3,5,7,11,13,17,19,23 therefore 2+3+5+7+11+13+17+19+23=100 then the average 100/9= 11 1/9. Answer is 11 1/9. 12.The average of 2,7,6 and x is 5 and the average of and the average of 18,1,6,x and y is 10 .what is the value of y? Solution: 2+7+6+x/4=5 =>15+x=20 =>x=5. 18+1+6+x+y/5=10 =>25+5+y=50 =>y=20. 13.The average of a non-zero number and its square is 5 times the number.The number is Solution: The number be x then x+x2/2=5x =>x2-9x=0 =>x(x-9)=0 therefore x=0 or x=9. The number is 9. 14.Nine persons went to a hotel for taking their meals . Eight of them spent Rs.12 each on their meals and the ninth spent Rs.8 then the average expenditure of all the nine. What was the total money spent by them? Solution: The average expenditure be x. then 8*12+(x+8)=9x =>96+x+8=9x. =>8x=104 =>x=13 Total money spent =9x=>9*13=117 Answer is Rs.117 15.The average weight of A.B.C is 45 Kgs.If the average weight of A and B be 40 Kgs and that of Band C be 43 Kgs. Find the weight of B? Solution: The weight of A,B,Care 45*3=135 Kgs. The weight of A,B are 40*2=80 Kgs. The weight of B,C are 43*2=86 Kgs. To get the Weight of B. (A+B)+(B+C)-(A+B+C)=80+86-135 B=31 kgs. Answer is 31 Kgs. 16.The sum of three consecutive odd number is 48 more than the average of these number .What is the first of these numbers? Solution: let the three consecutive odd numbers are x, x+2, x+4. By adding them we get x+x+2+x+4=3x+6. Then 3x+6-(3x+6)/3=38(given) =>2(3x+6)=38*3. =>6x+12=114 =>6x=102 =>x=17. Answer is 17. 17.A family consists of grandparents,parents and three grandchildren.

file:///E|/work/books/placement/09_Aptitude/average.html[1/28/2012 12:44:53 AM]

The average age of the grandparents is 67 years,that of parents is 35 years and that of the grand children is 6 years . What is the average age of the family? Solution: grandparents age is 67*2=134 parents age is 35*2=70 grandchildren age is 6*3=18 therefore age of family is 134+70+18=222 average is 222/7=31 5/7 years. Answer is 31 5/7 years.

Top
18.A library has an average of 510 visitors on Sundays and 240 on other days .The average number of visitors per day in a month 30 days beginning with a Sunday is? Solution: Here specified that month starts with Sunday so, in a month there are 5 Sundays. Therefore remaining days will be 25 days. 510*5+240*25=2550+6000 =8550 visitors. The average visitors are 8550/30=285. Answer is 285. 19.The average age of a class of 39 students is 15 years . If the age of the teacher be included ,then average increases by 3 months. Find the age of the teacher. Solution: Total age for 39 persons is 39*15=585 years. Now 40 persons is 40* 61/4=610 years (since 15 years 3 months=15 3/12=61/4) Age of the teacher =610-585 years =>25 years. Answer is 25 years. 20.The average weight of a 10 oarsmen in a boat is increases by 1.8 Kgs .When one of the crew ,who weighs 53 Kgs is replaced by new man. Find the weight of the new man. Solution: Weight of 10 oars men is increases by 1.8 Kgs so, 10*1.8=18 Kgs therefore 53+18=71 Kgs will be the weight of the man. Answer is 71 Kgs. 21.A bats man makes a score of 87 runs in the 17th inning and thus increases his average by 3. Find the average after 17th inning. Solution: Average after 17 th inning =x then for 16th inning is x-3. Therefore 16(x-3)+87 =17x =>x=87-48 =>x=39. Answer is 39. 22.The average age of a class is 15.8 years .The average age of boys in the class is 16.4 years while that of the girls is 15.4 years .What is the ratio of boys to girls in the class. Solution: Ratio be k:1 then k*16.4 + 1*15.4 = (k+1)*15.8 =>(16.4-15.8)k=15.8-15.4 =>k=0.4/0.6 =>k=2/3 therefore 2/3:1=>2:3 Answer is 2:3 23.In a cricket eleven ,the average of eleven players is 28 years .Out of these ,the average ages of three groups of players each are 25 years,28 years, and 30 years respectively. If in these groups ,the captain and the youngest player are not included and the captain is eleven years older than the youngest players , what is the age of the captain? Solution: let the age of youngest player be x then ,age of the captain =(x+11) therefore 3*25 + 3*28 + 3*30 + x + x+11=11*28 =>75+84+90+2x+11=308 =>2x=48 =>x=24. Therefore age of the captain =(x+11)= 24+11= 35 years. Answer is 35 years.

Top
24.The average age of the boys in the class is the number of girls in the class .If the ratio boys and girls in the class of 36 be 5:1, what the total of the age (in years) of the boys in Solution: Number of boys=36*5/6=30 Number of girls =6 Average age of boys =2*6=12 years Total age of the boys=30*12=360 years Answer is 360 years. 25.Five years ago, the average age of P and Q was 15 years ,average age of P,Q, and R today is 20 years,how old will R be after 10 years? Solution: Age of P and Q are 15*2=30 years Present age of P and Q is 30+5*2=40 years. Age of P Q and R is 20*3= 60 years. R ,present age is 60-40=20 years After 10 years =20+10=30 years. Answer is 30 years. twice of is the class?

file:///E|/work/books/placement/09_Aptitude/average.html[1/28/2012 12:44:53 AM]

26.The average weight of 3 men A,B and C is 84 Kgs. Another man D joins the group and the average now becomes 80 Kgs.If another man E whose weight is 3 Kgs more than that of D ,replaces A then the average weight B,C,D and E becomes 79 Kgs. The weight of A is. Solution:Total weight of A, B and C is 84 * 3 =252 Kgs. Total weight of A,B,C and Dis 80*4=320 Kgs Therefore D=320-252=68 Kgs. E weight (68+3)=71 kgs Total weight of B,C,D and E = 79*4=316 Kgs (A+B+C+D)-(B+C+D+E)=320-316 =4Kgs A-E=4Kgs A-71=4 kgs A=75 Kgs Answer is 75 kgs 27.A team of 8 persons joins in a shooting competition. The best marksman scored 85 points.If he had scored 92 points ,the average score for the team would have been 84.The team scored was. Solution: Here consider the total score be x. therefore x+92-85/8=84 =>x+7=672 =>x=665. Answer is 665 28.A man whose bowling average is 12.4,takes 5 wickets for 26 runs and there by decrease his average by 0.4. The number of wickets,taken by him before his last match is: Solution: Number of wickets taken before last match be x. therefore 12.4x26/x+5=12(since average decrease by 0.4 therefore 12.4-0.4=12) =>12.4x+2612x+60 =>0.4x=34 =>x=340/4 =>x=85. Answer is 85. 29.The mean temperature of Monday to Wednesday was 37 degrees and of Tuesday to Thursday was 34 degrees .If the temperature on Thursday was 4/5th that of Monday. The temperature on Thursday was: Solution: The total temperature recorded on Monday,Wednesday was 37*3=111. The total temperature recorded on Tuesday, Wednesday,Thursday was 34*3=102. and also given that Th=4/5M =>M=5/4Th (M+T+W)-(T+W+Th)=111-102=9 M-Th=9 5/4Th-Th=9 Th(1/4)=9 =>Th=36 degrees. 30. 16 children are to be divided into two groups A and B of 10 and 6 children. The average percent marks obtained by the children of group A is 75 and the average percent marks of all the 16 children is 76. What is the average percent marks of children of groups B? Solution: Here given average of group A and whole groups . So,(76*16)-(75*10)/6 =>1216-750/6 =>466/6=233/3=77 2/3 Answer is 77 2/3. 31.Of the three numbers the first is twice the second and the second is twice the third .The average of the reciprocal of the numbers is 7/72,the number are. Solution:Let the third number be x Let the second number be 2x. Let the first number be 4x. Therefore average of the reciprocal means 1/x+1/2x+1/4x=(7/72*3) 7/4x=7/24 =>4x=24 x=6. Therefore First number is 4*6=24. Second number is 2*6=12 Third number is 1*6=6 Answer is 24,12,6. 32.The average of 5 numbers is 7.When 3 new numbers are added the average of the eight numbers is 8.5. The average of the three new number is: Solution: Sum of three new numbers=(8*8.5-5*7)=33 Their average =33/3=11. Answer is 11. 33.The average temperature of the town in the first four days of a month was 58 degrees. The average for the second ,third,fourth and fifth days was 60 degree .If the temperature of the first and fifth days were in the ratio 7:8 then what is the temperature on the fifth day? Solution : Sum of temperature on 1st 2nd 3rd and 4th days =58*4=232 degrees.

file:///E|/work/books/placement/09_Aptitude/average.html[1/28/2012 12:44:53 AM]

Sum of temperature on 2nd 3rd 4th and 5th days =60*4=240 degrees Therefore 5th day temperature is 240-232=8 degrees. The ratio given for 1st and 5th days be 7x and 8x degrees then 8x-7x=8 =>x=8. therefore temperature on the 5th day =8x=8*8=64 degrees.

Back Back To Main

Top

Contact: 040-23000700

file:///E|/work/books/placement/09_Aptitude/average.html[1/28/2012 12:44:53 AM]

APTITUDE
Numbers H.C.F and L.C.M Decimal Fractions Simplification Square and Cube roots Average Problems on Numbers Problems on Ages Surds and Indices Percentage Profit and Loss Ratio And Proportions Partnership Chain Rule Time and Work Pipes and Cisterns Time and Distance Trains Boats and Streams Alligation or Mixture Simple Interest Compound Interest Logorithms Areas Volume and Surface area Races and Games of Skill Calendar Clocks Stocks ans Shares True Discount Bankers Discount Oddmanout and Series Data Interpretation probability Permutations and Combinations Puzzles

BANKER'S DISCOUNT
CONCEPT SIMPLE PROBLEMS MEDIUM PROBLEMS COMPLEX PROBLEMS

file:///E|/work/books/placement/09_Aptitude/banker1.html[1/28/2012 12:44:54 AM]

APTITUDE
Numbers H.C.F and L.C.M Decimal Fractions Simplification Square and Cube roots Average Problems on Numbers Problems on Ages Surds and Indices Percentage Profit and Loss Ratio And Proportions Partnership Chain Rule Time and Work Pipes and Cisterns Time and Distance Trains Boats and Streams Alligation or Mixture Simple Interest Compound Interest Logorithms Areas Volume and Surface area Races and Games of Skill Calendar Clocks Stocks ans Shares True Discount Bankers Discount Oddmanout and Series Data Interpretation probability Permutations and Combinations Puzzles BACK

COMPLEX PROBLEMS
1.A bill for Rs.6000 is drawn on July 14 at 5 months . It is discounted on 5th October at 10%.Find the bankers discount true discount, bankers gain and the money that the holder of the Sol:Face value of the bill = Rs.6000 date on which the bill was drawn = July 14 at 5 months nominally due date = December 14 legally due date = December 17 Date on which the bill was discounted = October 5 Unexpired time : Oct Nov Dec 26 + 30 + 17 = 73days =1/5 years B.D = S.I on Rs 6000 for 1/5 year = Rs (6000 * 10 * 1/5 * 1/100) = Rs 120 T.D = Rs(6000 * 10 *1/5)/(100 + (10 * 1/5)) = Rs. 117.64 B.G = (B.D) (T.D) = Rs(120 -117.64) = Rs 2.36 Money received by the holder of the bill = Rs(6000 120) = 5880 bill receives.

2. The bankers gain on a certain sum due

1 year hence

is 3/25 of the bankers discount .The rate percent is Sol:Let B.D = 1 then B.G = 3/25 T.D = (B.D B.G) = (1 3/25) = 22/25 sum = (1 * 22/25) / (1 22/25) = 22/3 S.I on Rs 22/3 for 1 year is 1. Rate = (100 * 1) / (22/3 * 3/2) = 9 1/9%

3. The bankers gain of a certain sum due 2 years hence at 10% per annum is Rs 24 .The percent worth is Sol:-

T.D = (B.G * 100) / (Rate * Time) 00) / (10 * 2) = 120. P.W = (100 *T.D) / (Rate * Time)

file:///E|/work/books/placement/09_Aptitude/bankercomplex.html[1/28/2012 12:44:54 AM]

= (100 * 120) /(10 * 2) = 600

BACK

file:///E|/work/books/placement/09_Aptitude/bankercomplex.html[1/28/2012 12:44:54 AM]

APTITUDE
Numbers H.C.F and L.C.M Decimal Fractions Simplification Square and Cube roots Average Problems on Numbers Problems on Ages Surds and Indices Percentage Profit and Loss Ratio And Proportions Partnership Chain Rule Time and Work Pipes and Cisterns Time and Distance Trains Boats and Streams Alligation or Mixture Simple Interest Compound Interest Logorithms Areas Volume and Surface area Races and Games of Skill Calendar Clocks Stocks ans Shares True Discount Bankers Discount Oddmanout and Series Data Interpretation probability Permutations and Combinations Puzzles BACK

CONCEPT
-> Suppose a merchant A buys goods worth Rs.10000 from another merchant B at a credit of say 5 months -> Then,B prepares a bill , called the bill of exchange -> A signs this bill & allows B to withdraw the amount from his bank account after exactly 5 months,the date exactly after 5 months is called Nominally due date -> Three days (grace days) are added to it get a date known as legally due date -> Suppose B wants to have money before legally due date then he can have the money from banker or a broker who deducts S.I on the face value (i.e., 10000) for the period from the date on which the bill was discounted (i.e paied by the banker) & the legally due date this amount is known as Bankers Discount -> Thus , B.D is the S.I on the face for the period from the date on which the bill was discounted and the legally due date -> Bankers Gain (B.G) = (B.D) (T.D) for the unexpired time Note:- When the date of the bill is not given,grace days are not to be added Formulae:(1)B.D = S.I on bill for unexpired time (2)B.G = (B.D) (T.D) = S.I on T.D = (T.D)^2 /P.W (3)T.D = sqrt(P.W * B.G) (4)B.D = (Amount * Rate * Time)/100 (5)T.D = (Amount * Rate * Time)/(100+(Rate * time) (6)Amount = (B.D * T.D)/(B.D T.D) (7)T.D = (B.G * 100)/(Rate * Time)
BACK

file:///E|/work/books/placement/09_Aptitude/bankerconcept.html[1/28/2012 12:44:55 AM]

APTITUDE
Numbers H.C.F and L.C.M Decimal Fractions Simplification Square and Cube roots Average Problems on Numbers Problems on Ages Surds and Indices Percentage Profit and Loss Ratio And Proportions Partnership Chain Rule Time and Work Pipes and Cisterns Time and Distance Trains Boats and Streams Alligation or Mixture Simple Interest Compound Interest Logorithms Areas Volume and Surface area Races and Games of Skill Calendar Clocks Stocks ans Shares True Discount Bankers Discount Oddmanout and Series Data Interpretation probability Permutations and Combinations Puzzles BACK

MEDIUM PROBLEMS
1.The Bankers discount on Rs 1650 due a certain time hence is Rs 165. find the true discount and the bankers gain. Sol :Sum = (B.D * T.D) / (B.D T.D) = (B.D * T.D) / B.G T.D/B.G = Sum/B.D =650/165 =10/1 Thus if B.G is Rs 1 ,T.D = Rs 10 if B.D is Rs 11 ,T.D = Rs 10 if B.D is Rs 165,T.D = Rs (10/11 * 165) = 150 B.G =Rs(165 150) = Rs 15 2.The Present worth of a bill due something hence is Rs 1110 and the true discount on the bill is Rs.110 . Find the bankes discount & the bankers gain. Sol:T.D = sqrt (P.W * B.G) B.G = (T.D)^2 /P.W = (110 * 110) / 1110 = 11 B.D = (T.D + B.G) = (110 + 11) = Rs 121 3.What rate percent does a man get for his money when in discounting Sol:Let the amount of the bill =100 Money deducted = 10 Money received by the holder of the bill = (100 10) = 90 S.I on Rs 90 for 10 months = 10 Rate = (100 * 10) / (90 * 10/12) = 13 1/3%
BACK

file:///E|/work/books/placement/09_Aptitude/bankermedium.html[1/28/2012 12:44:55 AM]

Bankers Discount
Formulae: i. Suppose a merchant A buys goods worth Rs.10000 from another merchant B at a credit of say 5 months ii.Then,B prepares a bill , called the bill of exchange iii. A signs this bill & allows B to withdraw the amount from his bank account after exactly 5 months,the date exactly after 5 months is called Nominally due date iv. Three days (grace days) are added to it get a date known as legally due date v.Suppose B wants to have money before legally due date then he can have the money from banker or a broker who deducts S.I on the face value (i.e., 10000) for the period from the date on which the bill was discounted (i.e paied by the banker) & the legally due date this amount is known as Bankers Discount vi.Thus , B.D is the S.I on the face for the period from the date on which the bill was discounted and the legally due date vii.Bankers Gain (B.G) = (B.D) (T.D) for the unexpired time Note: When the date of the bill is not given,grace days are not to be added Formulae: (1)B.D = S.I on bill for unexpired time (2)B.G = (B.D) (T.D) = S.I on T.D = (T.D)^2 /P.W (3)T.D = sqrt(P.W * B.G) (4)B.D = (Amount * Rate * Time)/100 (5)T.D = (Amount * Rate * Time)/(100+(Rate * time) (6)Amount = (B.D * T.D)/(B.D T.D) (7)T.D = (B.G * 100)/(Rate * Time)

Top
Simple Problems 1.If the true discount on a certain sum due 6 months hence at 15% is Rs 120.What is the bankers discount on the same for same time and the same rate. Sol:B.G = S.I on T.D = RS (120 * 15 * = 9 (B.D) (T.D) = 9 B.D = 120 + 9 =129 * 1/100)

2.The bankers discount on Rs 1800 at 12 % per annum is equal to the true discount on Rs 1872 for the same time at the same rate .Find the time. Sol:S.I on Rs 1800 = T.D on Rs 1872 P.W of Rs 1872 is Rs 1800 Rs 72 is S.I on Rs 1800 at 12% Time = (100 * 72)/(12 * 1800) = 1/3 years = 4 months

3.The bankers discount and true discount on a sum of money due 8 months hence are Rs.120 & Rs.110 resp. Find the sum & the rate per cent Sol:Sum = (B.D * T.D) / (B.D) (T.D) = (120 * 110) / (120 110) = 1320 Since B.D is S.I on sum due, so S.I on Rs 1320 for 8 months is Rs 120 Rate = (100 * 120) / (1320 * 2/3) = 13 7/11% Medium Problems 1.The Bankers discount on Rs 1650 due a certain time hence is Rs 165. find the true discount and the bankers gain. Sol :Sum = (B.D * T.D) / (B.D T.D) = (B.D * T.D) / B.G T.D/B.G = Sum/B.D =650/165 =10/1 Thus if B.G is Rs 1 ,T.D = Rs 10 if B.D is Rs 11 ,T.D = Rs 10 if B.D is Rs 165,T.D = Rs (10/11 * 165) = 150 B.G =Rs(165 150) = Rs 15 2.The Present worth of a bill due something hence is Rs 1110 and the true discount on the bill is Rs.110 . Find the bankes discount & the bankers gain. Sol:T.D = sqrt (P.W * B.G) B.G = (T.D)^2 /P.W

file:///E|/work/books/placement/09_Aptitude/bankersdiscount.html[1/28/2012 12:44:55 AM]

= = B.D = = =

(110 * 110) / 1110 11 (T.D + B.G) (110 + 11) Rs 121

3.What rate percent does a man get for his money when in discounting Sol:Let the amount of the bill =100 Money deducted = 10 Money received by the holder of the bill = (100 10) = 90 S.I on Rs 90 for 10 months = 10 Rate = (100 * 10) / (90 * 10/12) = 13 1/3%

Top
Complex Problems 1.A bill for Rs.6000 is drawn on July 14 at 5 months . It is discounted on 5th October at 10%.Find the bankers discount true discount, bankers gain and the money that the holder of the bill receives. Sol:Face value of the bill = Rs.6000 date on which the bill was drawn = July 14 at 5 months nominally due date = December 14 legally due date = December 17 Date on which the bill was discounted = October 5 Unexpired time : Oct Nov Dec 26 + 30 + 17 = 73days =1/5 years B.D = S.I on Rs 6000 for 1/5 year = Rs (6000 * 10 * 1/5 * 1/100) = Rs 120 T.D = Rs(6000 * 10 *1/5)/(100 + (10 * 1/5)) = Rs. 117.64 B.G = (B.D) (T.D) = Rs(120 -117.64) = Rs 2.36 Money received by the holder of the bill = Rs(6000 120) = 5880 2. The bankers gain on a certain sum due 1 year hence is 3/25 of the bankers discount .The rate percent is Sol:Let B.D = 1 then B.G = 3/25 T.D = (B.D B.G) = (1 3/25) = 22/25 sum = (1 * 22/25) / (1 22/25) = 22/3 S.I on Rs 22/3 for 1 year is 1. Rate = (100 * 1) / (22/3 * 3/2) = 9 1/9%

3. The bankers gain of a certain sum due 2 years hence at 10% per annum is Rs 24 .The percent worth is Sol:T.D = (B.G * 100) / (Rate * Time) 00) / (10 * 2) = 120. P.W = (100 *T.D) / (Rate * Time) = (100 * 120) /(10 * 2) = 600

Back Back To Main

Top

Contact: 040-23000700

file:///E|/work/books/placement/09_Aptitude/bankersdiscount.html[1/28/2012 12:44:55 AM]

APTITUDE
Numbers H.C.F and L.C.M Decimal Fractions Simplification Square and Cube roots Average Problems on Numbers Problems on Ages Surds and Indices Percentage Profit and Loss Ratio And Proportions Partnership Chain Rule Time and Work Pipes and Cisterns Time and Distance Trains Boats and Streams Alligation or Mixture Simple Interest Compound Interest Logorithms Areas Volume and Surface area Races and Games of Skill Calendar Clocks Stocks ans Shares True Discount Bankers Discount Oddmanout and Series Data Interpretation probability Permutations and Combinations Puzzles BACK

SIMPLE PROBLEM
1.If the true discount on a certain sum due 6 months hence at 15% is Rs 120.What is the bankers discount on the same for same time and the same rate. Sol:B.G = S.I on T.D = RS (120 * 15 * * 1/100) = 9 (B.D) (T.D) = 9 B.D = 120 + 9 =129 2.The bankers discount on Rs 1800 at 12 % per annum is equal to the true discount on Rs 1872 for the same time at the same rate .Find the time. Sol:S.I on Rs 1800 = T.D on Rs 1872 P.W of Rs 1872 is Rs 1800 Rs 72 is S.I on Rs 1800 at 12% Time = (100 * 72)/(12 * 1800) = 1/3 years = 4 months 3.The bankers discount and true discount on a sum of money due 8 months hence are Rs.120 & Rs.110 resp. Find the sum & the rate per cent Sol:Sum = (B.D * T.D) / (B.D) (T.D) = (120 * 110) / (120 110) = 1320 Since B.D is S.I on sum due, so S.I on Rs 1320 for 8 months is Rs 120 Rate = (100 * 120) / (1320 * 2/3) = 13 7/11%
BACK

file:///E|/work/books/placement/09_Aptitude/bankersimple.html[1/28/2012 12:44:56 AM]

APTITUDE
Numbers H.C.F and L.C.M Decimal Fractions Simplification Square and Cube roots Average Problems on Numbers Problems on Ages Surds and Indices Percentage Profit and Loss Ratio And Proportions Partnership Chain Rule Time and Work Pipes and Cisterns Time and Distance Trains Boats and Streams Alligation or Mixture Simple Interest Compound Interest Logorithms Areas Volume and Surface area Races and Games of Skill Calendar Clocks Stocks ans Shares True Discount Bankers Discount Oddmanout and Series Data Interpretation probability Permutations and Combinations Puzzles

BOATS AND STREAMS


CONCEPTS PROBLEMS

file:///E|/work/books/placement/09_Aptitude/boats.html[1/28/2012 12:44:56 AM]

Boats and Streams


Important facts: 1)In water, the direction along the stream is called down stream. 2)Direction against the stream is called upstream. 3)The speed of boat in still water is U km/hr and the speed of stream is V km/hr then speed down stream =U + V km/hr speed up stream = U V km/hr Formulae: If the speed down stream is A km/hr and the speed up stream is B km/hr then speed in still water = (A+B) km/hr rate of stream =1/2(A-B) km/hr Problems: 1. In one hour a boat goes 11 km long the stream and 5 km against the stream. The speed of the boat in still water is? Sol: Speed in still water = ( 11+5) km/hr= 8 kmph 2.A man can row 18 kmph in still water. It takes him thrice as long as row up as to row down the river. find the rate of stream. Sol: Let man's rate up stream be xkmph then, in still water =1/2[3x+x]=2x kmph so, 2x= 18, x=9 rate upstream =9kmph rate downstream =27 kmph rate of stream = [27-9] = 9kmph 3.A man can row 71/2kmph in still watre . if in a river running at 1.5 km an hour, if takes him 50 min to row to place and back. how far off is the place? Sol: speed down stream =7.5+1.5=9kmph speed upstream =7.5-1.5=6kmph let the required distence x km. then , x/9+x/6=50/60 = 2x+3x= 5/6*18 5x=15, x=3 Hence, the required distence is 3 km

4.A man can row 3 quarters of a km aganist the stream is 111/4 min. the speed of the man in still water is ? Sol: rate upstream = 750/625 m/sec =10/9 m/sec rate downstream =750/450 m/sec = 5/3 m/sec rate in still water =1/2[10/9+5/3] = 25/18 m/sec = 25/18*18/5=5 kmph 5.A boat can travel with a speed of 13 kmph in still water. if the speed of stream is 4 kmph,find the time taken by the boat to go 68 km downstream? Sol: Speed down stream = 13+4= 17 kmph time taken to travel 68km downstream =68/17 hrs = 4 hrs

Top
6.A boat takes 90 min less to travel 36 miles downstream then to travel the same distence upstream. if the speed of the boat in still water is 10 mph . The speed of the stream is : Sol: Let the speed of the stream be x mph . then, speed downstream = [10+x]mph speed upstream =[10-x] mph 36/[10+x] - 36/[10-x] = 90/60 =72x*60= 90[100-x2] (x+50)(x-2) =0 x=2 kmph 7.At his usual rowing rate, Rahul 12 miles down stream in a certain river in 6 hrs less than it takes him to travel the same distence upstream. but if he could double his usual rowing rate for his 24 miles roundthe down stream 12 miles would then take only one hour less than the up stream 12 miles. what is the speed of the current in miles per hours? Sol: Let the speed in still water be x mph and the speed of the curren be y mph. then, speed upstream = (x-y) speed downstream =(x+y)

file:///E|/work/books/placement/09_Aptitude/boatsandstreams.html[1/28/2012 12:44:56 AM]

12/(x-y) - 12/(x+y) = 6 6(x2 y2) m= 2xy => x2 y2 =4y -(1) and 12/(2x-y) - 12/(2x+y) =1 => 4x2 y2 = 24y x2= ( 24y + y2)/4 -->(2) from 1 and 2 we have 4y+ y2 =( 24y+y2)/4 y=8/3 mph y= 22/3 mph 8.There is a road beside a river. two friends started from a place A, moved to a temple situated at another place B and then returned to A again. one of them moves on a cycle at a speed of 12 kmph, while the other sails on a boat at a speed of 10 kmph . if the river flows at the speedof 4 kmph, which of the two friends will return to place A first ? Sol: Clearly, The cyclist moves both ways at a speed of 12 kmph so, average speed of the cyclist = 12 kmph the boat sailor moves downstream = (10+4) = 14 kmph upstream =(10-4) = 6 kmph So, average speed of the boat sailor =[ 2*14*6]/[14+6] kmph =42/5 kmph =8.4 kmph Since, the average speed of the cyclist is greater, he will return to A first. 9.A boat takes 19 hrs for travelling downstream from point A to point B. and coming back to a point C midway between A and B. if the velocity of the sream is 4 kmph. and the speed of the boat in still water is 14 kmph. what is the distence between A and B? Sol: speed downstream =14+4 =18 kmph speed upstream = 14 -4 = 10 kmph let the distence between A and B be x km. then, x/18 + (x/2)/10 = 19 x/18 + x/20 =19 19x/180 =19 =>x = 180km Hence, the distence between A and B bw 180 km

Back Back To Main

Top

Contact: 040-23000700

file:///E|/work/books/placement/09_Aptitude/boatsandstreams.html[1/28/2012 12:44:56 AM]

APTITUDE
Numbers H.C.F and L.C.M Decimal Fractions Simplification Square and Cube roots Average Problems on Numbers Problems on Ages Surds and Indices Percentage Profit and Loss Ratio And Proportions Partnership Chain Rule Time and Work Pipes and Cisterns Time and Distance Trains Boats and Streams Alligation or Mixture Simple Interest Compound Interest Logorithms Areas Volume and Surface area Races and Games of Skill Calendar Clocks Stocks ans Shares True Discount Bankers Discount Oddmanout and Series Data Interpretation probability Permutations and Combinations Puzzles BACK

BOATS AND STREAMS


Important facts: 1)In water, the direction along the stream is called down stream. 2)Direction against the stream is called upstream. 3)The speed of boat in still water is U km/hr and the speed of stream is V km/hr then speed down stream =U + V km/hr speed up stream = U V km/hr Formulas: If the speed down stream is A km/hr and the speed up stream is B km/hr then speed in still water = (A+B) km/hr rate of stream =1/2(A-B) km/hr
BACK

file:///E|/work/books/placement/09_Aptitude/boatsconcept.html[1/28/2012 12:44:57 AM]

APTITUDE
BACK

BOATS AND STREAMS


Examples: 1. In one hour a boat goes 11 km long the stream and 5 km against the stream. The speed of the boat in still water is? Sol: Speed in still water = ( 11+5) km/hr = 8 kmph 2.A man can row 18 kmph in still water. It takes him thrice as long as row up as to row down the river. find the rate of stream. Sol: Let man's rate up stream be xkmph then, in still water =1/2[3x+x]=2x kmph so, 2x= 18, x=9 rate upstream =9kmph rate downstream =27 kmph rate of stream = [27-9] = 9kmph 3.A man can row 71/2kmph in still watre . if in a river running at 1.5 km an hour, if takes him 50 min to row to place and back. how far off is the place? Sol: speed down stream =7.5+1.5=9kmph speed upstream =7.5-1.5=6kmph let the required distence x km. then , x/9+x/6=50/60 = 2x+3x= 5/6*18 5x=15, x=3 Hence, the required distence is 3 km 4.A man can row 3 quarters of a km aganist the stream is 111/4 min. the speed of the man in still water is ? Sol: rate upstream = 750/625 m/sec =10/9 m/sec rate downstream =750/450 m/sec = 5/3 m/sec rate in still water =1/2[10/9+5/3] = 25/18 m/sec = 25/18*18/5=5 kmph 5.A boat can travel with a speed of 13 kmph in still water. if the speed of stream is 4 kmph, find the time taken by the boat to go 68 km downstream? Sol: Speed down stream = 13+4= 17 kmph time taken to travel 68km downstream =68/17 hrs = 4 hrs 6.A boat takes 90 min less to travel 36 miles downstream then to travel the same distence upstream. if the speed of the boat in still water is 10 mph . the speed of the stream is : Sol: Let the speed of the stream be x mph . then, speed downstream = [10+x]mph speed upstream =[10-x] mph 36/[10+x] - 36/[10-x] = 90/60 =72x*60= 90[100-x2] (x+50)(x-2) =0 x=2 kmph 7.At his usual rowing rate, Rahul 12 miles down stream in a certain river in 6 hus less than it takes him to travel the same distence upstream. but if he could double his usual rowing rate for his 24 miles roundthe down stream 12 miles would then take only one hour less than the up stream 12 miles. what is the speed of the current in miles per hours? Sol: Let the speed in still water be x mph and the speed of the curren be y mph. then, speed upstream = (x-y) speed downstream =(x+y) 12/(x-y) - 12/(x+y) = 6 6(x2 y2) m= 2xy => x2 y2 =4y -(1) and 12/(2x-y) - 12/(2x+y) =1 => 4x2 y2 = 24y x2= ( 24y + y2)/4 -->(2) from 1 and 2 we have 4y+ y2 =( 24y+y2)/4 y=8/3 mph y= 22/3 mph 8.There is a road beside a river. two friends started from a place A, moved to a at another place B and then returned to A again. one of them moves on a cycle at kmph, while the other sails on a boat at a speed of 10 kmph . if the river flows kmph , which of the two friends will return to place A first ? Sol: Clearly, The cyclist moves both ways at a speed of 12 kmph so, average speed of the cyclist = 12 kmph the boat sailor moves downstream = (10+4) = 14 kmph upstream =(10-4) = 6 kmph So, average speed of the boat sailor =[ 2*14*6]/[14+6] kmph =42/5 kmph =8.4 kmph Sience, the average speed of the cyclist is greater, he will return to A temple situated a speed of 12 at the speedof 4

Numbers H.C.F and L.C.M Decimal Fractions Simplification Square and Cube roots Average Problems on Numbers Problems on Ages Surds and Indices Percentage Profit and Loss Ratio And Proportions Partnership Chain Rule Time and Work Pipes and Cisterns Time and Distance Trains Boats and Streams Alligation or Mixture Simple Interest Compound Interest Logorithms Areas Volume and Surface area Races and Games of Skill Calendar Clocks Stocks ans Shares True Discount Bankers Discount Oddmanout and Series Data Interpretation probabulity Permutations and Combinations Puzzles

first.

9.A boat takes 19 hrs for travelling downstream from point A to point B. and coming back to a point C midway between A and B. if the velocity of the sream is 4 kmph . and the speed of the boat in still water is 14 kmph. what is the distence between A and B? Sol: speed downstream =14+4 =18 kmph speed upstream = 14 -4 = 10 kmph let the distence between A and B be x km. then, x/18 + (x/2)/10 = 19 x/18 + x/20 =19 19x/180 =19 =>x = 180km Hence, the distence between A and B bw 180 km

file:///E|/work/books/placement/09_Aptitude/boatspro.html[1/28/2012 12:44:57 AM]

APTITUDE
Numbers H.C.F and L.C.M Decimal Fractions Simplification Square and Cube roots Average Problems on Numbers Problems on Ages Surds and Indices Percentage Profit and Loss Ratio And Proportions Partnership Chain Rule Time and Work Pipes and Cisterns Time and Distance Trains Boats and Streams Alligation or Mixture Simple Interest Compound Interest Logorithms Areas Volume and Surface area Races and Games of Skill Calendar Clocks Stocks ans Shares True Discount Bankers Discount Oddmanout and Series Data Interpretation probability Permutations and Combinations Puzzles BACK

BOATS AND STREAMS


PROBLEMS: 1. In one hour a boat goes 11 km long the stream and 5 km against the stream. The speed of the boat in still water is? Sol: Speed in still water = ( 11+5) km/hr = 8 kmph 2.A man can row 18 kmph in still water. It takes him thrice as long as row up as to row down the river. find the rate of stream. Sol: Let man's rate up stream be xkmph then, in still water =1/2[3x+x]=2x kmph so, 2x= 18, x=9 rate upstream =9kmph rate downstream =27 kmph rate of stream = [27-9] = 9kmph 3.A man can row 71/2kmph in still watre . if in a river running at 1.5 km an hour, if takes him 50 min to row to place and back. how far off is the place? Sol: speed down stream =7.5+1.5=9kmph speed upstream =7.5-1.5=6kmph let the required distence x km. then , x/9+x/6=50/60 = 2x+3x= 5/6*18 5x=15, x=3 Hence, the required distence is 3 km 4.A man can row 3 quarters of a km aganist the stream is 111/4 min. the speed of the man in still water is ? Sol: rate upstream = 750/625 m/sec =10/9 m/sec rate downstream =750/450 m/sec = 5/3 m/sec rate in still water =1/2[10/9+5/3] = 25/18 m/sec = 25/18*18/5=5 kmph 5.A boat can travel with a speed of 13 kmph in still water. if the speed of stream is 4 kmph,find the time taken by the boat to go 68 km downstream? Sol: Speed down stream = 13+4= 17 kmph time taken to travel 68km downstream =68/17 hrs = 4 hrs 6.A boat takes 90 min less to travel 36 miles downstream then to travel the same distence upstream. if the speed of the boat in still water is 10 mph . the speed of the stream is : Sol: Let the speed of the stream be x mph .

file:///E|/work/books/placement/09_Aptitude/boatsproblems.html[1/28/2012 12:44:57 AM]

then, speed downstream = [10+x]mph speed upstream =[10-x] mph 36/[10+x] - 36/[10-x] = 90/60 =72x*60= 90[100-x2] (x+50)(x-2) =0 x=2 kmph 7.At his usual rowing rate, Rahul 12 miles down stream in a certain river in 6 hrs less than it takes him to travel the same distence upstream. but if he could double his usual rowing rate for his 24 miles roundthe down stream 12 miles would then take only one hour less than the up stream 12 miles. what is the speed of the current in miles per hours? Sol: Let the speed in still water be x mph and the speed of the curren be y mph. then, speed upstream = (x-y) speed downstream =(x+y) 12/(x-y) - 12/(x+y) = 6 6(x2 y2) m= 2xy => x2 y2 =4y -(1) and 12/(2x-y) - 12/(2x+y) =1 => 4x2 y2 = 24y x2= ( 24y + y2)/4 -->(2) from 1 and 2 we have 4y+ y2 =( 24y+y2)/4 y=8/3 mph y= 22/3 mph 8.There is a road beside a river. two friends started from a place A, moved to a temple situated at another place B and then returned to A again. one of them moves on a cycle at a speed of 12 kmph, while the other sails on a boat at a speed of 10 kmph . if the river flows at the speedof 4 kmph , which of the two friends will return to place A first ? Sol: Clearly, The cyclist moves both ways at a speed of 12 kmph so, average speed of the cyclist = 12 kmph the boat sailor moves downstream = (10+4) = 14 kmph upstream =(10-4) = 6 kmph So, average speed of the boat sailor =[ 2*14*6]/[14+6] kmph =42/5 kmph =8.4 kmph Since, the average speed of the cyclist is greater, he will return to A first. 9.A boat takes 19 hrs for travelling downstream from point A to point B. and coming back to a point C midway between A and B. if the velocity of the sream is 4 kmph . and the speed of the boat in still water is 14 kmph. what is the distence between A and B? Sol: speed downstream =14+4 =18 kmph speed upstream = 14 -4 = 10 kmph let the distence between A and B be x km. then, x/18 + (x/2)/10 = 19 x/18 + x/20 =19 19x/180 =19 =>x = 180km Hence, the distence between A and B bw 180 km BACK

file:///E|/work/books/placement/09_Aptitude/boatsproblems.html[1/28/2012 12:44:57 AM]

Calenders
Important Facts and Formulae:

Back Back To Main

Top

Contact: 040-23000700

file:///E|/work/books/placement/09_Aptitude/calendar.html[1/28/2012 12:45:03 AM]

Calenders
Important Facts and Formulae: a) Odd Days : The number of days more than the complete number of weeks in a given period is number of odd days during that period. b) Leap year : Every year which is divisible by 4 is called a leap year. Thus each one of he year 1992,1996,2004,2008,..etc, is a eap year. Every 4th century is a leap year but no other century is a leap year thus each one of 400,800,1200,1600,2000,etc is a leap year. None of the 1900,2010,2020,2100,etc is a leap year. An year which is not a leap year is called Ordinary year. c)An ordinary year has 365 days. d) A leap year has 366 days. e)Counting of odd days : i)1 ordinary year = 365 days =52 weeks+1 day,Therefore An ordinary year has 1 Odd day. ii)One leap year = 366 days =52 weeks+2 days, Therefore a leap year has 2 Odd days. iii) 100 years = 76 ordinary years+ 24 leap years = [(76*52) weeks+76 days]+[(24*52)weeks+48 days] = 5200 weeks+124days=[5217 weeks+5 days] therefore 100 years contain 5 odd days iv)200 years contain 10(1week+3days), i.e 3 odd days v)300 years contain 15(2 weeks+1 day), i.e 1 odd day vi)400 years contain (20+1), i.e 3 weeks,so 0 Odd days similarly each one of 800,1200,1600,etc contains 0 odd days. Note:(7n+m) odd days , where m less than or equal to 7 is equivalent to m odd days ,thus ,8 odd days = 1 odd day etc. f) Some codes o remember the months and weeks: a) Week Sunday - 1 Monday - 2 Tuesday - 3 Wednesday - 4 Thursday - 5 Friday - 6 Saturday - 0 b) Month jan - 1 feb - 4 Mar - 4 Apr - 0 May - 2 june - 5 Simple problems: Shortcuts : This shortcut must be applied only starting with 19 series. Example: What day of the week on 17th june , 1998? Solution : 5 -> the given month code(august) 17 -> the given date 98->(19 th century after years) 24-> ((47/4) = 11 i.e how many leap years -------total = 144 ((144/7) = 20 and the remainder is 4) therefore in the above week table the no 4 code represents wednesday so the required day is wednesday. july - 0 Aug - 3 Sep - 6 Oct - 1 Nov - 4 Dec - 6

Top

Problem 1: The first republic day of the India was celebrated on 26th January,1950. It was Solution : 01 26 50 12 ---------total = 89 ((89/7) = 12 and the remainder is 5) therefore in the above week table represents the number 5 as thursday, so the required day was Thursday.

file:///E|/work/books/placement/09_Aptitude/calender.html[1/28/2012 12:45:03 AM]

Problem 2: Find on which day 15th august1947 ? Solution : 03 15 47 11 ---------total = 76 Then (76)/7 = 6 odd days 6 indicates friday in the above week table. Therefore required day is friday. Problem 3: Find on which day jan 26th 1956 ? Solution : 01 26 56 14 -1 (-1 indicates leapyear(i.e 1956),so 1 reduce from the total) --------total = 96 Then (96)/7 = 5 odd days 5 indicates thursday in the above week table Therefore our required day is Thursday. Problem 4: Today is friday after 62 days,it will be : Solution : Each day of the week is repeated after 7 days. so, after 63 days,it will be friday. Hence ,after 62 days, it will be thursday. Therefore the required day is thursday. Problem 5: Find the day of the week on 25th december,1995? Solution : 06 25 95 23 --------total = 149 Then (149)/7=(23)=2 odd days Therefore the required day is "Monday".

Top
Medium Problems Problem 1: jan 1, 1995 was a sunday.what day of the week lies on jan 1,1996? Solution : 01 01 96 24 -1(since 1996 was leap year) --------total = 121 Then (121)/7 = (17) = 2 odd days Therefore our required day wasMonday. Problem 2: On 8th feb,1995 it wednesday. The day of the week on 8th feb,1994 was? Solution : 04 08 94 23 --------total = 129 Then (129)/7 = (18) = 3 odd days Therefore the required day is Tuesday. Problem 3: may 6,1993 was thursday.what day of the week was on may 6,1992 ? Solution : 02 06 92 23 -1 ---------total = 122 Then (122)/7 = (17) = 3 odd days Therefore the required day is Tuesday Problem 4: jan 1, 1992 was wednesday. What day of the week was on jan 1,1993 ? Solution : 01 01 93 23 ---------total = 118

file:///E|/work/books/placement/09_Aptitude/calender.html[1/28/2012 12:45:03 AM]

Then (118)/7 = (16) = 6 odd days Therefore the required day is Friday. Problem 5: January 1,2004 was a thursday,what day of the week lies on jan ,2005? solution : The year 2004 being a leap year, it has 2 odd days. so, first day of the 2005 will be 2 days beyond thursday and so it will be saturday therefore the required day is Thursday. Problem 6: On 8th march,2005,wednesday falls what day of the week was it on 8th march,2004? Solution : the year 2004 being a leap year,it has 2 odd days. so, the day on8th march,2005 will be two days beyond the day on 8th march,2004.but 8th march,2005 is wednesday. so, 8th march,2004 is monday. Therefore the required day is Monday. Problem 7: what was the day of the week on 19th september ,1986 ? Solution : 06 19 86 21 --------total = 132 Then ((132/7 = 18 and the remainder is 6) In the above week table represents the number 6 is friday. Therefore the required day is Friday.

Top
Typical problems Problem 1: On what dates of october,1994 did monday fall ? Solution : 01 01 94 23 ------total = 119 Then (119)/7 = (17) = 0 odd days so the day is saturday Therefore in october first the day is saturday.so, the monday fell on 3rd october 1994.During october 1994, monday fell on 3rd ,10th,17th and 24th. Problem 2: How many days are there from 2nd january 1995 to 15 th march,1995 ? Solution : Jan + Feb + March 30 + 28 + 15 = 73 days

Problem 3: The year next to 1996 having the same calendar as that of 1996 is ? Solution : Starting with 1996 , we go on countig the number of odd days till the sum is divisible by 7. Year 1996 1997 1998 1999 2000 odd days 2 1 1 1 2 2 + 1 + 1 + 1 + 2 = 7 odd days i.e odd day. Therefore calendar for 2001 will be the same as that of 1995. Problem 4: The calendar for 1990 is same as for : Solution: count the number of days 1990 onwards to get 0 odd day. Year 1990 1991 1992 1993 1994 1995 oddd days 1 1 2 1 1 1 1 + 1 + 2 + 1 + 1 + 1 = 7 or 0 odd days Therefore calendar for 1990 is the same as for the year 1996. Problem 5: The day on 5th march of year is the same day on what date of the same year? Solution: In the given monthly code table represents the march code and november code both are same.that means any date in march is the same day of week as the corresponding date in november of that year, so the same day falls on 5th november.

Back Back To Main

Top

file:///E|/work/books/placement/09_Aptitude/calender.html[1/28/2012 12:45:03 AM]

Contact: 040-23000700

file:///E|/work/books/placement/09_Aptitude/calender.html[1/28/2012 12:45:03 AM]

Chain Rule
Important Facts: Direct Proportion: Two Quantities are said to be directly proportional, if on the increase (or decrease) of th one, the other increases(or decreases) to the same extent. Ex:(i) Cost is directly proportional to the number of articles. (More articles, More cost). (ii) Work done is directly proportional to the number of men working on it. (More men, more work). Indirect Proportion: Two Quantities are said to be indirectly proportional,if on the increase of the one , the other decreases to the same extent and vice - versa. Ex:(i) The time taken by a car covering a certain distance is inversely proportional to th speed of the car.(More speed, less is the time taken to cover the distance). (ii) Time taken to finish a work is inversely proportional to the number of persons working at it. (More persons, less is the time taken to finish a job). Nte: In solving Questions by chain rule, we compare every item with the term to be found out. Problems 1)If 15 toys cost Rs.234, what do 35 toys cost ? Sol: Let the required cost be Rs. x then more toys more cost(direct proportion) 15:35:: 234:x (15*x)=(234*35) x=(234*35) /(15)= 546 Rs

2)If 36 men can do a piece of work in 25hours, in how many hours will 15men do it ? Sol: Let the required number of hours be x. less men more hours(Indirect proportion). 15:36::25:x (15*x)=(36*25) x=(36*25) /15 x=60 For 15 men it takes 60 hours.

3)If 9 engines consume 24metric tonnes of coal, when each is working 8 hours a day, how much coal will be required for 8 engines, each running 13 hours a day, it being given that 3 engines of former type consume as much as 4 engines of latter type ? Sol: Let 3 engines of former type consume 1 unit in 1 hour. 4 engines of latter type consume 1 unit in 1 hour. 1 engine of former type consumes 1/3 unit in 1 hour. 1 engine of latter type consumes unit in 1 hour. Let required consumption of coal be x units. Less engines, less coal consumed.(direct) More working hours, more coal consumed(direct) Less rate of consumption, less coal consumed (direct) 9:8 8:13 1/3:1/4 :: 24:x

(9*8*(1/3)*x)=(8*13*(1/4)*24) 24x=624 x=26 metric tonnes.

Top
Complex Problems 1)A contract is to be completed in 46 days and 117 men were set to work, each working 8 hours a day. After 33 days, 4/7 of the work is completed. How many additional men may be employed so that the work may be completed in time, each man now working 9 hours a day ? Sol: 4/7 of work is completed . Remaining work=1 - 4/7 =3/7 Remaining period= 46 - 33 =13 days Less work, less men(direct proportion) less days, more men(Indirect proportion) More hours/day, less men(Indirect proportion)

file:///E|/work/books/placement/09_Aptitude/chainrule.html[1/28/2012 12:45:04 AM]

work 4/7:3/7 Days 13:33 hrs/day 9:8

::

117:x

(4/7)*13*9*x=(3/7)*33*8*117 x=(3*33*8*117) / (4*13*9) x=198 men So, additional men to be employed=198 - 117=81 2)A garrison had provisions for a certain number of days. After 10 days, 1/5 of the men desert and it is found that the provisions will now last just as long as before. How long was that ? Sol: Let initially there be x men having food for y days. After, 10 days x men had food for ( y - 10)days Also, (x - x/5) men had food for y days. x(y - 10)=(4x/5)*y => (x*y) - 50x=(4(x*y)/5) 5(x*y) - 4(x*y)=50x x*y=50x y=50

3)A contractor undertook to do a certain piece of work in 40 days. He engages 100 men at the beginning and 100 more after 35 days and completes the work in stipulated time. If he had not engaged the additional men, how many days behind schedule would it be finished ? Sol: 40 days - 35 days=5 days =>(100*35)+(200*5) men can finish the work in 1 day. 4500 men can finish it in 4500/100= 45 days This s 5 days behind the schedule.

4)12 men and 18 boys,working 7 hors a day, can do a piece if work in 60 days. If a man works equal to 2 boys, then how many boys will be required to help 21 men to do twice the work in 50 days, working 9 hours a day ? Sol: 1man =2 boys 12men+18boys=>(12*2+18)boys=42 boys let the required number of boys=x 21 men+x boys =>((21*2)+x) boys =>(42+x) boys less days, more boys(Indirect proportion) more hours per day, less boys(Indirect proportion) days 50:60 hrs/day 9:15/2 :: 42:(42+x) work 1:2 (50*9*1*(42+x))=60*(15/2)*2*42 (42+x)= (60*15*42)/(50*9)= 84 x=84 - 42= 42 =42 42 days behind the schedule it will be finished.

Back Back To Main

Top

Contact: 040-23000700

file:///E|/work/books/placement/09_Aptitude/chainrule.html[1/28/2012 12:45:04 AM]

APTITUDE
Numbers H.C.F and L.C.M Decimal Fractions Simplification Square and Cube roots Average Problems on Numbers Problems on Ages Surds and Indices Percentage Profit and Loss Ratio And Proportions Partnership Chain Rule Time and Work Pipes and Cisterns Time and Distance Trains Boats and Streams Alligation or Mixture Simple Interest Compound Interest Logorithms Areas Volume and Surface area Races and Games of Skill Calendar Clocks Stocks ans Shares True Discount Bankers Discount Oddmanout and Series Data Interpretation probability Permutations and Combinations Puzzles CHAIN RULE CONCEPT SIMPLE PROBLEMS COMPLEX PROBLEMS

file:///E|/work/books/placement/09_Aptitude/chainrule1.html[1/28/2012 12:45:04 AM]

APTITUDE
Numbers H.C.F and L.C.M Decimal Fractions Simplification Square and Cube roots Average Problems on Numbers Problems on Ages Surds and Indices Percentage Profit and Loss Ratio And Proportions Partnership Chain Rule Time and Work Pipes and Cisterns Time and Distance Trains Boats and Streams Alligation or Mixture Simple Interest Compound Interest Logorithms Areas Volume and Surface area Races and Games of Skill Calendar Clocks Stocks ans Shares True Discount Bankers Discount Oddmanout and Series Data Interpretation probability Permutations and Combinations Puzzles BACK

COMPLEX PROBLEMS
1)A contract is to be completed in 46 days and 117 men were set to work, each working 8 hours a day. After 33 days, 4/7 of the work is completed. How many additional men may be employed so that the work may be completed in time, each man now working 9 hours a day? Sol: 4/7 of work is completed . Remaining work=1- 4/7 =3/7 Remaining period= 46-33 =13 days Less work, less men(direct proportion) less days, more men(Indirect proportion) More hours/day, less men(Indirect proportion) work 4/7:3/7 Days 13:33 hrs/day 9:8 :: 117:x

(4/7)*13*9*x=(3/7)*33*8*117 x=(3*33*8*117) / (4*13*9) x=198 men So, additional men to be employed=198 -117=81 2)A garrison had provisions for a certain number of days. After 10 days, 1/5 of the men desert and it is found that the provisions will now last just as long as before. How long was that? Sol: Let initially there be x men having food for y days. After, 10 days x men had food for ( y-10)days Also, (x -x/5) men had food for y days. x(y-10)=(4x/5)*y => (x*y) -50x=(4(x*y)/5) 5(x*y)-4(x*y)=50x x*y=50x y=50

3)A contractor undertook to do a certain piece of work in 40 days. He engages 100 men at the beginning and 100 more after 35 days and completes the work in stipulated time. If he had not engaged the additional men, how many days behind schedule would it be finished? Sol: 40 days- 35 days=5 days =>(100*35)+(200*5) men can finish the work in 1 day. 4500 men can finish it in 4500/100= 45 days This s 5 days behind the schedule.

4)12 men and 18 boys,working 7 hors a day, can do a piece if work in 60 days. If a man works equal to 2 boys, then how many boys will be required to help 21 men to do twice the work in 50 days, working 9 hours a day? Sol: 1man =2 boys 12men+18boys=>(12*2+18)boys=42 boys let the required number of boys=x 21 men+x boys =>((21*2)+x) boys =>(42+x) boys less days, more boys(Indirect proportion) more hours per day, less boys(Indirect proportion) days 50:60 hrs/day 9:15/2 :: 42:(42+x) work 1:2 (50*9*1*(42+x))=60*(15/2)*2*42 (42+x)= (60*15*42)/(50*9)= 84 x=84-42= 42 =42 42 days behind the schedule it will be finished.

BACK

file:///E|/work/books/placement/09_Aptitude/chainrulecomplex.html[1/28/2012 12:45:04 AM]

file:///E|/work/books/placement/09_Aptitude/chainrulecomplex.html[1/28/2012 12:45:04 AM]

APTITUDE
Numbers H.C.F and L.C.M Decimal Fractions Simplification Square and Cube roots Average Problems on Numbers Problems on Ages Surds and Indices Percentage Profit and Loss Ratio And Proportions Partnership Chain Rule Time and Work Pipes and Cisterns Time and Distance Trains Boats and Streams Alligation or Mixture Simple Interest Compound Interest Logorithms Areas Volume and Surface area Races and Games of Skill Calendar Clocks Stocks ans Shares True Discount Bankers Discount Oddmanout and Series Data Interpretation probability Permutations and Combinations Puzzles BACK

CHAIN RULE
1.DIRECT PROPORTION: Two Quantities are said to be directly proportional, if on the increase (or decrease) of th one, the other increases(or decreases) to the same extent. Ex:(i) Cost is directly proportional to the number of articles.(More articles, More cost). (ii)Work done is directly proportional to the number of men working on it. (More men, more work). 2.INDIRECT PROPORTION: Two Quantities are said to be indirectly proportional,if on the increase of the one , the other decreases to the same extent and vice-versa. Ex:(i) The time taken by a car covering a certain distance is inversely proportional to th speed of the car.(More speed, less is the time taken to cover the distance). (ii)Time taken to finish a work is inversely proportional to the number of persons working at it. (More persons, less is the time taken to finish a job). NOTE: In solving Questions by chain rule, we compare every item with the term to be found out.

BACK

file:///E|/work/books/placement/09_Aptitude/chainruleconcept.html[1/28/2012 12:45:05 AM]

APTITUDE
Numbers H.C.F and L.C.M Decimal Fractions Simplification Square and Cube roots Average Problems on Numbers Problems on Ages Surds and Indices Percentage Profit and Loss Ratio And Proportions Partnership Chain Rule Time and Work Pipes and Cisterns Time and Distance Trains Boats and Streams Alligation or Mixture Simple Interest Compound Interest Logorithms Areas Volume and Surface area Races and Games of Skill Calendar Clocks Stocks ans Shares True Discount Bankers Discount Oddmanout and Series Data Interpretation probability Permutations and Combinations Puzzles BACK

SIMPLE PROBLEMS
1)If 15 toys cost Rs.234, what do 35 toys cost ? Sol: Let the required cost be Rs. x then more toys more cost(direct proportion) 15:35:: 234:x (15*x)=(234*35) x=(234*35) /(15)= 546 Rs

2)If 36 men can do a piece of work in 25hours, in how many hours will 15men do it? Sol: Let the required number of hours be x. less men more hours(Indirect proportion). 15:36::25:x (15*x)=(36*25) x=(36*25) /15 x=60 For 15 men it takes 60 hours.

3)If 9 engines consume 24metric tonnes of coal, when each is working 8 hours a day, how much coal will be required for 8 engines, each running 13 hours a day, it being given that 3 engines of former type consume as much as 4 engines of latter type? Sol: Let 3 engines of former type consume 1 unit in 1 hour. 4 engines of latter type consume 1 unit in 1 hour. 1 engine of former type consumes 1/3 unit in 1 hour. 1 engine of latter type consumes unit in 1 hour. Let required consumption of coal be x units. Less engines, less coal consumed.(direct) More working hours, more coal consumed(direct) Less rate of consumption, less coal consumed (direct) 9:8 8:13 1/3:1/4 :: 24:x

(9*8*(1/3)*x)=(8*13*(1/4)*24) 24x=624 x=26 metric tonnes.

BACK

file:///E|/work/books/placement/09_Aptitude/chainrulesimple.html[1/28/2012 12:45:05 AM]

APTITUDE
Numbers H.C.F and L.C.M Decimal Fractions Simplification Square and Cube roots Average Problems on Numbers Problems on Ages Surds and Indices Percentage Profit and Loss Ratio And Proportions Partnership Chain Rule Time and Work Pipes and Cisterns Time and Distance Trains Boats and Streams Alligation or Mixture Simple Interest Compound Interest Logorithms Areas Volume and Surface area Races and Games of Skill Calendar Clocks Stocks ans Shares True Discount Bankers Discount Oddmanout and Series Data Interpretation probability Permutations and Combinations Puzzles

COMPOUND INTEREST
CONCEPT SIMPLE PROBLEMS MEDIUM PROBLEMS COMPLEX PROBLEMS

file:///E|/work/books/placement/09_Aptitude/CI.html[1/28/2012 12:45:05 AM]

APTITUDE
Numbers H.C.F and L.C.M Decimal Fractions Simplification Square and Cube roots Average Problems on Numbers Problems on Ages Surds and Indices Percentage Profit and Loss Ratio And Proportions Partnership Chain Rule Time and Work Pipes and Cisterns Time and Distance Trains Boats and Streams Alligation or Mixture Simple Interest Compound Interest Logorithms Areas Volume and Surface area Races and Games of Skill Calendar Clocks Stocks ans Shares True Discount Bankers Discount Oddmanout and Series Data Interpretation probability Permutations and Combinations Puzzles BACK

COMPLEX PROBLEMS
1.A certain sum amounts to Rs.7350 in 2 yrs and to Rs.8575 in 3 yrs.Find the sum and rate%? sol: S.I. on Rs.7350 for 1yr =Rs.(8575-7350) =Rs.1225 S.I. on Rs.7350 for 2yrs=Rs.2*1225 =Rs.2450 PTR/100=2450 =>P*2*R/100=2450 Since S.I. on Rs.7350 for 1yr =Rs.(8575-7350) =Rs.1225 Rate R=100*1225/(7350*10 =16 2/3% Since it is C.I ,Let sum be Rs.X Then X[1+(R/100)]2=7350 =>X[1+(50/100)]2=7350 =>X=7350*(36/49) Sum=Rs.5400

2.If the difference between C.I compounded halfyearly and S.I on a sum at 10% per annum for one yr is Rs.25 the sum is sol: p[1+((R/2)/100)]2n-PTR/100=25 P[1+((10/2)/100)]2n-P*1*10/100=25 =>P=Rs.400

3.A man borrowed Rs.800 at 10 % per annum S.I and immediately lent the whole sum at 10% per annum C.I What does he gain at the end of 2yrs? sol: C.I.=Rs.[800[1+(10/100)2]-800]=Rs.168 S.I=Rs.[800*10*2/100]=Rs.160 Gain=C.I-S.I=Rs(168-160) =Rs.8

4.On what sum of money will be S.I for 3 yrs at 8% per annum be half of C.I on Rs.400 for 2 yrs at 10% per annum? sol: C.I on Rs.400 for 2yrs at 10%=Rs.[400*[1+(10/100)]2-400] =Rs.84 Required S.I =1/2*84=42/New S.I=Rs.42,Time=3yrs Rate=8% Sum=Rs.[100*42/(3*8)] =Rs.175

5.A sum of money placed at C.I doubles itself in 5yrs .It will amount to

file:///E|/work/books/placement/09_Aptitude/CIcomplex.html[1/28/2012 12:45:06 AM]

8 times itself in------------sol: p[1+(R/100)]5=2P =>[1+(R/100)]5=2 To become 8 times =>8P p[1+(R/100)]5=2^3P =[1+(R/100)]^(5*3) =[1+(R/100)]^15 n=15years

BACK

file:///E|/work/books/placement/09_Aptitude/CIcomplex.html[1/28/2012 12:45:06 AM]

APTITUDE
Numbers H.C.F and L.C.M Decimal Fractions Simplification Square and Cube roots Average Problems on Numbers Problems on Ages Surds and Indices Percentage Profit and Loss Ratio And Proportions Partnership Chain Rule Time and Work Pipes and Cisterns Time and Distance Trains Boats and Streams Alligation or Mixture Simple Interest Compound Interest Logorithms Areas Volume and Surface area Races and Games of Skill Calendar Clocks Stocks ans Shares True Discount Bankers Discount Oddmanout and Series Data Interpretation probability Permutations and Combinations Puzzles BACK

CONCEPT
Compound Interest:Sometimes it so happens that the borrower and the lender agree to fix up a certain unit of time ,say yearly or half-yearly or quarterly to settle the previous account. In such cases ,the amount after the first unit of time becomes the principal for the 2nd unit ,the amount after second unit becomes the principal for the 3rd unit and so on. After a specified period ,the difference between the amount and the money borrowed is called Compound Interest for that period. Formulae:

Let principal=p,Rate=R% per annum Time=nyears 1.When interest is compounded Annually, Amount=P[1+(R/100)]n 2.When interest is compounded Halfyearly, Amount=P[1+((R/2)100)]2n 3.When interest is compounded Quaterly, Amount=P[1+((R/4)100)]4n 4.When interest is compounded Annually,but time in fractions say 3 2/5 yrs Amount=P[1+(R/100)]3[1+((2R/5)/100)] 5.When rates are different for different years R1%,R2%,R3% for 1st ,2nd , 3rd yrs respectively Amount=P[1+(R1/100)][1+(R2/100)][1+(R3/100)] 6.Present Worth of Rs.X due n years hence is given by Present Worth=X/[1+(R/100)]n
BACK

file:///E|/work/books/placement/09_Aptitude/CIconcept.html[1/28/2012 12:45:06 AM]

APTITUDE
Numbers H.C.F and L.C.M Decimal Fractions Simplification Square and Cube roots Average Problems on Numbers Problems on Ages Surds and Indices Percentage Profit and Loss Ratio And Proportions Partnership Chain Rule Time and Work Pipes and Cisterns Time and Distance Trains Boats and Streams Alligation or Mixture Simple Interest Compound Interest Logorithms Areas Volume and Surface area Races and Games of Skill Calendar Clocks Stocks ans Shares True Discount Bankers Discount Oddmanout and Series Data Interpretation probability Permutations and Combinations Puzzles BACK

MEDIUM PROBLEMS
1.The difference between C.I and S.I. on a certain sum at 10% per annum for 2 yrs is Rs.631.find the sum Sol: MethodI: NOTE: a) For 2 yrs -------->sum=(1002D/R2) b) For 3 yrs -------->sum=(1003D/R2(300+R)) Sum=1002*631/102 =Rs.63100 MethodII: Let the sum be Rs.X,Then C.I.=X[1+(10/100)]2-X S.I=(X*10*2)/100 =X/5 C.I-S.I.=21X/100-X/5 =X/100 X/100=631 X=Rs.63100

2.If C.I on a certain sum for 2 yrs at 12% per annum is Rs.1590. What would be S.I? sol: C.I.=Amount-Principle Let P be X C.I=X[1+(12/100)]2-X =>784X/625-X=1590 =>X=Rs.6250 S.I=(6250*12*2)/100=Rs.1500

3.A sum of money amounts to Rs.6690 after 3 yrs and to Rs.10035b after 6 yrs on C.I .find the sum sol: For 3 yrs, Amount=P[1+(R/100)]3=6690-----------------------(1) For 6 yrs, Amount=P[1+(R/100)]6=10035----------------------(2) (1)/(2)------------[1+(R/100)]3=10035/6690 =3/2 [1+(R/100)]3=3/2-----------------(3) substitue (3) in (1) p*(3/2)=6690 =>p=Rs.4460 sum=Rs.4460

4.A sum of money doubles itself at C.I in 15yrs.In how many yrs will it become 8 times?

file:///E|/work/books/placement/09_Aptitude/CImedium.html[1/28/2012 12:45:06 AM]

sol: Compound Interest for 15yrs p[1+(R/100)]15 p[1+(R/100)]15=2P =>p[1+(R/100)]n=8P =>[1+(R/100)]n=8 =>[1+(R/100)]n=23 =>[1+(R/100)]n=[1+(R/100)]15*3 since [1+(R/100)] =2 n=45yrs

5.The amount of Rs.7500 at C.I at 4% per annum for 2yrs is sol: Iyear------------------7500+300(300------Interest on 7500) IIyear ----------------7500+300+12(12------------4% interest on 300) Amount=7500+300+300+12 =Rs.8112

6.The difference between C.I and S.I on a sum of money for 2 yrs at 121/2% per annum is Rs.150.the sum is sol: Sum=1002D/R2=( 1002*150) /(25/2)2=Rs.9600

7.If the S.I on sum of money at 15% per annum for 3yrs is Rs.1200, the C.I on the same sum for the same period at same rate is--------sol: S.I=1200 P*T*R/100=1200 P*3*5/100=1200 =>P=Rs.8000 C.I for Rs.8000 at 5% for 3 yrs is-------------8000+400 -------------8000+400+20 -------------8000+400+20+20+1 C.I =400+400+20+400+20+20+1 =Rs.1261

BACK

file:///E|/work/books/placement/09_Aptitude/CImedium.html[1/28/2012 12:45:06 AM]

APTITUDE
Numbers H.C.F and L.C.M Decimal Fractions Simplification Square and Cube roots Average Problems on Numbers Problems on Ages Surds and Indices Percentage Profit and Loss Ratio And Proportions Partnership Chain Rule Time and Work Pipes and Cisterns Time and Distance Trains Boats and Streams Alligation or Mixture Simple Interest Compound Interest Logorithms Areas Volume and Surface area Races and Games of Skill Calendar Clocks Stocks ans Shares True Discount Bankers Discount Oddmanout and Series Data Interpretation probability Permutations and Combinations Puzzles BACK

SIMPLE PROBLEMS
1.Find CI on Rs.6250 at 16% per annum for 2yrs ,compounded annually. Sol: Rate R=16,n=2,Principle=Rs.6250 Method1: Amount=P[1+(R/100)]n =6250[1+(16/100)]2 =Rs.8410 C.I=Amount-P =8410-6250 =Rs.2160 Method2: Iyear------------------6250+1000 \\Interest for 1st yr on 6250 II yr---------------6250+1000+160 \\Interest for I1yr on 1000 C.I.=1000+1000+160 =Rs.2160

2.Find C.I on Rs.16000 at 20% per annum for 9 months compounded quaterly Sol: MethodI: R=20% 12months------------------------20% => 3 months------------------------5% For 9 months,there are '3' 3months --------16000+800 --------16000+800+40 --------16000+800+40+10+2 =>Rs.2522 MethodII: Amount=P[1+(R/100)]n =16000[1+(5/100)]3 =Rs.18522 C.I=18522-16000 =Rs.2522

BACK

file:///E|/work/books/placement/09_Aptitude/CIsimple.html[1/28/2012 12:45:07 AM]

Clocks
General Concepts: The face or dial of a watch is a circle whose circumference is divided into 60 equal parts,called minute spaces. A clock has two hands, the Smaller one is called the hour hand or short hand while the larger one is called the minute hand or long hand. Important points: a) In every 60 minutes, the minute hand gains 55 minutes on the hour hand b)In every hour, both the hands coincide once ,i.e 0 degrees. c)the hands are in the same straight line when they are coincident or opposite to each other. i.e 0 degrees or 180 degrees. d)when the two hands are at right angles, they are 15 minute spaces apart,i.e 90 degrees. e)when the hands are in the opposite directions,they are 30 minute spaces apart,i.e 180 degrees. f)Angle traced by hour hand in 12hrs = 360 degrees. g)Angle traced by minute hand in 60 min = 360 degrees. If a watch or a clock indicated 8.15,when the correct time is 8, it is said to be 15 minutes too fast. On the other hand, if it indicates 7.45, when the correct time is 8,it is said to be 15 minutes slow. h)60 min --> 360 degrees 1 min --> 60 i)the hands of a clock coincide in a day or 24 hours is 22 times, in 12hours 11minutes. j)the hands of clock are straight in a day is 44 times . k)the hands of a clock at right angle in a day is 44 times . l)the hands of a clock in straight line but opposite in direction is 22 times per day

Top
Simple Problems: Type1: Find the angle between the hour hand and the minute hand of a clock when the time is 3.25 solution : In this type of problems the formulae is as follows 30*[hrs-(min/5)]+(min/2) In the above problem the given data is time is 3.25. that is applied in the formulae 30*[3-(25/5)]+(25/2)30*(15-25)/5+25/2 = 30*(-10/5)+25/2 = -300/5+25/2 = -600+(25/2)=-475/10=-47.5 i.e 47 1/20 therefore the required angle is 47 1/20. Note:The -sign must be neglected. Another shortcut for type1 is : The formulae is 6*x-(hrs*60+X)/2 Here x is the given minutes, so in the given problem the minutes is 25 minutes, that is applied in the given formulae 6*25-(3*60+25)/2 150-205/2 (300-205)/2=95/2 =47 1/20. therefore the required angle is 47 1/20. Type2: At what time between 2 and 3 o' clock will be the hands of a clock be together? Solution : In this type of problems the formulae is 5*x*(12/11) Here x is replaced by the first interval of given time. here i.e 2. In the above problem the given data is between 2 and 3 o' clock 5*2*12/11 =10*12/11=120/11=10 10/11min. Therefore the hands will coincide at 10 10/11 min.past2.

file:///E|/work/books/placement/09_Aptitude/clocks.html[1/28/2012 12:45:07 AM]

Another shortcut for type2 is: Here the clocks be together but not opposite to each other so the angle is 0 degrees. so the formulae is 6*x-(2*60+x)/2=06*x-(120+x)/2=012x-120-x=0 11x=120 x=120/11=10 10/11 therefore the hands will be coincide at 10 10/11 min.past2.

Top
Medium Problems Type3: At what time between 4 and 5 o'clock will the hands of a clock be at rightangle? Solution : In this type of problems the formulae is (5*x + or -15)*(12/11) Here x is replaced by the first interval of given time here i.e 4 Case 1 : (5*x + 15)*(12/11) (5*4 +15)*(12/11) (20+15)*(12/11) 35*12/11=420/11=38 2/11 min. Therefore they are right angles at 38 2/11 min .past4 Case 2 : (5*x-15)*(12/11) (5*4-15)*(12/11) (20-15)*(12/11) 5*12/11=60/11 min=5 5/11min Therefore they are right angles at 5 5/11 min.past4. Another shortcut for type 3 is: Here the given angle is right angle i.e 900. Case 1 : The formulae is 6*x-(hrs*60+x)/2=Given angle 6*x-(4*60+x)/2=90 6*x-(240+x)/2=90 12x-240-x=180 11x=180+240 11x=420 x=420/11= 38 2/11 min Therefore they are at right angles at 38 2/11 min. past4. Case 2 : The formulae is (hrs*60+x)/2-(6*x)=Given angle (4*60+x)/2-(6*x)=90 (240+x)/2-(6*x)=90 240+x-12x=180 -11x+240=180 240-180=11x x=60/11= 5 5/11 min Therefore they art right angles at 5 5/11 min past4. Type 4: Find at what time between 8 and 9 o'clock will the hands of a clock be in the same straight line but not together ? Solution : In this type of problems the formulae is (5*x-30)*12/11 x is replaced by the first interval of given time Here i.e 8 (5*8-30)*12/11 (40-30)*12/11 10*12/11=120/11 min=10 10/11 min. Therefore the hands will be in the same straight line but not together at 10 10/11 min.past 8. Another shortcut for type 4 is: Here the hands of a clock be in the same straight line but not together the angle is 180 degrees. The formulae is (hrs*60+x)/2-(6*x)=Given angle (8*60+x)/2-6*x=180 (480+x)/2-(6*x)=180 480+x-12*x=360 11x=480-360 x=120/11=10 10/11 min. therefore the hands will be in the same straight line but not together at 10 10/11 min. past8.

Top
Type 5: At what time between 5 and 6 o clock are the hands of a 3 minutes apart ? Solution : In this type of problems the formuae is (5*x+ or - t)*12/11 Here x is replaced by the first interval of given time here xis 5. t is spaces apart Case 1 : (5*x+t)*12/11 (5*5+3)*12/11 28*12/11 = 336/11=31 5/11 min therefore the hands will be 3 min .apart at 31 5/11 min.past5. Case 2 : (5*x-t)*12/11 (5*5-3)*12/11 (25-3)*12/11=24 min therefore the hands wi be 3 in apart at 24 min past 5. Typicalproblems problems: A watch which gains uniformly ,is 5 min,slow at 8 o'clock in the morning on sunday and it is 5 min.48 sec.fast at 8 p.m on following sunday. when was it correct? Solution :

file:///E|/work/books/placement/09_Aptitude/clocks.html[1/28/2012 12:45:07 AM]

Time from 8 am on sunday to 8 p.m on following sunday = 7 days 12 hours = 180 hours the watch gains (5+(5 4/5))min .or 54/5 min. in 180 hours Now 54/5 minare gained in 180 hours. Therefore 5 minutes are gained in(180*5/54*5)hours=83 hours20 min. =3 days11hrs20min. therefore watch is correct at 3 days 11 hours 20 minutes after 8 a.m of sunday therefore it wil be correct at 20 min.past 7 p.m on wednesday

Back Back To Main

Top

Contact: 040-23000700

file:///E|/work/books/placement/09_Aptitude/clocks.html[1/28/2012 12:45:07 AM]

APTITUDE
Numbers H.C.F and L.C.M Decimal Fractions Simplification Square and Cube roots Average Problems on Numbers Problems on Ages Surds and Indices Percentage Profit and Loss Ratio And Proportions Partnership Chain Rule Time and Work Pipes and Cisterns Time and Distance Trains Boats and Streams Alligation or Mixture Simple Interest Compound Interest Logorithms Areas Volume and Surface area Races and Games of Skill Calendar Clocks Stocks ans Shares True Discount Bankers Discount Oddmanout and Series Data Interpretation probability Permutations and Combinations Puzzles BACK

CLOCKS

General Concepts :

The face or dial of a watch is a circle whose circumference is divided into 60 equal parts,called minute spaces. A clock has two hands, the Smaller one is called the hour hand or short hand while the larger one is called the minute hand or long hand. Important points : a) In every 60 minutes, the minute hand gains 55 minutes on the hour hand b)In every hour, both the hands coincide once ,i.e 0 degrees. c)the hands are in the same straight line when they are coincident or opposite to each other. i.e 0 degrees or 180 degrees. d)when the two hands are at right angles, they are 15 minute spaces apart,i.e 90 degrees. e)when the hands are in the opposite directions,they are 30 minute spaces apart,i.e 180 degrees. f)Angle traced by hour hand in 12hrs = 360 degrees. g)Angle traced by minute hand in 60 min = 360 degrees. If a watch or a clock indicated 8.15,when the correct time is 8, it is said to be 15 minutes too fast. On the other hand , if it indicates 7.45,when the correct time is 8,it is said to be 15 minutes slow. h)60 min --> 360 degrees 1 min --> 60 i)the hands of a clock coincide in a day or 24 hours is 22 times ,in 12hours 11minutes. j)the hands of clock are straight in a day is 44 times . k)the hands of a clock at right angle in a day is 44 times . l)the hands of a clock in straight line but opposite in direction is 22 times per day
BACK

file:///E|/work/books/placement/09_Aptitude/clocksnotes.html[1/28/2012 12:45:07 AM]

APTITUDE
Numbers H.C.F and L.C.M Decimal Fractions Simplification Square and Cube roots Average Problems on Numbers Problems on Ages Surds and Indices Percentage Profit and Loss Ratio And Proportions Partnership Chain Rule Time and Work Pipes and Cisterns Time and Distance Trains Boats and Streams Alligation or Mixture Simple Interest Compound Interest Logorithms Areas Volume and Surface area Races and Games of Skill Calendar Clocks Stocks ans Shares True Discount Bankers Discount Oddmanout and Series Data Interpretation probability Permutations and Combinations Puzzles BACK

CLOCKS
Medium Problems

Type3 : At what time between 4 and 5 o'clock will the hands of a clock be at rightangle? Solution : In this type of problems the formulae is (5*x + or -15)*(12/11) Here x is replaced by the first interval of given time here i.e 4 Case 1 : (5*x + 15)*(12/11) (5*4 +15)*(12/11) (20+15)*(12/11) 35*12/11=420/11=38 2/11 min. Therefore they are right angles at 38 2/11 min .past4 Case 2 : (5*x-15)*(12/11) (5*4-15)*(12/11) (20-15)*(12/11) 5*12/11=60/11 min=5 5/11min Therefore they are right angles at 5 5/11 min.past4. Another shortcut for type 3 is :Here the given angle is right angle i.e 900. Case 1 : The formulae is 6*x-(hrs*60+x)/2=Given angle 6*x-(4*60+x)/2=90 6*x-(240+x)/2=90 12x-240-x=180 11x=180+240 11x=420 x=420/11= 38 2/11 min Therefore they are at right angles at 38 2/11 min. past4. Case 2 : The formulae is (hrs*60+x)/2-(6*x)=Given angle (4*60+x)/2-(6*x)=90 (240+x)/2-(6*x)=90 240+x-12x=180 -11x+240=180 240-180=11x x=60/11= 5 5/11 min Therefore they art right angles at 5 5/11 min past4. Type 4 : Find at what time between 8 and 9 o'clock will the hands of a clock be in the same straight line but not together ? Solution : In this type of problems the formulae is

file:///E|/work/books/placement/09_Aptitude/cmediumproblems.html[1/28/2012 12:45:08 AM]

(5*x-30)*12/11 x is replaced by the first interval of given time Here i.e 8 (5*8-30)*12/11 (40-30)*12/11 10*12/11=120/11 min=10 10/11 min. Therefore the hands will be in the same straight line but not together at 10 10/11 min.past 8. Another shortcut for type 4 is : Here the hands of a clock be in the same straight line but not together the angle is 180 degrees. The formulae is (hrs*60+x)/2-(6*x)=Given angle (8*60+x)/2-6*x=180 (480+x)/2-(6*x)=180 480+x-12*x=360 11x=480-360 x=120/11=10 10/11 min. therefore the hands will be in the same straight line but not together at 10 10/11 min. past8. Type 5 : At what time between 5 and 6 o clock are the hands of a 3 minutes apart ? Solution : In this type of problems the formuae is (5*x+ or - t)*12/11 Here x is replaced by the first interval of given time here xis 5. t is spaces apart Case 1 : (5*x+t)*12/11 (5*5+3)*12/11 28*12/11 = 336/11=31 5/11 min therefore the hands will be 3 min .apart at 31 5/11 min.past5. Case 2 : (5*x-t)*12/11 (5*5-3)*12/11 (25-3)*12/11=24 min therefore the hands wi be 3 in apart at 24 min past 5.

file:///E|/work/books/placement/09_Aptitude/cmediumproblems.html[1/28/2012 12:45:08 AM]

Compound Intrest
Important Facts and Formulae: Compound Interest: Sometimes it so happens that the borrower and the lender gree to fix up a certain unit of time ,say yearly or half-yearly or quarterly to settle the previous account. In such cases ,the amount after the first unit of time becomes the principal for the 2nd unit ,the amount after second unit becomes the principal for the 3rd unit and so on. After a specified period ,the difference between the amount and the money borrowed is called Compound Interest for that period. Formulae: Let principal=p,Rate=R% per annum Time=nyears 1.When interest is compounded Annually, Amount=P[1+(R/100)]n 2.When interest is compounded Halfyearly, Amount=P[1+((R/2)100)]2n 3.When interest is compounded Quaterly, Amount=P[1+((R/4)100)]4n 4.When interest is compounded Annually,but time in fractions say 3 2/5 yrs Amount=P[1+(R/100)]3[1+((2R/5)/100)] 5.When rates are different for different years R1%,R2%,R3% for 1st ,2nd ,3rd yrs respectively Amount=P[1+(R1/100)][1+(R2/100)][1+(R3/100)] 6.Present Worth of Rs.X due n years hence is given by Present Worth=X/[1+(R/100)]n Simple Problems 1.Find CI on Rs.6250 at 16% per annum for 2yrs ,compounded annually. Sol: Rate R=16,n=2,Principle=Rs.6250 Method1: Amount=P[1+(R/100)]n =6250[1+(16/100)]2 =Rs.8410 C.I=Amount-P =8410-6250 =Rs.2160 Method2: Iyear------------------6250+1000 \\Interest for 1st yr on 6250 II yr---------------6250+1000+160 \\Interest for I1yr on 1000 C.I.=1000+1000+160 =Rs.2160 2.Find C.I on Rs.16000 at 20% per annum for 9 months compounded quaterly Sol: MethodI: R=20% 12months------------------------20% => 3 months------------------------5% For 9 months,there are '3' 3months --------16000+800 --------16000+800+40 --------16000+800+40+10+2 =>Rs.2522 MethodII: Amount=P[1+(R/100)]n =16000[1+(5/100)]3 =Rs.18522 C.I=18522-16000 =Rs.2522

Top
Complex Problems 1.The difference between C.I and S.I. on a certain sum at 10% per annum for 2 yrs is Rs.631.find the sum Sol: MethodI: NOTE: a) For 2 yrs -------->sum=(1002D/R2) b) For 3 yrs -------->sum=(1003D/R2(300+R)) Sum=1002*631/102

file:///E|/work/books/placement/09_Aptitude/compoundintrest.html[1/28/2012 12:45:08 AM]

=Rs.63100 MethodII: Let the sum be Rs.X,Then C.I.=X[1+(10/100)]2-X S.I=(X*10*2)/100 =X/5 C.I-S.I.=21X/100-X/5 =X/100 X/100=631 X=Rs.63100 2.If C.I on a certain sum for 2 yrs at 12% per annum is Rs.1590. What would be S.I? sol: C.I.=Amount-Principle Let P be X C.I=X[1+(12/100)]2-X =>784X/625-X=1590 =>X=Rs.6250 S.I=(6250*12*2)/100=Rs.1500 3.A sum of money amounts to Rs.6690 after 3 yrs and to Rs.10035b after 6 yrs on C.I .find the sum sol: For 3 yrs, Amount=P[1+(R/100)]3=6690-----------------------(1) For 6 yrs, Amount=P[1+(R/100)]6=10035----------------------(2) (1)/(2)------------[1+(R/100)]3=10035/6690 =3/2 [1+(R/100)]3=3/2-----------------(3) substitue (3) in (1) p*(3/2)=6690 =>p=Rs.4460 sum=Rs.4460 4.A sum of money doubles itself at C.I in 15yrs.In how many yrs will it become 8 times? sol: Compound Interest for 15yrs p[1+(R/100)]15 p[1+(R/100)]15=2P =>p[1+(R/100)]n=8P =>[1+(R/100)]n=8 =>[1+(R/100)]n=23 =>[1+(R/100)]n=[1+(R/100)]15*3 since [1+(R/100)] =2 n=45yrs

5.The amount of Rs.7500 at C.I at 4% per annum for 2yrs is sol: Iyear------------------7500+300(300------Interest on 7500) IIyear ----------------7500+300+12(12------------4% interest on 300) Amount=7500+300+300+12 =Rs.8112 6.The difference between C.I and S.I on a sum of money for 2 yrs at 121/2% per annum is Rs.150.the sum is sol: Sum=1002D/R2=( 1002*150) /(25/2)2=Rs.9600 7.If the S.I on sum of money at 15% per annum for 3yrs is Rs.1200, the C.I on the same sum for the same period at same rate is-----sol: S.I=1200 P*T*R/100=1200 P*3*5/100=1200 =>P=Rs.8000 C.I for Rs.8000 at 5% for 3 yrs is-------------8000+400 -----8000+400+20 -------------8000+400+20+20+1 C.I =400+400+20+400+20+20+1 =Rs.1261

Back Back To Main

Top

Contact: 040-23000700

file:///E|/work/books/placement/09_Aptitude/compoundintrest.html[1/28/2012 12:45:08 AM]

APTITUDE
Numbers H.C.F and L.C.M Decimal Fractions Simplification Square and Cube roots Average Problems on Numbers Problems on Ages Surds and Indices Percentage Profit and Loss Ratio And Proportions Partnership Chain Rule Time and Work Pipes and Cisterns Time and Distance Trains Boats and Streams Alligation or Mixture Simple Interest Compound Interest Logorithms Areas Volume and Surface area Races and Games of Skill Calendar Clocks Stocks ans Shares True Discount Bankers Discount Oddmanout and Series Data Interpretation probability Permutations and Combinations Puzzles BACK

CLOCKS
Simple Problems Shortcuts in clocks : Type1: Find the angle between the hour hand and the minute hand of a clock when the time is 3.25 solution : In this type of problems the formulae is as follows 30*[hrs-(min/5)]+(min/2) In the above problem the given data is time is 3.25. that is applied in the formulae 30*[3-(25/5)]+(25/2)30*(15-25)/5+25/2 = 30*(-10/5)+25/2 = -300/5+25/2 = -600+(25/2)=-475/10=-47.5 i.e 47 1/20 therefore the required angle is 47 1/20. Note :the -sign must be neglected. Another shortcut for type1 is : The formulae is 6*x-(hrs*60+X)/2 Here x is the given minutes, so in the given problem the minutes is 25 minutes, that is applied in the given formulae 6*25-(3*60+25)/2 150-205/2 (300-205)/2=95/2 =47 1/20. therefore the required angle is 47 1/20. Type2 : At what time between 2 and 3 o' clock will be the hands of a clock be together? Solution : In this type of problems the formulae is 5*x*(12/11) Here x is replaced by the first interval of given time. here i.e 2. In the above problem the given data is between 2 and 3 o' clock 5*2*12/11 =10*12/11=120/11=10 10/11min. Therefore the hands will coincide at 10 10/11 min.past2. Another shortcut for type2 is : Here the clocks be together but not opposite to each other so the angle is 0 degrees. so the formulae is 6*x-(2*60+x)/2=06*x-(120+x)/2=012x-120-x=0 11x=120 x=120/11=10 10/11 therefore the hands will be coincide at 10 10/11 min.past2.

file:///E|/work/books/placement/09_Aptitude/csimpleproblems.html[1/28/2012 12:45:09 AM]

APTITUDE
Numbers H.C.F and L.C.M Decimal Fractions Simplification Square and Cube roots Average Problems on Numbers Problems on Ages Surds and Indices Percentage Profit and Loss Ratio And Proportions Partnership Chain Rule Time and Work Pipes and Cisterns Time and Distance Trains Boats and Streams Alligation or Mixture Simple Interest Compound Interest Logorithms Areas Volume and Surface area Races and Games of Skill Calendar Clocks Stocks ans Shares True Discount Bankers Discount Oddmanout and Series Data Interpretation probability Permutations and Combinations Puzzles BACK

CLOCKS
Typicalproblems problems : A watch which gains uniformly ,is 5 min,slow at 8 o'clock in the morning on sunday and it is 5 min.48 sec.fast at 8 p.m on following sunday. when was it correct? Solution : Time from 8 am on sunday to 8 p.m on following sunday = 7 days 12 hours = 180 hours the watch gains (5+(5 4/5))min .or 54/5 min. in 180 hours Now 54/5 minare gained in 180 hours. Therefore 5 minutes are gained in(180*5/54*5)hours=83 hours20 min. =3 days11hrs20min. therefore watch is correct at 3 days 11 hours 20 minutes after 8 a.m of sunday therefore it wil be correct at 20 min.past 7 p.m on wednesday

file:///E|/work/books/placement/09_Aptitude/ctypicalproblems.html[1/28/2012 12:45:09 AM]

APTITUDE
Numbers H.C.F and L.C.M Decimal Fractions Simplification Square and Cube roots Average Problems on Numbers Problems on Ages Surds and Indices Percentage Profit and Loss Ratio And Proportions Partnership Chain Rule Time and Work Pipes and Cisterns Time and Distance Trains Boats and Streams Alligation or Mixture Simple Interest Compound Interest Logorithms Areas Volume and Surface area Races and Games of Skill Calendar Clocks Stocks ans Shares True Discount Bankers Discount Oddmanout and Series Data Interpretation probability Permutations and Combinations Puzzles

DECIMAL FRACTIONS
1.Decimal fractions: Fractionin which denominations are powers of 10 are decimal fractions. 1 /10 = 0.1, 1 / 100 = 0.01 2.Convertion of Decimal into fraction:eg: 0.25 = 25/100 = 1/4 3.i) If numerator and denominator contain same number of decimal places, then we remove decimal sign. Thus, 1.84/2.99 =184/299

PROBLEMS: 1.0.75 =75/100 =3/4

2.Find porducts= 6.3204*100 = 632.04

3.2.61*1.3=261*13=3393 some of decimal places 2 +1 =3 = 3.393

4.If 1/3.718 =0.2689,then find value of 1/0.0003718 ? Sol: 10000/3.718 =10000*1/3.718 =10000*0.2689 = 2689

5.Find fractions : i) 0.37 = 37/99 ii)3.142857 =3+0.142857 =3 +142857/999999 = 3 142857/ 999999 iii) 0.17=17-1/90 =16/90=8/45 iv)0.1254 =1254 -12/9900 =1242/9900=69/550

6.Fraction 101 27/100000 Sol: 101+27/100000 =101+0.00027 =101.00027 7.If 47.2506 =4A + 7/B +2C + 5/D + 6E then 40+7+0.2+0.05+0.0006 Sol: compairing terms 4A= 40 => A=10 7/B = 7 => B=1 2C= 0.2=> C=0.1 5/D= 0.05=>D=5/0.05 =>5*100/5 =100 6E= 0.0006=> E= 0.0001

file:///E|/work/books/placement/09_Aptitude/dec.html[1/28/2012 12:45:09 AM]

5A + 3B+6C+ D+ 3E = 5*10+ 3*1+ 6*0.1 + 100+ 3*0.0001 =50+3+0.6+100+0.0003 =153.6003 8.4.036 divided by 0.04 Sol: 4.036/0.04 =4036/4 =100.9 9.[ 0.05/0.25 + 0.25/ 0.05]3 Sol: =>[5/25 + 25/5] = [1/5+ 5]3 =26/53 =5.23 = 140.603 10.The least among the following :a. 0.2 b.1/0.2 c. 0.2 d. 0.22 10/2 =5 0.2222 0.04 0.04 < 0.2 < 0.22 --------<5 Since 0.04 is least (0.2)2 is least. 11.Let F= 0.84181 Sol: when F is written as a fraction in lowest terms, denominator exceeds numerator by 84181 -841 /99000 = 83340/99000 =463/550 Required distence = (550 463) = 87

12.2 .75 + 3.78 Sol: [-2+0.75]+[-3+0.78] =-5+[0.75+0.78] = -5+1.53 =-5+1+0.53 = -4+0.53 = 4.53

13.the sum of first 20 terms of series is 1/5*6 +1/6*7+1/7*8------------Sol: [1/5 -1/6]+[1/6-1/7]+[1/7-1/8]+-----------------------= [1/5-1/25] =4/25=0.16

14.13 +23+ ------------+93 =2025 Sol: value of (0.11) 3+ (0.22) 3+---------(0.99)3 => (0.11) [1+2+--------+9] =0.001331*2025 =2.695275

15.(0.96)3 (0.1)3/ (0.96)2 +0.096 +(0.1)2 Sol: formula => a3 -b3/a2 +ab +b2 =a -b (0.96-0.1)=0.86

16.3.6*0.48*2.50 / 0.12*0.09*0.5 Sol: 36*48*250/12*9*5=800

17.find x/y = 0.04/1.5 = 4/150 =2/75

file:///E|/work/books/placement/09_Aptitude/dec.html[1/28/2012 12:45:09 AM]

find y-x/y+x (1- x/y) / (1+ x/y) 1 - 2/75 /1 +2/75 =73/77 18.0.3467+0.1333 Sol: 3467 -34/9900 + 1333-13/9900 = 3433 +1320/9900 = 4753/9900 = 4801 -48/9900 =0.4301

file:///E|/work/books/placement/09_Aptitude/dec.html[1/28/2012 12:45:09 AM]

Decimal Fractions
1.Decimal fractions: Fractionin which denominations are powers of 10 are decimal fractions. Example:1 /10 = 0.1, 1 / 100 = 0.01 2.Convertion of Decimal into fraction:Example: 0.25 = 25/100 = 1/4 3.i) If numerator and denominator contain same number of decimal places, then we remove decimal sign. Thus, 1.84/2.99 =184/299 Problems 1.0.75 =75/100 =3/4 2.Find porducts= 6.3204*100 = 632.04 3.2.61*1.3=261*13=3393 some of decimal places 2 +1 =3 sol: 3.393 4.If 1/3.718 =0.2689,then find value of 1/0.0003718 ? Sol: 10000/3.718 =10000*1/3.718 =10000*0.2689 = 2689 5.Find fractions : i) 0.37 = 37/99 ii)3.142857 =3+0.142857 =3 +142857/999999 = 3 142857/ 999999 iii) 0.17=17-1/90 =16/90=8/45 iv)0.1254 =1254 -12/9900 =1242/9900=69/550 6.Fraction 101 27/100000 Sol: 101+27/100000 =101+0.00027 =101.00027 7.If 47.2506 =4A + 7/B +2C + 5/D + 6E then 40+7+0.2+0.05+0.0006 Sol: compairing terms 4A= 40 => A=10 7/B = 7 => B=1 2C= 0.2=> C=0.1 5/D= 0.05=>D=5/0.05 =>5*100/5 =100 6E= 0.0006=> E= 0.0001 5A + 3B+6C+ D+ 3E = 5*10+ 3*1+ 6*0.1 + 100+ 3*0.0001 =50+3+0.6+100+0.0003 =153.6003

Top
8.4.036 divided by 0.04 Sol: 4.036/0.04 =4036/4 =100.9 9.[ 0.05/0.25 + 0.25/ 0.05]3 Sol: =>[5/25 + 25/5] = [1/5+ 5]3 =26/53 =5.23 = 140.603 10.The least among the following :a. 0.2 b.1/0.2 c. 0.2 d. 0.22 sol:10/2 =5 0.2222 0.04 0.04 < 0.2 < 0.22 --------<5 Since 0.04 is least (0.2)2 is least. 11.Let F= 0.84181 Sol: when F is written as a fraction in lowest terms, denominator exceeds numerator by 84181 -841 /99000 = 83340/99000 =463/550 Required distence = (550 463) = 87 12.2 .75 + 3.78 Sol: [-2+0.75]+[-3+0.78] =-5+[0.75+0.78]

file:///E|/work/books/placement/09_Aptitude/decimalfractions.html[1/28/2012 12:45:09 AM]

= -5+1.53 =-5+1+0.53 = -4+0.53 = 4.53 13.the sum of first 20 terms of series is 1/5*6 +1/6*7+1/7*8----Sol: [1/5 -1/6]+[1/6-1/7]+[1/7-1/8]+-----------------------= [1/5-1/25] =4/25=0.16 14.13 +23+ ------------+93 =2025 Sol: value of (0.11) 3+ (0.22) 3+---------(0.99)3 => (0.11) [1+2+--------+9] =0.001331*2025 =2.695275 15.(0.96)3 (0.1)3/ (0.96)2 +0.096 +(0.1)2 Sol: formula => a3 -b3/a2 +ab +b2 =a -b (0.96-0.1)=0.86 16.3.6*0.48*2.50 / 0.12*0.09*0.5 Sol: 36*48*250/12*9*5=800 17.find x/y = 0.04/1.5 = 4/150 =2/75 find y-x/y+x (1- x/y) / (1+ x/y) 1 - 2/75 /1 +2/75 =73/77 18.0.3467+0.1333 Sol: 3467 = = = -34/9900 + 1333-13/9900 3433 +1320/9900 4753/9900 4801 -48/9900 =0.4301

Back Back To Main

Top

Contact: 040-23000700

file:///E|/work/books/placement/09_Aptitude/decimalfractions.html[1/28/2012 12:45:09 AM]

APTITUDE
Numbers H.C.F and L.C.M Decimal Fractions Simplification Square and Cube roots Average Problems on Numbers Problems on Ages Surds and Indices Percentage Profit and Loss Ratio And Proportions Partnership Chain Rule Time and Work Pipes and Cisterns Time and Distance Trains Boats and Streams Alligation or Mixture Simple Interest Compound Interest Logorithms Areas Volume and Surface area Races and Games of Skill Calendar Clocks Stocks ans Shares True Discount Bankers Discount Oddmanout and Series Data Interpretation probability Permutations and Combinations Puzzles BACK

H.C.F AND L.C.M

Facts And Formulae: 1.Highest Common Factor:(H.C.F) or Greatest Common Meaure(G.C.M) : The H.C.F of two or more than is the greatest two numbers

number that divides each of them exactly. There are two methods : 1.Factorization method: Express each one

of the given numbers as the product of prime factors. The product of least powers of common prime factors gives HCF.

Example :

Find HCF of 26 * 32*5*7 4 ,

22 *3 5*5 2 * 76 , 2,5,7

2*52 *7 2

Sol: The prime numbers given common numbers are Therefore HCF is 22 * 5 *7 2 .

2.Division Method : Divide

the larger number by

smaller one. Now divide the divisor by remainder. Repeat the process of dividing preceding number last obtained till zero is obtained as number. The last divisor is HCF.

Example: Find HCF of 513, 1134, 1215 Sol: 1134) 1215(1 1134 ---------81)1134(14 81 ----------324 324 ----------0 ----------HCF of this two numbers is 81.

file:///E|/work/books/placement/09_Aptitude/hcf.html[1/28/2012 12:45:10 AM]

81)513(6 486 -------27)81(3 81 ----0 ---

HCF

of

81 and 513 is 27.

3.Least common multiple[LCM] : The least number which is divisible by each one of given numbers is LCM. There are two methods for this:

1.Factorization method

Resolve each one into product of prime factors. Then LCM is product of highest powers

of all factors.

2.Common Problems: 1.The

division method.

HCF of 2

numbers

is 11 and LCM is

693.If one

of numbers is 77.find other. Sol: Other number = 11 * 693/77=99.

2.Find largest number of 4 digits Sol: The largest number is 9999. LCM of 12,15,18,27 on dividing 9999 by Therefore number =9999 279 =9720. is 540.

divisible by

12,15,18,27

540 we get 279 as remainder.

3.Find least number which when divided by 20,25,35,40 leaves remainders 14,19,29,34. Sol:

file:///E|/work/books/placement/09_Aptitude/hcf.html[1/28/2012 12:45:10 AM]

2014=6 25-19=6 35-29=6 40-34=6

Therefore number =LCM of (20,25,35,40) - 6=1394

4.252

can be expressed as prime as :

2 2 3 3 prime factor is 2 *2 * 3 * 3

252 126 63 21 7 *7

5.1095/1168

when expressed in simple form is

1095)1168(1 1095 -----73)1095(15 73 --------365 365 --------0 ---------So, HCF is 73

Therefore 1095/1168 = 1095/73/1168/73= 15/16

6.GCD of 1.08,0.36,0.9 is Sol: HCF of 108,36,90 36)90(2 72 ---18)36(2 36 ---0 ---HCF is 18. HCF of 18 and 108 is 18 18)108(6 108

file:///E|/work/books/placement/09_Aptitude/hcf.html[1/28/2012 12:45:10 AM]

------0 -------Therefore HCF

=0.18

7.Three numbers are in ratio 1:2:3 and HCF is 12.Find numbers. Sol: Let the numbers be x. Three numbers are x,2x,3x Therefore HCF is 2x)3x(1 2x ----x)2x(2 2x -------0 ------------HCF is x so, x is 12

Therefore numbers are 12,24,36.

8.The Sol:

sum of two numbers is 216 and HCF is 27.

Let numbers are 27a + 27 b =216

a + b =216/27=8

Co-primes of 8 are (1,7) numbers=(27 * 1 ), (27 =27,89 *

and (3,5) 7)

9.LCM of two numbers is The sum of numbers is

48..The numbers are in ratio 2:3.

Sol: Let the number be x.

file:///E|/work/books/placement/09_Aptitude/hcf.html[1/28/2012 12:45:10 AM]

Numbers are 2x,3x LCM of 2x,3x is 6x

Therefore 6x=48 x=8. Numbers are 16 and 24 Sum=16 +24=40.

10.HCF and LCM of two numbers are

84 and 21.If ratio of

two numbers is 1:4.Then largest of two numbers is

Sol: Let the numbers be x,4x Then x * x2 =84 4x = 84 * 21

* 21 /4 x = 21

Largest number is 4 * 21.

11.HCF of two numbers is 23,and other factors of LCM 13,14.Largest number Sol: 23 * 14 is Largest number. is

are

12.The maximum number of students among and 910 pencils can be distributed each student Sol: HCF of 1001 and 910 910)1001(1 910 -----------91)910(10 910 -------0 --------Therefore HCF=91

them 1001 pens

in such a way that

gets same number of pens and pencils is ?

file:///E|/work/books/placement/09_Aptitude/hcf.html[1/28/2012 12:45:10 AM]

13.The least number which should be added to 2497 so that sum is divisible by 5,6,4,3 ?

Sol:

LCM

of

5,6,4,3

is 60.

On dividing

2497

by 60

we get 37 as remainder.

Therefore number to added is 60 37

=23.

Answer is 23.

14.The least number which is a perfect square and is each of numbers 16,20,24 is ?

divisible by

Sol:

LCM of 16,20,24

is 240.

2 * 2*2*2*3*5=240

To make it a perfect square multiply by 3 * 5

Therefore

240 * 3 * 5=3600

Answer is 3600.

file:///E|/work/books/placement/09_Aptitude/hcf.html[1/28/2012 12:45:10 AM]

H.C.F and L.C.M


Facts And Formulae: Highest Common Factor:(H.C.F) or Greatest Common Meaure(G.C.M) : The H.C.F of two or more than two numbers is the greatest number that divides each of them exactly. There are two methods : i.Factorization method: Express each one of the given numbers as the product of prime factors. The product of least powers of common prime factors gives HCF. Example : Solution: Find HCF of 26 * 32*5*74 , 22 *35*52 * 76 , 2*52 *72 The prime numbers given common numbers are 2,5,7 Therefore HCF is 22 * 5 *72 .

ii.Division Method : Divide the larger number by smaller one. Now divide the divisor by remainder. Repeat the process of dividing preceding number last obtained till zero is obtained as number. The last divisor is HCF. Example: Solution: 1134) 1215(1 1134 ---------81)1134(14 81 ----------324 324 ----------0 ----------HCF of this two numbers is 81. 81)513(6 486 -------27)81(3 81 ----0 --HCF of 81 and 513 is 27. Find HCF of 513, 1134, 1215

Least common multiple[LCM] : The least number which is divisible by each one of given numbers is LCM. There are two methods for this: i.Factorization method : Resolve each one into product of prime factors. Then LCM is product of highest powers of all factors. ii.Common Problems: 1.The HCF of 2 numbers is 77.find other. Sol: Other number = 11 * is 11 and LCM is 693/77=99. divisible by 12,15,18,27 693.If one of numbers division method.

Top

2.Find largest number of 4 digits

Sol: The largest number is 9999. LCM of 12,15,18,27 is 540. on dividing 9999 by 540 we get 279 as remainder. Therefore number =9999 279 =9720. 3.Find least number which when divided by 20,25,35,40 leaves remainders 14,19,29,34. Sol: 2014=6 25-19=6 35-29=6 40-34=6 Therefore number =LCM of (20,25,35,40) - 6=1394 4.252 can be expressed as prime as : 2 252 2 126

file:///E|/work/books/placement/09_Aptitude/hcfandlcm.html[1/28/2012 12:45:10 AM]

63 21 7 prime factor is 2 *2 * 3 * 3 *7 5.1095/1168 when expressed in simple form is 1095)1168(1 1095 -----73)1095(15 73 --------365 365 --------0 ---------So, HCF is 73 Therefore 1095/1168 = 1095/73/1168/73= 15/16 6.GCD of 1.08,0.36,0.9 is Sol: HCF of 108,36,90 36)90(2 72 ---18)36(2 36 ---0 ---HCF is 18. HCF of 18 and 108 is 18 18)108(6 108 ------0 -------Therefore HCF =0.18 7.Three numbers are in ratio 1:2:3 and HCF is 12.Find numbers. Sol: Let the numbers be x. Three numbers are x,2x,3x Therefore HCF is 2x)3x(1 2x ----x)2x(2 2x -------0 ------------HCF is x so, x is 12 Therefore numbers are 12,24,36.

3 3

Top
8.The Sol: sum of two numbers is 216 and HCF is 27. Let numbers are 27a + 27 b =216 a + b =216/27=8 Co-primes of 8 are (1,7) and (3,5) numbers=(27 * 1 ), (27 * 7) =27,89 9.LCM of two numbers is of numbers is Sol: Let the number be x. Numbers are 2x,3x LCM of 2x,3x is 6x Therefore 6x=48 x=8. Numbers are 16 and 24 Sum=16 +24=40. 10.HCF and LCM of two numbers are 84 and 21.If ratio of two numbers is 1:4.Then largest of two numbers is Sol: Let the numbers be x,4x Then x * 4x = 84 * 21 x2 =84 * 21 /4 x = 21 Largest number is 4 * 21. 11.HCF of two numbers is 23,and other factors of LCM Largest number is Sol: 23 * 14 is Largest number. 12.The maximum number of students among them 1001 pens and 910 pencils can be distributed in such a way that each student gets same number of pens and pencils is ? Sol: HCF of 1001 and 910 910)1001(1 910 -----------91)910(10 910 are 13,14. 48..The numbers are in ratio 2:3. The sum

file:///E|/work/books/placement/09_Aptitude/hcfandlcm.html[1/28/2012 12:45:10 AM]

Therefore

-------0 --------HCF=91

13.The least number which should be added to 2497 so that sum is divisible by 5,6,4,3 ? Sol: LCM of 5,6,4,3 is 60. On dividing 2497 by 60 we get 37 as remainder. Therefore number to added is 60 37 =23. Answer is 23. 14.The least number which is a perfect square and is each of numbers 16,20,24 is ? Sol: LCM of 16,20,24 is 240. 2 * 2*2*2*3*5=240 To make it a perfect square multiply by 3 * 5 Therefore 240 * 3 * 5=3600 Answer is 3600. divisible by

Back Back To Main

Top

Contact: 040-23000700

file:///E|/work/books/placement/09_Aptitude/hcfandlcm.html[1/28/2012 12:45:10 AM]

APTITUDE
Numbers H.C.F and L.C.M Decimal Fractions Simplification Square and Cube roots Average Problems on Numbers Problems on Ages Surds and Indices Percentage Profit and Loss Ratio And Proportions Partnership Chain Rule Time and Work Pipes and Cisterns Time and Distance Trains Boats and Streams Alligation or Mixture Simple Interest Compound Interest Logorithms Areas Volume and Surface area Races and Games of Skill Calendar Clocks Stocks ans Shares True Discount Bankers Discount Oddmanout and Series Data Interpretation probability Permutations and Combinations Puzzles

H.C.F AND L.C.M

Facts And Formulae: 1.Highest Common Factor:(H.C.F) or Greatest Common Meaure(G.C.M) : The H.C.F of two or more than is the greatest two numbers

number that divides each of them exactly. There are two methods : 1.Factorization method: Express each one

of the given numbers as the product of prime factors. The product of least powers of common prime factors gives HCF.

Example :

Find HCF of 26 * 32*5*7 4 ,

22 *3 5*5 2 * 76 , 2,5,7

2*52 *7 2

Sol: The prime numbers given common numbers are Therefore HCF is 22 * 5 *7 2 .

2.Division Method : Divide

the larger number by

smaller one. Now divide the divisor by remainder. Repeat the process of dividing preceding number last obtained till zero is obtained as number. The last divisor is HCF.

Example: Find HCF of 513, 1134, 1215 Sol: 1134) 1215(1 1134 ---------81)1134(14 81 ----------324 324 ----------0 ----------HCF of this two numbers is 81. 81)513(6 486

file:///E|/work/books/placement/09_Aptitude/hcfconcept.html[1/28/2012 12:45:11 AM]

-------27)81(3 81 ----0 ---

HCF

of

81 and 513 is 27.

3.Least common multiple[LCM] : The least number which is divisible by each one of given numbers is LCM. There are two methods for this:

1.Factorization method

Resolve each one into product of prime factors. Then LCM is product of highest powers

of all factors.

2.Common Problems: 1.The

division method.

HCF of 2

numbers

is 11 and LCM is

693.If one

of numbers is 77.find other. Sol: Other number = 11 * 693/77=99.

2.Find largest number of 4 digits Sol: The largest number is 9999. LCM of 12,15,18,27 on dividing 9999 by Therefore number =9999 279 =9720. is 540.

divisible by

12,15,18,27

540 we get 279 as remainder.

3.Find least number which when divided by 20,25,35,40 leaves remainders 14,19,29,34. Sol:

file:///E|/work/books/placement/09_Aptitude/hcfconcept.html[1/28/2012 12:45:11 AM]

2014=6 25-19=6 35-29=6 40-34=6

Therefore number =LCM of (20,25,35,40) - 6=1394

4.252

can be expressed as prime as :

2 2 3 3 prime factor is 2 *2 * 3 * 3

252 126 63 21 7 *7

5.1095/1168

when expressed in simple form is

1095)1168(1 1095 -----73)1095(15 73 --------365 365 --------0 ---------So, HCF is 73

Therefore 1095/1168 = 1095/73/1168/73= 15/16

6.GCD of 1.08,0.36,0.9 is Sol: HCF of 108,36,90 36)90(2 72 ---18)36(2 36 ---0 ---HCF is 18. HCF of 18 and 108 is 18 18)108(6 108 ------0

file:///E|/work/books/placement/09_Aptitude/hcfconcept.html[1/28/2012 12:45:11 AM]

-------Therefore HCF

=0.18

7.Three numbers are in ratio 1:2:3 and HCF is 12.Find numbers. Sol: Let the numbers be x. Three numbers are x,2x,3x Therefore HCF is 2x)3x(1 2x ----x)2x(2 2x -------0 ------------HCF is x so, x is 12

Therefore numbers are 12,24,36.

8.The Sol:

sum of two numbers is 216 and HCF is 27.

Let numbers are 27a + 27 b =216

a + b =216/27=8

Co-primes of 8 are (1,7) numbers=(27 * 1 ), (27 =27,89 *

and (3,5) 7)

9.LCM of two numbers is The sum of numbers is

48..The numbers are in ratio 2:3.

Sol: Let the number be x. Numbers are 2x,3x

file:///E|/work/books/placement/09_Aptitude/hcfconcept.html[1/28/2012 12:45:11 AM]

LCM of

2x,3x is 6x

Therefore 6x=48 x=8. Numbers are 16 and 24 Sum=16 +24=40.

10.HCF and LCM of two numbers are

84 and 21.If ratio of

two numbers is 1:4.Then largest of two numbers is

Sol: Let the numbers be x,4x Then x * x2 =84 4x = 84 * 21

* 21 /4 x = 21

Largest number is 4 * 21.

11.HCF of two numbers is 23,and other factors of LCM 13,14.Largest number Sol: 23 * 14 is Largest number. is

are

12.The maximum number of students among and 910 pencils can be distributed each student Sol: HCF of 1001 and 910 910)1001(1 910 -----------91)910(10 910 -------0 --------Therefore HCF=91

them 1001 pens

in such a way that

gets same number of pens and pencils is ?

file:///E|/work/books/placement/09_Aptitude/hcfconcept.html[1/28/2012 12:45:11 AM]

13.The least number which should be added to 2497 so that sum is divisible by 5,6,4,3 ?

Sol:

LCM

of

5,6,4,3

is 60.

On dividing

2497

by 60

we get 37 as remainder.

Therefore number to added is 60 37

=23.

Answer is 23.

14.The least number which is a perfect square and is each of numbers 16,20,24 is ?

divisible by

Sol:

LCM of 16,20,24

is 240.

2 * 2*2*2*3*5=240

To make it a perfect square multiply by 3 * 5

Therefore

240 * 3 * 5=3600

Answer is 3600.

BACK

file:///E|/work/books/placement/09_Aptitude/hcfconcept.html[1/28/2012 12:45:11 AM]

APTITUDE
Numbers H.C.F and L.C.M Decimal Fractions Simplification Square and Cube roots Average Problems on Numbers Problems on Ages Surds and Indices Percentage Profit and Loss Ratio And Proportions Partnership Chain Rule Time and Work Pipes and Cisterns Time and Distance Trains Boats and Streams Alligation or Mixture Simple Interest Compound Interest Logorithms Areas Volume and Surface area Races and Games of Skill Calendar Clocks Stocks ans Shares True Discount Bankers Discount Oddmanout and Series Data Interpretation probability Permutations and Combinations Puzzles

HIGHTS AND DISTENCES

file:///E|/work/books/placement/09_Aptitude/heights.html[1/28/2012 12:45:11 AM]

Aptitude Numbers H.C.F and L.C.M Decimal Fractions Simplification Square and Cube Roots Averages Problems on Numbers Problems on Ages Surds and Indices Percentages Profit and Loss Ratios and Proportions Partnership Chain Rule Time and Work Pipes and Cisterns Time and Distance Trains Boats and Streams Alligation or Mixtures Simple Intrest Compound Intrest Areas Races ans Games of Skill Calender Clocks True Discount Bankers Discount

file:///E|/work/books/placement/09_Aptitude/manual.html[1/28/2012 12:45:12 AM]

Oddmanout and Series Probability Permutations and Combinations Puzzles Reasoning Back To Main

file:///E|/work/books/placement/09_Aptitude/manual.html[1/28/2012 12:45:12 AM]

APTITUDE
Numbers H.C.F and L.C.M Decimal Fractions Simplification Square and Cube roots Average Problems on Numbers Problems on Ages Surds and Indices Percentage Profit and Loss Ratio And Proportions Partnership Chain Rule Time and Work Pipes and Cisterns Time and Distance Trains Boats and Streams Alligation or Mixture Simple Interest Compound Interest Logorithms Areas Volume and Surface area Races and Games of Skill Calendar Clocks Stocks ans Shares True Discount Bankers Discount Oddmanout and Series Data Interpretation probability Permutations and Combinations Puzzles

CALENDAR
BACK

Medium Problems
Problem 1 : jan 1, 1995 was a sunday.what day of the week lies on jan 1,1996? Solution : 01 01 96 24 -1(since 1996 was leap year) --------total = 121 Then (121)/7 = (17) = 2 odd days Therefore our required day wasMonday. Problem 2 : On 8th feb,1995 it wednesday. The day of the week on 8th feb,1994 was? Solution : 04 08 94 23 --------total = 129 Then (129)/7 = (18) = 3 odd days Therefore the required day is Tuesday. Problem 3 : may 6,1993 was thursday.what day of the week was on may 6,1992 ? Solution : 02 06 92 23 -1 ---------total = 122 Then (122)/7 = (17) = 3 odd days Therefore the required day is Tuesday Problem 4 : jan 1, 1992 was wednesday. What day of the week was on jan 1,1993 ? Solution : 01 01 93 23 ---------total = 118 Then (118)/7 = (16) = 6 odd days Therefore the required day is Friday. Problem 5: January 1,2004 was a thursday,what day of the week lies on jan ,2005? solution : the year 2004 being a leap year, it has 2 odd days. so,first day of the 2005 will be 2 days beyond thursday and so it will be saturday therefore the required day is Thursday. Problem 6 : On 8th march,2005,wednesday falls what day of the week was it on 8th march,2004? Solution : the year 2004 being a leap year,it has 2 odd days. so, the day on8th march,2005 will be two

file:///E|/work/books/placement/09_Aptitude/mediumproblems.html[1/28/2012 12:45:12 AM]

days beyond the day on 8th march,2004.but 8th march,2005 is wednesday. so,8th march,2004 is monday. Therefore the required day is Monday. Problem 7: what was the day of the week on 19th september ,1986 ? Solution : 06 19 86 21 --------total = 132 Then ((132/7 = 18 and the remainder is 6) In the above week table represents the number 6 is friday. Therefore the required day is Friday.
BACK

file:///E|/work/books/placement/09_Aptitude/mediumproblems.html[1/28/2012 12:45:12 AM]

APTITUDE
Numbers H.C.F and L.C.M Decimal Fractions Simplification Square and Cube roots Average Problems on Numbers Problems on Ages Surds and Indices Percentage Profit and Loss Ratio And Proportions Partnership Chain Rule Time and Work Pipes and Cisterns Time and Distance Trains Boats and Streams Alligation or Mixture Simple Interest Compound Interest Logorithms Areas Volume and Surface area Races and Games of Skill Calendar Clocks Stocks ans Shares True Discount Bankers Discount Oddmanout and Series Data Interpretation probability Permutations and Combinations Puzzles BACK

Prime Numbers :A natural number larger than unity is a prime number if it does not have other divisors except for itself and unity. Note:-Unity i e,1 is not a prime number. Properties Of Prime Numbers:->The lowest prime number is 2. ->2 is also the only even prime number. ->The lowest odd prime number is 3. ->The remainder when a prime number p>=5 s divided by 6 is 1 or 5.However, if a number on being divided by 6 gives a remainder 1 or 5 need not be prime. ->The remainder of division of the square of a prime number p>=5 divide by 24 is 1. ->For prime numbers p>3, p-1 is divided by 24. ->If a and b are any 2 odd primes then a-b is composite. Also a+b is composite. ->The remainder of the division of the square of a prime number p>=5 divided by 12 is 1. BACK

file:///E|/work/books/placement/09_Aptitude/nc1.html[1/28/2012 12:45:12 AM]

APTITUDE
Numbers H.C.F and L.C.M Decimal Fractions Simplification Square and Cube roots Average Problems on Numbers Problems on Ages Surds and Indices Percentage Profit and Loss Ratio And Proportions Partnership Chain Rule Time and Work Pipes and Cisterns Time and Distance Trains Boats and Streams Alligation or Mixture Simple Interest Compound Interest Logorithms Areas Volume and Surface area Races and Games of Skill Calendar Clocks Stocks ans Shares True Discount Bankers Discount Oddmanout and Series Data Interpretation probability Permutations and Combinations Puzzles BACK

Calendar

Main concepts : a) Odd Days : The number of days more than the complete number of weeks in a given period is number of odd days during that period. b) Leap year : Every year which is divisible by 4 is called a leap year. Thus each one of he year 1992,1996,2004,2008,2012,,etc, is a eap year. Every 4th century is a leap year but no other century is a leap year thus each one of 400,800,1200,1600,2000,etc is a leap year. None of the 1900,2010,2020,2100,etc is a leap year. An year which is not a leap year is called Ordinary year. c)An ordinary year has 365 days. d) A leap year has 366 days. e)Counting of odd days : i)1 ordinary year = 365 days =52 weeks+1 day,Therefore An ordinary year has 1 Odd day. ii)One leap year = 366 days =52 weeks+2 days, Therefore a leap year has 2 Odd days. iii) 100 years = 76 ordinary years+ 24 leap years = [(76*52) weeks+76 days]+[(24*52)weeks+48 days] = 5200 weeks+124days=[5217 weeks+5 days] therefore 100 years contain 5 odd days iv)200 years contain 10(1week+3days), i.e 3 odd days v)300 years contain 15(2 weeks+1 day), i.e 1 odd day vi)400 years contain (20+1), i.e 3 weeks,so 0 Odd days similarly each one of 800,1200,1600,2000,etc contains o odd days. Note: (7n+m) odd days , where m less than or equal to 7 is equivalent to m odd days ,thus ,8 odd days = 1 odd day etc. f) Some codes o remember the months and weeks: a) Week Sunday - 1 Monday - 2 Tuesday - 3 Wednesday - 4 Thursday - 5 Friday - 6 Saturday - 0 b) Month jan - 1 feb - 4 july - 0 Aug - 3

file:///E|/work/books/placement/09_Aptitude/notes.html[1/28/2012 12:45:13 AM]

Mar - 4 Apr - 0 May - 2 june - 5


BACK

Sep - 6 Oct - 1 Nov - 4 Dec - 6

file:///E|/work/books/placement/09_Aptitude/notes.html[1/28/2012 12:45:13 AM]

Numbers
Introduction: Natural Numbers: All positive integers are natural numbers. Ex 1,2,3,4,8,...... There are infinite natural numbers and number 1 is the least natural number. Based on divisibility there would be two types of natural numbers. They are Prime and composite. Prime Numbers: A natural number larger than unity is a prime number if it does not have other divisors except for itself and unity. Note:-Unity i e,1 is not a prime number. Properties Of Prime Numbers: ->The lowest prime number is 2. ->2 is also the only even prime number. ->The lowest odd prime number is 3. ->The remainder when a prime number p>=5 s divided by 6 is 1 or 5.However, if a number on being divided by 6 gives a remainder 1 or 5 need not be prime. ->The remainder of division of the square of a prime number p>=5 divide by 24 is 1. ->For prime numbers p>3, p - 1 is divided by 24. ->If a and b are any 2 odd primes then a - b is composite. Also a+b is composite. ->The remainder of the division of the square of a prime number p>=5 divided by 12 is 1. Process to Check A Number s Prime or not: Take the square root of the number. Round of the square root to the next highest integer call this number as Z. Check for divisibility of the number N by all prime numbers below Z. If there is no numbers below the value of Z which divides N then the number will be prime. Example 239 is prime or 239 lies between 15 or Prime numbers less than 239 is not divisible by is a prime number. Composite Numbers: The numbers which are not prime are known as composite numbers. Co-Primes: Two numbers a an b are said to be co - primes,if their H.C.F is 1. Example (2,3),(4,5),(7,9),(8,11)..... Place value or Local value of a digit in a Number: place value: Example 689745132 Place value of 2 is (2*1)=2 Place value of 3 is (3*10)=30 and so on. Face value: - It is the value of the digit itself at whatever place it may be. Example 689745132 Face value of 2 is 2. Face value of 3 is 3 and so on. not ? 16.Hence take the value of Z=16. 16 are 2,3,5,7,11 and 13. any of these. Hence we can conclude that 239

Top
Tests of Divisibility: Divisibility by 2: - A number is divisible by 2,if its unit's digit is any of 0,2,4,6,8. Example 84932 is divisible by 2,while 65935 is not. Divisibility by 3: - A number is divisible by 3,if the sum of its digits is divisible by 3. Example 1.592482 is divisible by 3,since sum of its digits 5+9+2+4+8+2=30 which is divisible by 3. Example 2. 864329 is not divisible by 3,since sum of its digits 8+6+4+3+2+9=32 which is not divisible by 3. Divisibility by 4: - A number is divisible by 4,if the number formed by last two digits is divisible by 4. Example 1. 892648 is divisible by 4,since the number formed by the last two digits is 48 divisible by 4.

file:///E|/work/books/placement/09_Aptitude/numbers.html[1/28/2012 12:45:13 AM]

Example 2.But 749282 is not divisible by 4,since the number formed by the last two digits is 82 is not divisible by 4. Divisibility by 5: - A number divisible by 5,if its unit's digit is either 0 or 5. Example 20820,50345 Divisibility by 6: - If the number is divisible by both 2 and 3. example 35256 is clearly divisible by 2 sum of digits =3+5+2+5+21,which is divisible by 3 Thus the given number is divisible by 6. Divisibility by 8: - A number is divisible by 8 if the last 3 digits of the number are divisible by 8. Divisibility by 11: - If the difference of the sum of the digits in the odd places and the sum of the digitsin the even places is zero or divisible by 11. Example 4832718 (8+7+3+4) - (1+2+8)=11 which is divisible by 11. Divisibility by 12: - All numbers divisible by 3 and 4 are divisible by 12. Divisibility by 7,11,13: - The difference of the number of its thousands and the remainder of its division by 1000 is divisible by 7,11,13. BASIC FORMULAE: ->(a+b)=a+b+2ab ->(a-b)=a+b - 2ab ->(a+b)- (a - b)=4ab ->(a+b)+(a - b)=2(a+b) ->a-b=(a+b)(a - b) ->(a-+b+c)=a+b+c+2(ab+b c+ca) ->a+b=(a+b)(a+b - ab) ->a-b=(a - b)(a+b+ab) ->a+b+c - 3a b c=(a+b+c)(a+b+c - ab - b c - ca) ->If a+b+c=0 then a+b+c=3a b c DIVISION ALGORITHM If we divide a number by another number ,then Dividend = (Divisor * quotient) + Remainder

Top
MULTIPLICATION BY SHORT CUT METHODS 1.Multiplication by distributive law: a)a*(b+c)=a*b+a*c b)a*(b-c)=a*b - a*c Example a)567958*99999=567958*(100000 - 1) 567958*100000 - 567958*1 56795800000 - 567958 56795232042 b)978*184+978*816=978*(184+816) 978*1000=978000 2.Multiplication of a number by 5n: - Put n zeros to the right of the multiplicand and divide the number so formed by 2n Example 975436*625=975436*54=9754360000/16=609647500. PROGRESSION: A succession of numbers formed and arranged in a definite order according to certain definite rule is called a progression. 1.Arithmetic Progression: - If each term of a progression differs from its preceding term by a constant. This constant difference is called the common difference of the A.P. The n th term of this A.P is Tn=a(n - 1)+d. The sum of n terms of A.P Sn=n/2[2a+(n - 1)d]. xImportant Results: a.1+2+3+4+5......................=n(n+1)/2. b.12+22+32+42+52......................=n(n+1)(2n+1)/6. c.13+23+33+43+53......................=n2(n+1)2/4 2.Geometric Progression: - A progression of numbers in which every term bears a constant ratio with ts preceding term. i.e a,a r,a r2,a r3............... In G.P Tn=a r n - 1 Sum of n terms Sn=a(1 - r n)/1 - r Problems 1.Simplify a.8888+888+88+8 b.11992-7823 - 456 Solution: a.8888 888 88 8 9872 b.11992-7823 - 456=11992 - (7823+456) =11992-8279=3713 2.What could be the maximum value of Q in the following equation ? 5PQ+3R7+2Q8=1114 Solution: 3 R 2 Q 5 7 8 P Q

file:///E|/work/books/placement/09_Aptitude/numbers.html[1/28/2012 12:45:13 AM]

11 1 4 2+P+Q+R=11 Maximum value of Q =11 - 2=9 (P=0,R=0) 3.Simplify: a.5793405*9999 b.839478*625 Solution: a. 5793405*9999=5793405*(10000 - 1) 57934050000 - 5793405=57928256595 b. 839478*625=839478*54=8394780000/16=524673750. 4.Evaluate 313*313+287*287 Solution: a+b=1/2((a+b)+(a - b)) 1/2(313+287) +(313 - 287)=1/2(600 +26 ) (360000+676)=180338

Top
5.Which of the following is a prime number ? a.241 b.337 c.391 Solution: a.241 16>241.Hence take the value of Z=16. Prime numbers less than 16 are 2,3,5,7,11 and 13. 241 is not divisible by any of these. Hence we can conclude that 241 is a prime number. b. 337 19>337.Hence take the value of Z=19. Prime numbers less than 16 are 2,3,5,7,11,13 and 17. 337 is not divisible by any of these. Hence we can conclude that 337 is a prime number. c. 391 20>391.Hence take the value of Z=20. Prime numbers less than 16 are 2,3,5,7,11,13,17 and 19. 391 is divisible by 17. Hence we can conclude that 391 is not a prime number. 6.Find the unit's digit n the product 2467 153 * 34172 ? Solution: Unit's digit in the given product=Unit's digit in 7 153 * 172 Now 7 4 gives unit digit 1 7 152 gives unit digit 1 7 153 gives 1*7=7.Also 172 gives 1 Hence unit's digit in the product =7*1=7. 7.Find the total number of prime factors in 411 *7 5 *112 ? Solution: 411 7 5 112= (2*2) 11 *7 5 *112 = 222 *7 5 *112 Total number of prime factors=22+5+2=29 8.Which of the following numbers s divisible by 3 ? a.541326 b.5967013 Solution: a. Sum of digits in 541326=5+4+1+3+2+6=21 divisible by 3. b. Sum of digits in 5967013=5+9+6+7+0+1+3=31 not divisible by 3. 9.What least value must be assigned to * so that th number 197*5462 is divisible by 9 ? Solution: Let the missing digit be x Sum of digits = (1+9+7+x+5+4+6+2)=34+x For 34+x to be divisible by 9 , x must be replaced by 2 The digit in place of x must be 2. 10.What least number must be added to 3000 to obtain a number exactly divisible by 19 ? Solution:On dividing 3000 by 19 we get 17 as remainder Therefore number to be added = 19 - 17=2. 11.Find the smallest number of 6 digits which is exactly divisible by 111 ? Solution:Smallest number of 6 digits is 100000 On dividing 10000 by 111 we get 100 as remainder Number to be added =111 - 100=11. Hence,required number =10011. 12.On dividing 15968 by a certain number the quotient is 89 and the remainder is 37.Find the divisor ? Solution:Divisor = (Dividend - Remainder)/Quotient =(15968-37) / 89 =179. 13.A number when divided by 342 gives a remainder 47.When the same number is divided by 19 what would be the remainder ? Solution:Number=342 K + 47 = 19 * 18 K + 19 * 2 + 9=19 ( 18K + 2) + 9. The given number when divided by 19 gives 18 K + 2 as quotient and 9 as remainder.

Top
14.A number being successively divided by 3,5,8 leaves remainders 1,4,7 respectively. Find the respective remainders if the order of divisors are reversed ? Solution:Let the number be x. 3 x 5 y - 1 8 z y=5z+4 = 5*15+4 = 79 x=3y+1 = 3*79+1=238 Now 8 238 5 29 - 6 3 5 - 4 1 - 2 Respective remainders are 6,4,2. - 4 1 - 7 z=8*1+7=15

file:///E|/work/books/placement/09_Aptitude/numbers.html[1/28/2012 12:45:13 AM]

15.Find the remainder when 231 is divided by 5 ? Solution:210 =1024.unit digit of 210 * 210 * 210 is 4 as 4*4*4 gives unit digit 4 unit digit of 231 is 8. Now 8 when divided by 5 gives 3 as remainder. 231 when divided by 5 gives 3 as remainder. 16.How many numbers between 11 and 90 are divisible by 7 ? Solution:The required numbers are 14,21,28,...........,84 This is an A.P with a=14,d=7. Let it contain n terms then T =84=a+(n - 1)d =14+(n-1)7 =7+7n 7n=77 =>n=11. 17.Find the sum of all odd numbers up to 100 ? Solution:The given numbers are 1,3,5.........99. This is an A.P with a=1,d=2. Let it contain n terms 1+(n - 1)2=99 =>n=50 Then required sum =n/2(first term +last term) =50/2(1+99)=2500. 18.How many terms are there in 2,4,6,8..........,1024 ? Solution:Clearly 2,4,6........1024 form a G.P with a=2,r=2 Let the number of terms be n then 2*2 n - 1=1024 2n-1 =512=29 n - 1=9 n=10. 19.2+22+23+24+25..........+28= ? Solution:Given series is a G.P with a=2,r=2 and n=8. Sum Sn=a(1 - r n)/1 - r=Sn=2(1 - 28)/1 - 2. =2*255=510. 20.A positive number which when added to 1000 gives a sum , which is greater than when it is multiplied by 1000.The positive integer is ? a.1 b.3 c.5 d.7 Solution:1000+N>1000N clearly N=1. 21.The sum of all possible two digit numbers formed from three different one digit natural numbers when divided by the sum of the original three numbers is equal to ? a.18 b.22 c.36 d. none Solution:Let the one digit numbers x,y,z Sum of all possible two digit numbers= =(10x+y)+(10x+z)+(10y+x)+(10y+z)+(10z+x)+(10z+y) = 22(x+y+z) Therefore sum of all possible two digit numbers when divided by sum of one digit numbers gives 22. 22.The sum of three prime numbers is 100.If one of them exceeds another by 36 then one of the numbers is ? a.7 b.29 c.41 d67. Solution:x+(x+36)+y=100 2x+y=64 Therefore y must be even prime which is 2 2x+2=64=>x=31. Third prime number =x+36=31+36=67. 23.A number when divided by the sum of 555 and 445 gives two times their difference as quotient and 30 as remainder .The number is ? a.1220 b.1250 c.22030 d.220030. Solution:Number=(555+445)*(555 - 445)*2+30 =(555+445)*2*110+30 =220000+30=220030. 24.The difference between two numbers s 1365.When the larger number is divided by the smaller one the quotient is 6 and the remainder is 15. The smaller number is ? a.240 b.270 c.295 d.360 Solution:Let the smaller number be x, then larger number =1365+x Therefore 1365+x=6x+15 5x=1350 => x=270 Required number is 270. 25.In doing a division of a question with zero remainder,a candidate took 12 as divisor instead of 21.The quotient obtained by him was 35. The correct quotient is ? a.0 b.12 c.13 d.20 Solution:Dividend=12*35=420. Now dividend =420 and divisor =21. Therefore correct quotient =420/21=20.

Back Back To Main

Top

Contact: 040-23000700
file:///E|/work/books/placement/09_Aptitude/numbers.html[1/28/2012 12:45:13 AM]

file:///E|/work/books/placement/09_Aptitude/numbers.html[1/28/2012 12:45:13 AM]

APTITUDE
Numbers H.C.F and L.C.M Decimal Fractions Simplification Square and Cube roots Average Problems on Numbers Problems on Ages Surds and Indices Percentage Profit and Loss Ratio And Proportions Partnership Chain Rule Time and Work Pipes and Cisterns Time and Distance Trains Boats and Streams Alligation or Mixture Simple Interest Compound Interest Logorithms Areas Volume and Surface area Races and Games of Skill Calendar Clocks Stocks ans Shares True Discount Bankers Discount Oddmanout and Series Data Interpretation probability Permutations and Combinations Puzzles BACK

NUMBER SYSTEMS
Natural Numbers:All positive integers are natural numbers. Ex 1,2,3,4,8,...... There are infinite natural numbers and number 1 is the least natural number. Based on divisibility there would be two types of natural numbers. They are Prime and composite.

Prime Numbers :A natural number larger than unity is a prime number if it does not have other divisors except for itself and unity. Note:-Unity i e,1 is not a prime number.

Properties Of Prime Numbers:->The lowest prime number is 2. ->2 is also the only even prime number. ->The lowest odd prime number is 3. ->The remainder when a prime number p>=5 s divided by 6 is 1 or 5.However, if a number on being divided by 6 gives a remainder 1 or 5 need not be prime.
->The remainder of division of the square of a prime number p>=5 divide by 24 is 1. ->For prime numbers p>3, p-1 is divided by 24. ->If a and b are any 2 odd primes then a-b is composite. Also a+b is composite. ->The remainder of the division of the square of a prime number p>=5 divided by 12 is 1.

Process to Check A Number s Prime or not:Take the square root of the number. Round of the square root to the next highest integer call this number as Z. Check for divisibility of the number N by all prime numbers below Z. If there is no numbers below the value of Z which divides N then the number will be prime. Example 239 is prime or not? 239 lies between 15 or 16.Hence take the value of Z=16. Prime numbers less than 16 are 2,3,5,7,11 and 13. 239 is not divisible by any of these. Hence we can conclude that 239 is a prime number.

Composite Numbers:The numbers which are not prime are known as composite numbers.

Co-Primes:-

file:///E|/work/books/placement/09_Aptitude/numbersconcept.html[1/28/2012 12:45:14 AM]

Two numbers a an b are said to be co-primes,if their H.C.F is 1. Example (2,3),(4,5),(7,9),(8,11).....

Place value or Local value of a digit in a Number:


place value: Example 689745132 Place value of 2 is (2*1)=2 Place value of 3 is (3*10)=30 and so on. Face value:-It is the value of the digit itself at whatever place it may be. Example 689745132 Face value of 2 is 2. Face value of 3 is 3 and so on.

Tests of Divisibility:-

Divisibility by 2:-A number is divisible by 2,if its unit's digit is any of 0,2,4,6,8. Example 84932 is divisible by 2,while 65935 is not. Divisibility by 3:-A number is divisible by 3,if the sum of its digits is divisible by 3. Example 1.592482 is divisible by 3,since sum of its digits 5+9+2+4+8+2=30 which is divisible by 3. 2.864329 is not divisible by 3,since sum of its digits 8+6+4+3+2+9=32 which is not divisible by 3. Divisibility by 4:-A number is divisible by 4,if the number formed by last two digits is divisible by 4. Example 1.892648 is divisible by 4,since the number formed by the last two digits is 48 divisible by 4. 2.But 749282 is not divisible by 4,since the number formed by the last two digits is 82 is not divisible by 4. Divisibility by 5:-A number divisible by 5,if its unit's digit is either 0 or 5. Example 20820,50345 Divisibility by 6:-If the number is divisible by both 2 and 3. example 35256 is clearly divisible by 2 sum of digits =3+5+2+5+21,which is divisible by 3 Thus the given number is divisible by 6. Divisibility by 8:-A number is divisible by 8 if the last 3 digits of the number are divisible by 8. Divisibility by 11:-If the difference of the sum of the digits in the odd places and the sum of the digitsin the even places is zero or divisible by 11. Example 4832718 (8+7+3+4) - (1+2+8)=11 which is divisible by 11. Divisibility by 12:-All numbers divisible by 3 and 4 are divisible by 12. Divisibility by 7,11,13:-The difference of the number of its thousands and the remainder of its division by 1000 is divisible by 7,11,13.

BASIC FORMULAE: ->(a+b)=a+b+2ab ->(a-b)=a+b-2ab ->(a+b)-(a-b)=4ab ->(a+b)+(a-b)=2(a+b) ->a-b=(a+b)(a-b) ->(a-+b+c)=a+b+c+2(ab+b c+ca) ->a+b=(a+b)(a+b-ab) ->a-b=(a-b)(a+b+ab) ->a+b+c-3a b c=(a+b+c)(a+b+c-ab-b c-ca) ->If a+b+c=0 then a+b+c=3a b c
DIVISION ALGORITHM If we divide a number by another number ,then

file:///E|/work/books/placement/09_Aptitude/numbersconcept.html[1/28/2012 12:45:14 AM]

Dividend = (Divisor * quotient) + Remainder

MULTIPLICATION BY SHORT CUT METHODS

1.Multiplication by distributive law: a)a*(b+c)=a*b+a*c b)a*(b-c)=a*b-a*c Example a)567958*99999=567958*(100000-1) 567958*100000-567958*1 56795800000-567958 56795232042 b)978*184+978*816=978*(184+816) 978*1000=978000 2.Multiplication of a number by 5n:-Put n zeros to the right of the multiplicand and divide the number so formed by 2n Example 975436*625=975436*54=9754360000/16=609647500.

PROGRESSION:A succession of numbers formed and arranged in a definite order according to certain definite rule is called a progression. 1.Arithmetic Progression:-If each term of a progression differs from its preceding term by a constant. This constant difference is called the common difference of the A.P. The n th term of this A.P is Tn=a(n-1)+d. The sum of n terms of A.P Sn=n/2[2a+(n-1)d].

xImportant Results:
a.1+2+3+4+5......................=n(n+1)/2. b.12+22+32+42+52......................=n(n+1)(2n+1)/6. c.13+23+33+43+53......................=n2(n+1)2/4 2.Geometric Progression:-A progression of numbers in which every term bears a constant ratio with ts preceding term. i.e a,a r,a r2,a r3............... In G.P Tn=a r n-1 Sum of n terms Sn=a(1-r n)/1-r BACK

file:///E|/work/books/placement/09_Aptitude/numbersconcept.html[1/28/2012 12:45:14 AM]

APTITUDE
Numbers H.C.F and L.C.M Decimal Fractions Simplification Square and Cube roots Average Problems on Numbers Problems on Ages Surds and Indices Percentage Profit and Loss Ratio And Proportions Partnership Chain Rule Time and Work Pipes and Cisterns Time and Distance Trains Boats and Streams Alligation or Mixture Simple Interest Compound Interest Logorithms Areas Volume and Surface area Races and Games of Skill Calendar Clocks Stocks ans Shares True Discount Bankers Discount Oddmanout and Series Data Interpretation probability Permutations and Combinations Puzzles BACK

NUMBER SYSTEMS

SOLVED PROBLEMS

1.Simplify a.8888+888+88+8 b.11992-7823-456 Solution: a.8888 888 88 8 9872 b.11992-7823-456=11992-(7823+456) =11992-8279=3713 2.What could be the maximum value of Q in the following equation? 5PQ+3R7+2Q8=1114 Solution: 5 P Q 3 R 7 2 Q 8 11 1 4 2+P+Q+R=11 Maximum value of Q =11-2=9 (P=0,R=0) 3.Simplify: a.5793405*9999 b.839478*625 Solution: a. 5793405*9999=5793405*(10000-1) 57934050000-5793405=57928256595 b. 839478*625=839478*54=8394780000/16=524673750. 4.Evaluate 313*313+287*287 Solution: a+b=1/2((a+b)+(a-b)) 1/2(313+287) +(313-287)=1/2(600 +26 ) (360000+676)=180338 5.Which of the following is a prime number? a.241 b.337 c.391 Solution: a.241 16>241.Hence take the value of Z=16. Prime numbers less than 16 are 2,3,5,7,11 and 13. 241 is not divisible by any of these. Hence we can conclude that 241 is a prime number. b. 337 19>337.Hence take the value of Z=19. Prime numbers less than 16 are 2,3,5,7,11,13 and 17. 337 is not divisible by any of these. Hence we can conclude that 337 is a prime number. c. 391 20>391.Hence take the value of Z=20. Prime numbers less than 16 are 2,3,5,7,11,13,17 and 19. 391 is divisible by 17. Hence we can conclude that 391 is not a prime number. 6.Find the unit's digit n the product 2467 153 * 34172?

file:///E|/work/books/placement/09_Aptitude/numbersproblems.html[1/28/2012 12:45:14 AM]

Solution: Unit's digit in the given product=Unit's digit in 7 153 * 172 Now 7 4 gives unit digit 1 7 152 gives unit digit 1 7 153 gives 1*7=7.Also 172 gives 1 Hence unit's digit in the product =7*1=7. 7.Find the total number of prime factors in 411 *7 5 *112 ? Solution: 411 7 5 112= (2*2) 11 *7 5 *112 = 222 *7 5 *112 Total number of prime factors=22+5+2=29

8.Which of the following numbers s divisible by 3? a.541326 b.5967013 Solution: a. Sum of digits in 541326=5+4+1+3+2+6=21 divisible by 3. b. Sum of digits in 5967013=5+9+6+7+0+1+3=31 not divisible by 3.

9.What least value must be assigned to * so that th number 197*5462 is divisible by 9? Solution: Let the missing digit be x Sum of digits = (1+9+7+x+5+4+6+2)=34+x For 34+x to be divisible by 9 , x must be replaced by 2 The digit in place of x must be 2.

10.What least number must be added to 3000 to obtain a number exactly divisible by 19? Solution:On dividing 3000 by 19 we get 17 as remainder Therefore number to be added = 19-17=2. 11.Find the smallest number of 6 digits which is exactly divisible by 111? Solution:Smallest number of 6 digits is 100000 On dividing 10000 by 111 we get 100 as remainder Number to be added =111-100=11. Hence,required number =10011. 12.On dividing 15968 by a certain number the quotient is 89 and the remainder is 37.Find the divisor? Solution:Divisor = (Dividend-Remainder)/Quotient =(15968-37) / 89 =179.

13.A number when divided by 342 gives a remainder 47.When the same number is divided by 19 what would be the remainder? Solution:Number=342 K + 47 = 19 * 18 K + 19 * 2 + 9=19 ( 18K + 2) + 9. The given number when divided by 19 gives 18 K + 2 as quotient and 9 as remainder.

file:///E|/work/books/placement/09_Aptitude/numbersproblems.html[1/28/2012 12:45:14 AM]

14.A number being successively divided by 3,5,8 leaves remainders 1,4,7 respectively. Find the respective remainders if the order of divisors are reversed? Solution:Let the number be x. 3 x 5 y -1 8 z -4 1 - 7 z=8*1+7=15

y=5z+4 = 5*15+4 = 79 x=3y+1 = 3*79+1=238 Now 8 5 3 1 238 29 - 6 5 -4 -2

Respective remainders are 6,4,2. 15.Find the remainder when 231 is divided by 5? Solution:210 =1024.unit digit of 210 * 210 * 210 is 4 as 4*4*4 gives unit digit 4 unit digit of 231 is 8. Now 8 when divided by 5 gives 3 as remainder. 231 when divided by 5 gives 3 as remainder.

16.How many numbers between 11 and 90 are divisible by 7? Solution:The required numbers are 14,21,28,...........,84 This is an A.P with a=14,d=7. Let it contain n terms then T =84=a+(n-1)d =14+(n-1)7 =7+7n 7n=77 =>n=11.

17.Find the sum of all odd numbers up to 100? Solution:The given numbers are 1,3,5.........99. This is an A.P with a=1,d=2. Let it contain n terms 1+(n-1)2=99 =>n=50 Then required sum =n/2(first term +last term) =50/2(1+99)=2500.

18.How many terms are there in 2,4,6,8..........,1024?

file:///E|/work/books/placement/09_Aptitude/numbersproblems.html[1/28/2012 12:45:14 AM]

Solution:Clearly 2,4,6........1024 form a G.P with a=2,r=2 Let the number of terms be n then 2*2 n-1=1024 2n-1 =512=29 n-1=9 n=10.

19.2+22+23+24+25..........+28=? Solution:Given series is a G.P with a=2,r=2 and n=8. Sum Sn=a(1-r n)/1-r=Sn=2(1-28)/1-2. =2*255=510.

20.A positive number which when added to 1000 gives a sum , which is greater than when it is multiplied by 1000.The positive integer is? a.1 b.3 c.5 d.7 Solution:1000+N>1000N clearly N=1. 21.The sum of all possible two digit numbers formed from three different one digit natural numbers when divided by the sum of the original three numbers is equal to? a.18 b.22 c.36 d. none Solution:Let the one digit numbers x,y,z Sum of all possible two digit numbers= =(10x+y)+(10x+z)+(10y+x)+(10y+z)+(10z+x)+(10z+y) = 22(x+y+z) Therefore sum of all possible two digit numbers when divided by sum of one digit numbers gives 22. 22.The sum of three prime numbers is 100.If one of them exceeds another by 36 then one of the numbers is? a.7 b.29 c.41 d67. Solution:x+(x+36)+y=100 2x+y=64 Therefore y must be even prime which is 2 2x+2=64=>x=31. Third prime number =x+36=31+36=67. 23.A number when divided by the sum of 555 and 445 gives two times their difference as quotient and 30 as remainder .The number is? a.1220 b.1250 c.22030 d.220030. Solution:Number=(555+445)*(555-445)*2+30 =(555+445)*2*110+30 =220000+30=220030. 24.The difference between two numbers s 1365.When the larger number is divided by the smaller one the quotient is 6 and the remainder is 15. The smaller number is? a.240 b.270 c.295 d.360 Solution:Let the smaller number be x, then larger number =1365+x Therefore 1365+x=6x+15 5x=1350 => x=270 Required number is 270. 25.In doing a division of a question with zero remainder,a candidate

file:///E|/work/books/placement/09_Aptitude/numbersproblems.html[1/28/2012 12:45:14 AM]

took 12 as divisor instead of 21.The quotient obtained by him was 35. The correct quotient is? a.0 b.12 c.13 d.20 Solution:Dividend=12*35=420. Now dividend =420 and divisor =21. Therefore correct quotient =420/21=20.

BACK

file:///E|/work/books/placement/09_Aptitude/numbersproblems.html[1/28/2012 12:45:14 AM]

APTITUDE
Numbers H.C.F and L.C.M Decimal Fractions Simplification Square and Cube roots Average Problems on Numbers Problems on Ages Surds and Indices Percentage Profit and Loss Ratio And Proportions Partnership Chain Rule Time and Work Pipes and Cisterns Time and Distance Trains Boats and Streams Alligation or Mixture Simple Interest Compound Interest Logorithms Areas Volume and Surface area Races and Games of Skill Calendar Clocks Stocks ans Shares True Discount Bankers Discount Oddmanout and Series Data Interpretation probability Permutations and Combinations Puzzles BACK

ODD MAN OUT & SERIES


EXAMPLE PROBLEMS:
1.Find the missing term in the series: 4,-8,16,-32,64,( ) SOLUTION: The terms are doubled and change the sign. So the next term is -128 2.16,33,65,131,261,( )

SOUTION: The terms are doubled and 1 is added. So 261*1+1=522+1=523 So the missing term is 523. 3.2,6,12,20,30,42,56,( )

SOLUTION: The pattern is 1 * 2 , 2 * 3 , 3 * 4 , 4 * 5 , 5 * 6 , 6 * 7 , 7 * 8 , 8 * 9 . So the series is 2,6,12,20,30,42,56,72. So 72 is the missing term. 4. 8,24,12,36,18,54,( )

SOLUTION: Numbers are alternatively multiplied by 3 and divided by 2. So the next term is 54 / 2 = 27. 5. 165,195,255,285,345,( )

SOLUTION: Each number is 15 multiplied by a prime number. i.e the series is 15*11,15*13,15*17,15*19,15*23,15*29. So series is 165,195,255,285,345,435. So 435 is the missing term. 6. 7,16,63,124,215,342,( ).

SOLUTION: Numbers are 23 -1,33-1,43-1,....................so 83-1=511. So 511 is the missing term. 7.2,4,12,48,240,( )

SOLUTION: Go on multiplying by 2,3,4,5,6. So the last term in the series is 240*6=1440. 8.8,7,11,12,14,17,17,22,( )

SOLUTION: There are two series 8,11,14,17,20 and 7,12,17,22 So increasing by 3 and 5.So 20 the missing term. 9.71,76,69,74,67,72,( )

SOLUTION: Alternately add 5 and subtract 7. so the series is 71+5=76 10.2,5,9,19,37 SOLUTION: Second number is one more than twice the first,Third number is one less than twice the second,Forth is one more than twice the third and so on. So the next number is 2 * 37 + 1 = 74+1 = 75. 11. Find the wrong number in the given series. 3,8,15,24,34,48,63 SOLUTION: The difference between consecutive terms are respectively 5,7,9,11,13. So 34 is the wrong number in the series. 12. 125,106,88,76,65,58,53 SOLUTION: Subtract 24,21,18,15,12,9 from the numbers to get the next number. So 128 is wrong.

13. 1,1,2,6,24,96,720

file:///E|/work/books/placement/09_Aptitude/oddcomplex.html[1/28/2012 12:45:15 AM]

SOLUTION: Multiply with 1,2,3,4,5,6 to get the next number. So 96 is wrong. 14 . 32,36,41,61,86,122,171,235 SOLUTION: Second term = First term + 22 Third term = Second term + 32 Fourth term = Third term + 42 Fifth term = Forth term + 52 Sixth term = Fifth term + 62 Seventh term = Sixth term + 72 So the third term should be 45 instead of 41. 15 . 15,16,34,105,424,2124,12576 SOLUTION: Second term = First term * 1 + 1 = 16 Third term = Second term * 2 + 2 = 34 Forth term = Third Term * 3 + 3 = 105 Fifth term = Forth term * 4 + 4 = 424 Sixth term = Fifth term * 5 + 5 =2125 Seventh term = Sixth term * 6 + 6 = 12576. So 2124 is wrong. 16 . 40960,10240,2560,640,200,40,10 SOLUTION: Go on dividing by 4 ,the series will be 40960,10240,2560,640,160,40,10. So 200 is wrong. 17.7,8,18,57,228,1165,6996 SOLUTION: Let the numbers be A,B,C,D,E,F,G then A,A*1+1,B*2+2,C*3+3,............ so 288 is wrong. 18. 19,26,33,46,59,74,91 SOLUTION: Go on adding 7,9,11,13,15,17. So 33 is wrong. 19 . 10,26,74,218,654,1946,5834. SOLUTION: Second term = first term * 3 4 = 26. Third term = Second term * 3 4 =74 Forth term = Third term * 3 4 =218 Fifth term = Forth term * 3 4 =650 So 654 is wrong .

file:///E|/work/books/placement/09_Aptitude/oddcomplex.html[1/28/2012 12:45:15 AM]

APTITUDE
Numbers H.C.F and L.C.M Decimal Fractions Simplification Square and Cube roots Average Problems on Numbers Problems on Ages Surds and Indices Percentage Profit and Loss Ratio And Proportions Partnership Chain Rule Time and Work Pipes and Cisterns Time and Distance Trains Boats and Streams Alligation or Mixture Simple Interest Compound Interest Logorithms Areas Volume and Surface area Races and Games of Skill Calendar Clocks Stocks ans Shares True Discount Bankers Discount Oddmanout and Series Data Interpretation probability Permutations and Combinations Puzzles BACK

ODD MAN OUT & SERIES


In any type of problems,a set of numbers is given in such a way that each one except one satiesfies a particular definite property.The one which does not satisfy that characteristic is to be taken out. Some important properties of numbers are 1.Prime Number Series Example: 2,3,5,7,11,................. 2.Even Number Series Example: 2,4,6,8,10,12,............ 3.Odd Number Series: Example: 1,3,5,7,9,11,................... 4.Perfect Squares: Example: 1,4,9,16,25,.............. 5.Perfect Cubes: Example: 1,8,27,64,125,................. 6.Multiples of Number Series: Example: 3,6,9,12,15,..............are multiples of 3 7.Numbers in Arthimetic Progression(A.P): Example: 13,11,9,7................ 8.Numbers in G.P: Example: 48,12,3,..... SOME MORE PROPERTIES: 1. If any series starts with 0,3,.....,generally the relation will be (n2-1). 2. If any series starts with 0,2,.....,generally the relation will be (n2-n). 3. If any series starts with 0,6,.....,generally the relation will be (n3-n). 4. If 36 is found in the series then the series will be in n2 relation. 5. If 35 is found in the series then the series will be in n2-1 relation. 6. If 37 is found in the series then the series will be in n2+1 relation. 7. If 125 is found in the series then the series will be in n3 relation. 8. If 124 is found in the series then the series will be in n3-1 relation. 9. If 126 is found in the series then the series will be in n3+1 relation. 10. If 20,30 found in the series then the series will be in n2-n relation. 11. If 60,120,210,........... is found as series then the series will be in n3-n relation. 12. If 222,............ is found then relation is n3+n 13. If 21,31,.......... is series then the relation is n2-n+1. 14. If 19,29,.......... is series then the relation is n2-n-1. 15. If series starts with 0,3,............ the series will be on n2-1 relation. given below :

file:///E|/work/books/placement/09_Aptitude/oddcont.html[1/28/2012 12:45:15 AM]

Oddman Out and Series


Introduction: In any type of problems,a set of numbers is given in such a way that each one except one satiesfies a particular definite property.The one which does not satisfy that characteristic is to be taken out. Some important properties of numbers are given below : 1.Prime Number Series Example: 2,3,5,7,11,.............. 2.Even Number Series Example: 2,4,6,8,10,12,........... 3.Odd Number Series: Example: 1,3,5,7,9,11,........... 4.Perfect Squares: Example: 1,4,9,16,25,............ 5.Perfect Cubes: Example: 1,8,27,64,125,................. 6.Multiples of Number Series: Example: 3,6,9,12,15,..............are multiples of 3 7.Numbers in Arthimetic Progression(A.P): Example: 13,11,9,7................ 8.Numbers in G.P: Example: 48,12,3,..... Some More Properties: 1. If any series starts with 0,3,.....,generally the relation will be (n2 - 1). 2. If any series starts with 0,2,.....,generally the relation will be (n2 - n). 3. If any series starts with 0,6,.....,generally the relation will be (n3 - n). 4. If 36 is found in the series then the series will be in n2 relation. 5. If 35 is found in the series then the series will be in n2-1 relation. 6. If 37 is found in the series then the series will be in n2+1 relation. 7. If 125 is found in the series then the series will be in n3 relation. 8. If 124 is found in the series then the series will be in n3 - 1 relation. 9. If 126 is found in the series then the series will be in n3+1 relation. 10. If 20,30 found in the series then the series will be in n2 - n relation. 11. If 60,120,210,........... is found as series then the series will be in n3 - n relation. 12. If 222,............ is found then relation is n3+n 13. If 21,31,.......... is series then the relation is n2 - n+1. 14. If 19,29,.......... is series then the relation is n2 - n - 1. 15. If series starts with 0,3,............ the series will be on n2-1 relation.

Top
Problems 1.Find the odd one out.3,5,7,12,17,19 SOLUTION: The above series except 12 all elements are odd numbers.so 12 is the odd one. 2.Find the odd one out.1,4,9,16,23,25,36 SOLUTION: In the above series all elements except 23 are perfect sqares. so 23 is odd one. 3.Find the odd one out.41,43,47,53,61,71,73,81 SOLUTION: In the above series all elements except 81 are prime numbers. so 81 is odd one. 4.Find the odd one out.1,4,9,16,20,36,49 SOLUTION: In the above series all elements except 20 are perfect squares. So 20 is odd one. 5.Find the odd one out.8,27,64,100,125,216,343 SOLUTION: In the above series all elements except 100 are perfect cubes. so 100 is odd one. 6. Find the odd one out.1,5,14,30,50,55,99 SOLUTION: In the above series all elements in the pattern like 12,

file:///E|/work/books/placement/09_Aptitude/oddmanoutandseries.html[1/28/2012 12:45:15 AM]

12+22,12+22+32,................. pattern,so odd one.

But

50 is

not in this

7.Find the odd one out.835,734,642,751,853,981,532 SOLUTION: In the above series,the difference between third and first digit of each element is equal to its middle digit.But 751 is not in this pattern,so odd one. 8.Find the odd one out.385,4462,572,396,427,672,264 SOLUTION: In the above series,the sum of first and third digit of each element is equal to its middle digit.But 427is not in this pattern,so odd one. 9.Find the odd one out.331,482,551,263,383,242,111 SOLUTION: In the above series,the product of first and third digit of each element is equal to its middle digit. But 383 is not in this pattern,so odd one. 10. Find the odd one out.2,5,10,17,26,37,50,64 SOLUTION: In the above series,the elements are in the pattern of x2+1, Where x is 1,2,3,4,5,6,7.but 82+1 is not equal to 64.It is 65.64 is odd one. 11.Find the odd one out. 19,28,39,52,67,84,102 SOLUTION: In the above series,the elements are in the pattern of x2+3, Where x is 4,5,6,7,8,9,10.but 102+3 is not equal to 102. It is 103.so 102 is odd one.

Top
12.Find the odd one out.253,136,352,460,324,613,244 SOLUTION: In the above series,the elements are in the pattern of x2+3, Where x is 4,5,6,7,8,9,10.but 102+3 is not equal to 102.It is 103.so 102 is odd one. 13.Find the odd one out. 2,5,10,50,500,5000 SOLUTION: In the above series,the 1st term * 2nd term = 2nd term * 3rd term = 3rd term * 4th term = But 50*500=25000 which so 5000 is odd one. pattern as follows: 3rd term 4th term 5th term is not equal to 5000.

14.Find the odd one out. 582,605,588,611,634,617,600 SOLUTION: In the above series, alternatively 23 is added and 17 is subtracted from the terms. So 634 is odd one. 15.Find the odd one out.46080,3840,384,48,24,2,1 SOLUTION: In the above series,the terms are successively divided by 12,10,8,6,..... so 24 is not in this pattern. so 24 is odd one. 16.Find the odd one out.5,16,6,16,7,16,9 SOLUTION: In the above series,the terms at odd places are 5,6,7,8....... and at even places is 16. So 9 is odd one. 17.Find the odd one out.6,13,18,25,30,37,40 SOLUTION: In the above series,the difference between two successive terms from the beginning are 7,5,7,5......... so 40 is odd one. 18.Find the odd one out.56,72,90,110,132,150 SOLUTION: The above series as follows: 7*8,8*9,9*10,10*11,11*12,12*13. So it will be 56,72,90,110,132,156 so 150 is wrong. 19.Find the odd one out. 1,2,6,15,31,56,91 SOLUTION: Add 1square ,2square ,....,6square so 91 is wrong. to the terms.

20.Find the odd one out.105,85,60,30,0, - 45, - 90 SOLUTION: Subtract 20,25,30,35,40,45 So 0 is odd one. from the terms.

21.Find out the odd one out.3,10,21,36,55,70,105 SOLUTION: The pattern in the series is 1*3, 2*5, 3*7, 4*9, 5*11, 6*13, 7*15. So the series will be 3,10,21,36,55,78,105. So 70 is wrong term in the series.

Top
22.Find out the odd one out. 4,9,19,39,79,160,319 SOLUTION: Double the number and add 1 to it. So the series will be 4,9,39,79,159,319. So 160 is wrong. 23.Find out the odd one out.10,14,28,32,64,68,132. SOLUTION:

file:///E|/work/books/placement/09_Aptitude/oddmanoutandseries.html[1/28/2012 12:45:15 AM]

Alternatively add 4 and double the next term. So 132 is wrong. COmplex Problems 1.Find the missing term in the series: 4,-8,16, - 32,64,( ) SOLUTION: The terms are doubled and change the sign. So the next term is - 128 2.16,33,65,131,261,( SOUTION: The terms are doubled and 1 is added. So 261*1+1=522+1=523 So the missing term is 523. 3.2,6,12,20,30,42,56,( ) )

SOLUTION: The pattern is 1 * 2 , 2 * 3 , 3 * 4 , 4 * 5 , 5 * 6 , 6 * 7 , 7 * 8 , 8 * 9 . So the series is 2,6,12,20,30,42,56,72. So 72 is the missing term. 4. 8,24,12,36,18,54,( )

SOLUTION: Numbers are alternatively multiplied by 3 and divided by 2. So the next term is 54 / 2 = 27. 5. 165,195,255,285,345,( )

SOLUTION: Each number is 15 multiplied by a prime number. i.e the series is 15*11,15*13,15*17,15*19,15*23,15*29. So series is 165,195,255,285,345,435. So 435 is the missing term.

Top
6. 7,16,63,124,215,342,( ). SOLUTION: Numbers are 23 - 1,33 - 1,43 - 1,....................so 83 - 1=511. So 511 is the missing term. 7.2,4,12,48,240,( SOLUTION: Go on multiplying by 2,3,4,5,6. So the last term in the series is 240*6=1440. 8.8,7,11,12,14,17,17,22,( SOLUTION: There are two series 8,11,14,17,20 and 7,12,17,22 So increasing by 3 and 5.So 20 the missing term. 9.71,76,69,74,67,72,( SOLUTION: Alternately add 5 and subtract 7. so the series is 71+5=76 10.10.2,5,9,19,37 SOLUTION: Second number is one more than twice the first,Third number is one less than twice the second,Forth is one more than twice the third and so on. So the next number is 2 * 37 + 1 = 74+1 = 75. 11. Find the wrong number in the given series. 3,8,15,24,34,48,63 SOLUTION: The difference between consecutive terms are respectively 5,7,9,11,13. So 34 is the wrong number in the series. 12. 125,106,88,76,65,58,53 ) ) )

SOLUTION: Subtract 24,21,18,15,12,9 from the numbers to get the next number. So 128 is wrong. 13. 1,1,2,6,24,96,720 SOLUTION: Multiply with 1,2,3,4,5,6 to get the next number. So 96 is wrong. 14 . 32,36,41,61,86,122,171,235 SOLUTION: Second term = First term + 22 Third term = Second term + 32 Fourth term = Third term + 42 Fifth term = Forth term + 52 Sixth term = Fifth term + 62 Seventh term = Sixth term + 72 So the third term should be 45 instead of 41. 15 . 15,16,34,105,424,2124,12576 SOLUTION: Second term = First term * 1 + 1 = 16 Third term = Second term * 2 + 2 = 34 Forth term = Third Term * 3 + 3 = 105 Fifth term = Forth term * 4 + 4 = 424 Sixth term = Fifth term * 5 + 5 =2125 Seventh term = Sixth term * 6 + 6 = 12576. So 2124 is wrong.

file:///E|/work/books/placement/09_Aptitude/oddmanoutandseries.html[1/28/2012 12:45:15 AM]

16 . 40960,10240,2560,640,200,40,10 SOLUTION: Go on dividing by 4 ,the series will be 40960,10240,2560,640,160,40,10. So 200 is wrong. 17.7,8,18,57,228,1165,6996 SOLUTION: Let the numbers be A,B,C,D,E,F,G then A,A*1+1,B*2+2,C*3+3,............ so 288 is wrong. 18. 19,26,33,46,59,74,91 SOLUTION: Go on adding 7,9,11,13,15,17. So 33 is wrong. 19 . 10,26,74,218,654,1946,5834. SOLUTION: Second term = first term * 3 4 = 26. Third term = Second term * 3 4 =74 Forth term = Third term * 3 4 =218 Fifth term = Forth term * 3 4 =650 So 654 is wrong .

Back Back To Main

Top

Contact: 040-23000700

file:///E|/work/books/placement/09_Aptitude/oddmanoutandseries.html[1/28/2012 12:45:15 AM]

APTITUDE
Numbers H.C.F and L.C.M Decimal Fractions Simplification Square and Cube roots Average Problems on Numbers Problems on Ages Surds and Indices Percentage Profit and Loss Ratio And Proportions Partnership Chain Rule Time and Work Pipes and Cisterns Time and Distance Trains Boats and Streams Alligation or Mixture Simple Interest Compound Interest Logorithms Areas Volume and Surface area Races and Games of Skill Calendar Clocks Stocks ans Shares True Discount Bankers Discount Oddmanout and Series Data Interpretation probability Permutations and Combinations Puzzles BACK

ODD MAN OUT & SERIES


EXAMPLE PROBLEMS:
13.Find the odd one out. 2,5,10,50,500,5000 SOLUTION: In the above series,the pattern as follows: 1st term * 2nd term = 3rd term 2nd term * 3rd term = 4th term 3rd term * 4th term = 5th term But 50*500=25000 which is not equal to 5000. so 5000 is odd one. 14.Find the odd one out. 582,605,588,611,634,617,600 SOLUTION: In the above series, alternatively 23 is added and 17 is from the terms. So 634 is odd one. 15.Find the odd one out. 46080,3840,384,48,24,2,1 SOLUTION: In the above series,the terms are successively divided by 12,10,8,6,..... so 24 is not in this pattern. so 24 is odd one. 16.Find the odd one out. 5,16,6,16,7,16,9 SOLUTION: In the above series,the terms at odd places are 5,6,7,8.......and at even places is 16. So 9 is odd one. 17.Find the odd one out. 6,13,18,25,30,37,40 SOLUTION: In the above series,the difference between two successive terms from are 7,5,7,5......... so 40 is odd one. 18.Find the odd one out. 56,72,90,110,132,150 SOLUTION: The above series as follows: 7*8,8*9,9*10,10*11,11*12,12*13. So it will be 56,72,90,110,132,156 so 150 is wrong. 19.Find the odd one out. 1,2,6,15,31,56,91 SOLUTION: Add 1square ,2square ,....,6square so 91 is wrong. 20.Find the odd one out. 105,85,60,30,0,-45,-90 SOLUTION: Subtract 20,25,30,35,40,45 So 0 is odd one. 21.Find out the odd one out. 3,10,21,36,55,70,105 SOLUTION: The pattern in the series is 1*3, 2*5, 3*7, 4*9, 5*11, 6*13, 7*15. So the series will be 3,10,21,36,55,78,105. So 70 is wrong term in the series. 22.Find out the odd one out. 4,9,19,39,79,160,319 SOLUTION: Double the number and add 1 to it. So the series will be 4,9,39,79,159,319. So 160 is wrong. from the terms. to the terms. subtracted

the beginning

file:///E|/work/books/placement/09_Aptitude/oddmedium.html[1/28/2012 12:45:16 AM]

23.Find out the odd one out. 10,14,28,32,64,68,132. SOLUTION: Alternatively add 4 and double the next term. So 132 is wrong.

file:///E|/work/books/placement/09_Aptitude/oddmedium.html[1/28/2012 12:45:16 AM]

APTITUDE
Numbers H.C.F and L.C.M Decimal Fractions Simplification Square and Cube roots Average Problems on Numbers Problems on Ages Surds and Indices Percentage Profit and Loss Ratio And Proportions Partnership Chain Rule Time and Work Pipes and Cisterns Time and Distance Trains Boats and Streams Alligation or Mixture Simple Interest Compound Interest Logorithms Areas Volume and Surface area Races and Games of Skill Calendar Clocks Stocks ans Shares True Discount Bankers Discount Oddmanout and Series Data Interpretation probability Permutations and Combinations Puzzles

ODDMAN OUT AND SERIES


CONCEPT SIMPLE PROBLEMS MEDIUM PROBLEMS COMPLEX PROBLEMS

file:///E|/work/books/placement/09_Aptitude/oddseries.html[1/28/2012 12:45:16 AM]

APTITUDE
Numbers H.C.F and L.C.M Decimal Fractions Simplification Square and Cube roots Average Problems on Numbers Problems on Ages Surds and Indices Percentage Profit and Loss Ratio And Proportions Partnership Chain Rule Time and Work Pipes and Cisterns Time and Distance Trains Boats and Streams Alligation or Mixture Simple Interest Compound Interest Logorithms Areas Volume and Surface area Races and Games of Skill Calendar Clocks Stocks ans Shares True Discount Bankers Discount Oddmanout and Series Data Interpretation probability Permutations and Combinations Puzzles BACK

ODD MAN OUT & SERIES


EXAMPLE PROBLEMS:
1.Find the odd one out. 3,5,7,12,17,19 SOLUTION: In the above series except 12 all elements are odd numbers.so 12 is the odd one. 2. Find the odd one out. 1,4,9,16,23,25,36 SOLUTION: In the above series all elements except 23 are perfect sqares.so 23 is odd one. 3. Find the odd one out. 41,43,47,53,61,71,73,81 SOLUTION: In the above series all elements except 81 are prime numbers.so odd one. 4.Find the odd one out. 1,4,9,16,20,36,49 SOLUTION: In the above series all elements except 20 are perfect odd one. 5.Find the odd one out. 8,27,64,100,125,216,343 SOLUTION: In the above series all elements except 100 are perfect cubes.so 100 is odd one. 6. Find the odd one out. 1,5,14,30,50,55,99 SOLUTION: In the above series all elements in the pattern like 12, 12+22,12+22+32,................. But 50 is not in this pattern,so odd one. 7.Find the odd one out. 835,734,642,751,853,981,532 SOLUTION: In the above series,the difference between third and first digit of each element is equal to its middle digit.But 751 is not in this pattern,so odd one. 8. Find the odd one out. 385,4462,572,396,427,672,264 SOLUTION: In the above series,the sum of first and third digit of each element is equal to its middle digit.But 427is not in this pattern,so odd one. 9. Find the odd one out. 331,482,551,263,383,242,111 squares.So 20 is 81 is

SOLUTION: In the above series,the product of first and third digit of each element is equal to its middle digit. But 383 is not in this pattern,so odd one. 10. Find the odd one out. 2,5,10,17,26,37,50,64 SOLUTION: In the above series,the elements are in the pattern of x2+1,Where is 1,2,3,4,5,6,7.but 82+1 is not equal to 64.It is 65.64 is odd one. 11. Find the odd one out. 19,28,39,52,67,84,102 x x

SOLUTION: In the above series,the elements are in the pattern of x2+3,Where is 4,5,6,7,8,9,10.but 102+3 is not equal to 102.It is 103.so 102 is odd one. 12.Find the odd one out. 253,136,352,460,324,613,244 SOLUTION: In the above series,the elements are in the pattern of x2+3,Where is 4,5,6,7,8,9,10.but 102+3 is not equal to 102.It is 103.so 102 is odd one.

file:///E|/work/books/placement/09_Aptitude/oddsimple.html[1/28/2012 12:45:17 AM]

APTITUDE
Numbers H.C.F and L.C.M Decimal Fractions Simplification Square and Cube roots Average Problems on Numbers Problems on Ages Surds and Indices Percentage Profit and Loss Ratio And Proportions Partnership Chain Rule Time and Work Pipes and Cisterns Time and Distance Trains Boats and Streams Alligation or Mixture Simple Interest Compound Interest Logorithms Areas Volume and Surface area Races and Games of Skill Calendar Clocks Stocks ans Shares True Discount Bankers Discount Oddmanout and Series Data Interpretation probability Permutations and Combinations Puzzles

BACK

PARTNERSHIP
Important Facts: Partnership:When two or more than two persons run a business
jointly, they are called partners and the deal is known as partnership. Ratio of Division of Gains: 1.When the investments of all the partners are do the same time, the gain or loss is distributed among the partners in the ratio of their investments. Suppose A and B invest Rs x and Rs y respectively for a year in a business, then at the end of the year: (A's share of profit):(B's share of profit)=x:y 2.When investments are for different time periods, then equivalent capitals are calculated for a unit of time by taking (capital*number of units of time). Now gain or loss is divided in the ratio of these capitals. Suppose A invests Rs x for p months and B invests Rs y for q months, then (A's share of profit):(B's share of profit)=xp:yq 3.Working and sleeping partners:A partner who manages the business is known as working partner and the one who simply invests the money is a sleeping partner. FORMULAE 1.When investments of A and B are Rs x and Rs y for a year in a business ,then at the end of the year (A's share of profit):(B's share of profit)=x:y 2.When A invests Rs x for p months and B invests Rs y for q months,then A's share profit:B's share of profit=xp:yq BACK

file:///E|/work/books/placement/09_Aptitude/partnerconcepts.html[1/28/2012 12:45:17 AM]

APTITUDE
Numbers H.C.F and L.C.M Decimal Fractions Simplification Square and Cube roots Average Problems on Numbers Problems on Ages Surds and Indices Percentage Profit and Loss Ratio And Proportions Partnership Chain Rule Time and Work Pipes and Cisterns Time and Distance Trains Boats and Streams Alligation or Mixture Simple Interest Compound Interest Logorithms Areas Volume and Surface area Races and Games of Skill Calendar Clocks Stocks ans Shares True Discount Bankers Discount Oddmanout and Series Data Interpretation probability Permutations and Combinations Puzzles

BACK

PARTNERSHIP
Difficult problems: 1.P and Q started a business investing Rs 85,000 and Rs 15,000 respectively. In what ratio the profit earned after 2 years be divided between P and Q respectively? Sol: 85,000*2 : 15,000*2 17*2 : 3*2=34:6 2.A,B and C started a business by investing Rs 1,20,000, Rs 1,35,000 and Rs 1,50,000.Find the share of each ,out of an annual profit of Rs 56,700? Sol: Ratio of shares of A,B and C=Ratio of their investments
120,000:135,000:150,000 =8:9:10 A's share=Rs 56,700*(8/27)=Rs 16,800 B's share =Rs 56,700*(9/27)=Rs 18,900 C's share =Rs 56,700*(10/27)=Rs 21,000

3.3 milkman A,B,C rented a pasture A grazed his 45 cows for 12 days B grazed his 36 cows for 15 days and c 60 cows for 10 days.If b's share of rent was Rs 540 What is the total rent? Sol:
45*12:36*15:60*10 =9:9:10 9 parts is equal to Rs 540 then one part is equal to Rs 60 total rent=60*28=Rs 1680

4.Ramu and Krishna entered into a partnership with Rs 50,000 and Rs 60,000, after 4 months Ramu invested Rs 25,000 more while Krishna withdraw Rs 20,000 . Find the share of Ramu in the annual profit of Rs 289,000. Sol: Ramu : Krishna=50,000*4+75,000*8:60,000*4+40,000*8 =10:7 Ramu's annual profit=289000*(10/17)=Rs 170,000

5.A,B,C enter into partnership .A invests 3 times as much as B invests and B invests two third of what C invests. At the end of the year ,the profit earned is Rs 6600. what is the share of B? Sol:
let C's capital =Rs x B's capital=Rs (2/3)*x A's capital =3*(2/3)*x=Rs 2x ratio of their capitals=2x:(2/3)*x:x =6x:2x;3x B's share =Rs 6600(2/11)=Rs 1200

file:///E|/work/books/placement/09_Aptitude/partnerdifficult.html[1/28/2012 12:45:17 AM]

6.A,B and C enter into a partnership by investing in the ratio of 3:2:4. After one year ,B invests another Rs 2,70,000 and C,at the end of 2 years, also invests Rs 2,70,000.At the end of 3 years ,profit are shared in the ratio of 3:4:5.Find the initial investment of each? Sol: Initial investments of A,B,c be Rs 3x, Rs 2x, Rs 4x then for 3 years (3x*36):[(2x*12)+(2x+270000)*24]:[(4x*24)+(4x+270000)*12]=3:4:5 108x:(72x+640,000):(144x+3240000)=3:4:5 108x:72x+6480000:144x+3240000=3:4:5 (108x)/(72x+6480000)=3/4 432x=216x+19440000 216x=19440000 x=Rs 90000 A's initial investment=3x=3*90,000=Rs 2,70,000 B's initial investment=2x=2*90,000=Rs 1,80,000 C's initial investment=4x=4*90,000=Rs 3,60,000 BACK

file:///E|/work/books/placement/09_Aptitude/partnerdifficult.html[1/28/2012 12:45:17 AM]

Partnership
Important Facts: Partnership:When two or more than two persons run a business jointly, they are called partners and the deal is known as partnership. Ratio of Division of Gains: 1.When the investments of all the partners are do the same time, the gain or loss is distributed among the partners in the ratio of their investments. Suppose A and B invest Rs x and Rs y respectively for a year in a business, then at the end of the year: (A's share of profit):(B's share of profit)=x:y 2.When investments are for different time periods, then equivalent capitals are calculated for a unit of time by taking (capital*number of units of time). Now gain or loss is divided in the ratio of these capitals. Suppose A invests Rs x for p months and B invests Rs y for q months, then (A's share of profit):(B's share of profit)=xp:yq 3.Working and sleeping partners:A partner who manages the business is known as working partner and the one who simply invests the money is a sleeping partner. Formulae 1.When investments of A and B are Rs x and Rs y for a year in a business ,then at the end of the year (A's share of profit):(B's share of profit)=x:y 2.When A invests Rs x for p months and B invests Rs y for q months, then A's share profit:B's share of profit=xp:yq Short cuts: 1.In case of 3 A,B,C investments then individual share is to be found then A=16000 , B=32,000 , C=40,000 Sol: A:B:C=16:32:40 =2:4:5` then individual share can be easily known. 2.If business mans A contributes for 5 months and B contributes for 9 months then share of B in the total profit of Rs 26,8000 ,A = Rs 15000, B =Rs 12000 Sol: 15000*5 : 12000*9 25 : 36 for 36 parts=268000*(36/61) =Rs 158.16 Back Difficult problems:

Top

1.P and Q started a business investing Rs 85,000 and Rs 15,000 respectively. In what ratio the profit earned after 2 years be divided between P and Q respectively? Sol: 85,000*2 : 15,000*2 17*2 : 3*2=34:6

2.A,B and C started a business by investing Rs 1,20,000, Rs 1,35,000 and Rs 1,50,000.Find the share of each ,out of an annual profit of Rs 56,700? Sol: Ratio of shares of A,B and C=Ratio of their investments 120,000:135,000:150,000 =8:9:10 A's share=Rs 56,700*(8/27)=Rs 16,800 B's share =Rs 56,700*(9/27)=Rs 18,900 C's share =Rs 56,700*(10/27)=Rs 21,000 3.3 milkman A,B,C rented a pasture A grazed his 45 cows for 12 days B grazed his 36 cows for 15 days and c 60 cows for 10 days.If b's share of rent was Rs 540 What is the total rent? Sol: 45*12:36*15:60*10 =9:9:10 9 parts is equal to Rs 540 then one part is equal to Rs 60 total rent=60*28=Rs 1680

4.Ramu and Krishna entered into a partnership with Rs 50,000 and Rs 60,000, after 4 months Ramu invested Rs 25,000 more while Krishna withdraw Rs 20,000 . Find the share of Ramu in the annual profit of Rs 289,000. Sol: Ramu : Krishna=50,000*4+75,000*8:60,000*4+40,000*8

file:///E|/work/books/placement/09_Aptitude/partnership.html[1/28/2012 12:45:18 AM]

=10:7 Ramu's annual profit=289000*(10/17)=Rs 170,000 5.A,B,C enter into partnership .A invests 3 times as much as B invests and B invests two third of what C invests. At the end of the year ,the profit earned is Rs 6600. what is the share of B? Sol: let C's capital =Rs x B's capital=Rs (2/3)*x A's capital =3*(2/3)*x=Rs 2x ratio of their capitals=2x:(2/3)*x:x =6x:2x;3x B's share =Rs 6600(2/11)=Rs 1200

6.A,B and C enter into a partnership by investing in the ratio of 3:2:4. After one year ,B invests another Rs 2,70,000 and C,at the end of 2 years, also invests Rs 2,70,000.At the end of 3 years ,profit are shared in the ratio of 3:4:5.Find the initial investment of each? Sol: Initial investments of A,B,c be Rs 3x, Rs 2x, Rs 4x then for 3 years (3x*36):[(2x*12)+(2x+270000)*24]:[(4x*24)+(4x+270000)*12]=3:4:5 108x:(72x+640,000):(144x+3240000)=3:4:5 108x:72x+6480000:144x+3240000=3:4:5 (108x)/(72x+6480000)=3/4 432x=216x+19440000 216x=19440000 x=Rs 90000 A's initial investment=3x=3*90,000=Rs 2,70,000 B's initial investment=2x=2*90,000=Rs 1,80,000 C's initial investment=4x=4*90,000=Rs 3,60,000

Back Back To Main

Top

Contact: 040-23000700

file:///E|/work/books/placement/09_Aptitude/partnership.html[1/28/2012 12:45:18 AM]

APTITUDE
Numbers H.C.F and L.C.M Decimal Fractions Simplification Square and Cube roots Average Problems on Numbers Problems on Ages Surds and Indices Percentage Profit and Loss Ratio And Proportions Partnership Chain Rule Time and Work Pipes and Cisterns Time and Distance Trains Boats and Streams Alligation or Mixture Simple Interest Compound Interest Logorithms Areas Volume and Surface area Races and Games of Skill Calendar Clocks Stocks ans Shares True Discount Bankers Discount Oddmanout and Series Data Interpretation probability Permutations and Combinations Puzzles

PARTNERSHIP
Short cuts: 1.In case of 3 A,B,C investments then individual share is to be found then A=16000 , B=32,000 , C=40,000 Sol: A:B:C=16:32:40 =2:4:5` then individual share can be easily known. 2.If business mans A contributes for 5 months and B contributes for 9 months then share of B in the total profit of Rs 26,8000 ,A = Rs 15000 ,B =Rs 12000 Sol: 15000*5 : 12000*9 25 : 36 for 36 parts=268000*(36/61) =Rs 158.16
BACK

file:///E|/work/books/placement/09_Aptitude/partnershortcuts.html[1/28/2012 12:45:18 AM]

APTITUDE
Numbers H.C.F and L.C.M Decimal Fractions Simplification Square and Cube roots Average Problems on Numbers Problems on Ages Surds and Indices Percentage Profit and Loss Ratio And Proportions Partnership Chain Rule Time and Work Pipes and Cisterns Time and Distance Trains Boats and Streams Alligation or Mixture Simple Interest Compound Interest Logorithms Areas Volume and Surface area Races and Games of Skill Calendar Clocks Stocks ans Shares True Discount Bankers Discount Oddmanout and Series Data Interpretation probability Permutations and Combinations Puzzles

PERCENTAGES
CONCEPT SIMPLE PROBLEMS MEDIUM PROBLEMS COMPLEX PROBLEMS

file:///E|/work/books/placement/09_Aptitude/percent.html[1/28/2012 12:45:18 AM]

Problems on Percentages
Simple problems: 1 . Express the following a) 56% SOLUTION: 56/100=14/25 b) 4% SOLUTION: 4/100=1/25 c) 0.6% SOLUTION: 0.6/100=6/1000=3/500 d) 0.08% SOLUTION: 0.08/100=8/10000=1/1250 2. a) Express the following as decimals 6% SOLUTION: 6% = 6/100=0.06 b) 0.04% SOLUTION: 0.04% = 0.04/100=0.0004 3 . Express the following as rate percent. i).23/36 SOLUTION: = (23/36*100) % = 63 8/9% ii).6 SOLUTION: 6 =27/4 (27/4 *100) % =675 % 4. Evaluate the following: 28% of 450 + 45% of 280 ? SOLUTION: =(28/100) *450 + (45/100) *280 = 28 * 45 / 5 = 252 5. 2 is what percent of 50? SOLUTION: Formula : (IS / OF ) *100 % = 2/50 *100 = 4% 6. is what percent of 1/3? SOLUTION: =( ) / (1/3) *100 % = 3/2 *100 % = 150 % 7. What percent of 2 Metric tonnes is 40 Quintals? as a fraction.

SOLUTION: 1 metric tonne =10 Quintals So required percentage=(40/(2*10))*100% = 200% 8. Find the missing figure . i) ? % of 25 = 2.125 SOLUTION : Let x% of 25 = 2.125.then (x/100) *25 =2.125 x = 2.125 * 4 = 8.5 ii) 9% of ? =6.3 SOLUTION: Let 9 % of x = 6.3. Then 9/100 of x= 6.3 so x = 6.3 *100/7 = 70. 9. Which is the greatest in 16 2/3 %, 2/15,0.17? SOLUTION: 16 2/3 % = 50/3 % =50/3 * 1/100 =1/6 = 0.166 2 / 15 =0.133 So 0.17 is greatest number in the given series. 10.If the sales tax be reduced from 3 % to 3 1/3 % ,

file:///E|/work/books/placement/09_Aptitude/percentage.html[1/28/2012 12:45:19 AM]

then what difference does it make to a person who purchases an article with marked price of RS 8400? SOLUTION: Required difference = 3 % of 8400 3 1/3 % of 8400 =(7/2-10/3)% of 8400 =1/6 % of 8400 = 1/6* 1/100* 8400 = Rs 14. 11. A rejects 0.08% of the meters as defective .How many will he examine to reject 2? SOLUTION: Let the number of meters to be examined be x. Then 0.08% of x=2. 0.08/100*x= 2 x= 2 * 100/0.08 =2 * 100 * 100/8 = 2500 12.65 % of a number is 21 less than 4/5 of that number. What is the number? SOLUTION: Let the number be x. 4/5 x- (65% of x) = 21 4/5x 65/100 x=21 15x=2100 x=140

13. Difference of two numbers is 1660.If 7.5 % of one number is 12.5% of the other number. Find two numbers? SOLUTION: Let the two numbers be x and y. 7.5% of x=12.5% of y' So 75x=125 y 3x=5y x=5/3y. Now x-y=1660 5/3y-y=1660 2/3y=1660 y=2490 So x= 2490+1660 =4150. So the numbers are 4150 , 1660. 14. In expressing a length 81.472 KM as nearly as possible with 3 significant digits ,Find the % error? SOLUTION: Error= 81.5-81.472=0.028 So the required percentage = 0.028/81.472*100% = 0.034% Top

15. In an election between two persons ,75% of the voters cast their votes out of which 2% are invalid. A got 9261 which 75% of the total valid votes. Find total number of votes? SOLUTION: Let x be the total votes. valid votes are 98% of 75% of x. So 75%(98%(75% of x))) = 9261 ==> 75/100 *98 /100 * 75 100 *x = 9261 x= 1029 * 4 *100 *4 / 9 = 16800 So total no of votes = 16800 16 . A's maths test had 75 problems i.e 10 arithmetic, 30 algebra and 35 geometry problems.Although he answered 70% of arithmetic ,40% of algebra and 60 % of geometry problems correctly he didn't pass the test because he got less than 60% of the problems right. How many more questions he would have needed to answer correctly to get a 60% passing grade. SOLUTION: 70% of 10 =70/100 * 10 =7 40% of 30 = 40 / 100 * 30 = 12 60 % of 35 = 60 / 100 *35 = 21 So correctly attempted questions = 7 + 12 + 21 =40. Questions to be answered correctly for 60% grade =60% of 75 = 60/100 *75 =45. So required questions=45-40 = 5 17 . If 50% of (x y) = 30% of (x + y) then what percent of x is y ? SOLUTION: 50/100(x-y) =30/100(x+y) (x-y)= 3/10(x+y) 5x-5y=3x+3y x=4y So Required percentage =y/x*100 %

file:///E|/work/books/placement/09_Aptitude/percentage.html[1/28/2012 12:45:19 AM]

=y/4y *100 % = 25%. 18 .If the price of tea is increased by 20% ,find how much percent must a householder reduce her consumption of tea so as not to increase the expenditure? SOLUTION: Reduction in consumption= R/(100+R) *100% =20/120 *100 = 16 2/3 % 19.The population of a town is 176400 . If it increases at the rate of 5% per annum ,what will be the population 2 years hence? What was it 2 years ago? SOLUTION: Population After 2 years = 176400[1+5/100]2 =176400 * 21/20 *21/20 =194481 Population 2 years ago = 176400/(1+5/100)2 = 176400 * 20/21 *20/ 21 =160000 20.1 liter of water is add to 5 liters of a 20 % solution of alcohol in water . Find the strength of alcohol in new solution? SOLUTION: Alcohol in 5 liters = 20% of 5 =1 liter Alcohol in 6 liters of new mixture = 1liter So % of alcohol is =1/6 *100=16 2/3% 21.If A earns 33 1/3 more than B .Then B earns less than A by what percent? SOLUTION: 33 1/3 =100 / Required Percentage = (100/3)/(100 + (100/3)) *100 % = 100/400 *100 = 25 % 22.A school has only three classes which contain 40,50,60 students respectively.The pass percent of these classes are 10, 20 and 10 respectively . Then find the pass percent in the school. SOLUTION: Number of passed candidates = 10/100*40+20/100 *50+10/100 * 60 =4+10+6 =20 Total students in school = 40+50+60 =150 So required percentage = 20/150 *100 = 40 /3 =13 1/3 % 23. There are 600 boys in a hostel . Each plays either hockey or football or both .If 75% play hockey and 45 % play football ,Find how many play both? SOLUTION: n(A)=75/100 *600 =450 n(B) = 45/100 *600 = 270 n(A^B)=n(A) + n(B) n(AUB) =450 + 270 -600 =120 So 120 boys play both the games. 24.A bag contains 600 coins of 25p denomination and 1200 coins of 50p denomination. If 12% of 25p coins and 24 % of 50p coins are removed, Find the percentage of money removed from the bag ? SOLUTION: Total money = (600 * 25/100 +1200 *50/100) =Rs 750 25p coins removed = 12/100 *600 =72 50p coins removed = 24/100 *1200 =288

file:///E|/work/books/placement/09_Aptitude/percentage.html[1/28/2012 12:45:19 AM]

So money removed =72 *1/4 +288 *1/2 = Rs 162 So required percentage=162/750 *100 =21 .6% 25. P is six times as large as Q.Find the percent that Q is less than P? SOLUTION: Given that P= 6Q So Q is less than P by 5Q. Required percentage= 5Q/P*100 % =5/6 * 100 % =83 1/3% 26.For a sphere of radius 10 cm ,the numerical value of surface area is what percent of the numerical value of its volume? SOLUTION: Surface area = 4 *22/7 *r2 = 3/r(4/3 * 22/7 * r3) =3/r * VOLUME Where r = 10 cm So we have S= 3/10 V =3/10 *100 % of V = 30 % of V So surface area is 30 % of Volume. 27. A reduction of 21 % in the price of wheat enables a person to buy 10 .5 kg more for Rs 100.What is the reduced price per kg. SOLUTION: Let the original price = Rs x/kg Reduced price =79/100x /kg ==> 100/(79x/100)-100/x =10.5 ==> 10000/79x-100/x=10.5 ==> 10000-7900=10.5 * 79 x ==> x= 2100/10.5 *79 So required price = Rs (79/100 *2100/10.5 *79) /kg = Rs 2 per kg. 28.The length of a rectangle is increased by 60 % . By what percent would the width have to be decreased to maintain the same area? SOLUTION: Let the length =l,Breadth= b. Let the required decrease in breadth then 160/100 l *(100-x)/100 b=lb 160(100-x)=100 *100 or 100-x =10000/160 =125/2 so x = 100-125/2 =75/2=37.5 be x %

Back Back To Main

Top

Contact: 040-23000700

file:///E|/work/books/placement/09_Aptitude/percentage.html[1/28/2012 12:45:19 AM]

APTITUDE
Numbers H.C.F and L.C.M Decimal Fractions Simplification Square and Cube roots Average Problems on Numbers Problems on Ages Surds and Indices Percentage Profit and Loss Ratio And Proportions Partnership Chain Rule Time and Work Pipes and Cisterns Time and Distance Trains Boats and Streams Alligation or Mixture Simple Interest Compound Interest Logorithms Areas Volume and Surface area Races and Games of Skill Calendar Clocks Stocks ans Shares True Discount Bankers Discount Oddmanout and Series Data Interpretation probability Permutations and Combinations Puzzles

PERMUTATIONS AND COMBINATIONS


CONCEPT PROBLEMS

file:///E|/work/books/placement/09_Aptitude/percom1.html[1/28/2012 12:45:19 AM]

APTITUDE
Numbers H.C.F and L.C.M Decimal Fractions Simplification Square and Cube roots Average Problems on Numbers Problems on Ages Surds and Indices Percentage Profit and Loss Ratio And Proportions Partnership Chain Rule Time and Work Pipes and Cisterns Time and Distance Trains Boats and Streams Alligation or Mixture Simple Interest Compound Interest Logorithms Areas Volume and Surface area Races and Games of Skill Calendar Clocks Stocks ans Shares True Discount Bankers Discount Oddmanout and Series Data Interpretation probability Permutations and Combinations Puzzles BACK

CONCEPT
Formulae:-> Factorial Notation :- Let n be positive integer.Then ,factorial n dentoed by n! is defined as n! = n(n-1)(n-2). . . . . . . .3.2.1 eg:- 5! = (5 * 4* 3 * 2 * 1) = 120 0! = 1 ->Permutations :- The different arrangements of a given number of things by taking some or all at a time,are called permutations. eg:- All permutations( or arrangements)made with the letters a,b,c by taking two at a time are (ab,ba,ac,ca,bc,cb) ->Numbers of permutations :- Number of all permutations of n things , taken r at a time is given by nPr = n(n-1)(n-2). . .. . . (n-r+1) = n! / (n-r)! ->An Important Result :- If there are n objects of which p1 are alike of one kind ; p2 are alike of another kind ; p3 are alike of third kind and so on and pr are alike of rth kind, such that (p1+p2+. . . . . . . . pr) = n Then,number of permutations of these n objects is: n! / (p1!).(p2!). . . . .(pr!) ->Combinations :- Each of different groups or selections which can be formed by taking some or all of a number of objects,is called a combination. eg:- Suppose we want to select two out of three boys A,B,C . then ,possible selection are AB,BC & CA. Note that AB and BA represent the same selection. -> Number of Combination :- The number of all combination of n things taken r at atime is: nCr Note that : nCn = 1 and nC0 =1 An Important Result : nCr = nC(n-r)
BACK

= n! / (r!)(n-r)! = n(n-1)(n-2). . . . . . . tor factors / r!

file:///E|/work/books/placement/09_Aptitude/percomconcept.html[1/28/2012 12:45:20 AM]

APTITUDE
Numbers H.C.F and L.C.M Decimal Fractions Simplification Square and Cube roots Average Problems on Numbers Problems on Ages Surds and Indices Percentage Profit and Loss Ratio And Proportions Partnership Chain Rule Time and Work Pipes and Cisterns Time and Distance Trains Boats and Streams Alligation or Mixture Simple Interest Compound Interest Logorithms Areas Volume and Surface area Races and Games of Skill Calendar Clocks Stocks ans Shares True Discount Bankers Discount Oddmanout and Series Data Interpretation probability Permutations and Combinations Puzzles BACK

PERCENTAGES

EXAMPLE PROBLEMS:
22. A school has only three classes which contain 40,50,60 students respectively . The pass percent of these classes are 10, 20 and 10 respectively . Then find the pass percent in the school. SOLUTION: Number of passed candidates = 10/100*40+20/100 *50+10/100 * 60 =4+10+6 =20 Total students in school = 40+50+60 =150 So required percentage = 20/150 *100 = 40 /3 =13 1/3 % 23. There are 600 boys in a hostel . Each plays either hockey or football or both .If 75% play hockey and 45 % play football ,Find how many play both? SOLUTION: n(A)=75/100 *600 =450 n(B) = 45/100 *600 = 270 n(A^B)=n(A) + n(B) n(AUB) =450 + 270 -600 =120 So 120 boys play both the games.

24.A bag contains 600 coins of 25p denomination and 1200 coins of 50p denomination. If 12% of 25p coins and 24 % of 50p coins are removed, Find the percentage of money removed from the bag ? SOLUTION: Total money = (600 * 25/100 +1200 *50/100) =Rs 750 25p coins removed = 12/100 *600 =72 50p coins removed = 24/100 *1200 =288 So money removed =72 *1/4 +288 *1/2 = Rs 162 So required percentage=162/750 *100 =21 .6% 25. P is six times as large as Q.Find the percent that Q is less than P? SOLUTION: Given that P= 6Q So Q is less than P by 5Q. Required percentage= 5Q/P*100 % =5/6 * 100 % =83 1/3% 26.For a sphere of radius 10 cm ,the numerical value of surface area is what percent of the numerical value of its volume? SOLUTION: Surface area = 4 *22/7 *r2 = 3/r(4/3 * 22/7 * r3) =3/r * VOLUME Where r = 10 cm So we have S= 3/10 V =3/10 *100 % of V = 30 % of V So surface area is 30 % of Volume. 27. A reduction of 21 % in the price of wheat enables a person to buy 10 .5 kg more for Rs 100.What is the reduced price per kg. SOLUTION: Let the original price = Rs x/kg Reduced price =79/100x /kg ==> 100/(79x/100)-100/x =10.5 ==> 10000/79x-100/x=10.5 ==> 10000-7900=10.5 * 79 x ==> x= 2100/10.5 *79 So required price = Rs (79/100 *2100/10.5 *79) /kg = Rs 2 per kg. 28.The length of a rectangle is increased by 60 % .By what percent would the width have to be decreased to maintain the same area? SOLUTION: Let the length =l,Breadth= b. Let the required decrease in breadth be x % then 160/100 l *(100-x)/100 b=lb 160(100-x)=100 *100 or 100-x =10000/160

file:///E|/work/books/placement/09_Aptitude/percomplex.html[1/28/2012 12:45:20 AM]

=125/2 so x = 100-125/2 =75/2 =37.5

file:///E|/work/books/placement/09_Aptitude/percomplex.html[1/28/2012 12:45:20 AM]

APTITUDE
Numbers H.C.F and L.C.M Decimal Fractions Simplification Square and Cube roots Average Problems on Numbers Problems on Ages Surds and Indices Percentage Profit and Loss Ratio And Proportions Partnership Chain Rule Time and Work Pipes and Cisterns Time and Distance Trains Boats and Streams Alligation or Mixture Simple Interest Compound Interest Logorithms Areas Volume and Surface area Races and Games of Skill Calendar Clocks Stocks ans Shares True Discount Bankers Discount Oddmanout and Series Data Interpretation probability Permutations and Combinations Puzzles BACK

PROBLEMS
1.Evaluate 30!/28! Sol:30!/28! = 30 * 29 * (28!) / (28!) = 30 * 29 =870

2.Find the value of 60P3 Sol:- 60P3 = 60! / (60 3)! = 60! / 57! = (60 * 59 *58 * (57!) )/ 57! = 60 * 59 *58 = 205320 3. Find the value of 100C98 Sol:100C98 50C 50 = 100C100-98) = 100 * 99 / 2 *1 = 4950 50C50 = 1

4.How many words can be formed by using all the letters of the word DAUGHTR so that vowels always come together & vowels are never together? Sol:- (i) Given word contains 8 different letters When the vowels AUE are always together we may suppose them to form an entity ,treated as one letter then the letter to be arranged are DAHTR(AUE) these 6 letters can be arranged in 6p6 = 6! = 720 ways The vowels in the group (AUE) may be arranged in 3! = 6 ways Required number of words = 760 * 6 =4320 (ii)Total number of words formed by using all the letters of the given words 8! = 8 * 7 * 6 * 5 * 4 * 3 * 2 * 1 = 40320 Number of words each having vowels together = 760 * 6 = 4320 Number of words each having vowels never together = 40320 4320 = 36000 5.In how many ways can a cricket eleven be chosen out of a batch of 15 players. Sol:- Required number of ways = 15C 11 = 15C (15-11) = 15 C 4 15C4 = 15 * 14 * 13 * 12 / 4 * 3 * 2 *1 = 1365 6.In how many a committee of 5 members can be selected from 6men 5 ladies consisting of 3 men and 2 ladies Sol:(3 men out of 6) and (2 ladies out of 5) are to be chosen Required number of ways =(6C3 * 5C2) = 200 7.How many 4-letter word with or without meaning can be formed out of the letters of the word 'LOGARITHMS' if repetition of letters is not allowed Sol:'LOGARITHMS' contains 10 different letters Required number of words = Number of arrangements of 100 letters taking 4 at a time = 10P4 = 10 * 9 * 8 * 7 = 5040

file:///E|/work/books/placement/09_Aptitude/percomproblems.html[1/28/2012 12:45:20 AM]

8.In how many ways can the letter of word 'LEADER' be arranged Sol:The word 'LEADER' contains 6 letters namely 1L,2E,1A,1D and 1R Required number of ways = 6! / (1!)(2!)(1!)(1!)(1!) = 6 * 5 * 4 * 3 * 2 *1 / 2 * 1 =360 9.How many arrangements can be made out of the letters of the word 'MATHEMATICS' be arranged so that the vowels always come together Sol:In the word ' MATHEMATICS' we treat vowels AEAI as one letter thus we have MTHMTCS(AEAI) now we have to arrange 8 letters out of which M occurs twice ,T occurs twice & the rest are different Number of ways of arranging these letters = 8! / (2!)(2!) = 10080 now AEAI has 4 letters in which A occurs 2 times and the rest are different Number of ways of arranging these letters = 4! / 2! = 12 Required number of words = (10080 * 12) = 120960 10.In how many different ways can the letter of the word 'DETAIL' be arranged in such a way that the vowels occupy only the odd positions Sol:These are 6 letters in the given word , out of which there are 3 vowels and 3 consonants Let us mark these positions as under (1)(2) (3) (4)(5)(6) now 3 vowels can be placed at any of the three places out of 4 marked 1,3,5 Number of ways of arranging the vowels = 3P3 = 3! =6 Also,the 3 consonants can be arranged at the remaining 3 positions Number of arrangements = 3P3 = 6 Total number of ways = (6 * 6) =36 11.How many 3 digit numbers can be formed from the digits 2,3,5,6,7 and 9 which are divisible by 5 and none of the digits is repeated? Sol:- Since each desired number is divisible by 5, so we much have 5 at the unit place. The hundreds place can now be filled by any of the remaining 4 digits .so, there 4 ways of filling it. Required number of numbers = (1 * 5 * 4) = 20 12.In how many ways can 21 books on English and 19 books on Hindi be placed in a row on a self so that two books on Hindi may not be together? Sol:In order that two books on Hindi are never together, we must place all these books as under: X E X E X . . . . . . . . . . X E X Where E denotes the position of an English and X that of a Hindi book. Since there are 21 books on English,the number of places marked X are therefore 22. Now, 19 places out of 22 can be chosen in 22 C 19 = 22 C 3 =22 * 21 * 20 / 3 * 2 *1 Hence the required number of ways = 1540 13.Out of 7 constants and 4 vowels how many words of 3 consonants and 2 vowels can be formed? Sol:Number of ways of selecting (3 consonants out of 7) and (2 vowels out of 4) = 7C3 * 4C2 = 210 Number of groups each having 3 consonants and 2 vowels = 210 Each group contains 5 letters Number of ways of arranging 5 letters among themselves = 5! = (5 * 4 * 3 * 2 * 1) = 210 Required number of words = (210 * 210)

file:///E|/work/books/placement/09_Aptitude/percomproblems.html[1/28/2012 12:45:20 AM]

= 25200
BACK

file:///E|/work/books/placement/09_Aptitude/percomproblems.html[1/28/2012 12:45:20 AM]

APTITUDE
Numbers H.C.F and L.C.M Decimal Fractions Simplification Square and Cube roots Average Problems on Numbers Problems on Ages Surds and Indices Percentage Profit and Loss Ratio And Proportions Partnership Chain Rule Time and Work Pipes and Cisterns Time and Distance Trains Boats and Streams Alligation or Mixture Simple Interest Compound Interest Logorithms Areas Volume and Surface area Races and Games of Skill Calendar Clocks Stocks ans Shares True Discount Bankers Discount Oddmanout and Series Data Interpretation probability Permutations and Combinations Puzzles BACK

PERCENTAGES
By a certain percent, we mean that many hundredths. Thus x per cent means x hundredths written as x%. So x% = x/100. EXAMPLE: 1.20% = 20/100 = 1/5 2.48% = 48/100 = 12/25 To express a/b as a percent ,we have

a/b=(a/b*100)%

Thus 1/4=(1/4*100)%=25% 0.6=6/10=(3/5*100)%=60% If the price of commodity increases by r%,then the reduction in consumption so as not to increase the expenditure is given as follows [r/(100+r)*100]% If the price of commodity decreased by r%,then the increase in consumption so as not to decrease the expenditure is given as follows [r/(100-r)*100]%

RESULT ON POPULATION:
Let the population of a town be P now and suppose it increases at the rate R% per annum then i)Population after n years = P[1+(R/100)]n ii) Population n years ago= P/[1+(R/100)]n

RESULT ON DEPRECIATION:
Let the present value of a machine be P. Suppose it depriciates at the rate of R% per annum, then i)Value of machine after n years = P[1-(R/100)]n ii)Value of machine after n years = P/[1-(R/100)]n

IMPORTANT
i) If A is R% more than B,then B is less than A by [(R/(100+R))*100]% A is R% less than B,then B is more than A by [(R/(100-R))*100]%

ii)If

file:///E|/work/books/placement/09_Aptitude/percont.html[1/28/2012 12:45:21 AM]

APTITUDE
Numbers H.C.F and L.C.M Decimal Fractions Simplification Square and Cube roots Average Problems on Numbers Problems on Ages Surds and Indices Percentage Profit and Loss Ratio And Proportions Partnership Chain Rule Time and Work Pipes and Cisterns Time and Distance Trains Boats and Streams Alligation or Mixture Simple Interest Compound Interest Logorithms Areas Volume and Surface area Races and Games of Skill Calendar Clocks Stocks ans Shares True Discount Bankers Discount Oddmanout and Series Data Interpretation probability Permutations and Combinations Puzzles BACK

PERCENTAGES

EXAMPLE PROBLEMS:
13. Difference of two numbers is 1660.If 7.5 % of one number is 12.5% of the other number. Find two numbers? SOLUTION: Let the two numbers be x and y. 7.5% of x=12.5% of y So 75x=125 y 3x=5y x=5/3y. Now x-y=1660 5/3y-y=1660 2/3y=1660 y=2490 So x= 2490+1660 =4150. So the numbers are 4150 , 1660. 14. In expressing a length 81.472 KM as nearly as possible with 3 significant digits ,Find the % error? SOLUTION: Error= 81.5-81.472=0.028 So the required percentage = 0.028/81.472*100% = 0.034% 15. In an election between two persons ,75% of the voters cast their votes out of which 2% are invalid. A got 9261 which 75% of the total valid votes. Find total number of votes? SOLUTION: Let x be the total votes. valid votes are 98% of 75% of x. So 75%(98%(75% of x))) = 9261 ==> 75/100 *98 /100 * 75 100 *x = 9261 x= 1029 * 4 *100 *4 / 9 = 16800 So total no of votes = 16800 16 . A's maths test had 75 problems i.e 10 arithmetic, 30 algebra and 35 geometry problems. Although he answered 70% of arithmetic , 40% of algebra and 60 % of geometry problems correctly he didn't pass the test because he got less than 60% of the problems right. How many more questions he would have needed to answer correctly to get a 60% passing grade. SOLUTION: 70% of 10 =70/100 * 10 =7 40% of 30 = 40 / 100 * 30 = 12 60 % of 35 = 60 / 100 *35 = 21 So correctly attempted questions = 7 + 12 + 21 =40. Questions to be answered correctly for 60% grade =60% of 75 = 60/100 *75 =45. So required questions=45-40 = 5 17 . If 50% of (x y) = 30% of (x + y) then what percent of x is y ? SOLUTION: 50/100(x-y) =30/100(x+y) (x-y)= 3/10(x+y) 5x-5y=3x+3y x=4y So Required percentage =y/x*100 % =y/4y *100 % = 25%. 18 . If the price of tea is increased by 20% ,find how much percent must a householder reduce her consumption of tea so as not to increase the expenditure? SOLUTION: Reduction in consumption= R/(100+R) *100% =20/120 *100 = 16 2/3 % 19.The population of a town is 176400 . If it increases at the rate of 5% per annum ,what will be the population 2 years hence? What was it 2 years ago? SOLUTION: Population After 2 years = 176400[1+5/100]2 =176400 * 21/20 *21/20 =194481

file:///E|/work/books/placement/09_Aptitude/permedium.html[1/28/2012 12:45:21 AM]

Population 2 years ago = 176400/(1+5/100)2 = 176400 * 20/21 *20/ 21 =160000 20.1 liter of water is add to 5 liters of a 20 % solution of alcohol in water . Find the strength of alcohol in new solution? SOLUTION: Alcohol in 5 liters = 20% of 5 =1 liter Alcohol in 6 liters of new mixture = 1liter So % of alcohol is =1/6 *100=16 2/3% 21. If A earns 33 1/3 more than B .Then B earns less than A by what percent? SOLUTION: 33 1/3 =100 / 3 Required Percentage = (100/3)/(100 + (100/3)) *100 % = 100/400 *100 = 25 %

file:///E|/work/books/placement/09_Aptitude/permedium.html[1/28/2012 12:45:21 AM]

Permutations and Combinations


Formulae: Factorial Notation: Let n be positive integer.Then ,factorial n dentoed by n! is defined as n! = n(n-1)(n-2). . . . . . . .3.2.1 eg:- 5! = (5 * 4* 3 * 2 * 1) = 120 0! = 1 Permutations: The different arrangements of a given number of things by taking some or all at a time,are called permutations. eg:- All permutations( or arrangements)made with the letters a,b,c by taking two at a time are (ab,ba,ac,ca,bc,cb) Numbers of permutations: Number of all permutations of n things, taken r at a time is given by nPr = n(n-1)(n-2). . .. . . (n-r+1) = n! / (n-r)! An Important Result: If there are n objects of which p1 are alike of one kind; p2 are alike of another kind ; p3 are alike of third kind and so on and pr are alike of rth kind, such that (p1+p2+. . . . . . . . pr) = n Then,number of permutations of these n objects is: n! / (p1!).(p2!). . . . .(pr!) Combinations: Each of different groups or selections which can be formed by taking some or all of a number of objects,is called a combination. eg:- Suppose we want to select two out of three boys A,B,C . then ,possible selection are AB,BC & CA. Note that AB and BA represent the same selection. Number of Combination: The number of all combination of n things taken r at atime is: nCr = n! / (r!)(n-r)! = n(n-1)(n-2). . . . . . . tor factors / r! Note: nCn = 1 and nC0 =1 An Important Result: nCr = nC(n-r)

Top
Problems 1.Evaluate 30!/28! Sol:30!/28! = 30 * 29 * (28!) = 30 * 29 =870 / (28!)

2.Find the value of 60P3 Sol:- 60P3 = = = = 60! / (60 3)! = 60! / 57! (60 * 59 *58 * (57!) )/ 57! 60 * 59 *58 205320

3. Find the value of 100C98,50C 50 Sol:100C98 = 100C100-98) = 100 * 99 / 2 *1 = 4950 50C50 = 1

4.How many words can be formed by using all the letters of the word DAUGHTR so that vowels always come together & vowels are never together? Sol:(i) Given word contains 8 different letters When the vowels AUE are always together we may suppose them to form an entity ,treated as one letter then the letter to be arranged are DAHTR(AUE) these 6 letters can be arranged in 6p6 = 6! = 720 ways The vowels in the group (AUE) may be arranged in 3! = 6 ways Required number of words = 760 * 6 =4320 (ii)Total number of words formed by using all the letters of the given words 8! = 8 * 7 * 6 * 5 * 4 * 3 * 2 * 1 = 40320 Number of words each having vowels together = 760 * 6 = 4320 Number of words each having vowels never together

file:///E|/work/books/placement/09_Aptitude/permutationsandcombinations.html[1/28/2012 12:45:21 AM]

= 40320 4320 = 36000 5.In how many ways can a cricket eleven be chosen out of a batch of 15 players. Sol:- Required number of ways = 15C 11 = 15C (15-11) = 15 C 4 15C4 = 15 * 14 * 13 * 12 / 4 * 3 * 2 *1 = 1365 6.In how many a committee of 5 members can be selected from 6men 5 ladies consisting of 3 men and 2 ladies Sol:(3 men out of 6) and (2 ladies out of 5) are to be chosen Required number of ways =(6C3 * 5C2) = 200

7.How many 4-letter word with or without meaning can be formed out of the letters of the word 'LOGARITHMS' if repetition of letters is not allowed Sol:'LOGARITHMS' contains 10 different letters Required number of words = Number of arrangements of 100 letters taking 4 at a time = 10P4 = 10 * 9 * 8 * 7 = 5040

8.In how many ways can the letter of word 'LEADER' be arranged Sol:The word 'LEADER' contains 6 letters namely 1L,2E,1A,1D and 1R Required number of ways = 6! / (1!)(2!)(1!)(1!)(1!) = 6 * 5 * 4 * 3 * 2 *1 / 2 * 1 =360

Top
9.How many arrangements can be made out of the letters of the word 'MATHEMATICS' be arranged so that the vowels always come together Sol:In the word ' MATHEMATICS' we treat vowels AEAI as one letter thus we have MTHMTCS(AEAI) now we have to arrange 8 letters out of which M occurs twice ,T occurs twice & the rest are different Number of ways of arranging these letters = 8! / (2!)(2!) = 10080 now AEAI has 4 letters in which A occurs 2 times and the rest are different Number of ways of arranging these letters = 4! / 2! = 12 Required number of words = (10080 * 12) = 120960 10.In how many different ways can the letter of the word 'DETAIL' be arranged in such a way that the vowels occupy only the odd positions Sol:These are 6 letters in the given word , out of which there are 3 vowels and 3 consonants Let us mark these positions as under

(1)(2) (3) (4)(5)(6) now 3 vowels can be placed at any of the three places out of 4 marked 1,3,5 Number of ways of arranging the vowels = 3P3 = 3! =6 Also,the 3 consonants can be arranged at the remaining 3 positions Number of arrangements = 3P3 = 6 Total number of ways = (6 * 6) =36 11.How many 3 digit numbers can be formed from the digits 2,3,5,6,7 and 9 which are divisible by 5 and none of the digits is repeated? Sol:- Since each desired number is divisible by so we much have 5 at the unit place. can now be filled by any of the remaining 4 4 ways of filling it. Required number of numbers = (1 * 5 * = 20 12.In how many ways can 21 books on English and be placed in a row on a self so that two books be together? Sol:5, The hundreds place digits .so, there 4) 19 books on Hindi on Hindi may not

In order that two books on Hindi are never together, we must place all these books as under: X E X E X . . . . . . . . . . X E X Where E denotes the position of an English and X that of a Hindi book. Since there are 21 books on English,the number of places marked X are therefore 22. Now, 19 places out of 22 can be chosen in 22 C 19 = 22 C 3 =22 * 21 * 20 / 3 * 2 *1 Hence the required number of ways = 1540

13.Out of 7 constants and 4 vowels how many words of 3 consonants and 2 vowels can be formed? Sol:Number of ways of selecting (3 consonants out of 7) and (2 vowels out of 4) = 7C3 * 4C2 = 210 Number of groups each having 3 consonants and 2 vowels = 210 Each group contains 5 letters

file:///E|/work/books/placement/09_Aptitude/permutationsandcombinations.html[1/28/2012 12:45:21 AM]

Number of ways of arranging 5 letters among themselves = 5! = (5 * 4 * 3 * 2 * 1) = 210 Required number of words = (210 * 210) = 25200

Back Back To Main

Top

Contact: 040-23000700

file:///E|/work/books/placement/09_Aptitude/permutationsandcombinations.html[1/28/2012 12:45:21 AM]

APTITUDE
Numbers H.C.F and L.C.M Decimal Fractions Simplification Square and Cube roots Average Problems on Numbers Problems on Ages Surds and Indices Percentage Profit and Loss Ratio And Proportions Partnership Chain Rule Time and Work Pipes and Cisterns Time and Distance Trains Boats and Streams Alligation or Mixture Simple Interest Compound Interest Logorithms Areas Volume and Surface area Races and Games of Skill Calendar Clocks Stocks ans Shares True Discount Bankers Discount Oddmanout and Series Data Interpretation probability Permutations and Combinations Puzzles BACK

PERCENTAGES
EXAMPLE PROBLEMS:

EXAPLE PROBLEMS:

1 . Express the following as a fraction. a) 56% SOLUTION: 56/100=14/25 b) 4% SOLUTION: 4/100=1/25 c) 0.6% SOLUTION: 0.6/100=6/1000=3/500 d) 0.08% SOLUTION: 0.08/100=8/10000=1/1250 2.Express the following as decimals a) 6% SOLUTION: 6% = 6/100=0.06 b) 0.04% SOLUTION: 0.04% = 0.04/100=0.0004 3 . Express the following as rate percent. i).23/36 SOLUTION: = (23/36*100) % = 63 8/9% ii).6 SOLUTION: 6 =27/4 (27/4 *100) % =675 % 4.Evaluate the following: 28% of 450 + 45% of 280 ? SOLUTION: =(28/100) *450 + (45/100) *280 = 28 * 45 / 5 = 252 5.2 is what percent of 50? SOLUTION: Formula : (IS / OF ) *100 % = 2/50 *100 = 4% 6. is what percent of 1/3? SOLUTION: =( ) / (1/3) *100 % = 3/2 *100 % = 150 % 7.What percent of 2 Metric tonnes is 40 Quintals? SOLUTION: 1 metric tonne =10 Quintals So required percentage = (40/(2*10)) *100 % = 200% 8.Find the missing figure . i) ? % of 25 = 2.125 SOLUTION : Let x% of 25 = 2.125. then (x/100) *25 =2.125 x = 2.125 * 4 = 8.5 ii) 9% of ? =6.3 SOLUTION: Let 9 % of x = 6.3. Then 9/100 of x= 6.3 so x = 6.3 *100/7 = 70. 9.Which is the greatest in 16 2/3 %, 2/15,0.17? SOLUTION: 16 2/3 % = 50/3 %

file:///E|/work/books/placement/09_Aptitude/persimple.html[1/28/2012 12:45:22 AM]

=50/3 * 1/100 =1/6 = 0.166 2 / 15 =0.133 So 0.17 is greatest number in the given series. 10.If the sales tax be reduced from 3 % to 3 1/3 % ,then what difference does it make to a person who purchases an article with marked price of RS 8400? SOLUTION: Required difference = 3 % of 8400 3 1/3 % of 8400 =(7/2-10/3)% of 8400 =1/6 % of 8400 = 1/6* 1/100* 8400 = Rs 14. 11. A rejects 0.08% of the meters as defective .How many will he examine to reject 2? SOLUTION: Let the number of meters to be examined be x. Then 0.08% of x=2. 0.08/100*x= 2 x= 2 * 100/0.08 =2 * 100 * 100/8 = 2500 12.65 % of a number is 21 less than 4/5 of that number. What is the number? SOLUTION: Let the number be x. 4/5 x- (65% of x) = 21 4/5x 65/100 x=21 15x=2100 x=140

>

file:///E|/work/books/placement/09_Aptitude/persimple.html[1/28/2012 12:45:22 AM]

APTITUDE
Numbers H.C.F and L.C.M Decimal Fractions Simplification Square and Cube roots Average Problems on Numbers Problems on Ages Surds and Indices Percentage Profit and Loss Ratio And Proportions Partnership Chain Rule Time and Work Pipes and Cisterns Time and Distance Trains Boats and Streams Alligation or Mixture Simple Interest Compound Interest Logorithms Areas Volume and Surface area Races and Games of Skill Calendar Clocks Stocks ans Shares True Discount Bankers Discount Oddmanout and Series Data Interpretation probability Permutations and Combinations Puzzles

PUZZLES
PUZZLES REASONING

file:///E|/work/books/placement/09_Aptitude/pinkivijji_puzzles.html[1/28/2012 12:45:22 AM]

Pipes and Cisterns


Important Facts: 1.INLET:A pipe connected with a tank or cistern or a reservoir, that fills it, it is known as Inlet. OUTLET:A pipe connected with a tank or a cistern or a reservoir, emptying it, is known as Outlet. 2. i) If a pipe can fill a tank in x hours, then : part filled in 1 hour=1/x. ii)If a pipe can empty a tank in y hours, then : part emptied in 1 hour=1/y. iii)If a pipe can fill a tank in x hours and another pipe can empty the full tank in y hours( where y>x), then on opening both the pipes, the net part filled in 1 hour=(1/x -1/y). iv)If a pipe can fill a tank in x hours and another pipe can empty the full tank in y hours( where x>y), then on opening both the pipes, the net part filled in 1 hour=(1/y -1/x). v) If two pipes A and B can fill a tank in x hours and y hours respectively. If both the pipes are opened simultaneously, part filled by A+B in 1 hour= 1/x +1/y.

Top
Simple Problems 1)Two pipes A& B can fill a tank in 36 hours and 45 hours respectively. If both the pipes are opened simultaneously, how much time will be taken to fill the tank? Sol: Part filled Part filled Part filled Hence, both the by A in 1 hour=1/36 by B in 1 hour= 1/45; by (A+B)'s in 1 hour=1/36 +1/45= 9/180 =1/20 pipes together will fill the tank in 20 hours.

2)Two pipes can fill a tank in 10 hours & 12 hours respectively. While 3rd pipe empties the full tank n 20 hours. If all the three pipes operate simultaneously,in how much time will the tank be filled? Sol: Net part filled in 1 hour=1/10 +1/12 -1/20 =8/60=2/15 The tank be filled in 15/2hours= 7 hrs 30 min

3)A cistern can be filled by a tap in 4 hours while it can be emptied by another tap in 9 hours. If both the taps are opened simultaneously, then after how much time will the cistern get filled? Sol: Net part filled in 1 hour= 1/4 -1/9= 5/36 Therefore the cistern will be filled in 36/5 hours or 7.2 hours.

4)If two pipes function simultaneously, the reservoir will be filled in 12 days.One pipe fills the reservoir 10 hours faster than the other. How many hours does it take the second pipe to fill the reservoir. Sol: Let the reservoir be filled by the 1st pipe in x hours. The second pipe will fill it in (x+10) hours 1/x + (1/(x+10))= 1/12 => (2x+10)/((x)*(x+10))= 1/12 => x=20 So, the second pipe will take 30 hours to fill the reservoir.

5)A cistern has two taps which fill it in 12 min and 15 min respectively. There is also a waste pipe in the cistern. When all the three are opened, the empty cistern is full in 20 min. How long will the waste pipe take to empty the full cistern? Sol: Work done by a waste pipe in 1 min =1/20 -(1/12+1/15)= -1/10 (-ve means emptying)

6)A tap can fill a tank in 6 hours. After half the tank is filled, three more similar taps are opened. What is the total time taken to fill the tank completely? Sol: Time taken by one tap to fill the half of the tank =3 hours Part filled by the four taps in 1 hour=4/6=2/3 Remaining part=1 -1/2=1/2 Therefore, 2/3:1/2::1:x or x=(1/2)*1*(3/2)=3/4 hours. i.e 45 min So, total time taken= 3hrs 45min.

7)A water tank is two-fifth full. Pipe A can fill a tank in 10 min. And B can empty it in 6 min. If both pipes are open, how long will it take to empty or fill the tank completely ? Sol: Clearly, pipe B is faster than A and So, the tank will be emptied. Part to be emptied=2/5. Part emptied by (A+B) in 1 min= 1/6 -1/10=1/15

file:///E|/work/books/placement/09_Aptitude/pipesandcisterns.html[1/28/2012 12:45:22 AM]

Therefore, 1/15:2/5::1:x or x=((2/5)*1*15)=6 min. So, the tank be emptied in 6 min. 8)Bucket P has thrice the capacity as Bucket Q. It takes 60 turns for Bucket P to fill the empty drum. How many turns it will take for both the buckets P&Q, having each turn together to fill the empty drum? Sol: Let the capacity of P be x lit. Then capacity of Q=x/3 lit Capacity of the drum=60x lit Required number of turns= 60x/(x+(x/3))= 60x*3/4x=45

Top
Complex Problems 1)Two pipes can fill a cistern in 14 hours and 16 hours respectively. The pipes are opened simultaneously and it is found that due to leakage in the bottom it took 32min more to fill the cistern. When the cistern is full, in what time will the leak empty it? Sol: Work done by the two pipes in 1 hour= 1/14+1/16=15/112 Time taken by these two pipes to fill the tank=112/15 hrs. Due to leakage, time taken = 7 hrs 28 min+ 32 min= 8 hours Therefore, work done by (two pipes + leak) in 1 hr= 1/8 work done by leak n 1 hour=15/112 -1/8=1/112 Leak will empty full cistern n 112 hours.

2)Two pipes A&B can fill a tank in 30 min. First, A&B are opened. After 7 min, C also opened. In how much time, the tank s full. Sol: Part filled n 7 min = 7*(1/36+1/45)=7/20 Remaining part= 1-7/20=13/20 Net part filled in 1 min when A,B and C are opened=1/36 +1/45- 1/30=1/60 Now, 1/60 part is filled in 1 min. 13/20 part is filled n (60*13/20)=39 min Total time taken to fill the tank=39+7=46 min

3)Two pipes A&B can fill a tank in 24 min and 32 min respectively. If both the pipes are opened simultaneously, after how much time B should be closed so that the tank is full in 18 min. Sol: Let B be closed after x min, then part filled by (A+B) in x min+ part filled by A in (18-x) min=1 x(1/24+1/32) +(18-x)1/24 =1 => x=8 Hence B must be closed after 8 min.

4)Two pipes A& B together can fill a cistern in 4 hours. Had they been opened separately, then B would have taken 6 hours more than A to fill the cistern. How much time will be taken by A to fill the cistern separately? Sol: Let the cistern be filled by pipe A alone in x hours. Pipe B will fill it in x+6 hours 1/x + 1/x+6=1/4 Solving this we get x=6. Hence, A takes 6 hours to fill the cistern separately.

5)A tank is filled by 3 pipes with uniform flow. The first two pipes operating simultaneously fill the tan in the same time during which the tank is filled by the third pipe alone. The 2nd pipe fills the tank 5 hours faster than first pipe and 4 hours slower than third pipe. The time required by first pipe is : Sol: Suppose, first pipe take x hours to fill the tank then B & C will take (x-5) and (x-9) hours respectively. Therefore, 1/x +1/(x-5) =1/(x-9) On solving, x=15 Hence, time required by first pipe is 15 hours.

Top
6)A large tanker can be filled by two pipes A& B in 60min and 40 min respectively. How many minutes will it take to fill the tanker from empty state if B is used for half the time & A and B fill it together for the other half? Sol: Part filled by (A+B) n 1 min=(1/60 +1/40)=1/24 Suppose the tank is filled in x minutes Then, x/2(1/24+1/40)=1 => (x/2)*(1/15)=1 => x=30 min.

7)Two pipes A and B can fill a tank in 6 hours and 4 hours respectively. If they are opened on alternate hours and if pipe A s opened first, in how many hours, the tank shall be full. Sol: (A+B)'s 2 hours work when opened alternatively =1/6+1/4 =5/12 (A+B)'s 4 hours work when opened alternatively=10/12=5/6 Remaining part=1 -5/6=1/6. Now, it is A's turn and 1/6 part is filled by A in 1 hour. So, total time taken to fill the tank=(4+1)= 5 hours.

8)Three taps A,B and C can fill a tank in 12, 15 and 20 hours respectively. If A is open all the time and B and C are open for one hour each alternatively, the tank will be full in. Sol: (A+B)'s 1 hour's work=1/12+1/15=9/60=3/20 (A+C)'s 1 hour's work=1/20+1/12=8/60=2/15 Part filled in 2 hours=3/20+2/15=17/60 Part filled in 2 hours=3/20+2/15= 17/60 Part filled in 6 hours=3*17/60 =17/20 Remaining part=1 -17/20=3/20 Now, it is the turn of A & B and 3/20 part is filled by A& B in 1 hour. Therefore, total time taken to fill the tank=6+1=7 hours filling m3. The filling than it as well as for emptying a tank. emptying capacity of the tank is capacity and the pump needs 8 needs to fill it. What is the

9)A Booster pump can be used for The capacity of the tank is 2400 10 m3 per minute higher than its minutes lesser to empty the tank filling capacity of the pump?

file:///E|/work/books/placement/09_Aptitude/pipesandcisterns.html[1/28/2012 12:45:22 AM]

Sol:

Let, the filling capacity of the pump be x m3/min Then, emptying capacity of the pump=(x+10) m3/min. So,2400/x 2400/(x+10) = 8 on solving x=50.

10)A leak in the bottom of a tank can empty the full tan in 8 hr. An inlet pipe fills water at the rate of 6 lits a minute. When the tank is full, the inlet is opened and due to the leak, the tank is empty in 12 hrs. How many liters does the cistern hold? Sol: Work done by the inlet in 1 hr= 1/8 -1/12=1/24 Work done by the inlet in i min= (1/24)*(1/60)=1/1440 Therefore, Volume of 1/1440 part=6 lit Volume of whole=(1440*6) lit=8640 lit.

11)Two pipes A and B can fill a cistern in 37 min and 45 minutes respectively. Both the pipes are opened. The cistern will be filled in just half an hour, if the pipe B is turned off after: sol: Let B be turned off after x min. Then, Part filled by (A+B) in x min+ part filled by A in (30-x)min=1 Therefore, x(2/75+1/45)+(30-x)(2/75)=1 11x/225 + (60-2x)/75=1 11x+ 180-6x=225 x=9. So, B must be turned off after 9 minutes.

Back Back To Main

Top

Contact: 040-23000700

file:///E|/work/books/placement/09_Aptitude/pipesandcisterns.html[1/28/2012 12:45:22 AM]

APTITUDE
Numbers H.C.F and L.C.M Decimal Fractions Simplification Square and Cube roots Average Problems on Numbers Problems on Ages Surds and Indices Percentage Profit and Loss Ratio And Proportions Partnership Chain Rule Time and Work Pipes and Cisterns Time and Distance Trains Boats and Streams Alligation or Mixture Simple Interest Compound Interest Logorithms Areas Volume and Surface area Races and Games of Skill Calendar Clocks Stocks ans Shares True Discount Bankers Discount Oddmanout and Series Data Interpretation probability Permutations and Combinations Puzzles BACK

COMPLEX PROBLEMS
1)Two pipes can fill a cistern in 14 hours and 16 hours respectively. The pipes are opened simultaneously and it is found that due to leakage in the bottom it took 32min more to fill the cistern. When the cistern is full, in what time will the leak empty it? Sol: Work done by the two pipes in 1 hour= 1/14+1/16=15/112 Time taken by these two pipes to fill the tank=112/15 hrs. Due to leakage, time taken = 7 hrs 28 min+ 32 min= 8 hours Therefore, work done by (two pipes + leak) in 1 hr= 1/8 work done by leak n 1 hour=15/112 -1/8=1/112 Leak will empty full cistern n 112 hours.

2)Two pipes A&B can fill a tank in 30 min. First, A&B are opened. After 7 min, C also opened. In how much time, the tank s full. Sol: Part filled n 7 min = 7*(1/36+1/45)=7/20 Remaining part= 1-7/20=13/20 Net part filled in 1 min when A,B and C are opened=1/36 +1/451/30=1/60 Now, 1/60 part is filled in 1 min. 13/20 part is filled n (60*13/20)=39 min Total time taken to fill the tank=39+7=46 min 3)Two pipes A&B can fill a tank in 24 min and 32 min respectively. If both the pipes are opened simultaneously, after how much time B should be closed so that the tank is full in 18 min. Sol: Let B be closed after x min, then part filled by (A+B) in x min+ part filled by A in (18-x) min=1 x(1/24+1/32) +(18-x)1/24 =1 => x=8 Hence B must be closed after 8 min.

4)Two pipes A& B together can fill a cistern in 4 hours. Had they been opened separately, then B would have taken 6 hours more than A to fill the cistern. How much time will be taken by A to fill the cistern separately? Sol: Let the cistern be filled by pipe A alone in x hours. Pipe B will fill it in x+6 hours 1/x + 1/x+6=1/4 Solving this we get x=6. Hence, A takes 6 hours to fill the cistern separately. 5)A tank is filled by 3 pipes with uniform flow. The first two pipes operating simultaneously fill the tan in the same time during which the tank is filled by the third pipe alone. The 2nd pipe fills the tank 5 hours faster than first pipe and 4 hours slower than third pipe. The time required by first pipe is : Sol: Suppose, first pipe take x hours to fill the tank then B & C will take (x-5) and (x-9) hours respectively. Therefore, 1/x +1/(x-5) =1/(x-9) On solving, x=15 Hence, time required by first pipe is 15 hours. 6)A large tanker can be filled by two pipes A& B in 60min and 40 min respectively. How many minutes will it take to fill the tanker from empty state if B is used for half the time & A and B fill it together for the other half? Sol: Part filled by (A+B) n 1 min=(1/60 +1/40)=1/24 Suppose the tank is filled in x minutes Then, x/2(1/24+1/40)=1 => (x/2)*(1/15)=1 => x=30 min.

7)Two pipes A and B can fill a tank in 6 hours and 4 hours respectively. If they are opened on alternate hours and if pipe A s opened first, in how many hours, the tank shall be full. Sol: (A+B)'s 2 hours work when opened alternatively =1/6+1/4 =5/12

file:///E|/work/books/placement/09_Aptitude/pipesandcisternscomplex.html[1/28/2012 12:45:23 AM]

(A+B)'s 4 hours work when opened alternatively=10/12=5/6 Remaining part=1 -5/6=1/6. Now, it is A's turn and 1/6 part is filled by A in 1 hour . So, total time taken to fill the tank=(4+1)= 5 hours. 8)Three taps A,B and C can fill a tank in 12, 15 and 20 hours respectively. If A is open all the time and B and C are open for one hour each alternatively, the tank will be full in. Sol: (A+B)'s 1 hour's work=1/12+1/15=9/60=3/20 (A+C)'s 1 hour's work=1/20+1/12=8/60=2/15 Part filled in 2 hours=3/20+2/15=17/60 Part filled in 2 hours=3/20+2/15= 17/60 Part filled in 6 hours=3*17/60 =17/20 Remaining part=1 -17/20=3/20 Now, it is the turn of A & B and 3/20 part is filled by A& B in 1 Therefore, total time taken to fill the tank=6+1=7 hours 9)A Booster pump can be used for The capacity of the tank is 2400 10 m3 per minute higher than its lesser to empty the tank than it of the pump? Sol: filling as well as for emptying a tank. m3. The emptying capacity of the tank is filling capacity and the pump needs 8 minutes needs to fill it. What is the filling capacity

hour.

Let, the filling capacity of the pump be x m3/min Then, emptying capacity of the pump=(x+10) m3/min. So,2400/x 2400/(x+10) = 8 on solving x=50.

10)A leak in the bottom of a tank can empty the full tan in 8 hr. An inlet pipe fills water at the rate of 6 lits a minute. When the tank is full, the inlet is opened and due to the leak, the tank is empty in 12 hrs. How many liters does the cistern hold? Sol: Work done by the inlet in 1 hr= 1/8 -1/12=1/24 Work done by the inlet in i min= (1/24)*(1/60)=1/1440 Therefore, Volume of 1/1440 part=6 lit Volume of whole=(1440*6) lit=8640 lit.

11)Two pipes A and B can fill a cistern in 37 min and 45 minutes respectively. Both the pipes are opened. The cistern will be filled in just half an hour, if the pipe B is turned off after: sol: Let B be turned off after x min. Then, Part filled by (A+B) in x min+ part filled by A in (30-x)min=1 Therefore, x(2/75+1/45)+(30-x)(2/75)=1 11x/225 + (60-2x)/75=1 11x+ 180-6x=225 x=9. So, B must be turned off after 9 minutes.

BACK

file:///E|/work/books/placement/09_Aptitude/pipesandcisternscomplex.html[1/28/2012 12:45:23 AM]

APTITUDE
Numbers H.C.F and L.C.M Decimal Fractions Simplification Square and Cube roots Average Problems on Numbers Problems on Ages Surds and Indices Percentage Profit and Loss Ratio And Proportions Partnership Chain Rule Time and Work Pipes and Cisterns Time and Distance Trains Boats and Streams Alligation or Mixture Simple Interest Compound Interest Logorithms Areas Volume and Surface area Races and Games of Skill Calendar Clocks Stocks ans Shares True Discount Bankers Discount Oddmanout and Series Data Interpretation probability Permutations and Combinations Puzzles BACK

PIPES AND CISTERNS


1.INLET:A pipe connected with a tank or cistern or a reservoir, that fills it, it is known as Inlet. OUTLET:A pipe connected with a tank or a cistern or a reservoir, emptying it, is known as Outlet. 2. i) If a pipe can fill a tank in x hours, then : part filled in 1 hour=1/x. ii)If a pipe can empty a tank in y hours, then : part emptied in 1 hour=1/y. iii)If a pipe can fill a tank in x hours and another pipe can empty the full tank in y hours( where y>x), then on opening both the pipes, the net part filled in 1 hour=(1/x -1/y). iv)If a pipe can fill a tank in x hours and another pipe can empty the full tank in y hours( where x>y), then on opening both the pipes, the net part filled in 1 hour=(1/y -1/x). v) If two pipes A and B can fill a tank in x hours and y hours respectively. If both the pipes are opened simultaneously, part filled by A+B in 1 hour= 1/x +1/y.
BACK

file:///E|/work/books/placement/09_Aptitude/pipesandcisternsconcept.html[1/28/2012 12:45:23 AM]

APTITUDE
Numbers H.C.F and L.C.M Decimal Fractions Simplification Square and Cube roots Average Problems on Numbers Problems on Ages Surds and Indices Percentage Profit and Loss Ratio And Proportions Partnership Chain Rule Time and Work Pipes and Cisterns Time and Distance Trains Boats and Streams Alligation or Mixture Simple Interest Compound Interest Logorithms Areas Volume and Surface area Races and Games of Skill Calendar Clocks Stocks ans Shares True Discount Bankers Discount Oddmanout and Series Data Interpretation probability Permutations and Combinations Puzzles BACK

SIMPLE PROBLEMS
1)Two pipes A& B can fill a tank in 36 hours and 45 hours respectively. If both the pipes are opened simultaneously, how much time will be taken to fill the tank? Sol: Part filled Part filled Part filled Hence, both by A in 1 hour=1/36 by B in 1 hour= 1/45; by (A+B)'s in 1 hour=1/36 +1/45= 9/180 =1/20 the pipes together will fill the tank in 20 hours.

2)Two pipes can fill a tank in 10 hours & 12 hours respectively. While 3rd pipe empties the full tank n 20 hours. If all the three pipes operate simultaneously,in how much time will the tank be filled? Sol: Net part filled in 1 hour=1/10 +1/12 -1/20 =8/60=2/15 The tank be filled in 15/2hours= 7 hrs 30 min

3)A cistern can be filled by a tap in 4 hours while it can be emptied by another tap in 9 hours. If both the taps are opened simultaneously, then after how much time will the cistern get filled? Sol: Net part filled in 1 hour= 1/4 -1/9= 5/36 Therefore the cistern will be filled in 36/5 hours or 7.2 hours.

4)If two pipes function simultaneously, the reservoir will be filled in 12 days.One pipe fills the reservoir 10 hours faster than the other. How many hours does it take the second pipe to fill the reservoir. Sol: => => Let the reservoir be filled by the 1st pipe in x hours. The second pipe will fill it in (x+10) hours 1/x + (1/(x+10))= 1/12 (2x+10)/((x)*(x+10))= 1/12 x=20 So, the second pipe will take 30 hours to fill the reservoir.

5)A cistern has two taps which fill it in 12 min and 15 min respectively. There is also a waste pipe in the cistern. When all the three are opened, the empty cistern is full in 20 min. How long will the waste pipe take to empty the full cistern? Sol: Work done by a waste pipe in 1 min=1/20 -(1/12+1/15)= -1/10 (-ve means emptying) 6)A tap can fill a tank in 6 hours. After half the tank is filled, three more similar taps are opened. What is the total time taken to fill the tank completely? Sol: Time taken by one tap to fill the half of the tank =3 hours Part filled by the four taps in 1 hour=4/6=2/3 Remaining part=1 -1/2=1/2 Therefore, 2/3:1/2::1:x or x=(1/2)*1*(3/2)=3/4 hours. i.e 45 min So, total time taken= 3hrs 45min.

7)A water tank is two-fifth full. Pipe A can fill a tank in 10 min. And B can empty it in 6 min. If both pipes are open, how long will it take to empty or fill the tank completely ? Sol: Clearly, pipe B is faster than A and So, the tank will be emptied. Part to be emptied=2/5. Part emptied by (A+B) in 1 min= 1/6 -1/10=1/15 Therefore, 1/15:2/5::1:x or x=((2/5)*1*15)=6 min. So, the tank be emptied in 6 min.

8)Bucket P has thrice the capacity as Bucket Q. It takes 60 turns for Bucket P to fill the empty drum. How many turns it will take for both the buckets P&Q, having each turn together to fill the empty drum? Sol: Let the capacity of P be x lit.

file:///E|/work/books/placement/09_Aptitude/pipesandcisternssimple.html[1/28/2012 12:45:24 AM]

Then capacity of Q=x/3 lit Capacity of the drum=60x lit Required number of turns= 60x/(x+(x/3))= 60x*3/4x=45
BACK

file:///E|/work/books/placement/09_Aptitude/pipesandcisternssimple.html[1/28/2012 12:45:24 AM]

APTITUDE
Numbers H.C.F and L.C.M Decimal Fractions Simplification Square and Cube roots Average Problems on Numbers Problems on Ages Surds and Indices Percentage Profit and Loss Ratio And Proportions Partnership Chain Rule Time and Work Pipes and Cisterns Time and Distance Trains Boats and Streams Alligation or Mixture Simple Interest Compound Interest Logorithms Areas Volume and Surface area Races and Games of Skill Calendar Clocks Stocks ans Shares True Discount Bankers Discount Oddmanout and Series Data Interpretation probability Permutations and Combinations Puzzles

PROBLEMS ON NUMBERS
SIMPLE PROBLEMS MEDIUM PROBLEMS COMPLEX PROBLEMS

file:///E|/work/books/placement/09_Aptitude/pnumbers.html[1/28/2012 12:45:24 AM]

APTITUDE
Numbers H.C.F and L.C.M Decimal Fractions Simplification Square and Cube roots Average Problems on Numbers Problems on Ages Surds and Indices Percentage Profit and Loss Ratio And Proportions Partnership Chain Rule Time and Work Pipes and Cisterns Time and Distance Trains Boats and Streams Alligation or Mixture Simple Interest Compound Interest Logorithms Areas Volume and Surface area Races and Games of Skill Calendar Clocks Stocks ans Shares True Discount Bankers Discount Oddmanout and Series Data Interpretation probability Permutations and Combinations Puzzles BACK

PROBLEMS ON NUMBERS

SOLVED PROBLEMS
Complex Problems: 24.Six bells commence tolling together and toll at intervals of 2,4,6,8,10,12 seconds respectively. In 30 minutes how many times do they toll together? Solution: To find the time that the bells will toll together we have to take L.C.M of 2,4,6,8,10,12 is 120. So,the bells will toll together after every 120 seconds i e, 2 minutes In 30 minutes they will toll together [30/2 +1]=16 times 25.The sum of two numbers is 15 and their geometric mean is 20% lower than their arithmetic mean. Find the numbers? a.11,4 Solution:
Sum of the two numbers is a+b=15. their A.M = a+b / 2 and G.M = (ab) 1/2 Given G.M = 20% lower than A.M =80/100 A.M (ab) 1/2 =4/5 (ab) 1/2 =6 ab=36 =>b=36/a a+b=15 a+36/a=15 a2+36=15a a2-15a+36=0 a2-3a-12a+36=0 a(a-3)-12(a-3)=0 a=12 or 3. a+b/2 = 2*15/5= 6

b.12,3

c.13,2

d.10,5

If a=3 and a+b=15 then b=12. If a=12 and a+b=15 then b=3.

file:///E|/work/books/placement/09_Aptitude/pnumberscomplex.html[1/28/2012 12:45:24 AM]

Ans (b). 26.When we multiply a certain two digit number by the sum of its digits 405 is achieved. If we multiply the number written in reverse order of the same digits by the sum of the digits,we get 486.Find the number? a.81 b.45 c.36 d. none

Solution: Let the number be x y. When we multiply the number by the sum of its digit 405 is achieved. (10x+y)(x+y)=405....................1 If we multiply the number written in reverse order by its sum of digits we get 486. (10y+x)(x+y)=486......................2 dividing 1 and 2 (10x+y)(x+y)/(10y+x)(x+y) = 405/486. 10x+y / 10y+x = 5/6. 60x+6y = 50y+5x 55x=44y 5x = 4y. From the above condition we conclude that the above condition is satisfied by the second option i e b. 45. Ans (b). 27.Find the HCF and LCM of the polynomials x2-5x+6 and x2-7x+10? a.(x-2),(x-2)(x-3)(x-5) b.(x-2),(x-2)(x-3) c.(x-3),(x-2)(x-3)(x-5) d. none Solution: The given polynomials are x2-5x+6=0................1 x2-7x+10=0...............2 we have to find the factors of the polynomials
x2-5x+6 x2-2x-3x+6 x(x-2)-3(x-2) (x-3)(x-2) and x2-7x+10 x2-5x-2x+10 x(x-5)-2(x-5) (x-2)(x-5)

From the above factors of the polynomials we can easily find the HCF as (x-3)and LCM as (x-2)(x-3)(x-5). Ans (c) 28.The sum of all possible two digit numbers formed from

file:///E|/work/books/placement/09_Aptitude/pnumberscomplex.html[1/28/2012 12:45:24 AM]

three different one digit natural numbers when divided by the sum of the original three numbers is equal to? a.18 b.22 c.36 d. none

Solution: Let the one digit numbers x,y,z Sum of all possible two digit numbers =(10x+y)+(10x+z)+(10y+x)+(10y+z)+(10z+x)+(10z+y) = 22(x+y+z) Therefore sum of all possible two digit numbers when divided by sum of one digit numbers gives 22. 29.A number being successively divided by 3,5,8 leaves remainders 1,4,7 respectively. Find the respective remainders if the order of divisors are reversed? Solution: Let the number be x.
3 5 8 y z 1 1 4 7 x

z=8*1+7=15 y=5z+4 = 5*15+4 = 79 x=3y+1 = 3*79+1=238


Now 8 238 5 3 29 5 1 6 4 2

Respective remainders are 6,4,2. 30.The arithmetic mean of two numbers is smaller by 24 than the larger of the two numbers and the GM of the same numbers exceeds by 12 the smaller of the numbers. Find the numbers? a.6,54 b.8,56 c.12,60 d.7,55 Solution: Let the numbers be a,b where a is smaller and b is larger number. The AM of two numbers is smaller by 24 than the larger of the two numbers. AM=b-24 AM of two numbers is a+b/2. a+b/2 = b-24 a+b = 2b-48 a = b-48...................1 The GM of the two numbers exceeds by 12 the smaller of the numbers

file:///E|/work/books/placement/09_Aptitude/pnumberscomplex.html[1/28/2012 12:45:24 AM]

GM = a+12
GM of two numbers is (ab) 1/2 (ab) 1/2 = a+12 ab = a2+144+24a from 1 b=a+48 a(a+48)= a2+48a = a2+144+24a a2+144+24a

24a=144=>a=6 Therefore b=a+48=54.

Ans (a). 31.The sum of squares of the digits constituting a positive two digit number is 13,If we subtract 9 from that number we shall get a number written by the same digits in the reverse order. Find the number? a.12 b.32 c.42 d.52.

Solution: Let the number be x y. the sum of the squares of the digits of the number is 13 x2+y2=13 If we subtract 9 from the number we get the number in reverse order x y-9=y x. 10x+y-9=10y+x. 9x-9y=9 x-y=1 (x-y)2 =x2+y2-2x y 1 =13-2x y 2x y = 12 x y = 6 =>y=6/x x-y=1 x-6/x=1 x2-6=x x2-x-6=0 x+2x-3x-6=0 x(x+2)-3(x+2)=0 x=3,-2. If x=3 and x-y=1 then y=2. If x=-2 and x-y=1 then y=-3. Therefore the number is 32. Ans (b). 32.If we add the square of the digit in the tens place of the positive two digit number to the product of the digits of that number we get 52,and if we add the square of the digit in the unit's place to the same product of the digits we get 117.Find the two digit number?

file:///E|/work/books/placement/09_Aptitude/pnumberscomplex.html[1/28/2012 12:45:24 AM]

a.18 b.39 c.49 d.28 Solution: Let the digit number be x y Given that if we add square of the digit in the tens place of a number to the product of the digits we get 52. x2+x y=52. x(x+y)=52....................1 Given that if we add the square of the digit in the unit's plac e to the product is 117. y2+x y= 117 y(x+y)=117.........................2 dividing 1 and 2 x(x+y)/y(x+y) = 52/117=4/9 x/y=4/9 from the options we conclude that the two digit number is 49 because the condition is satisfied by the third option. Ans (c) 33.The denominators of an irreducible fraction is greater than the numerator by 2.If we reduce the numerator of the reciprocal fraction by 3 and subtract the given fraction from the resulting one,we get 1/15.Find the given fraction? Solution: Let the given fraction be x / (x+2) because given that denominator of the fraction is greater than the numerator by 2 1 [(x 1/(x+2))/3] = 1/15. 1 (x2+2x-1) /3(x+2) = 1/15 (3x+6-x2-2x+1)/3(x+2) = 1/15 (7-x2+2x)/(x+2) = 1/5 -5x2+5x+35 = x+2 5x2-4x-33 = 0 5x2-15x+11x-33 = 0 5x(x-3)+11(x-3) = 0 (5x+11)(x-3) = 0 Therefore x=-11/5 or 3 Therefore the fraction is x/(x+2) = 3/5. 34.Three numbers are such that the second is as much lesser than the third as the first is lesser than the second. If the product of the two smaller numbers is 85 and the product of two larger numbers is 115. Find the middle number? Solution: Let the three numbers be x,y,z Given that z y = y x 2y = x+z.....................1 Given that the product of two smaller numbers is 85 x y = 85................2 Given that the product of two larger numbers is 115 y z = 115...............3 Dividing 2 and 3 x y /y z = 85/115 x / z = 17 / 23

file:///E|/work/books/placement/09_Aptitude/pnumberscomplex.html[1/28/2012 12:45:24 AM]

From 1 2y = x+z 2y = 85/y + 115/y 2y2 = 200 y2 = 100 y = 10 35.If we divide a two digit number by the sum of its digits we get 4 as a quotient and 3 as a remainder. Now if we divide that two digit number by the product of its digits we get 3 as a quotient and 5 as a remainder . Find the two digit number? Solution: Let the two digit number is x y. Given that x y / (x+y) quotient=4 and remainder = 3 we can write the number as x y = 4(x+y) +3...........1 Given that x y /(x*y) quotient = 3 and remainder = 5 we can write the number as x y = 3 x*y +5...............2 By trail and error method For example take x=1,y=2 1............12=4(2+3)+3 =4*3+3 ! =15 let us take x=2 y=3 1..............23=4(2+3)+3 =20+3 =23 2.............23=3*2*3+5 =18+5 =23 the above two equations are satisfied by x=2 and y=3 Therefore the required number is 23. 36.First we increased the denominator of a positive fraction by 3 and then it by 5.The sum of the resulting fractions proves to be equal to 2/3. Find the denominator of the fraction if its numerator is 2. Solution: Let us assume the fraction is x/y First we increasing the denominator by 3 we get x/(y-3) Then decrease it by 5 we get the fraction as x/(y-5) Given that the sum of the resulting fraction is 2/3 x/(y+3) + x/(y-5) = 2/3 Given numerator equal to 2 2*[ 1/y+3 + 1/y-5] =2/3 (y-5+y+3) / (y-3)(y+5) =1/3 6y 6 = y2-5y+3y-15 y2-8y-9 = 0 y2-9y+y-9 = 0 y(y-9)+1(y-9) = 0 Therefore y =-1 or 9.

file:///E|/work/books/placement/09_Aptitude/pnumberscomplex.html[1/28/2012 12:45:24 AM]

37.If we divide a two digit number by a number consisting of the same digits written in the reverse order,we get 4 as quotient and 15 as a remainder. If we subtract 1 from the given number we get the sum of the squares of the digits constituting that number. Find the number? a.71 b.83 c.99 d. none

Solution: Let the number be x y. If we divide 10x+y by a number in reverse order i e,10y+x we get 4 as quotient and 15 as remainder. We can write as 10x+y = 4(10y+x)+15......................1 If we subtract 1 from the given number we get square of the digits 10x+y = x2+y2.....................................2 By using above two equations and trail and error method we get the required number. From the options also we can solve the problem. In this no option is satisfied so answer is d. Ans (d) BACK

file:///E|/work/books/placement/09_Aptitude/pnumberscomplex.html[1/28/2012 12:45:24 AM]

APTITUDE
Numbers H.C.F and L.C.M Decimal Fractions Simplification Square and Cube roots Average Problems on Numbers Problems on Ages Surds and Indices Percentage Profit and Loss Ratio And Proportions Partnership Chain Rule Time and Work Pipes and Cisterns Time and Distance Trains Boats and Streams Alligation or Mixture Simple Interest Compound Interest Logorithms Areas Volume and Surface area Races and Games of Skill Calendar Clocks Stocks ans Shares True Discount Bankers Discount Oddmanout and Series Data Interpretation probability Permutations and Combinations Puzzles BACK

PROBLEMS ON NUMBERS SOLVED PROBLEMS


Medium Problems:

12.The difference between two numbers 1365.When the larger number is divided by the smaller one the quotient is 6 and the remainder is 15.The smaller number is? a.240 b.270 c.295 d.360 Solution: Let the smaller number be x, then larger number =1365+x Therefore 1365+x=6x+15 5x=1350 => x=270 Required number is 270. 13.Find the remainder when 231 is divided by 5? Solution: 210 =1024. unit digit of 210 * 210 * 210 is 4 as 4*4*4 gives unit digit 4 unit digit of 231 is 8. Now 8 when divided by 5 gives 3 as remainder. 231 when divided by 5 gives 3 as remainder. 14.The largest four digit number which when divided by 4,7 or 13 leaves a remainder of 3 in each case is? a.8739 b.9831 c.9834 d.9893. Solution: Greatest number of four digits is 9999 L.C.M of 4,7, and 13=364. On dividing 9999 by 364 remainder obtained is 171. Therefore greatest number of four digits divisible by 4,7,13 =9999-171=9828. Hence required number=9828+3=9831. Ans (b). 15.What least value must be assigned to * so that th number 197*5462 is divisible by 9? Solution: Let the missing digit be x Sum of digits = (1+9+7+x+5+4+6+2)=34+x For 34+x to be divisible by 9 , x must be replaced by 2 The digit in place of x must be 2. 16.Find the smallest number of 6 digits which is exactly divisible by 111? Solution: Smallest number of 6 digits is 100000

file:///E|/work/books/placement/09_Aptitude/pnumbersmedium.html[1/28/2012 12:45:25 AM]

On dividing 10000 by 111 we get 100 as remainder Number to be added =111-100=11. Hence,required number =10011. 17.A number when divided by 342 gives a remainder 47.When the same number is divided by 19 what would be the remainder? Solution: Number=342 K + 47 = 19 * 18 K + 19 * 2 + 9=19 ( 18K + 2) + 9. The given number when divided by 19 gives 18 K + 2 as quotient and 9 as remainder. 18.In doing a division of a question with zero remainder,a candidate took 12 as divisor instead of 21.The quotient obtained by him was 35. The correct quotient is? a.0 b.12 c.13 d.20 Solution: Dividend=12*35=420. Now dividend =420 and divisor =21. Therefore correct quotient =420/21=20. 19.If a number is multiplied by 22 and the same number is added to it then we get a number that is half the square of that number. Find the number. a.45 b.46 c.47 d. none Solution: Let the required number be x. Given that x*22+x = 1/2 x2 23x = 1/2 x2 x = 2*23=46 Ans (b) 20.Find the number of zeros in the factorial of the number 18? Solution: 18! contains 15 and 5,which combined with one even number gives zeros. Also 10 is also contained in 18! which will give additional zero .Hence 18! contains 3 zeros and the last digit will always be zero. 21.The sum of three prime numbers is 100.If one of them exceeds another by 36 then one of the numbers is? a.7 b.29 c.41 d67. Solution: x+(x+36)+y=100 2x+y=64 Therefore y must be even prime which is 2 2x+2=64=>x=31. Third prime number =x+36=31+36=67. 22.A number when divided by the sum of 555 and 445 gives two times their difference as quotient and 30 as remainder . The number is?

file:///E|/work/books/placement/09_Aptitude/pnumbersmedium.html[1/28/2012 12:45:25 AM]

a.1220 b.1250 c.22030 d.220030. Solution: Number=(555+445)*(555-445)*2+30 =(555+445)*2*110+30 =220000+30=220030. 23.The difference of 1025-7 and 1024+x is divisible by 3 for x=? a.3 b.2 c.4 d.6 Solution: The difference of 1025-7 and 1024+x is =(1025-7)-(1024-x) =1025-7-1024-x =10.1024-7 -1024-x =1024(10-1)-(7-x) =1024*9-(7+x) The above expression is divisible by 3 so we have to replace x with 2. Ans (b). BACK

file:///E|/work/books/placement/09_Aptitude/pnumbersmedium.html[1/28/2012 12:45:25 AM]

APTITUDE
Numbers H.C.F and L.C.M Decimal Fractions Simplification Square and Cube roots Average Problems on Numbers Problems on Ages Surds and Indices Percentage Profit and Loss Ratio And Proportions Partnership Chain Rule Time and Work Pipes and Cisterns Time and Distance Trains Boats and Streams Alligation or Mixture Simple Interest Compound Interest Logorithms Areas Volume and Surface area Races and Games of Skill Calendar Clocks Stocks ans Shares True Discount Bankers Discount Oddmanout and Series Data Interpretation probability Permutations and Combinations Puzzles BACK

PROBLEMS ON NUMBERS SOLVED PROBLEMS


Simple problems: 1.What least number must be added to 3000 to obtain a number exactly divisible by 19? Solution: On dividing 3000 by 19 we get 17 as remainder Therefore number to be added = 19-17=2. 2.Find the unit's digit n the product 2467 153 * 34172? Solution: Unit's digit in the given product=Unit's digit in 7 153 * 172 Now 7 4 gives unit digit 1 7 152 gives unit digit 1 7 153 gives 1*7=7.Also 172 gives 1 Hence unit's digit in the product =7*1=7. 3.Find the total number of prime factors in 411 *7 5 *112 ? Solution: 411 7 5 112= (2*2) 11 *7 5 *112 = 222 *7 5 *112 Total number of prime factors=22+5+2=29 4.The least umber of five digits which is exactly divisible by 12,15 and 18 is? a.10010 b.10015 c.10020 d.10080 Solution: Least number of five digits is 10000 L.C.Mof 12,15,18 s 180. On dividing 10000 by 180,the remainder is 100. Therefore required number=10000+(180-100) =10080. Ans (d). 5.The least number which is perfect square and is divisible by each of the numbers 16,20 and 24 is? a.1600 b.3600 c.6400 d.14400 Solution: The least number divisible by 16,20,24 = L.C.M of 16,20,24=240 =2*2*2*2*3*5 To make it a perfect square it must be multiplied by 3*5. Therefore required number =240*3*5=3600.

file:///E|/work/books/placement/09_Aptitude/pnumberssimple.html[1/28/2012 12:45:25 AM]

Ans (b). 6.A positive number which when added to 1000 gives a sum , which is greater than when it is multiplied by 1000. The positive integer is? a.1 b.3 c.5 d.7 Solution: 1000+N>1000N clearly N=1. 7.How many numbers between 11 and 90 are divisible by 7? Solution: The required numbers are 14,21,28,...........,84. This is an A.P with a=14,d=7. Let it contain n terms then T =84=a+(n-1)d =14+(n-1)7 =7+7n 7n=77 =>n=11. 8.Find the sum of all odd numbers up to 100? Solution: The given numbers are 1,3,5.........99. This is an A.P with a=1,d=2. Let it contain n terms 1+(n-1)2=99 =>n=50 Then required sum =n/2(first term +last term) =50/2(1+99)=2500. 9.How many terms are there in 2,4,6,8..........,1024? Solution: Clearly 2,4,6........1024 form a G.P with a=2,r=2 Let the number of terms be n then 2*2 n-1=1024 2n-1 =512=29 n-1=9 n=10. 10.2+22+23+24+25..........+28=? Solution: Given series is a G.P with a=2,r=2 and n=8. Sum Sn=a(1-r n)/1-r=Sn=2(1-28)/1-2. =2*255=510. 11.Find the number of zeros in 27!? Solution: Short cut method : number of zeros in 27!=27/5 + 27/25 =5+1=6zeros. BACK

file:///E|/work/books/placement/09_Aptitude/pnumberssimple.html[1/28/2012 12:45:25 AM]

Probability
Introduction: Experiment: An operation which can produce some well-defined outcome is called an experiment. Random Experiment: An experiment in which all possible out comes are known and the exact output cannot be predicted in advance is called a random experiment. EX: 1) Rolling an unbiased dice. 2) Tossing a fair coin. 3) Drawing a card from a pack of well-shuffled cards . 4)Picking up a ball of certain colour from a bag containing balls of different colours. Details: 1) When we thrown a coin ,then either a Head(H) or a Tail(T) appears. 2)A dice is a solid cube ,having 6 faces,marked 1,2,3,4,5,6 respectively. When we throw a die ,the outcome is the number that appears on its upper face. 3)A pack of cards has 52 cards. It has 13 cards of each suit,namely spades,clubs,hearts and diamonds. Cards of spades and clubs are balck cards. Cards of hearts and diamonds are red cards. There are four honours of each suit. These are Aces,Kings,queens and Jacks. These are called Face cards. Sample Space: When we perform an experiment ,then the set of S of all possible outcomes is called the Sample space . EX: 1)In tossing a coin S= {H,T}. 2)If two coins are tossed then S= {HH,HT,TH,TT}. 3)In rolling a dice ,we have S={1,2,3,4,5,6}. Event:Any subset of a sample space is called an Event. Probability of occurrence of an Event: Let S be the sample space. Let E be the Event. Then E cS i.e E is subset of S then probability of E p(E) =n(E)/n(S). Reults on Probability: 1)P(S) =1. 2)0 < P(E) < 1 probability of an event lies between 0 and 1. Max value of probability of an event is one. 3)P()=0. 4)For any events A and B we have . P(AUB) =P(A) +P(B) -P(AnB). 5)If A denotes (not -A) then P(A) =1-P(A) P(A)+P(A) =1.

Top
Problems: 1)An biased die is tossed.Find the probability of getting a multiple of 3? Sol: Here we have sample space S={1,2,3,4,5,6}. Let E be the event of getting a multiple of 3. Then E={3,6}. P(E) =n(E)/n(S). n(E) =2, n(S) =6. P(E) =2/6 P(E) =1/3. 2)In a simultaneous throw of a pair of dice,find the probability of getting a total more than 7? Sol: Here we have sample space n(S) =6*6 =36. Let E be the event of getting a total more than 7. ={(1,6),(2,5),(3,4),(4,3)(5,2),(6,1)(2,6),(3,5),(4,4), (5,3),(6,2),(4,5),(5,4), (5,5),(4,6),(6,4)} n(E) =15 P(E) = n(E)/n(S) = 15/36. P(E) = 5/12. 3)A bag contains 6 white and 4 black balls .Two balls are

file:///E|/work/books/placement/09_Aptitude/probability.html[1/28/2012 12:45:26 AM]

drawn at random .Find the probability that they are of the same colour? Sol: Let S be the sample space. Number of ways for drawing two balls out of 6 white and 4 red balls = 10C2 =10!/(8!*2!) = 45. n(S) =45. Let E =event of getting both balls of the same colour. Then n(E) =number of ways of drawing ( 2balls out of 6) or (2 balls out of 4). = 6C2 +4C2 = 6!/(4!*2!) + 4!/(2! *2!) = 6*5/2 +4 *3/2 =15+6 =21. P(E) =n(E)/n(S) =21/45 =7/45. 4)Two dice are thrown together .What is the probability that the sum of the number on the two faces is divisible by 4 or 6? Sol: n(S) = 6*6 =36. E be the event for getting the sum of the number on the two faces is divisible by 4 or 6. E={(1,3)(1,5)(2,4?)(2,2)(3,5)(3,3)(2,6)(3,1)(4,2)(4,4) (5,1)(5,3)(6,2)(6,6)} n(E) =14. Hence P(E) =n(E)/n(S) = 14/36. P(E) = 7/18 5)Two cards are drawn at random from a pack of 52 cards What is the probability that either both are black or both are queens? Sol: total number of ways for choosing 2 cards from 52 cards is =52C2 =52 !/(50!*2!) = 1326. Let A= event of getting bothe black cards. Let B= event of getting bothe queens AnB=Event of getting queens of black cards n(A) =26C2. We have 26 black cards from that we have to choose 2 cards. n(A) =26C2=26!/(24!*2!) = 26*25/2=325 from 52 cards we have 4 queens. n(B) = 4C2 = 4!/(2!* 2!) =6 n(AnB) =2C2. =1 P(A) = n(A) /n(S) =325/1326 P(B) = n(B)/n(S) = 6/1326 P(A n B) = n(A n B)/n(S) = 1/1326 P(A u B) = P(A) +P(B) -P(AnB) = 325/1326 + 6/1326 -1/1326 = 330/1326 P(AuB) = 55/221 6)Two diced are tossed the probability that the total score is a prime number? Number of total ways n(S) =6 * 6 =36 E =event that the sum is a prime number. Then E={(1,1)(1,2)(1,4)(1,6)(2,1)(2,3)(2,5)(3,2)(3,4)(4,1) (4,3)(5,2)(5,6)(6,1)(6,5)} n(E) =15 P(E) =n(E)/n(S) = 15/36 P(E) = 5/12

Top
7)Two dice are thrown simultaneously .what is the probability of getting two numbers whose product is even? Sol : In a simultaneous throw of two dice ,we have n(S) = 6*6 = 36 E=Event of getting two numbers whose product is even E={(1,2)(1,4)(1,6)(2,1)(2,2)(2,3)(2,4)(2,5)(2,6)(3,2) (3,4)(3,6)(4,1)(4,2)(4,3)(4,4)(4,5)(4,6)(5,2)(5,4)(5,6)(6,1) (6,2)(6,3)(6,4)(6,5)(6,6)} n(E) = 27 P(E) = n(E)/n(S) = 27 /36 P(E) =3/4 probability of getting two numbers whose product is even is equals to 3/4. 8)In a lottery ,there are 10 prozes and 25 blanks.A lottery is drawn at random. what is the probability of getting a prize ? Sol: By drawing lottery at random ,we have n(S) =10C1+25C1 = 10+25 = 35. E =event of getting a prize. n(E) =10C1 =10 out of 10 prozes we have to get into one prize .The number of ways 10C1. n(E) =10 n(S) =35 P(E) =n(E)/n(S) =10/35 = 2/7 Probability is 2/7. 9)In a class ,30 % of the students offered English,20 % offered Hindi and 10 %offered Both.If a student is offered at random, what is the probability that he has offered English or Hindi? Sol:English offered students =30 %.

file:///E|/work/books/placement/09_Aptitude/probability.html[1/28/2012 12:45:26 AM]

Hindi offered students =20% Both offered students =10 % Then only english offered students E =30 -10 =20 % only Hindi offered students S =20 -10 % = 10 % All the students =100% =E +S +E or S 100 =20 +10 + E or S +E and S Hindi or English offered students =100 -20-10-10 =60 % Probability that he has offered English or Hindi =60/100= 2/5 10) A box contains 20 electricbulbs ,out of which 4 are defective, two bulbs are chosen at random from this box.What is the probability that at least one of these is defective ? Sol: out of 20 bulbs ,4 bulbs are defective. 16 bulbs are favourable bulbs. E = event for getting no bulb is defective. n(E) =16 C 2 out of 16 bulbs we have to choose 2 bulbs randomly .so the number of ways =16 C 2 n(E) =16 C2 n(S) =20 C 2 P(E) =16 C2/20C2 = 12/19 probability of at least one is defective + probability of one is non defective =1 P(E) + P(E) =1 12/19 +P(E) =1 P(E) =7/19 11)A box contains 10 block and 10 white balls.What is the probability of drawing two balls of the same colour? Sol: Total number of balls =10 +10 =20 balls Let S be the sample space. n(S) =number of ways drawing 2 balls out of 20 = 20 C2 = 20 !/(18! *2!) = 190. Let E =event of drawing 2 balls of the same colour. n(E) =10C2+ 10C2 = 2(10 C2) = 90 P(E) =n(E)/n(S) P(E) =90/190 = 9/19 12) A bag contains 4 white balls ,5 red and 6 blue balls .Three balls are drawn at random from the bag.What is the probability that all of them are red ? Sol: Let S be the sample space. Then n(S) =number of ways drawing 3 balls out of 15. =15 C3. =455 Let E =event of getting all the 3 red balls. n(E) = 5 C3 =5C2 = 10 P(E) =n(E) /n(S) =10/455 =2/91.

Top
13)From a pack of 52 cards,one card is drawn at random.What is the probability that the card is a 10 or a spade? Sol: Total no of cards are 52. These are 13 spades including tne and there are 3 more tens. n(E) =13+3 = 16 P(E) =n(E)/n(S). =16/52 P(E) =4/13. 14) A man and his wife appear in an interview for two vacancies in the same post.The probability of husband's selection is 1/7 and the probabililty of wife's selection is 1/5.What is the probabililty that only one of them is selected? Sol: let A =event that the husband is selected. B = event that the wife is selected. E = Event for only one of them is selected. P(A) =1/7 and p(B) =1/5. P(A') =Probability of husband is not selected is =1-1/7=6/7 P(B') =Probaility of wife is not selected =1-1/5=4/7 P(E) =P[(A and B') or (B and A')] = P(A and B') +P(B and A') = P(A)P(B') + P(B)P(A') = 1/7*4/5 + 1/5 *6/7 P(E) =4/35 +6/35=10/35 =2/7 15)one card is drawn at random from a pack of 52 cards.What is the probability that the card drawn is a face card? Sol: There are 52 cards,out of which there 16 face cards. P(getting a face card) =16/52 = 4/13 16) The probability that a card drawn from a pack of 52 cards will be a diamond or a king? Sol: In 52 cards 13 cards are diamond including one king there are 3 more kings. E event of getting a diamond or a king. n(E) =13 +3 = 16

file:///E|/work/books/placement/09_Aptitude/probability.html[1/28/2012 12:45:26 AM]

P(E) =n(E) /n(S) =16/52 =4/13 17) Two cards are drawn together from apack of 52 cards.What is the probability that one is a spade and one is a heart ? Sol: S be the sample space the n (S) =52C2 =52*51/2 =1326 let E =event of getting 2 kings out of 4 kings n(E) =4C2 = 6 P(E) =n(E)/n(S) =6/1326 =1/221 18) Two cards are drawn together from a pack of 52 cards.What is the probability that one is a spade and one is a heart? Sol: Let S be the sample space then n(S) =52C2 =1326 E = Event of getting 1 spade and 1 heart. n(E) =number of ways of choosing 1 spade out of 13 and 1 heart out of 13. = 13C1*13C1 =169 P(E)= n(E)/n(S) =169/1326 =13/102. 19) Two cards are drawn from a pack of 52 cards .What is the probability that either both are Red or both are Kings? Sol: S be the sample space. n(S) =The number of ways for drawing 2 cards from 52 cards. n(S) =52C2 =1326 E1 be the event of getting bothe red cards. E2 be the event of getting both are kings. E1nE2 =Event of getting 2 kings of red cards. We have 26 red balls.From 26 balls we have to choose 2 balls. n(E1) =26C2 = 26*25/2 =325 We have 4 kings .out of 4 kings,we have to choosed 2 balls. n(E2) =4C2 =6 n(E1nE2) =2C2 =1 P(E1) = n(E1)/n(S) =325/1326 P(E2) =n(E2)/n(S) =6/1326 P(E1nE2) =n(E1nE2)/n(S) =1/1326 P(both red or both kings) = P(E1UE2) = P(E1) +P(E2)-P(E1nE2) =325/1326 +6/1326 -1/1326 =330/1326 =55/221

Back Back To Main

Top

Contact: 040-23000700

file:///E|/work/books/placement/09_Aptitude/probability.html[1/28/2012 12:45:26 AM]

APTITUDE
Numbers H.C.F and L.C.M Decimal Fractions Simplification Square and Cube roots Average Problems on Numbers Problems on Ages Surds and Indices Percentage Profit and Loss Ratio And Proportions Partnership Chain Rule Time and Work Pipes and Cisterns Time and Distance Trains Boats and Streams Alligation or Mixture Simple Interest Compound Interest Logorithms Areas Volume and Surface area Races and Games of Skill Calendar Clocks Stocks ans Shares True Discount Bankers Discount Oddmanout and Series Data Interpretation probability Permutations and Combinations Puzzles BACK

PROBABILITY

EXPERIMENT:An operation which can produce some well-defined outcome is called an experiment. RANDOM EXPERIMENT : An experiment in which all possible out comes are known and the exact output cannot be predicted in advance is called a random experiment. EX: 1) Rolling an unbiased dice. 2) Tossing a fair coin. 3) Drawing a card from a pack of well-shuffled cards . 4)Picking up a ball of certain colour from a bag containing balls of different colours. DETAILS: 1) When we thrown a coin ,then either a Head(H) or a Tail(T) appears. 2)A dice is a solid cube ,having 6 faces,marked 1,2,3,4,5,6 respectively. When we throw a die ,the outcome is the number that appears on its upper face. 3)A pack of cards has 52 cards. It has 13 cards of each suit,namely spades,clubs,hearts and diamonds. Cards of spades and clubs are balck cards. Cards of hearts and diamonds are red cards. There are four honours of each suit. These are Aces,Kings,queens and Jacks. These are called Face cards. SAMPLE SPACE:When we perform an experiment ,then the set of S of all possible outcomes is called the Sample space . EX: 1)In tossing a coin S= {H,T}. 2)If two coins are tossed then S= {HH,HT,TH,TT}. 3)In rolling a dice ,we have S={1,2,3,4,5,6}.

EVENT: Any subset of a sample space is called an Event. Probability of occurrence of an Event: Let S be the sample space. Let E be the Event. Then E cS i.e E is subset of S then probability of E p(E) =n(E)/n(S).

Reults on Probability: 1)P(S) =1. 2)0 < P(E) < 1 probability of an event lies between 0 and 1. Max value of probability of an event is one. 3)P()=0. 4)For any events A and B we have . P(AUB) =P(A) +P(B) -P(AnB).
file:///E|/work/books/placement/09_Aptitude/probabilityconcept.html[1/28/2012 12:45:26 AM]

5)If A denotes (not -A) then P(A) =1-P(A) P(A)+P(A) =1.

BACK

file:///E|/work/books/placement/09_Aptitude/probabilityconcept.html[1/28/2012 12:45:26 AM]

APTITUDE
Numbers H.C.F and L.C.M Decimal Fractions Simplification Square and Cube roots Average Problems on Numbers Problems on Ages Surds and Indices Percentage Profit and Loss Ratio And Proportions Partnership Chain Rule Time and Work Pipes and Cisterns Time and Distance Trains Boats and Streams Alligation or Mixture Simple Interest Compound Interest Logorithms Areas Volume and Surface area Races and Games of Skill Calendar Clocks Stocks ans Shares True Discount Bankers Discount Oddmanout and Series Data Interpretation probability Permutations and Combinations Puzzles BACK

PROBABILITY

PROBLEMS: 1)An biased die is tossed.Find the probability of getting a multiple of 3? Sol: Here we have sample space S={1,2,3,4,5,6}. Let E be the event of getting a multiple of 3. Then E={3,6}. P(E) =n(E)/n(S). n(E) =2, n(S) =6. P(E) =2/6 P(E) =1/3. 2)In a simultaneous throw of a pair of dice,find the probability of getting a total more than 7? Sol: Here we have sample space n(S) =6*6 =36. Let E be the event of getting a total more than 7. ={(1,6),(2,5),(3,4),(4,3)(5,2),(6,1)(2,6),(3,5),(4,4),(5,3),(6,2),(4,5),(5,4), (5,5),(4,6),(6,4)} n(E) =15 P(E) = n(E)/n(S) = 15/36. P(E) = 5/12. 3)A bag contains 6 white and 4 black balls .Two balls are drawn at random .Find the probability that they are of the same colour? Sol: Let S be the sample space. Number of ways for drawing two balls out of 6 white and 4 red balls = 10C2 =10!/(8!*2!) = 45. n(S) =45. Let E =event of getting both balls of the same colour. Then n(E) =number of ways of drawing ( 2balls out of 6) or (2 balls out of 4). = 6C2 +4C2 = 6!/(4!*2!) + 4!/(2! *2!) = 6*5/2 +4 *3/2 =15+6 =21. P(E) =n(E)/n(S) =21/45 =7/45. 4)Two dice are thrown together .What is the probability that the sum of the number on the two faces is divisible by 4 or 6? Sol: n(S) = 6*6 =36. E be the event for getting the sum of the number on the two faces is divisible by 4 or 6. E={(1,3)(1,5)(2,4?)(2,2)(3,5)(3,3)(2,6)(3,1)(4,2)(4,4)(5,1)(5,3)(6,2)(6,6)} n(E) =14.

file:///E|/work/books/placement/09_Aptitude/probabilityproblems.html[1/28/2012 12:45:27 AM]

Hence P(E) =n(E)/n(S) = 14/36. P(E) = 7/18 5)Two cards are drawn at random from a pack of 52 cards What is the probability that either both are black or both are queens? Sol: total number of ways for choosing 2 cards from 52 cards is =52C2 =52 !/(50!*2!) = 1326. Let A= event of getting bothe black cards. Let B= event of getting bothe queens AnB=Event of getting queens of black cards n(A) =26C2. We have 26 black cards from that we have to choose 2 cards. n(A) =26C2=26!/(24!*2!) = 26*25/2=325 from 52 cards we have 4 queens. n(B) = 4C2 = 4!/(2!* 2!) =6 n(AnB) =2C2. =1 P(A) = n(A) /n(S) =325/1326 P(B) = n(B)/n(S) = 6/1326 P(A n B) = n(A n B)/n(S) = 1/1326 P(A u B) = P(A) +P(B) -P(AnB) = 325/1326 + 6/1326 -1/1326 = 330/1326 P(AuB) = 55/221 6)Two diced are tossed the probability that the total score is a prime number? Number of total ways n(S) =6 * 6 =36 E =event that the sum is a prime number. Then E={(1,1)(1,2)(1,4)(1,6)(2,1)(2,3)(2,5)(3,2)(3,4)(4,1)(4,3)(5,2)(5,6)(6,1)(6,5)} n(E) =15 P(E) =n(E)/n(S) = 15/36 P(E) = 5/12 7)Two dice are thrown simultaneously .what is the probability of getting two numbers whose product is even? Sol : In a simultaneous throw of two dice ,we have n(S) = 6*6 = 36 E =Event of getting two numbers whose product is even E ={(1,2)(1,4)(1,6)(2,1)(2,2)(2,3)(2,4)(2,5)(2,6)(3,2)(3,4)(3,6)(4,1) (4,2)(4,3)(4,4)(4,5)(4,6)(5,2)(5,4)(5,6)(6,1)(6,2)(6,3)(6,4)(6,5)(6,6)} n(E) = 27 P(E) = n(E)/n(S) = 27 /36 P(E) =3/4 probability of getting two numbers whose product is even is equals to 3/4.

8)In a lottery ,there are 10 prozes and 25 blanks.A lottery is drawn at random. what is the probability of getting a prize ? Sol: By drawing lottery at random ,we have n(S) =10C1+25C1 = 10+25 = 35. E =event of getting a prize. n(E) =10C1 =10

file:///E|/work/books/placement/09_Aptitude/probabilityproblems.html[1/28/2012 12:45:27 AM]

out of 10 prozes we have to get into one prize .The number of ways 10C1. n(E) =10 n(S) =35 P(E) =n(E)/n(S) =10/35 = 2/7 Probability is 2/7. 9)In a class ,30 % of the students offered English,20 % offered Hindi and 10 %offered Both.If a student is offered at random ,wha5t is the probability that he has offered English or Hindi? English offered students =30 %. Hindi offered students =20% Both offered students =10 % Then only english offered students E =30 -10 =20 % only Hindi offered students S =20 -10 % = 10 % All the students =100% =E +S +E or S 100 =20 +10 + E or S +E and S Hindi or English offered students =100 -20-10-10 =60 % Probability that he has offered English or Hindi =60/100 = 2/5 10) A box contains 20 electricbulbs ,out of which 4 are defective ,two bulbs are chosen at random from this box.What is the probability that at least one of these is defective ? Sol: out of 20 bulbs ,4 bulbs are defective. 16 bulbs are favourable bulbs. E = event for getting no bulb is defective. n(E) =16 C 2 out of 16 bulbs we have to choose 2 bulbs randomly .so the number of ways =16 C 2 n(E) =16 C2 n(S) =20 C 2 P(E) =16 C2/20C2 = 12/19 probability of at least one is defective + probability of one is non defective =1 P(E) + P(E) =1 12/19 +P(E) =1 P(E) =7/19 11)A box contains 10 block and 10 white balls.What is the probability of drawing two balls of the same colour? Sol: Total number of balls =10 +10 =20 balls Let S be the sample space. n(S) =number of ways drawing 2 balls out of 20 = 20 C2 = 20 !/(18! *2!) = 190. Let E =event of drawing 2 balls of the same colour. n(E) =10C2+ 10C2 = 2(10 C2) = 90 P(E) =n(E)/n(S) P(E) =90/190 = 9/19 12) A bag contains 4 white balls ,5 red and 6 blue balls .Three balls are drawn at

file:///E|/work/books/placement/09_Aptitude/probabilityproblems.html[1/28/2012 12:45:27 AM]

random from the bag.What is the probability that all of them are red ? Sol: Let S be the sample space. Then n(S) =number of ways drawing 3 balls out of 15. =15 C3. =455 Let E =event of getting all the 3 red balls. n(E) = 5 C3 =5C2 = 10 P(E) =n(E) /n(S) =10/455 =2/91.

13)From a pack of 52 cards,one card is drawn at random.What is the probability that the card is a 10 or a spade? Sol: Total no of cards are 52. These are 13 spades including tne and there are 3 more tens. n(E) =13+3 = 16 P(E) =n(E)/n(S). =16/52 P(E) =4/13. 14) A man and his wife appear in an interview for two vacancies in the same post. The probability of husband's selection is 1/7 and the probabililty of wife's selection is 1/5. What is the probabililty that only one of them is selected? Sol: let A =event that the husband is selected. B = event that the wife is selected. E = Event for only one of them is selected. P(A) =1/7 and p(B) =1/5. P(A') =Probability of husband is not selected is =1-1/7=6/7 P(B') =Probaility of wife is not selected =1-1/5=4/7 P(E) =P[(A and B') or (B and A')] = P(A and B') +P(B and A') = P(A)P(B') + P(B)P(A') = 1/7*4/5 + 1/5 *6/7 P(E) =4/35 +6/35=10/35 =2/7 15)one card is drawn at random from a pack of 52 cards.What is the probability that the card drawn is a face card? Sol: There are 52 cards,out of which there 16 face cards. P(getting a face card) =16/52 = 4/13 16) The probability that a card drawn from a pack of 52 cards will be a diamond or a king? Sol: In 52 cards 13 cards are diamond including one king there are 3 more kings. E event of getting a diamond or a king. n(E) =13 +3 = 16 P(E) =n(E) /n(S) =16/52 =4/13 17) Two cards are drawn together from apack of 52 cards.What is the probability that one is a spade and one is a heart ? Sol: S be the sample space the n (S) =52C2 =52*51/2

file:///E|/work/books/placement/09_Aptitude/probabilityproblems.html[1/28/2012 12:45:27 AM]

=1326 let E =event of getting 2 kings out of 4 kings n(E) =4C2 =6 P(E) =n(E)/n(S) =6/1326 =1/221 18) Two cards are drawn together from a pack of 52 cards.What is the probability that one is a spade and one is a heart? Sol: Let S be the sample space then n(S) =52C2 =1326 E = Event of getting 1 spade and 1 heart. n(E) =number of ways of choosing 1 spade out of 13 and 1 heart out of 13. = 13C1*13C1 =169 P(E)= n(E)/n(S) =169/1326 =13/102. 19) Two cards are drawn from a pack of 52 cards .What is the probability that either both are Red or both are Kings? Sol: S be the sample space. n(S) =The number of ways for drawing 2 cards from 52 cards. n(S) =52C2 =1326 E1 be the event of getting bothe red cards. E2 be the event of getting both are kings. E1nE2 =Event of getting 2 kings of red cards. We have 26 red balls.From 26 balls we have to choose 2 balls. n(E1) =26C2 = 26*25/2 =325 We have 4 kings .out of 4 kings,we have to choosed 2 balls. n(E2) =4C2 =6 n(E1nE2) =2C2 =1 P(E1) = n(E1)/n(S) =325/1326 P(E2) =n(E2)/n(S) =6/1326 P(E1nE2) =n(E1nE2)/n(S) =1/1326 P(both red or both kings) = P(E1UE2) = P(E1) +P(E2)-P(E1nE2) =325/1326 +6/1326 -1/1326 =330/1326 =55/221
BACK

file:///E|/work/books/placement/09_Aptitude/probabilityproblems.html[1/28/2012 12:45:27 AM]

Problems on Ages
Simple problems:

1.The present age of a father is 3 years more than three times the age of his son.Three years hence,fathers age will be 10 years more than twice the age of the son.Find the present age of the father. Solution: Let the present age be 'x' years. Then father's present age is 3x+3 years. Three years hence (3x+3)+3=2(x+3)+10 x=10 Hence father's present age = 3x+3 = 33 years. 2. One year ago the ratio of Ramu & Somu age was 6:7respectively. Four years hence their ratio would become 7:8. How old is Somu. Solution: Let us assume Ramu &Somu ages are x &y respectively. One year ago their ratio was 6:7 i.e x-1 / y-1 = 7x-6y=1 Four years hence their ratios,would become 7:8 i.e x-4 / y-4 = 7 / 8 8x-7y=-4 From the above two equations we get y= 36 years. i.e Somu present age is 36 years. 3. The total age of A &B is 12 years more than the total age of B&C. C is how many year younger than A. Solution: From the given data A+B = 12+(B+C) A+B-(B+C) = 12 A-C=12 years. C is 12 years younger than A

4. The ratio of the present age of P & Q is 6:7. If Q is 4 years old than P. what will be the ratio of the ages of P & Q after 4 years. Solution: The present age of P & Q is 6:7 i.e P / Q = 6 / 7 Q is 4 years old than P i.e Q = P+4. P/ P+4 = 6/7 7P-6P = 24, P = 24 , Q = P+4 =24+4 = 28 After 4 years the ratio of P &Q is P+4:Q+4 24+4 : 28+4 = 28:32 = 7:8

5. The ratio of the age of a man & his wife is 4:3.After 4 years this ratio will be 9:7. If the time of marriage the ratio was 5:3, then how many years ago were they married. Solution: The age of a man is 4x . The age of his wife is 3x. After 4 years their ratio's will be 9:7 i.e 4x+4 / 3x+4 = 9 / 7 28x-27x=36-28 x = 8. Age of a man is 4x = 4*8 = 32 years. Age of his wife is 3x = 3*8 = 24 years. Let us assume 'y' years ago they were married , the ratio was 5:3 ,i.e 32-y / 24-y = 5/ 3 y=12 years i.e 12 years ago they were married Top 6. Sneh's age is 1/6th of her father's age.Sneh's father's age will be twice the age of Vimal's age after 10 years. If Vimals eight birthday was celebrated two years before,then what is Sneh's present age. a) 6 2/3 years b) 24 years c) 30 years d) None of the above

Solution:

Assume Snehs age is 'x' years. Assume her fathers age is 'y' years. Snehs age is 1/6 of her fathers age i.e x = y /6. Fathers age will be twice of Vimal's age after 10 years.

file:///E|/work/books/placement/09_Aptitude/problemsonages.html[1/28/2012 12:45:27 AM]

i.e y+10 = 2( V+10)( where 'V' is the Vimal's age) Vimal's eight birthday was celebrated two years before, Then the Vimal's present age is 10 years. Y+10 = 2(10+10) Y=30 years. Sneh's present age x = y/6 x = 30/6 = 5 years. Sneh's present age is 5 years. 7.The sum of the ages of the 5 children's born at the intervals of 3 years each is 50 years what is the age of the youngest child. a) 4 years Solution: b) 8 years c) 10 years d)None of the above

Let the age of the children's be x ,x+3, x+6, x+9, x+12. x+(x+3)+(x+6)+(x+9)+(x+12) = 50 5x+30 = 50 5x = 20 x=4. Age of the youngest child is x = 4 years.

8. If 6 years are subtracted from the present age of Gagan and the remainder is divided by 18,then the present age of his grandson Anup is obtained. If Anup is 2 years younger to Madan whose age is 5 years,then what is Gagan's present age. a) 48 years Solution: b)60 years c)84 years d)65 years

Let us assume Anup age (x-6) x-6 x=60

Gagan present age is 'x' years. = 5-2 = 3 years. / 18 = 3 = 54 years

9.My brother is 3 years elder to me. My father was 28 years of age when my sister was born while my father was 26 years of age when i was born. If my sister was 4 years of age when my brother was born,then what was the age my father and mother respectively when my brother was born. a) 32 yrs, 23yrs b)32 yrs, 29yrs c)35 yrs,29yrs d)35yrs,33 yrs

Solution: My brother was born 3 years before I was born & 4 years after my sister was born. Father's age when brother was born = 28+4 = 32 years. Mother's age when brother was born = 26-3 = 23 years.

Back Back To Main

Top

Contact: 040-23000700

file:///E|/work/books/placement/09_Aptitude/problemsonages.html[1/28/2012 12:45:27 AM]

Problems on Numbers
Simple problems: 1.What least number must be added to 3000 to obtain a number exactly divisible by 19? Solution: On dividing 3000 by 19 we get 17 as remainder Therefore number to be added = 19-17=2. 2.Find the unit's digit n the product 2467 153 * 34172? Solution: Unit's digit in the given product=Unit's digit in 7 153 * 172 Now 7 4 gives unit digit 1 7 152 gives unit digit 1 7 153 gives 1*7=7.Also 172 gives 1 Hence unit's digit in the product =7*1=7. 3.Find the total number of prime factors in 411 *7 5 *112 ? Solution: 411 7 5 112= (2*2) 11 *7 5 *112 = 222 *7 5 *112 Total number of prime factors=22+5+2=29 4.The least umber of five digits which is exactly divisible by 12,15 and 18 is? a.10010 b.10015 c.10020 d.10080 Solution: Least number of five digits is 10000 L.C.Mof 12,15,18 s 180. On dividing 10000 by 180,the remainder is 100. Therefore required number=10000+(180-100) =10080. Ans (d). 5.The least number which is perfect square and is divisible by each of the numbers 16,20 and 24 is? a.1600 b.3600 c.6400 d.14400 Solution: The least number divisible by 16,20,24 = L.C.M of 16,20,24=240 =2*2*2*2*3*5 To make it a perfect square it must be multiplied by 3*5. Therefore required number =240*3*5=3600. Ans (b). 6.A positive number which when added to 1000 gives a sum , which is greater than when it is multiplied by 1000. The positive integer is? a.1 b.3 c.5 d.7 Solution: 1000+N>1000N clearly N=1. 7.How many numbers between 11 and 90 are divisible by 7? Solution: The required numbers are 14,21,28,...........,84. This is an A.P with a=14,d=7. Let it contain n terms then T =84=a+(n-1)d =14+(n-1)7 =7+7n 7n=77 =>n=11. 8.Find the sum of all odd numbers up to 100? Solution: The given numbers are 1,3,5.........99. This is an A.P with a=1,d=2. Let it contain n terms 1+(n-1)2=99 =>n=50 Then required sum =n/2(first term +last term) =50/2(1+99)=2500. 9.How many terms are there in 2,4,6,8..........,1024? Solution: Clearly 2,4,6........1024 form a G.P with a=2,r=2 Let the number of terms be n then 2*2 n-1=1024 2n-1 =512=29 n-1=9 n=10.

file:///E|/work/books/placement/09_Aptitude/problemsonnumbers.html[1/28/2012 12:45:28 AM]

10.2+22+23+24+25..........+28=? Solution: Given series is a G.P with a=2,r=2 and n=8. Sum Sn=a(1-r n)/1-r=Sn=2(1-28)/1-2. =2*255=510. 11.Find the number of zeros in 27!? Solution: Short cut method : number of zeros in 27!=27/5 + 27/25 =5+1=6zeros.

Top
Medium Problems: 12.The difference between two numbers 1365.When the larger number is divided by the smaller one the quotient is 6 and the remainder is 15.The smaller number is? a.240 b.270 c.295 d.360 Solution: Let the smaller number be x, then larger number =1365+x Therefore 1365+x=6x+15 5x=1350 => x=270 Required number is 270. 13.Find the remainder when 231 is divided by 5? Solution: 210 =1024. unit digit of 210 * 210 * 210 is 4 as 4*4*4 gives unit digit 4 unit digit of 231 is 8. Now 8 when divided by 5 gives 3 as remainder. 231 when divided by 5 gives 3 as remainder. 14.The largest four digit number which when divided by 4,7 or 13 leaves a remainder of 3 in each case is? a.8739 b.9831 c.9834 d.9893. Solution: solution: Greatest number of four digits is 9999 L.C.M of 4,7, and 13=364. On dividing 9999 by 364 remainder obtained is 171. Therefore greatest number of four digits divisible by 4,7,13 =9999-171=9828. Hence required number=9828+3=9831. Ans (b). 15.What least value must be assigned to * so that th number 197*5462 is divisible by 9? Solution: Let the missing digit be x Sum of digits = (1+9+7+x+5+4+6+2)=34+x For 34+x to be divisible by 9 , x must be replaced by 2 The digit in place of x must be 2. 16.Find the smallest number of 6 digits which is exactly divisible by 111? Solution: Smallest number of 6 digits is 100000 On dividing 10000 by 111 we get 100 as remainder Number to be added =111-100=11. Hence,required number =10011. 17.A number when divided by 342 gives a remainder 47.When the same number is divided by 19 what would be the remainder? Solution: Number=342 K + 47 = 19 * 18 K + 19 * 2 + 9=19 ( 18K + 2) + 9. The given number when divided by 19 gives 18 K + 2 as quotient and 9 as remainder. 18.In doing a division of a question with zero remainder,a candidate took 12 as divisor instead of 21.The quotient obtained by him was 35. The correct quotient is? a.0 b.12 c.13 d.20 Solution: Dividend=12*35=420. Now dividend =420 and divisor =21. Therefore correct quotient =420/21=20. 19.If a number is multiplied by 22 and the same number is added to it then we get a number that is half the square of that number. Find the number. a.45 b.46 c.47 d. none Solution: Let the required number be x. Given that x*22+x = 1/2 x2 23x = 1/2 x2 x = 2*23=46 Ans (b) 20.Find the number of zeros in the factorial of the number 18? Solution: 18! contains 15 and 5,which combined with one even number gives zeros. Also 10 is also contained in 18! which will give additional zero .Hence 18! contains 3 zeros and the last digit will always be zero. 21.The sum of three prime numbers is 100.If one of them

file:///E|/work/books/placement/09_Aptitude/problemsonnumbers.html[1/28/2012 12:45:28 AM]

exceeds another by 36 then one of the numbers is? a.7 b.29 c.41 d67. Solution: x+(x+36)+y=100 2x+y=64 Therefore y must be even prime which is 2 2x+2=64=>x=31. Third prime number =x+36=31+36=67. 22.A number when divided by the sum of 555 and 445 gives two times their difference as quotient and 30 as remainder . The number is? a.1220 b.1250 c.22030 d.220030. Solution: Number=(555+445)*(555-445)*2+30 =(555+445)*2*110+30 =220000+30=220030. 23.The difference of 1025-7 and 1024+x is divisible by 3 for x=? a.3 b.2 c.4 d.6 Solution: The difference of 1025-7 and 1024+x is =(1025-7)-(1024-x) =1025-7-1024-x =10.1024-7 -1024-x =1024(10-1)-(7-x) =1024*9-(7+x) The above expression is divisible by 3 so we have to replace x with 2. Ans (b).

Top
Complex Problems: 24.Six bells commence tolling together and toll at intervals of 2,4,6,8,10,12 seconds respectively. In 30 minutes how many times do they toll together? Solution: To find the time that the bells will toll together we have to take L.C.M of 2,4,6,8,10,12 is 120. So,the bells will toll together after every 120 seconds i e, 2 minutes In 30 minutes they will toll together [30/2 +1]=16 times 25.The sum of two numbers is 15 and their geometric mean is 20% lower than their arithmetic mean. Find the numbers? a.11,4 b.12,3 c.13,2 d.10,5 Solution: Sum of the two numbers is a+b=15. their A.M = a+b / 2 and G.M = (ab)1/2 Given G.M = 20% lower than A.M =80/100 A.M (ab)1/2=4/5 a+b/2 = 2*15/5= 6 (ab)1/2=6 ab=36 =>b=36/a a+b=15 a+36/a=15 a2+36=15a a2-15a+36=0 a2-3a-12a+36=0 a(a-3)-12(a-3)=0 a=12 or 3. If a=3 and a+b=15 then b=12. If a=12 and a+b=15 then b=3. Ans (b). 26.When we multiply a certain two digit number by the sum of its digits 405 is achieved. If we multiply the number written in reverse order of the same digits by the sum of the digits,we get 486.Find the number? a.81 b.45 c.36 d. none Solution: Let the number be x y. When we multiply the number by the sum of its digit 405 is achieved. (10x+y)(x+y)=405....................1 If we multiply the number written in reverse order by its sum of digits we get 486. (10y+x)(x+y)=486......................2 dividing 1 and 2 (10x+y)(x+y)/(10y+x)(x+y) = 405/486. 10x+y / 10y+x = 5/6. 60x+6y = 50y+5x 55x=44y 5x = 4y. From the above condition we conclude that the above condition is satisfied by the second option i e b. 45. Ans (b). 27.Find the HCF and LCM of the polynomials x2-5x+6 and x2-7x+10? a.(x-2),(x-2)(x-3)(x-5) b.(x-2),(x-2)(x-3) c.(x-3),(x-2)(x-3)(x-5) d. none Solution: The given polynomials are x2-5x+6=0................1 x2-7x+10=0...............2 we have to find the factors of the polynomials x2-5x+6 and x2-7x+10 x2-2x-3x+6 x2-5x-2x+10

file:///E|/work/books/placement/09_Aptitude/problemsonnumbers.html[1/28/2012 12:45:28 AM]

x(x-2)-3(x-2) x(x-5)-2(x-5) (x-3)(x-2) (x-2)(x-5) From the above factors of the polynomials we can easily find the HCF as (x-3)and LCM as (x-2)(x-3)(x-5). Ans (c) 28.The sum of all possible two digit numbers formed from three different one digit natural numbers when divided by the sum of the original three numbers is equal to? a.18 b.22 c.36 d. none Solution: Let the one digit numbers x,y,z Sum of all possible two digit numbers =(10x+y)+(10x+z)+(10y+x)+(10y+z)+(10z+x)+(10z+y) = 22(x+y+z) Therefore sum of all possible two digit numbers when divided by sum of one digit numbers gives 22. 29.A number being successively divided by 3,5,8 leaves remainders 1,4,7 respectively. Find the respective remainders if the order of divisors are reversed? Solution: Let the number be x. 5 8 z=8*1+7=15 y=5z+4 = 5*15+4 = 79 x=3y+1 = 3*79+1=238 Now 8 5 3 3 y z 1 - x - 1 4 - 7

238 - 6 4 - 2

29 5 1 Respective remainders are 6,4,2.

30.The arithmetic mean of two numbers is smaller by 24 than the larger of the two numbers and the GM of the same numbers exceeds by 12 the smaller of the numbers. Find the numbers? a.6,54 b.8,56 c.12,60 d.7,55 Solution: Let the numbers be a,b where a is smaller and b is larger number. The AM of two numbers is smaller by 24 than the larger of the two numbers. AM=b-24 AM of two numbers is a+b/2. a+b/2 = b-24 a+b = 2b-48 a = b-48...................1 The GM of the two numbers exceeds by 12 the smaller of the numbers GM = a+12 GM of two numbers is (ab)1/2 (ab) 1/2= a+12 ab = a2+144+24a from 1 b=a+48 a(a+48)= a2+144+24a a2+48a = a2+144+24a 24a=144=>a=6 Therefore b=a+48=54. Ans (a). 31.The sum of squares of the digits constituting a positive two digit number is 13,If we subtract 9 from that number we shall get a number written by the same digits in the reverse order. Find the number? a.12 b.32 c.42 d.52. Solution: Let the number be x y. the sum of the squares of the digits of the number is 13 x2+y2=13 If we subtract 9 from the number we get the number in reverse order x y-9=y x. 10x+y-9=10y+x. 9x-9y=9 x-y=1 (x-y)2 =x2+y2-2x y 1 =13-2x y 2x y = 12 x y = 6 =>y=6/x x-y=1 x-6/x=1 x2-6=x x2-x-6=0 x+2x-3x-6=0 x(x+2)-3(x+2)=0 x=3,-2. If x=3 and x-y=1 then y=2. If x=-2 and x-y=1 then y=-3. Therefore the number is 32. Ans (b).

Top
32.If we add the square of the digit in the tens place of the positive two digit number to the product of the digits of that number we get 52,and if we add the square of the digit in the unit's place to the same product of the digits we get 117.Find the two digit number? a.18 b.39 c.49 d.28 Solution:

file:///E|/work/books/placement/09_Aptitude/problemsonnumbers.html[1/28/2012 12:45:28 AM]

Let the digit number be x y Given that if we add square of the digit in the of a number to the product of the digits we get x2+x y=52. x(x+y)=52....................1 Given that if we add the square of the digit in e to the product is 117. y2+x y= 117 y(x+y)=117.........................2 dividing 1 and 2 x(x+y)/y(x+y) = 52/117=4/9 x/y=4/9 from the options we conclude that the two digit because the condition is satisfied by the third Ans (c)

tens place 52. the unit's plac

number is 49 option.

33.The denominators of an irreducible fraction is greater than the numerator by 2.If we reduce the numerator of the reciprocal fraction by 3 and subtract the given fraction from the resulting one,we get 1/15.Find the given fraction? Solution: Let the given fraction be x / (x+2) because given that denominator of the fraction is greater than the numerator by 2 1 [(x 1/(x+2))/3] = 1/15. 1 (x2+2x-1) /3(x+2) = 1/15 (3x+6-x2-2x+1)/3(x+2) = 1/15 (7-x2+2x)/(x+2) = 1/5 -5x2+5x+35 = x+2 5x2-4x-33 = 0 5x2-15x+11x-33 = 0 5x(x-3)+11(x-3) = 0 (5x+11)(x-3) = 0 Therefore x=-11/5 or 3 Therefore the fraction is x/(x+2) = 3/5. 34.Three numbers are such that the second is as much lesser than the third as the first is lesser than the second. If the product of the two smaller numbers is 85 and the product of two larger numbers is 115. Find the middle number? Solution: Let the three numbers be x,y,z Given that z y = y x 2y = x+z.....................1 Given that the product of two smaller numbers is 85 x y = 85................2 Given that the product of two larger numbers is 115 y z = 115...............3 Dividing 2 and 3 x y /y z = 85/115 x / z = 17 / 23 From 1 2y = x+z 2y = 85/y + 115/y 2y2 = 200 y2 = 100 y = 10 35.If we divide a two digit number by the sum of its digits we get 4 as a quotient and 3 as a remainder. Now if we divide that two digit number by the product of its digits we get 3 as a quotient and 5 as a remainder . Find the two digit number? Solution: Let the two digit number is x y. Given that x y / (x+y) quotient=4 and remainder = 3 we can write the number as x y = 4(x+y) +3...........1 Given that x y /(x*y) quotient = 3 and remainder = 5 we can write the number as x y = 3 x*y +5...............2 By trail and error method For example take x=1,y=2 1............12=4(2+3)+3 =4*3+3 ! =15 let us take x=2 y=3 1..............23=4(2+3)+3 =20+3 =23 2.............23=3*2*3+5 =18+5 =23 the above two equations are satisfied by x=2 and y=3 Therefore the required number is 23. 36.First we increased the denominator of a positive fraction by 3 and then it by 5.The sum of the resulting fractions proves to be equal to 2/3. Find the denominator of the fraction if its numerator is 2. Solution: Let us assume the fraction is x/y First we increasing the denominator by 3 we get x/(y-3) Then decrease it by 5 we get the fraction as x/(y-5) Given that the sum of the resulting fraction is 2/3 x/(y+3) + x/(y-5) = 2/3 Given numerator equal to 2 2*[ 1/y+3 + 1/y-5] =2/3 (y-5+y+3) / (y-3)(y+5) =1/3 6y 6 = y2-5y+3y-15 y2-8y-9 = 0 y2-9y+y-9 = 0 y(y-9)+1(y-9) = 0 Therefore y =-1 or 9.

file:///E|/work/books/placement/09_Aptitude/problemsonnumbers.html[1/28/2012 12:45:28 AM]

37.If we divide a two digit number by a number consisting of the same digits written in the reverse order,we get 4 as quotient and 15 as a remainder. If we subtract 1 from the given number we get the sum of the squares of the digits constituting that number. Find the number? a.71 b.83 c.99 d. none Solution: Let the number be x y. If we divide 10x+y by a number in reverse order i e,10y+x we get 4 as quotient and 15 as remainder. We can write as 10x+y = 4(10y+x)+15......................1 If we subtract 1 from the given number we get square of the digits 10x+y = x2+y2.....................................2 By using above two equations and trail and error method we get the required number. From the options also we can solve the problem. In this no option is satisfied so answer is d. Ans (d)

Back Back To Main

Top

Contact: 040-23000700

file:///E|/work/books/placement/09_Aptitude/problemsonnumbers.html[1/28/2012 12:45:28 AM]

Profit and Loss


Important Facts: Cost Price: The price at which an article is purchased, is called its cost price,abbreviated as C.P. Selling Price: The price at which an article is sold, is called its selling price,abbreviated as S.P. Profit or Gain: If S.P. Is greater than C.P. The seller is said to have a profit or gain. Loss:if S.P. Is less than C.P., the seller is said to have incurred a loss. Formulae 1.Gain=(S.P-C.P) 2.Loss=(C.P-S.P) 3.Loss or Gain is always reckoned on C.P. 4.Gain%=(gain*100)/C.P 5.Loss%=(loss*100)/C.P 6.S.P=[(100+gain%)/100]*C.P 7.S.P=[(100-loss%)/100]*C.P 8.C.P=(100*S.P)/(100+gain%) 9.C.P=(100*S.P)/(100-loss%) 10.If an article is sold at a gain of say,35%,then S.P=135% of C.P. 11.If an article is sold at a loss of say,35%,then S.P=65% of C.P. 12.When a person sells two similar items, one at a gain of say, x%,and the other at a loss of x%,then the seller always incurs a loss given by Loss%=[common loss and gain %/10]2=(x/10)2 13.If a trader professes to sell his goods at cost price,but uses false weight,then Gain%=[(error/(true value-error))*100]% 14.Net selling price=Marked price-Discount

Top
Simple Problems 1.A man buys an article for Rs.27.50 and sells it for Rs.28.60 Find the gain percent. Sol: C.P=Rs 27.50 S.P=Rs 28.60 then Gain=S.P-C.P=28.60-27.50=Rs 1.10 Gain%=(gain*100)/C.P% =(1.10*100)/27.50%=4%

2.If a radio is purchased for Rs 490 and sold for Rs 465.50 Find the loss%? Sol: C.P=Rs 490 S.P=Rs 465.50 Loss=C.P-S.P=490-465.50=Rs 24.50 Loss%=(loss*100)/C.P% =(24.50*100)/490%=5%

3.Find S.P when C.P=Rs 56.25 and Gain=20% Sol: S.P=[(100+gain%)/100]*C.P S.P=[(100+20)/100]56.25=Rs 67.50

4.Find S.P when C.P=Rs 80.40,loss=5% Sol: S.P=[(100-loss%)/100]*C.P S.P=[(100-5)/100]*80.40=Rs 68.34

5.Find C.P when S.P=Rs 40.60,gain=16%? Sol: C.P=(100*S.P)/(100+gain%) C.P=(100*40.60)/(100+16)=Rs 35

6.Find C.P when S.P=Rs 51.70 ,loss=12%? Sol: C.P=(100*S.P)/(100-loss%) C.P=(100*51.70)/(100-12)=Rs 58.75

7.A person incurs 5% loss by selling a watch for Rs 1140 . At what price should the watch be sold to earn 5% profit? Sol: Let the new S.P be Rs x then, (100-loss%):(1st S.P)=(100+gain%):(2nd S.P) (100-5)/1140=(100+5)/x x=(105*1140)/95=Rs 1260

8.If the cost price is 96% of the selling price,then what is the profit percent? Sol: let S.P=Rs 100 then C.P=Rs 96 profit=S.P-C.P=100-96=Rs 4 profit%=(profit*C.P)/100%

file:///E|/work/books/placement/09_Aptitude/profitandloss.html[1/28/2012 12:45:28 AM]

=(4*96)/100=4.17% 9.A discount dealer professes to sell his goods at cost price but uses a weight of 960 gms for a Kg weight .Find his gain %? Sol: Gain%=[(error*100)/(true value-error)]% =[(40*100)/1000-40)]%=25/6%

10.A man sold two flats for Rs 675,958 each .On one he gains 16% while on the other he losses 16%.How much does he gain or lose in the whole transaction? Sol: loss%=[common loss or gain%/10]2=(16/10)2=2.56%

11.A man sold two cows at Rs 1995 each. On one he lost 10% and on the other he gained 10%.what his gain or loss percent? Sol: If loss% and gain% is equal to 10 then there is no loss or no gain.

12.The price of an article is reduced by 25% in order to restore the must be increased by ? Sol: [x/(100-x)]*100 =[25/(100-25)]*100 =(25/75)*100=100/3%

13.Two discounts of 40% and 20% equal to a single discount of? Sol: {[(100-40)/100]*[(100-20)/100]}%=(60*80)/(100*100)% =48% single discount is equal to (100-48)%=52%

Top
Difficult Problems 1.The cost of an article including the sales tax is Rs 616.The rate of sales tax is 10%,if the shopkeeper has made a profit of 12%,then the cost price of the article is? Sol: 110% of S.P=616 S.P=(616*100)/110=Rs 560 C.P=(100*S.P)/(100+gain%) C.P. =(100*560)/(100+12)=Rs 500

2.Sam purchased 20 dozens of toys at the rate of 375 Rs per dozen. He sold each one of then at the rate of Rs 33.What was his percentage profit? Sol: C.P of one toy=Rs 375/12=Rs 31.25 S.P of one toy=Rs 33 profit=S.P-C.P=33-31.25=Rs 1.75 profit %=(profit/C.P)*100 =(1.75/31.25)*100 profit% =5.6%

3.Two third of consignment was sold at a profit of 5% and the remainder at a loss of 2%.If the total was Rs 400,the value of the consignment was? Sol: let the value total total total value be Rs x of 2/3=2x/3, value of 1/3=x/3 S.P value be Rs x S.P=[(105% of 2x/3)+(98% of x/3)] =(105*2x)/(100*3)+(98/100)+x/3 =308x/300 (308x/300)-x=400 8x/300=400 x=(300*400)/8=Rs 15000

4.Kunal bought a suitcase with 15% discount on the labelled price. He sold the suitcase for Rs 2880 with 20% profit on the labelled price .At what price did he buy the suitcase? Sol: let the labelled price be Rs x then 120% of x=2880 x=(2880*100)/120=Rs 2400 C.P=85% of the 2400 (85*2400)/100=Rs 2040

5.A tradesman gives 4% discount on the marked price and gives article free for buying every 15 articles and thus gains 35%. The marked price is above the cost price by Sol: let the C.P of each article be Rs 100 then C.P of 16 articles=Rs (100*16)=Rs 1600 S.P of 15 articles =1600*(135/100)=Rs 2160 S.P of each article =2160/15=Rs 144 If S.P is Rs 96, marked price =Rs 100 If S.P is Rs 144,marked price =(100/96)*144=Rs 15000 therefore marked price=50% above C.P

6.By selling 33m of cloth ,one gains the selling price of 11m.Find the gain percent? Sol: gain=S.P of 33m-C.P of 33m =11m of S.P S.P of 22m=C.P of 33m let C.P of each meter be Rs 1,then C.P of 22m=Rs 22 S.P of 22m=Rs 33 gain=S.P-C.P=33-22=Rs 11 gain%=(gain/C.P)*100 =(11/22)*100=50%

Top
7.The price of a jewel, passing through three hands, rises on the whole by 65%.if the first and second sellers earned 20% and 25% profit respectively,find the percentage profit earned by the third seller?

file:///E|/work/books/placement/09_Aptitude/profitandloss.html[1/28/2012 12:45:28 AM]

Sol:

let the original price of the jewel be Rs P and let the profit earned by the third seller be x% then (100+x)% of 125% of P=165% of P [(100+x)/100]*(125/100)*(120/100)*P=(165/100)*P 100+x=(165*100*100)/(125*120) 100+x=110 x=10%

8.When a producer allows 36% commission on the retail price of his product ,he earns a profit of 8.8%.what would be his profit percent if the commission is reduced by 24% Sol: let retail price =Rs 100 commission=Rs 36 S.P=retail price-commission=100-36=Rs 64 But profit=8.8% C.P=(100*C.P)/(gain+100)=(100*64)/(100+8.8)=Rs 1000/17 new commission=Rs 12 new S.P=100-12=Rs 88 gain=88-(1000/17)=Rs 496/17 gain%=gain*100/C.P =(496*17*100)(17*1000) gain%=49.6%

9.Vikas bought paper sheets for Rs 7200 and spent Rs 200 on transport. Paying Rs 600,he had 330 boxes made,which he sold at Rs 28 each. His profit percentage is Sol: total investments=7200+200+600=Rs 8000 total receipt=330*28=Rs 9240 gain=S.P-C.P =total receipt-total investments gain=9240-8000=Rs 1240 gain% =gain*100/C.P=1240*100/8000=15.5%

10.A person earns 15% on investment but loses 10% on another investment .If the ratio of the two investments be 3:5 ,what is the gain or loss on the two investments taken together? Sol: let the investments be 3x and 5x then total investment=8x total receipt=115% of 3x+90% of 5x =115*3x/100+90*5x/100=7.95x loss=C.P-S.P=8x-7.95x=0.05x loss%=.05x*100/8x=0.625%

11.The profit earned by selling an article for Rs 900 is double the loss incurred when the same article is sold for Rs 490 .At what price should the article be sold to make 25% profit? Sol: let C.P be Rs x 900-x=2(x-450) 3x=1800 x=Rs 600 C.P=Rs 600 , gain required=25% S.P=(100+gain%)*C.P/100 S.P=(100+25)*600/100=Rs 750

12.If an article is sold at 5% gain instead of 5% loss,the seller gets Rs 6.72 more. The C.P of the article is? Sol: let C.P be Rs x 105% of x-95% of x=6.72 (105/100)*x-(95/100)*x=6.72 x/10=6.72 x=Rs 67.21.

Back Back To Main

Top

Contact: 040-23000700

file:///E|/work/books/placement/09_Aptitude/profitandloss.html[1/28/2012 12:45:28 AM]

APTITUDE
Numbers H.C.F and L.C.M Decimal Fractions Simplification Square and Cube roots Average Problems on Numbers Problems on Ages Surds and Indices Percentage Profit and Loss Ratio And Proportions Partnership Chain Rule Time and Work Pipes and Cisterns Time and Distance Trains Boats and Streams Alligation or Mixture Simple Interest Compound Interest Logorithms Areas Volume and Surface area Races and Games of Skill Calendar Clocks Stocks ans Shares True Discount Bankers Discount Oddmanout and Series Data Interpretation probability Permutations and Combinations Puzzles BACK

PROFIT
Important Facts:

AND

LOSS

Cost Price:The price at which an article is purchased , is called its cost price,abbreviated as C.P. Selling Price:The price at which an article is sold , is called its selling price,abbreviated as S.P. Profit or Gain:If S.P. Is greater than C.P. The seller is said to have a profit or gain. Loss:if S.P. Is less than C.P., the seller is said to have incurred a loss.

FORMULAE
1.Gain=(S.P-C.P) 2.Loss=(C.P-S.P) 3.Loss or Gain is always reckoned on C.P. 4.Gain%=(gain*100)/C.P 5.Loss%=(loss*100)/C.P 6.S.P=[(100+gain%)/100]*C.P 7.S.P=[(100-loss%)/100]*C.P 8.C.P=(100*S.P)/(100+gain%) 9.C.P=(100*S.P)/(100-loss%) 10.If an article is sold at a gain of say,35%,then S.P=135% of C.P. 11.If an article is sold at a loss of say,35%,then S.P=65% of C.P. 12.When a person sells two similar items, one at a gain of say,x%,and the other at a loss of x%,then the seller always incurs a loss given by Loss%=[common loss and gain %/10]2=(x/10)2 13.If a trader professes to sell his goods at cost price,but uses false weight,then Gain%=[(error/(true value-error))*100]% 14.Net selling price=Marked price-Discount

BACK

file:///E|/work/books/placement/09_Aptitude/profitconcepts.html[1/28/2012 12:45:29 AM]

APTITUDE
Numbers H.C.F and L.C.M Decimal Fractions Simplification Square and Cube roots Average Problems on Numbers Problems on Ages Surds and Indices Percentage Profit and Loss Ratio And Proportions Partnership Chain Rule Time and Work Pipes and Cisterns Time and Distance Trains Boats and Streams Alligation or Mixture Simple Interest Compound Interest Logorithms Areas Volume and Surface area Races and Games of Skill Calendar Clocks Stocks ans Shares True Discount Bankers Discount Oddmanout and Series Data Interpretation probability Permutations and Combinations Puzzles BACK

PROFIT AND LOSS


DIFFICULT PROBLEMS
1.The of an article including the sales tax is Rs 616.The rate of sales tax is 10%,if the shopkeeper has made a profit of 12%,then the cost price of the article is? Sol: 110% of S.P=616 S.P=(616*100)/110=Rs 560 C.P=(100*S.P)/(100+gain%) C.P. =(100*560)/(100+12)=Rs 500 2.Sam purchased 20 dozens of toys at the rate of 375 Rs per dozen .He sold each one of then at the rate of Rs 33.What was his percentage profit? Sol: C.P of one toy=Rs 375/12=Rs 31.25 S.P of one toy=Rs 33 profit=S.P-C.P=33-31.25=Rs 1.75 profit %=(profit/C.P)*100 =(1.75/31.25)*100 profit% =5.6% 3.Two third of consignment was sold at a profit of 5% and the remainder at a loss of 2%.If the total was Rs 400,the value of the consignment was? Sol: let the total value be Rs x value of 2/3=2x/3, value of 1/3=x/3 total S.P value be Rs x total S.P=[(105% of 2x/3)+(98% of x/3)] =(105*2x)/(100*3)+(98/100)+x/3 =308x/300 (308x/300)-x=400 8x/300=400 x=(300*400)/8=Rs 15000 4.Kunal bought a suitcase with 15% discount on the labelled price. He sold the suitcase for Rs 2880 with 20% profit on the labelled price .At what price did he buy the suitcase? Sol: let the labelled price be Rs x then 120% of x=2880 x=(2880*100)/120=Rs 2400 C.P=85% of the 2400 (85*2400)/100=Rs 2040 5.A tradesman gives 4% discount on the marked price and gives article free for buying every 15 articles and thus gains 35%.The marked price is above the cost price by Sol: let the C.P of each article be Rs 100 then C.P of 16 articles=Rs (100*16)=Rs 1600 S.P of 15 articles =1600*(135/100)=Rs 2160 S.P of each article =2160/15=Rs 144 If S.P is Rs 96, marked price =Rs 100 If S.P is Rs 144,marked price =(100/96)*144=Rs 15000 therefore marked price=50% above C.P 6.By selling 33m of cloth ,one gains the selling price of 11m.Find the gain percent? Sol: gain=S.P of 33m-C.P of 33m =11m of S.P S.P of 22m=C.P of 33m let C.P of each meter be Rs 1,then C.P of 22m=Rs 22 S.P of 22m=Rs 33 gain=S.P-C.P=33-22=Rs 11 gain%=(gain/C.P)*100 =(11/22)*100=50% 7.The price of a jewel, passing through three hands, rises on the whole by 65%.if the first and second sellers earned 20% and 25% profit respectively,find the percentage profit earned by the third seller? Sol: let the original price of the jewel be Rs P and let the profit earned by the third seller be x% then (100+x)% of 125% of P=165% of P [(100+x)/100]*(125/100)*(120/100)*P=(165/100)*P

file:///E|/work/books/placement/09_Aptitude/profitdifficult.html[1/28/2012 12:45:29 AM]

100+x=(165*100*100)/(125*120) 100+x=110 x=10% 8.When a producer allows 36% commission on the retail price of his product ,he earns a profit of 8.8%.what would be his profit percent if the commission is reduced by 24% Sol: let retail price =Rs 100 commission=Rs 36 S.P=retail price-commission=100-36=Rs 64 But profit=8.8% C.P=(100*C.P)/(gain+100)=(100*64)/(100+8.8)=Rs 1000/17 new commission=Rs 12 new S.P=100-12=Rs 88 gain=88-(1000/17)=Rs 496/17 gain%=gain*100/C.P =(496*17*100)(17*1000) gain%=49.6% 9.Vikas bought paper sheets for Rs 7200 and spent Rs 200 on transport. Paying Rs 600,he had 330 boxes made,which he sold at Rs 28 each. His profit percentage is Sol: total investments=7200+200+600=Rs 8000 total receipt=330*28=Rs 9240 gain=S.P-C.P =total receipt-total investments gain=9240-8000=Rs 1240 gain% =gain*100/C.P=1240*100/8000=15.5% 10.A person earns 15% on investment but loses 10% on another investment .If the ratio of the two investments be 3:5 ,what is the gain or loss on the two investments taken together? Sol: let the investments be 3x and 5x then total investment=8x total receipt=115% of 3x+90% of 5x =115*3x/100+90*5x/100=7.95x loss=C.P-S.P=8x-7.95x=0.05x loss%=.05x*100/8x=0.625% 11.The profit earned by selling an article for Rs 900 is double the loss incurred when the same article is sold for Rs 490 .At what price should the article be sold to make 25% profit? Sol: let C.P be Rs x 900-x=2(x-450) 3x=1800 x=Rs 600 C.P=Rs 600 , gain required=25% S.P=(100+gain%)*C.P/100 S.P=(100+25)*600/100=Rs 750 12.If an article is sold at 5% gain instead of 5% loss,the seller gets Rs 6.72 more. The C.P of the article is? Sol: let C.P be Rs x 105% of x-95% of x=6.72 (105/100)*x-(95/100)*x=6.72 x/10=6.72 x=Rs 67.21.

BACK

file:///E|/work/books/placement/09_Aptitude/profitdifficult.html[1/28/2012 12:45:29 AM]

APTITUDE
Numbers H.C.F and L.C.M Decimal Fractions Simplification Square and Cube roots Average Problems on Numbers Problems on Ages Surds and Indices Percentage Profit and Loss Ratio And Proportions Partnership Chain Rule Time and Work Pipes and Cisterns Time and Distance Trains Boats and Streams Alligation or Mixture Simple Interest Compound Interest Logorithms Areas Volume and Surface area Races and Games of Skill Calendar Clocks Stocks ans Shares True Discount Bankers Discount Oddmanout and Series Data Interpretation probability Permutations and Combinations Puzzles BACK

PROFIT AND LOSS


SIMPLE PROBLEMS
1.A man buys an article for Rs.27.50 and sells it for Rs.28.60 Find the gain percent. Sol: C.P=Rs 27.50 S.P=Rs 28.60 then Gain=S.P-C.P=28.60-27.50=Rs 1.10 Gain%=(gain*100)/C.P% =(1.10*100)/27.50%=4% 2.If a radio is purchased for Rs 490 and sold for Rs 465.50 Find the loss%? Sol: C.P=Rs 490 S.P=Rs 465.50 Loss=C.P-S.P=490-465.50=Rs 24.50 Loss%=(loss*100)/C.P% =(24.50*100)/490%=5% 3.Find S.P when C.P=Rs 56.25 and Gain=20% Sol: S.P=[(100+gain%)/100]*C.P S.P=[(100+20)/100]56.25=Rs 67.50 4.Find S.P when C.P=Rs 80.40,loss=5% Sol: S.P=[(100-loss%)/100]*C.P S.P=[(100-5)/100]*80.40=Rs 68.34 5.Find C.P when S.P=Rs 40.60,gain=16%? Sol: C.P=(100*S.P)/(100+gain%) C.P=(100*40.60)/(100+16)=Rs 35 6.Find C.P when S.P=Rs 51.70 ,loss=12%? Sol: C.P=(100*S.P)/(100-loss%) C.P=(100*51.70)/(100-12)=Rs 58.75 7.A person incurs 5% loss by selling a watch for Rs 1140 . At what price should the watch be sold to earn 5% profit? Sol: Let the new S.P be Rs x then, (100-loss%):(1st S.P)=(100+gain%):(2nd S.P) (100-5)/1140=(100+5)/x x=(105*1140)/95=Rs 1260 8.If the cost price is 96% of the selling price,then what is the profit percent? Sol: let S.P=Rs 100 then C.P=Rs 96 profit=S.P-C.P=100-96=Rs 4 profit%=(profit*C.P)/100% =(4*96)/100=4.17% 9.A discount dealer professes to sell his goods at cost price but uses a weight of 960 gms for a Kg weight .Find his gain %? Sol: Gain%=[(error*100)/(true value-error)]% =[(40*100)/1000-40)]%=25/6% 10.A man sold two flats for Rs 675,958 each .On one he gains 16% while on the other he losses 16%.How much does he gain or lose in the whole transaction? Sol: loss%=[common loss or gain%/10]2=(16/10)2=2.56% 11.A man sold two cows at Rs 1995 each. On one he lost 10% and on the other he gained 10%.what his gain or loss percent? Sol: If loss% and gain% is equal to 10 then there is no loss or no gain. 12.The price of an article is reduced by 25% in order to restore the must be increased by ? Sol: [x/(100-x)]*100 =[25/(100-25)]*100 =(25/75)*100=100/3% 13.Two discounts of 40% and 20% equal to a single discount of? Sol: {[(100-40)/100]*[(100-20)/100]}%=(60*80)/(100*100)% =48% single discount is equal to (100-48)%=52%

BACK
file:///E|/work/books/placement/09_Aptitude/profitsimple.html[1/28/2012 12:45:30 AM]

file:///E|/work/books/placement/09_Aptitude/profitsimple.html[1/28/2012 12:45:30 AM]

Puzzles
Introduction: Puzzles are dealt in a detailed manner with certain solutions. Different puzzles are gathered from ShakuntalaDevis puzzle books. Keeping in mind certain puzzles for Infosys some reasoning problems are also dealt. Puzzle name at the top of each problem will give a brief idea regarding the mode of application. SELECTING A CANDIDATE For an advertisement of six local posts,twelve persons applied for the job.Can you tell in how many different ways the selection can be made? Solution: 6^12 SET OF BAT AND BALL When I wanted to buy a bat and ball, the shopkeeper said they would together cost Rs.3.75.But I did not want to buy a ball. The shopkeeper said that bat would cost 75paise more than the ball.What was the cost of bat and the ball? Soluton: Given that bat and ball together cost Rs.3.75 = 375paise Let cost of the ball alone be x. Given cost of the bat is 75p greater than cost of the ball. So cost of the bat = x+75 x+x+75 = 375 2x = 375 75 2x = 300 x = 150p Hence cost of the ball = Rs.1.50 =>Cost of the bat = 1.50 + 75 = Rs.2.25 PLAYING CHILDREN A group of boys and girls are playing.15 boys leave.There remain 2 girls for each boy.Then 45 girls leave.There remain 5 boys for each girl.How many boys were in the orginal group? Solution: Let B and G represent no.of boys and girls in the original group respectively. G ---------> 2 B-15 ----------> 1 G/B-15 = 2/1 i.e., 2 girls are left for 15 boys who are alone. G-45 -------------------->1 B-15 ------------------------->5 5 boys are left out when 15 girls are alone. =>G/B-15=2/1 --------------------------------(1) =>G-45/B-15 =1/5 ----------------------------(2) (1) & (2) => G = 2B-30 5G 225 = B - 15 5 ( 2B 30 ) = B 15 + 225 10B = B 15 + 225 + 150 9B = 360 B = 40 (1)=> G/40-15 = 2 G=50 girls.

Top
Problems 1.Reshma appeared for a maths exam.She was given 100 problems to solve.She tried to solve all of them correctly but some went wrong.But she scored 85. Her score was calculated by subtracting two times th no.of wrong answers from the no.of correct answers. How many problems did Reshma do correctly? Soluton: Assume W as wrong answers and R as correct answers Given total no.of questions as 100 R+W=100 ---------------------------(1) Score is calculated by subtracting 2 times wrong answers(2W) from right answers(R) and given as 85 R-2W=85 -------------------------------(2) (2)-(1) R-2W=85 R+W=100 --------------------W=5 Hence,100-5=95 is the no.of correct answers of Reshma.

file:///E|/work/books/placement/09_Aptitude/puzzles.html[1/28/2012 12:45:30 AM]

2.A RUNNNG RACE Sneha,Shilpa,Sushma join a running race.The distance is 1500 metres. Sneha beats Shilpa by 30 metres and Sushma by 100 metres.By how much could Shilpa beat Sushma over the full distance if they both ran as before? Solution: Total distance covered by Sneha=1500m Shilpa=1500-30=1470 Sneha =1500-100=1400 Distance covered by Shilpa=1500*1400/1470=1428.6 Distance to be covered by Shilpa to beat Sushma over full distance 1500-1428.6=71.4m 3.FILLING A CISTERN Pipe S1 can fill a cistern in 2 hours and pipe S2 in 3 hours.Pipe S3 can empty it in 5 hours.Supposing all the pipes are turned on when the cistern is competely empty,how long will it take to fill? Solution: S1 fills cistern in 1/2 hours S2 fills cistern in 1/3 hours S3 empties it in 1/5 hours A the pipes S1,S2,S3 working i.e.,filling the cistern 1/2+1/3-1/5=15+10-6/30=25-6/30=19/30 No.of hours to fill=30/19=1 11\19hours. 4.SEQUENCE PROBLEMS What are the next two terms in the sequence?1,1,5,17,61,217......... Solution: The order in this cases is Tn=3*Tn-1 +2*Tn-2 = 3(217)+2(61) = 773 Tn+1=3(773)+2(271) =2319+542 =2753 5.SEQUENCES What are the next two terms in the sequence? 1,1,5,17,61,217................. Solution: Tn=3Tn-1+2Tn-2 =3(217)+2(61) =773 Tn+1=3(773)+2(217)=2753 6.What are the next three terms to the series? 1+3+7+15+31+63........... Solution: Actual term is 2exp n-1. The next three terms are: 2exp7-1=127 2exp8-1=255 2exp9-1=511 7.A PROBLEM OF SHOPPING Samsrita went out for shopping by taking with her Rs.15/- in one rupee notes and 20p coins.On return she had as many one rupee notes as she originally had and as many 20p coins as she had one rupee notes.She came back with 1/3rd with what she had.How much did Samsrita spend and how much did she take? Solution: Let x be no.of rupee notes y be no.of 20pcoins. So,when going for shopping 100x+20y paise were there with Samsrita. On return she had 100y+20x paise. Also it is given that she had 1/3 rd of the orginal amount. 1/3(10x+20y)=100y+20x =>4x=280y =>x=7y y=1 => x=7 total =7.20 <15 y=2 =>x=14 total=14.40~=15 y=3 =>x=21 total=21.60 >>15 Hence the suitable value nearer to the amunt is 14.40 and so is the amount Samsrita carried with her. 1/3(1440)=480paise. Rs.4.80/- is amount spent by Samsrita. 8.A PUZZLE OF CULTURAL GROUPS Literary,Dramatic,Musical,Dancing and Painting are the 5 groups of a club.Literary group meets every other day,dramatic every third day,musical every fourth day,dancing every fifth day,painting every sixth day.Five groups meet on NewYears day of 1975 and starting from that day they met regularly on schedule. How many times did all the 5 groups meet on same day in first quarter excluding Jan1,1975.How many days did none of them met? Solution: LCM of 2,3,4,5,6 is 60. Hence excluding Jan1,1975 they met on every 61st day. 60/2=30 60/3=20 60/4=15 60/5=12 60/6=10 Literary meet for 30 2 day intervals. Dramatic meet for 20 3 day intervals. Musical meet for 15 4 day intervals. Dancing meet for 12 5 day intervals. Paintng meet for 10 6 day intervals. First quartr implies 3 months with 90 days. so inorder to a nswer that how many days do they don/t meet atleast once in first quarter is got by rounding all other categories. By counting all the intervals for other groups no.of days in

file:///E|/work/books/placement/09_Aptitude/puzzles.html[1/28/2012 12:45:30 AM]

Jan 8,Feb 7,Mar 9. Total is 24. 9.STOLEN MANGOES Three naughty boys stole some mangoes from a garden.Among them one counted and ate one.From the remainder he took precise third and went back to sleep. After sometime second boy woke up,counted the mangoes,ate one,took an exact third of the remaining and went back to sleep. After sometime third boy also did the same.In the morning they found one which was rotten and hence threw it away from the remainder,they made an exact division.How many mangoes did they steal? Solution: Let the noof mangoes be x After the first boy had eaten noof mangoes =x-1 After taking 1/3 rd of remaining it is 2x-2/3 Second boy ate one and tok 1/3 then it is 2(2x-2/3 -1)=4x-10/9 Third boy ate and tok 1/3 as 2(4x-19)/27=8x-38/27 Deducting the rotten one from remaining noogf mangoes left =8x-38/27=8x-765/27 This is divided among the three equally 8x-65/27=3n 8x=81n+65 Let n be equal to odd number 2b+1 8x=81(2b+1)+65 4x=81b+73 Let b=2c+1 4x=81(2c+1)+73 2x=81c+77 Let c=2d+1 x=81d+79 Least value of x for d=0 is 79 for d=1 is 160 for d=3 is 241 On verfication,79-1=78/3=26 Hence 79 is the correct answer.

Top
10.AN ELECTION PROBLEM My club had a problem recently.They had to appoint a Ssecretary from among the men and a joint secretary from among the women. We have a membership of 12 men and 10 women.In how many ways can the selection be made? Solution: As per the permutations and combinatins concept of mathematics, out of 12 men one selected as secretary can be done in 12c1 ways out of 10 women one selected as joint secretary can be done in 10c1 ways Hence one secrtary and one joint secretary is 12*10=120 11.SNAPPING A PLANE A plane has a span of 12 metres.It was photographed as it was flying directly overhead with a camera with a depth of 12cm.In the photo the span of the plane was same.Can you tell how higher was the plane when it was snapped? Solution: Actual span of the plane was 12m Span of the plane n photograph was 800m Depth of the plane is 12000m=12cm Hence,height of the plane when photographed be x 12000:800 = x:12 x=180m 12.A THRST PROBLEM Pramatha and Pranathi went camping.They took their own water in bg plastic bottles. Pramatha got thirsty and drank half the water in her bottle.A little later on she drank 1/3 f what was left.Sometime afterwards she drank 1/4 of what remained and so on Pranathi also had a bottle of the same size.She drank half the bottle at the first instance ,half of what remained when she drank next and so on. Aftr each took 10 drins ,the water Pramatha left was how many times greatr than the water Pranathi had left? Soltuion: Pramatha for the first drink 1/2 for the second drink 1/3 for the third drink 1/4 She drank 10 times and hence by the end of the 10th drink 1/11 of water she had in the bottle was over. Pranathi for the first drink 1/2 for the second drin 1/4 for the third drink 1/8 So Pranathi as per the given information has drunk 1/1024 of water she had in the bottle. Water left for Pramatha/Water left for Pranathi=1/11 / 1/1024 =1024/11 13.NAME OF THE EXCHANGE In GreatBritain some years ago the first threeletters of a telephone number usd to indicate the name of the exchangeHow many such arrangements of 3 letters is it possible to devise from the 26 letters of the alphabet? Solution: For permutations the no.of ways to select is npr=n!/(n-r)! Hence out of 26 letters the possible outcomes are 26p3=26!/23!=15600 14.VALUE OF THE SERIES Take a good look at the following series. 1-1/3+1/5-1/7+1/9-1/11+1/13.................... Find the value of the series and multiply the answer by 4.You will notce that a well-known vale approximates this product.Even more interestng is that as you add more terms the approximation becomes closer. Solution: Ths is an Arithmetic progresson with value .76 when two terms

file:///E|/work/books/placement/09_Aptitude/puzzles.html[1/28/2012 12:45:30 AM]

added becomes .77 and multiplidd by 4 it becomes 3.04 and 3.08 repeatition it is 3.14 which pi value adjusted to 2 decimals. 15.PLANTING TREES If you wished to plant some trees so that each was equidistant from every pther tree,what is the largest number you would plant? Solution: From the above informatin,as per equidstant formula of triangle,it is an equilateral triangle. Planting at all the three corners only 3 can be planted. The centroid is the middle point placed exactly equidistant from all the corners. Hence 4 plants can be planted at euqidistant. 16.LENGTH OF A TRAIN A train is travelling at the speed of 96 kmph.It takes 3 seconds to enter a tunnel and 30 seconds more to pass thorugh it completely. What is the length of the train and the tunnel? Solution: Speed of the train=95*5/18 m/sec Time taken=3ssec Length of the train=96*5/18*3=80m Length of the tunnel=96*5/18*30=800m 17.A GAME OF BILLIARDS Rajv,Sanjiv,Vinay were playing a game of Billiards.Rajiv can give Sanjiv 10 points in 50 and Sanjiv can give 10 points in 50.How many points in 50 must Rajiv gve Sanjiv to make an even game? Solution: Rajiv 50 Sanjiv 40 Sanjiv 50 Vinay 40 Sanjiv 40 Vinay 40*40/50=32 Rajiv 50 Sanjiv 40 Vinay 32 Rajiv gains 18 points than Vinay (50-32=18) 18.WOMEN AT CLUB SOCIALS Women outnumbered men by 16 at a club social.Seventimes the no.of women exceeds nine times the no.of men by 32.What was the number of men and women at club? Solution: Let W and M be the no.of women and men respectively. Given W=M+16.................(1) 7W=9M+32................(2) 7*(1)=>7W=7M+112........(3) (3)-(2) =>M=40 W=56

Top
19.FILLING WINE IN BARRELS A friend of mine in London has a very nice cellar.He has two large barrels in the cellar.The larger barrel is mostly empty.But the smaller barrel is only 5/6 th full f wine while it can hold 536 litres.Supposing he empties the smaller barrel and fills the bigger barrel to find that the wine fills only 4/9 of it.How much wine would the larger barrel hold when full? Solution: 5/6-------------------536 4/9-------------------? 5/6*536=4/9*x =>x=1005 litrs 20.WEIGHT OF A BRICK We have a brick of regular size.It weighs 4 kilograms.How much do you think asmaller brick four times small, but made of the same material weigh? Solution: The weight of the given brick = 4 Kilograms = 4000 grams It is given that the smaller brick's volume is 4 times smaller than the given one. The volume of smaller brick = 4 * 4 * 4 = 64 times smaller The smaller brick's weight = 4000/64 = 62.5 grams 21.A JUMPING FROG A frog starts climbing a 30 ft wall. Each hour it climbs 3 ft and slips back 2. How many days does it take him to reach the top and get out? Solution: Hours Feets 1 3ft - 2ft = 1ft 2 3ft - 2ft = 1ft 3 3ft - 2ft = 1ft : : : : 27 3ft - 2ft = 1 ft At the end of 27th hour the frog climbs 27fts and on 28th hour it climbs the remaining 3fts and comes out. 22.QUESTION OF PROBABILITY My friend Parveen teaches at a school. One day she conducted a test for 3 of her students and when they handed back the test papers, they had forgotten to write their names. Parveen returned the papers to the students at random.What is the probability that none of the 3 students will get the right paper?

file:///E|/work/books/placement/09_Aptitude/puzzles.html[1/28/2012 12:45:30 AM]

Solution: The possible combinations for the given condition are: 1 2 3 condition met 1 2 3 No 1 3 2 Yes 2 1 3 No 2 3 1 Yes 3 1 2 Yes 3 2 1 No The required probability = 2/6 = 1/3 23.MATHEMATICAL ODDITY In the 20th century there are only seven years whose numbers are a mathematical oddity because their numbers signify a prime number. The first one of its kind was the year 1951.Can you name the other sin? Solution: 1973,1979,1987,1993,1997,1999. 24.DOWN THE ESCALATOR Recently,while in London,I decided to walk down the escalator of a tube station.I did some quick calculation in my mind.I found that if I wa down 26 steps, I require 30 seconds to reach the bottom. However ,if I am able to step down 34 stairs I would only require 18 secionds to get to the bottom. If the time is measured from the moment the top step begins to descend to the time I step off the last step at the bottom , can you yel the height of the stairway in steps? Solution: Given that after walking 26 steps time needed to reach the bottom is 30 seconds ---(1) Similarly after walking 34 steps, it needs 18 seconds more to reach the bottom ---(2) from (1) & (2) it is clear that for (34-26) steps it took (30-18) seconds. i.e; 12 seconds for 8 steps in 1 second --------? (1*8)/12=2/3 steps/sec. i.e; 2 steps in 3 seconds for 30 seconds -----------how many steps? => (30*2)/3=20 steps. Finally Total number of steps = 26 + (steps covered in 30 seconds) =26+20=46 steps. 25.A COMPUTING PROBLEM Compute (100-1)(100-2)(100-3).................(100+1)(100+2)(100+3)=? Solution: =>(100-1)(100-2)(100-3).........(100-100)(100+1)(100+2)(100+3) =>(100-1)(100-2)(100-3).........(0)(100+1)(100+2)(100+3) =0 26.A CIRCLE AND A TRIANGLE What do you call a circle which passes through the vertices of of a triangle? Solution: Circumscribed.The meaning to circumscribe is to describe a figure round another so as to touch it at points without cutting.This is exactly what takes place with the circumscribed circle. To find the center of such a circle,we have to bisect the sides of a triangle and errect perpendiculars which are concurrent at the circumcentre. The radius r of the circumscribed circle of the triangle ABC is given by R=a/2SinA=b/2SinB=c/2Sinc 27.MISSING TERMS 48,60,58,72,68,104....... Here is a sequenc.Can you find the two missing terms? Solution: The odd terms are in the decimal system and differ by 10.And each even term is the preceeding odd term expressed in the octonamy system.78-8=70,remainder 6:9:8=1,remainder 1.Therefore the next two terms are: 78,116

Top
28.PACKETS OF CANDY If 6 men can pack 6 packets of candy in 6 minutes.How many are required to pack 60 packets in 60 minutes? Solution: Given that for 6 men to pack 6packets of candy it takes 6 miutes i.e., for 1 man to pack 1 packet it takes 1 miute. Hence,for 60 packets to be packed in 60 miutes we need 60 men. 29.A PROBLEM OF WEIGHT In my neighbourhood lives a man who weighs 200 pounds.He has two sons.They both weigh 100 pounds each.On a festival day they decide to go across the river on a boat to vissit some relations. But the boat could carry a maximum load at only 200 pounds.Yes they managed to come across the river by boat.How did they? Solution: Let us assume that c1 ----------------->first son c2 ---------------->second son f ------------------>father First the two sons c1,c2rowed across the river and c1 stayed behind while c2 returned in the boat to his father.The son remained behind while the father crossed the river.Then the other son brought back and the two brothers c1,c2 rowed over together.

file:///E|/work/books/placement/09_Aptitude/puzzles.html[1/28/2012 12:45:30 AM]

30.A PROBLEM OF CANDY BARS Recently I attended a birthday party.All the children in the party were given candy bars.All the children got 3 candy bars each except the child sitting in the end.She got only 2 candy bars.If only child had been given 2 candy bars there would have been 8 candy bars remaining.How many candy bars were there altogether to begin with? Solution: Suppose that there were x children at the party. If we distribute the candies in the above mentioned ways,then the resulting expressions 3(x-1)+2---------------------(1) 2x+8-------------------------(2) 3(x-1)+2=2x+8 3x-3+2=2x+8 3x-1=2x+8 x=9 Therefore the no.of candies for distribution 2x+8=2*9+8=18+8=26 31.FIND OUT THE SUM What is the sum of all numbers between 100 and 1000 which are divisible by 14? Solution: Let us assume that the sum is S s=112+126+........+994 s=14(8+9+10+.....................+71) s=14(8+71)(71-8+1)/2=7(79)(64)=35392 32.WALKING ALL THE WAY One day I decided to walk all the way from Banglore to Tumkur. I started exactly at noon and some one I know in Tumkur decided to walk all the way from Tumkur to Banglore and she started exactly at 2 PM on the same day. We met on the Banglore - Tumkur road at 5 past four and we both reached our destination at exactly the same time.At what time did we both arrive? Solution: There fore Total time = 2:00 PM + 3:55 PM + 1:55 PM = 7:50PM 33.THE TRAINS AND THE FALCON Two trains start from two opposite directions towards each other. The stations from which they start are 50 miles apart.Both the trains start at the same time towards the other train. As soon as it reaches the second one, it fies back to the first train and so on and so forth. It continues to do so, flying bacwards and forwards from one train to other until the trains meet. Both the trains travel at a speed of 25 miles per hour,and the bird flies at 100 miles per hour. How many miles will the falcon have flown before the trains meet? Solution: The trains travel at 25 miles per hour. Hence they will meet after travelling for one hour and the falcon also must have been flyingfor one hour. Since it travels at 100 miles per hour the bird must have flown 100 miles 34.VALUE OF 'S' S434S0, what number must be substituted with to make it divisible by 36? Solution: To be divisible by 36, the number has to be divided by 4 and 9 To be divisible by 4 , the number 'S' must be an even number and to be divisible by 9, the sum of all the digits of the number must be either equal to 9 or a multiple of 9 i.e; S + 4 + 3 + 4 + S + 0 = 9n The only digit that meets these two condition is 8 35.HEIGHT OF A ROOM Given the floor area of a room as 24 feet by 48 feet,and the space diagonal of the room as 56 feet,Can you find the height of the room? Solution: We know that, Volume of a cube =l pow(2) + b pow(2) + h Here the values of l & b are given. We also know that (diagonal)pow(2) = (length)pow(2) + => (x)pow(2) = 24 pow(2) + 48 => x = 24squareroot(5); Therefore,volume= h pow(2) + x pow(2) =56 => h=16 Therefore,height of the room=16 ft. pow(2). (breadth)pow(2). pow(2) pow(2)

36.A QUESTION OF DISTANCE It was a beautiful sunny morning. The air was fresh and a mild wind was blowing against my wind screen I was driving from Banglore to Brindavan Gardens. It took me 1 hour and 30 miutes to complete the journey. After lunch I returned to Banglore. I drove for 90 minutes.How do you explain it? Solution: 90 minutes = 1 hour 30 minutes. Hence,the driving time there and back is absolutely the same because 90 minutes and 1 hour and 30 minutes are one and the samething. 37.ARRANGE THE DIGITS: Arrange the digits 1,2,3,4,5,6,7,8,9 in order from left to right and use only + or _ signs so as to produce a result of 100? Sol: 123-45-67+89

file:///E|/work/books/placement/09_Aptitude/puzzles.html[1/28/2012 12:45:30 AM]

Top
38.DIVISION OF 45: Can you divide the number 45 into four parts such that when 2 is added to the first part, 2 is subtracted from the second part, 2 is multiplied by the third part, and the fourth part is divided by 2. All the four results to be the same number. Sol: Let us take A,B,C,D are the four equal parts and their sum is equal to 45. A+B+C+D = 45 Given that, A+2 = B-2 = C*2 = D/2 ----------------(1) =>A = B-4 ; C = (B-2)/2 ; D = 2(B-2) =>B-4 + [(B-2)/2] + 2(B-2) = 45 =>B=12 :. A=8 , C=5 , D=20 And condition (1) is satisfied. i .e; 8+2 = 12-2 = 5*2 = 20/2 39.SPECIAL NUMBER: What is the special about the number 1729? Sol: This is popularly known as Ramanujan's number. This is the known number that is a sum of two cubes in two different ways. i .e; (10*10*10) + (9*9*9) = 1729 ( 12*12*12) + (1*1*1) = 1729 40.PRICE OF A BOTTLE: A bottle and its cork together cost Rs 1.10, and the bottle costs Rs 1.00 more than its cork. What is the price of the bottle? Sol: Let us assume that, B = Price of the bottle C = Price of the cork It is given that, B + C = Rs 1.10 -------------------(1) and B C = Rs 1.00 -------------------(2) From the equations (1) and (2) it is clear that B = Rs 1.05 C = Rs 0.05 41.A QUESTION OF DISTANCE: It was a beautiful sunny morning. The air was fresh and a mild wind was blowing against my wind screen. I was driving from Banglore to Brindavan Gardens. It took me one hour and 30 minutes to complete the journey. After lunch I returned to banglore. I drove for 90 minutes. How do you explain it? Sol: 90 minutes = 1 hour 30 minutes. Hence, the driving time there and back is absolutely same because 90 minutes and 1 hour 30 minutes are one and the same. 42.FOR THE CHARITIES: One day when I was walking on the road in New Delhi, a group of boys approached me for donation for their poor boys' fund. I gave them a Rupee more than half the money I had in my purse. I must have walked a few more yards when a group of women approached me for donation, for an orphanage. I gave them 2 Rupees more than half the money I had in my purse. Then after a few yards I was approached by a religious group for a donation to the temple they were building. I gave them 3 Rupees more than half of what I had in my purse. At last I returned to my hotel room, I found that I had only one Rupee remaining in my purse. How much money did I have in my purse when I started? Sol: Suppose that the money in his purse when he started = x --------(1) For poor boys fund he gave x/2 + 1 Rs/- -----------------(2) i .e; 1 Rupee more than half the amount he had. Now he left with [x - (x/2 + 1)] =(x-2)/2 Rs/For Orphanage he gave [(x/2 1)/2] +2 = (x+6)/4 Rs/- -----(3) Now he left with [(x-2)/2] [(x+6)/4] = (x-10)/4 Rs/For temple building he gave [(x-10)/4]/2 + 3 = (x+14)/8 Rs/- ---(4) Now he left with [(x-10)/4] [(x+14)/8] = (x-34)/8 Rs/Finally he had 1 Rupee in his purse. i .e; Actual amount Expended amount = 1 :. from (1),(2),(3) and (4) we have x-{ [ (x+2)/2 ] + [ (x+6)/4 ] + [ (x+14)/8 ] } = 1 =>x-34=8 => x=42 :. The original amount in his purse at the beginning = Rs 42/43.A PAIR OF PALLINDROMES Multiply 21978 by 4.Comment about the result? Solution: 21978*4=87912. If we clearly observe the two numbers 21978 and 87912, the resultant number ie; 87912 is the reverse number of the number 219780. There fore these two numbers are a pair of paindromes. 44.A COMPUTING PROBLEM Compute: [5-2/(4-5)]pow(2). Solution: [5-2/(4-5)]pow(2) =[5-2/(-1)]pow(2) =[5+2]pow(2)=49.

file:///E|/work/books/placement/09_Aptitude/puzzles.html[1/28/2012 12:45:30 AM]

45.CONTINUE THE SERIES 1,3,6,10.Name the next three numbers in the series. Solution: The series is +2,+3,+4,----------------There fore next three numbers are: 10+5,10+5+6,10+5+6+7 = 15,21,28. 46.NAME FIVE TERMS OF ANOTHER SERIES These are the numbers that are the first five terms of a series that add upto 150.Can you name five terms of another series without fractons that add upto 153? [ex: 10,20,30,40,50. sum=150.] Solution: Each term in this series is a factorial, in other words,the product of a the numbers from 1 to that particular term considered. The first five terms of the series are,there fore 1,2,6,24,120. Their sum is 153 and are factorials of 1,2,3,4,5 respectively. 47.FIND OUT THE TIME What does 1408 hours mean? Solution: 1408 hours is actually 8 minutes past 2 PM. This is the system of twenty-four-hour cock.Writing the hours and minutes this way is a sensible means of avoiding confusion between AM and PM. 48.FIND OUT TTHE PATTERN What do you think the pattern is?6,24,60,120,210,336, Solution: The series is 1.2.3, 2.3.4, 3.4.5, 4.5.6, 5.6.7, 6.7.8,---------The next numbers would be 7.8.9, 8.9.10,------(or) 504, 720,-----------------------

Top
49.THE TRAIN AND THE CYCLIST A railway track runs parallel to a road until a bend brings the road to a level crossing. A cyclist rides along to work along the road every day at a constant speed of 12 miles per hour. He normally meets a train that travels in the same direction at the crossing One day he was late by 25 minutes and met the train 6 miles ahead of the level crossing. Can you figure out the speed of the train? Solution: Suppose that the train and the cyclist meet everyday at the crossing at 8:00A.M. i. e; starts at 7:00A.M Since the cyclist is late by 25 minutes, he starts at 7:25A.M As his speed is 12 miles per hour, he reaches the crossing at 7:25A.M + 1 Hour = 8:25A.M By 8:30A.M the train is 6 miles ahead of the cyclist The difference between their timings = 8:30A.M 8:25A.M = 5 Minutes The difference between their distances = 6 Miles Therefore,the train travels 6 miles in 5 minutes In 1 minute it travels ---------------------? = [(1 * 6) /5] * 60 = 72 Miles/hour 50.HEIGHT OF THE PALM TREE A palm tree was 90 cm high, when it was planted. It grows by an equal number of cm each year, and at the end of the seventh year it was one ninth taller than at the end of the sixth year. Can you tell how tall was the tree at the end of the twelfth year? Solution: Suppose that the tree grows x cm each year Height of the tree at the end of the sixth year = (90 + 6x) cm Growth in seventh year is, X = 1/9(90 + 6x) cm x = 10 + 2x/3 x = 30 Therefore the height of the tree at the end of the twelfth year=(90+12*30)=450cm 51.PROBLEM OF AGE Recently I attended a cocktail party. There was a beautiful young lady, who seemed very vitty and intelligent. She was posed a question, how old are you? . She answered , my age 3 years hence munltiply by 3 and from that subtracted 3 times my age 3 years ago will give you my exact age? How od is the lady? Solution: Let the age be x Age after 3 years wi be (x + 3) Age before 3 years = (x 3) Hence 3(x + 3) 3(x 3) = x x = 3x + 9 3x + 9 x = 18 Therefore the age of the lady = 18 years 52.CONSECUTIVE NATURASL NUMBERS There are two consecutive natural numbers whose product is equal to the product of three consecutive natural numbers,for example x(x+1) = y(y+1)(y+2).What are the two numbers? Solution: 14 * 15 = 5 * 6 * 7 53.SOME GLUTTON A man sitting beside me at a hotel ate idlis one after the other by

file:///E|/work/books/placement/09_Aptitude/puzzles.html[1/28/2012 12:45:30 AM]

ordering plate by plate. He said to me after drinking some water the last one I ate was my hundredth idli in last five days. Each day I ate 6 more than the previous day. Can you tell me how many he ate yesterday? Soluton: First day the number of idlis he ate be x Second day the count is (x+6) Third day ------------------------- (x+12) Fourth day ------------------------(x+18) Fifth day ---------------------------(x+24) Total is 5x + 6(1+2+3+4) =100 5x + 60 =100 x = 8 Day Idlis 1 8 2 14 3 20 4 26 5 32 So, on fourth day the number of idlis the man ate were 26.

Back Back To Main

Top

Contact: 040-23000700

file:///E|/work/books/placement/09_Aptitude/puzzles.html[1/28/2012 12:45:30 AM]

APTITUDE
Numbers H.C.F and L.C.M Decimal Fractions Simplification Square and Cube roots Average Problems on Numbers Problems on Ages Surds and Indices Percentage Profit and Loss Ratio And Proportions Partnership Chain Rule Time and Work Pipes and Cisterns Time and Distance Trains Boats and Streams Alligation or Mixture Simple Interest Compound Interest Logorithms Areas Volume and Surface area Races and Games of Skill Calendar Clocks Stocks ans Shares True Discount Bankers Discount Oddmanout and Series Data Interpretation probability Permutations and Combinations Puzzles BACK

PUZZLES
Puzzles are dealt in a detailed manner with certain solutions. Different puzzles are gathered from ShakuntalaDevis puzzle books. Keeping in mind certain puzzles for Infosys some reasoning problems are also dealt. Puzzle name at the top of each problem will give a brief idea regarding the mode of application.

SELECTING A CANDIDATE For an advertisement of six local posts,twelve persons applied for the job.Can you tell in how many different ways the selection can be made? Solution: 6^12

SET OF BAT AND BALL When I wanted to buy a bat and ball ,the shopkeeper said they would together cost Rs.3.75.But I did not want to buy a ball.The shopkeeper said that bat would cost 75paise more than the ball.What was the cost of bat and the ball? Soluton: Given that bat and ball together cost Rs.3.75 = 375paise Let cost of the ball alone be x. Given cost of the bat is 75p greater than cost of the ball. So cost of the bat = x+75 x+x+75 = 375 2x = 375 75 2x = 300 x = 150p Hence cost of the ball = Rs.1.50 =>Cost of the bat = 1.50 + 75 = Rs.2.25

PLAYING CHILDREN A group of boys and girls are playing.15 boys leave.There remain 2 girls for each boy.Then 45 girls leave.There remain 5 boys for each girl.How many boys were in the orginal group? Solution: Let B and G represent no.of boys and girls in the original group respectively.

file:///E|/work/books/placement/09_Aptitude/puzzles1.html[1/28/2012 12:45:31 AM]

G ---------> 2 B-15 ----------> 1 G/B-15 = 2/1 i.e., 2 girls are left for 15 boys who are alone. G-45 -------------------->1 B-15 ------------------------->5 5 boys are left out when 15 girls are alone. =>G/B-15=2/1 =>G-45/B-15 =1/5 (1) & (2) => G = 2B-30 5G 225 = B - 15 5 ( 2B 30 ) = B 15 + 225 10B = B 15 + 225 + 150 9B = 360 B = 40 (1)=> G/40-15 = 2 G=50 girls. --------------------------------(1) ----------------------------(2)

SOLVNG PROBLEMS Reshma appeared for a maths exam.She was given 100 problems to solve.She tried to solve all of them correctly but some went wrong.But she scored 85. Her score was calculated by subtracting two times th no.of wrong answers from the no.of correct answers.How many problems did Reshma do correctly? Soluton: Assume W as wrong answers and R as correct answers

Given total no.of questions as 100 R+W=100 ---------------------------(1)

Score is calculated by subtracting 2 times wrong answers(2W) from right answers(R) and given as 85 R-2W=85 -------------------------------(2) (2)-(1) R-2W=85 R+W=100 --------------------W=5 Hence,100-5=95 is the no.of correct answers of Reshma.

A RUNNNG RACE Sneha,Shilpa,Sushma join a running race.The distance is 1500 metres.Sneha beats Shilpa by 30 metres and Sushma by 100 metres.By how much could Shilpa beat

file:///E|/work/books/placement/09_Aptitude/puzzles1.html[1/28/2012 12:45:31 AM]

Sushma over the full distance if they both ran as before? Solution: Total distance covered by Sneha=1500m Shilpa=1500-30=1470 Sneha =1500-100=1400 Distance covered by Shilpa=1500*1400/1470=1428.6 Distance to be covered by Shilpa to beat Sushma over full distance 1500-1428.6=71.4m

FILLING A CISTERN Pipe S1 can fill a cistern in 2 hours and pipe S2 in 3 hours.Pipe S3 can empty it in 5 hours.Supposing all the pipes are turned on when the cistern is competely empty,how long will it take to fill? Solution: S1 fills cistern in 1/2 hours S2 fills cistern in 1/3 hours S3 empties it in 1/5 hours A the pipes S1,S2,S3 working i.e.,filling the cistern 1/2+1/3-1/5=15+10-6/30=25-6/30=19/30 No.of hours to fill=30/19=1 11\19hours.

SEQUENCE PROBLEMS What are the next two terms in the sequence?1,1,5,17,61,217................. Solution: The order in this cases is Tn=3*Tn-1 +2*Tn-2 = 3(217)+2(61) = 773 Tn+1=3(773)+2(271) =2319+542 =2753

SEQUENCES What are the next two terms in the sequence? 1,1,5,17,61,217................. Solution: Tn=3Tn-1+2Tn-2 =3(217)+2(61) =773 Tn+1=3(773)+2(217)=2753

file:///E|/work/books/placement/09_Aptitude/puzzles1.html[1/28/2012 12:45:31 AM]

What are the next three terms to the series? 1+3+7+15+31+63........... Solution: Actual term is 2exp n-1. The next three terms are: 2exp7-1=127 2exp8-1=255 2exp9-1=511

A PROBLEM OF SHOPPING Samsrita went out for shopping by taking with her Rs.15/- in one rupee notes and 20p coins.On return she had as many one rupee notes as she originally had and as many 20p coins as she had one rupee notes.She came back with 1/3rd with what she had.How much did Samsrita spend and how much did she take? Solution: Let x be no.of rupee notes y be no.of 20pcoins. So,when going for shopping 100x+20y paise were there with Samsrita. On return she had 100y+20x paise. Also it is given that she had 1/3 rd of the orginal amount. 1/3(10x+20y)=100y+20x =>4x=280y =>x=7y y=1 => x=7 total =7.20 <15 y=2 =>x=14 total=14.40~=15 y=3 =>x=21 total=21.60 >>15 Hence the suitable value nearer to the amunt is 14.40 and so is the amount Samsrita carried with her. 1/3(1440)=480paise. Rs.4.80/- is amount spent by Samsrita.

A PUZZLE OF CULTURAL GROUPS Literary,Dramatic,Musical,Dancing and Painting are the 5 groups of a club.Literary group meets every other day,dramatic every third day,musical every fourth day,dancing every fifth day,painting every sixth day.Five groups meet on NewYears day of 1975 and starting from that day they met regularly on schedule. How many times did all the 5 groups meet on same day in first quarter excluding Jan1,1975.How many days did none of them met? Solution: LCM of 2,3,4,5,6 is 60. Hence excluding Jan1,1975 they met on every 61st day. 60/2=30 60/3=20 60/4=15 60/5=12 60/6=10 Literary meet for 30 2 day intervals.

Dramatic meet for 20 3 day intervals. Musical meet for 15 4 day intervals.

file:///E|/work/books/placement/09_Aptitude/puzzles1.html[1/28/2012 12:45:31 AM]

Dancing meet for 12 5 day intervals. Paintng meet for 10 6 day intervals. First quartr implies 3 months with 90 days. So inorder to a nswer that how many days do they don/t meet atleast once in first quarter is got by rounding all other categories. By counting all the intervals for other groups no.of days in Jan 8,Feb 7,Mar 9. Total is 24.

STOLEN MANGOES Three naughty boys stole some mangoes from a garden.Among them one counted and ate one.From the remainder he took precise third and went back to sleep. After sometime second boy woke up,counted the mangoes,ate one,took an exact third of the remaining and went back to sleep. After sometime third boy also did the same.In the morning they found one which was rotten and hence threw it away from the remainder,they made an exact division.How many mangoes did they steal? Solution: Let the noof mangoes be x After the first boy had eaten noof mangoes =x-1 After taking 1/3 rd of remaining it is 2x-2/3 Second boy ate one and tok 1/3 then it is 2(2x-2/3 -1)=4x-10/9 Third boy ate and tok 1/3 as 2(4x-19)/27=8x-38/27 Deducting the rotten one from remaining noogf mangoes left=8x-38/27=8x765/27 This is divided among the three equally 8x-65/27=3n 8x=81n+65 Let n be equal to odd number 2b+1 8x=81(2b+1)+65 4x=81b+73 Let b=2c+1 4x=81(2c+1)+73 2x=81c+77 Let c=2d+1 x=81d+79 Least value of x for d=0 is 79 for d=1 is 160 for d=3 is 241 On verfication,79-1=78/3=26 Hence 79 is the correct answer.

AN ELECTION PROBLEM

file:///E|/work/books/placement/09_Aptitude/puzzles1.html[1/28/2012 12:45:31 AM]

My club had a problem recently.They had to appoint a Ssecretary from among the men and a joint secretary from among the women. We have a membership of 12 men and 10 women.In how many ways can the selection be made? Solution: As per the permutations and combinatins concept of mathematics, out of 12 men one selected as secretary can be done in 12c1 ways out of 10 women one selected as joint secretary can be done in 10c1 ways Hence one secrtary and one joint secretary is 12*10=120

SNAPPING A PLANE A plane has a span of 12 metres.It was photographed as it was flying directly overhead with a camera with a depth of 12cm.In the photo the span of the plane was same.Can you tell how higher was the plane when it was snapped? Solution: Actual span of the plane was 12m Span of the plane n photograph was 800m Depth of the plane is 12000m=12cm Hence,height of the plane when photographed be x 12000:800 = x:12 x=180m

A THRST PROBLEM Pramatha and Pranathi went camping.They took their own water in bg plastic bottles. Pramatha got thirsty and drank half the water in her bottle.A little later on she drank 1/3 f what was left.Sometime afterwards she drank 1/4 of what remained and so on Pranathi also had a bottle of the same size.She drank half the bottle at the first instance ,half of what remained when she drank next and so on. Aftr each took 10 drins ,the water Pramatha left was how many times greatr than the water Pranathi had left? Soltuion: Pramatha for the first drink 1/2 for the second drink 1/3 for the third drink 1/4 She drank 10 times and hence by the end of the 10th drink 1/11 of water she had in the bottle was over. Pranathi for the first drink 1/2 for the second drin 1/4 for the third drink 1/8 So Pranathi as per the given information has drunk 1/1024 of water she had in the bottle. Water left for Pramatha/Water left for Pranathi=1/11 / 1/1024 =1024/11

file:///E|/work/books/placement/09_Aptitude/puzzles1.html[1/28/2012 12:45:31 AM]

NAME OF THE EXCHANGE In GreatBritain some years ago the first threeletters of a telephone number usd to indicate the name of the exchangeHow many such arrangements of 3 letters is it possible to devise from the 26 letters of the alphabet? Solution: For permutations the no.of ways to select is npr=n!/(n-r)! Hence out of 26 letters the possible outcomes are 26p3=26!/23!=15600

VALUE OF THE SERIES Take a good look at the following series. 1-1/3+1/5-1/7+1/9-1/11+1/13.................... Find the value of the series and multiply the answer by 4.You will notce that a well-known vale approximates this product.Even more interestng is that as you add more terms the approximation becomes closer. Solution: Ths is an Arithmetic progresson with value .76 when two terms adde becomes .77 and multiplidd by 4 it becomes 3.04 and 3.08 repeatition it is 3.14 which pi value adjusted to 2 decimals.

PLANTING TREES If you wished to plant some trees so that each was equidistant from every pther tree,what is the largest number you would plant? Solution: From the above informatin,as per equidstant formula of triangle,it is an equilateral triangle. Planting at all the three corners only 3 can be planted. The centroid is the middle point placed exactly equidistant from all the corners. Hence 4 plants can be planted at euqidistant.

LENGTH OF A TRAIN A train is travelling at the speed of 96 kmph.It takes 3 seconds to enter a tunnel and 30 seconds more to pass thorugh it completely.What is the length of the train and the tunnel? Solution: Speed of the train=95*5/18 m/sec Time taken=3ssec Length of the train=96*5/18*3=80m Length of the tunnel=96*5/18*30=800m

file:///E|/work/books/placement/09_Aptitude/puzzles1.html[1/28/2012 12:45:31 AM]

A GAME OF BILLIARDS Rajv,Sanjiv,Vinay were playing a game of Billiards.Rajiv can give Sanjiv 10 points in 50 and Sanjiv can give 10 points in 50.How many points in 50 must Rajiv gve Sanjiv to make an even game? Solution: Rajiv Sanjiv Sanjiv 50 50 40 Sanjiv Vinay 40 40

Vinay 40*40/50=32 Sanjiv 40 Vinay 32

Rajiv 50

Rajiv gains 18 points than Vinay (50-32=18)

WOMEN AT CLUB SOCIALS Women outnumbered men by 16 at a club social.Seventimes the no.of women exceeds nine times the no.of men by 32.What was the number of men and women at club? Solution: Let W and M be the no.of women and men respectively.

Given W=M+16.................(1) 7W=9M+32................(2) 7*(1)=>7W=7M+112........(3) (3)-(2) =>M=40 W=56

FILLING WINE IN BARRELS A friend of mine in London has a very nice cellar.He has two large barrels in the cellar.The larger barrel is mostly empty.But the smaller barrel is only 5/6 th full f wine while it can hold 536 litres.Supposing he empties the smaller barrel and fills the bigger barrel to find that the wine fills only 4/9 of it.How much wine would the larger barrel hold when full? Solution: 5/6-------------------536 4/9-------------------? 5/6*536=4/9*x =>x=1005 litrs

WEIGHT OF A BRICK We have a brick of regular size.It weighs 4 kilograms.How much do you think asmaller brick four times small, but made of the same material weigh? Solution: The weight of the given brick = 4 Kilograms = 4000 grams It is given that the smaller brick's volume is 4 times smaller than the given one. The volume of smaller brick = 4 * 4 * 4 = 64 times smaller The smaller brick's weight = 4000/64 = 62.5 grams

file:///E|/work/books/placement/09_Aptitude/puzzles1.html[1/28/2012 12:45:31 AM]

A JUMPING FROG A frog starts climbing a 30 ft wall. Each hour it climbs 3 ft and slips back 2. How many days does it take him to reach the top and get out? Solution: Hours 1 2 3 : : 27 Feets 3ft - 2ft = 1ft 3ft - 2ft = 1ft 3ft - 2ft = 1ft : : 3ft - 2ft = 1 ft

At the end of 27th hour the frog climbs 27fts and on 28th hour it climbs the remaining 3fts and comes out.

QUESTION OF PROBABILITY My friend Parveen teaches at a school. One day she conducted a test for 3 of her students and when they handed back the test papers, they had forgotten to write their names. Parveen returned the papers to the students at random.What is the probability that none of the 3 students will get the right paper? Solution: The possible combinations for the given condition are: 1 1 1 2 2 3 3 2 2 3 1 3 1 2 3 3 2 3 1 2 1 condition met No Yes No Yes Yes No

The required probability = 2/6 = 1/3

MATHEMATICAL ODDITY In the 20th century there are only seven years whose numbers are a mathematical oddity because their numbers signify a prime number.The first one of its kind was the year 1951.Can you name the other sin? Solution: 1973,1979,1987,1993,1997,1999.

DOWN THE ESCALATOR Recently,while in London,I decided to walk down the escalator of a tube station.I did some quick calculation in my mind.I found that if I wa down 26 steps, I require 30 seconds to reach the bottom.However ,if I am able to step

file:///E|/work/books/placement/09_Aptitude/puzzles1.html[1/28/2012 12:45:31 AM]

down 34 stairs I would only require 18 secionds to get to the bottom. If the time is measured from the moment the top step begins to descend to the time I step off the last step at the bottom , can you yel the height of the stairway in steps? Solution: Given that after walking 26 steps time needed to reach the bottom is 30 seconds ---(1) Similarly after walking 34 steps, it needs 18 seconds more to reach the bottom from (1) & (2) it is clear that for (34-26) steps it took (30-18) seconds. i.e; 12 seconds for 8 steps in 1 second --------? (1*8)/12=2/3 steps/sec. i.e; 2 steps in 3 seconds for 30 seconds -----------how many steps? ---(2)

=> (30*2)/3=20 steps. Finally Total number of steps = 26 + (steps covered in 30 seconds)=26+20=46 steps.

A COMPUTING PROBLEM Compute (100-1)(100-2)(100-3).........................(100+1)(100+2)(100+3)=? Solution: =>(100-1)(100-2)(100-3).........(100-100)(100+1)(100+2)(100+3) =>(100-1)(100-2)(100-3).........(0)(100+1)(100+2)(100+3) =0

A CIRCLE AND A TRIANGLE What do you call a circle which passes through the vertices of of a triangle? Solution: Circumscribed.The meaning to circumscribe is to describe a figure round another so as to touch it at points without cutting.This is exactly what takes place with the circumscribed circle. To find the center of such a circle,we have to bisect the sides of a triangle and errect perpendiculars which are concurrent at the circumcentre. The radius r of the circumscribed circle of the triangle ABC is given by R=a/2SinA=b/2SinB=c/2Sinc

MISSING TERMS 48,60,58,72,68,104....... Here is a sequenc.Can you find the two missing terms? Solution:

file:///E|/work/books/placement/09_Aptitude/puzzles1.html[1/28/2012 12:45:31 AM]

The odd terms are in the decimal system and differ by 10.And each even term is the preceeding odd term expressed in the octonamy system.78-8=70,remainder 6:9:8=1,remainder 1. Therefore the next two terms are: 78,116

PACKETS OF CANDY If 6 men can pack 6 packets of candy in 6 minutes.How many are required to pack 60 packets in 60 minutes? Solution: Given that for 6 men to pack 6packets of candy it takes 6 miutes.i.e., for 1 man to pack 1 packet it takes 1 miute. Hence,for 60 packets to be packed in 60 miutes we need 60 men.

A PROBLEM OF WEIGHT In my neighbourhood lives a man who weighs 200 pounds.He has two weigh 100 pounds each.On a festival day they decide to go across boat to vissit some relations.But the boat could carry a maximum 200 pounds.Yes they managed to come across the river by boat.How Solution: Let us assume that c1 ----------------->first son c2 ---------------->second son f ------------------>father First the two sons c1,c2rowed across the river and c1 stayed behind while c2 returned in the boat to his father. The son remained behind while the father crossed the river. Then the other son brought back and the two brothers c1,c2 rowed over together. sons.They both the river on a load at only did they?

A PROBLEM OF CANDY BARS Recently I attended a birthday party.All the children in the party were given candy bars.All the children got 3 candy bars each except the child sitting in the end.She got only 2 candy bars.If only child had been given 2 candy bars there would have been 8 candy bars remaining.How many candy bars were there altogether to begin with? Solution: Suppose that there were x children at the party. If we distribute the candies in the above mentioned ways,then the resulting expressions 3(x-1)+2---------------------(1) 2x+8-------------------------(2) 3(x-1)+2=2x+8 3x-3+2=2x+8 3x-1=2x+8 x=9 Therefore the no.of candies for distribution 2x+8=2*9+8=18+8=26

file:///E|/work/books/placement/09_Aptitude/puzzles1.html[1/28/2012 12:45:31 AM]

FIND OUT THE SUM What is the sum of all numbers between 100 and 1000 which are divisible by 14? Solution: Let us assume that the sum is S s=112+126+........+994 s=14(8+9+10+.....................+71) s=14(8+71)(71-8+1)/2=7(79)(64)=35392

WALKING ALL THE WAY One day I decided to walk all the way from Banglore to Tumkur.I started exactly at noon and some one I know in Tumkur decided to walk all the way from Tumkur to Banglore and she started exactly at 2 PM on the same day. We met on the Banglore - Tumkur road at 5 past four and we both reached our destination at exactly the same time.At what time did we both arrive? Solution: There fore Total time = 2:00 PM + 3:55 PM + 1:55 PM = 7:50PM

THE TRAINS AND THE FALCON Two trains start from two opposite directions towards each other.The stations from which they start are 50 miles apart.Both the trains start at the same time towards the other train. As soon as it reaches the second one, it fies back to the first train and so on and so forth. It continues to do so, flying bacwards and forwards from one train to other until the trains meet. Both the trains travel at a speed of 25 miles per hour,and the bird flies at 100 miles per hour. How many miles will the falcon have flown before the trains meet? Solution: The trains travel at 25 miles per hour. Hence they will meet after travelling for one hour and the falcon also must have been flyingfor one hour. Since it travels at 100 miles per hour the bird must have flown 100 miles

VALUE OF 'S' S434S0, what number must be substituted with to make it divisible by 36? Solution: To be divisible by 36, the number has to be divided by 4 and 9 To be divisible by 4 , the number 'S' must be an even number and to be divisible by 9, the sum of all the digits of the number must be either equal to 9 or a multiple of 9 i.e; S + 4 + 3 + 4 + S + 0 = 9n The only digit that meets these two condition is 8

HEIGHT OF A ROOM Given the floor area of a room as 24 feet by 48 feet,and the space diagonal of

file:///E|/work/books/placement/09_Aptitude/puzzles1.html[1/28/2012 12:45:31 AM]

the room as 56 feet,Can you find the height of the room? Solution: We know that, Volume of a cube =l pow(2) + b pow(2) + h pow(2). Here the values of We also know that (diagonal)pow(2) => (x)pow(2) = (length)pow(2) + (breadth)pow(2). = 24 pow(2) + 48 pow(2) l & b are given.

=> x = 24squareroot(5); Therefore,volume= h pow(2) + x pow(2) =56 pow(2) => h=16 Therefore,height of the room=16 ft.

A QUESTION OF DISTANCE It was a beautiful sunny morning. The air was fresh and a mild wind was blowing against my wind screen I was driving from Banglore to Brindavan Gardens. It took me 1 hour and 30 miutes to complete the journey. After lunch it? Solution: 90 minutes = 1 hour 30 minutes. Hence,the driving time there and back is absolutely the same because 90 minutes and 1 hour and 30 minutes are one and the samething. I returned to Banglore. I drove for 90 minutes.How do you explain

ARRANGE THE DIGITS: Arrange the digits 1,2,3,4,5,6,7,8,9 in order from left to right and use only + or _ signs so as to produce a result of 100?

Sol: 123-45-67+89

DIVISION OF 45: Can you divide the number 45 into four parts such that when 2 is added to the first part, 2 is subtracted from the second part, 2 is multiplied by the third part, and the fourth part is divided by 2.All the four results to be the same number.

Sol: Let us take A,B,C,D are the four equal parts and their sum is equal to 45. A+B+C+D = 45 Given that, A+2 = B-2 = C*2 = D/2 ----------------(1) =>A = B-4 ; C = (B-2)/2 ; D = 2(B-2)

file:///E|/work/books/placement/09_Aptitude/puzzles1.html[1/28/2012 12:45:31 AM]

=>B-4 + [(B-2)/2] + 2(B-2) = 45 =>B=12 :. A=8 , C=5 , D=20 And condition (1) is satisfied. i .e; 8+2 = 12-2 = 5*2 = 20/2

SPECIAL NUMBER: What is the special about the number 1729?

Sol: This is popularly known as Ramanujan's number. This is the known number that is a sum of two cubes in two different ways. i .e; (10*10*10) + (9*9*9) = 1729 ( 12*12*12) + (1*1*1) = 1729

PRICE OF A BOTTLE: A bottle and its cork together cost Rs 1.10, and the bottle costs Rs 1.00 more than its cork. What is the price of the bottle?

Sol: Let us assume that, B = Price of the bottle C = Price of the cork It is given that, B + C = Rs 1.10 -------------------(1) and B C = Rs 1.00 -------------------(2)

From the equations (1) and (2) it is clear that B = Rs 1.05 C = Rs 0.05

A QUESTION OF DISTANCE: It was a beautiful sunny morning. The air was fresh and a mild wind was blowing against my wind screen. I was driving from Banglore to Brindavan Gardens. It took me one hour and 30 minutes to complete the journey. After lunch I returned to banglore. I drove for 90 minutes. How do you explain it?

Sol: 90 minutes = 1 hour 30 minutes. Hence, the driving time there and back is absolutely same because 90 minutes and 1 hour 30 minutes are one and the same.

file:///E|/work/books/placement/09_Aptitude/puzzles1.html[1/28/2012 12:45:31 AM]

FOR THE CHARITIES: One day when I was walking on the road in New Delhi, a group of boys approached me for donation for their poor boys' fund. I gave them a Rupee more than half the money I had in my purse. I must have walked a few more yards when a group of women approached me for donation, for an orphanage. I gave them 2 Rupees more than half the money I had in my purse. Then after a few yards I was approached by a religious group for a donation to the temple they were building. I gave them 3 Rupees more than half of what I had in my purse. At last I returned to my hotel room, I found that I had only one Rupee remaining in my purse. How much money did I have in my purse when I started?

Sol: Suppose that the money in his purse when he started = x -------------------(1) For poor boys fund he gave x/2 + 1 Rs/-(2) -------------------------------------

i .e; 1 Rupee more than half the amount he had. Now he left with [x - (x/2 + 1)] =(x-2)/2 Rs/For Orphanage he gave [(x/2 1)/2] +2 = (x+6)/4 Rs/Now he left with [(x-2)/2] [(x+6)/4] = (x-10)/4 Rs/For temple building he gave [(x-10)/4]/2 + 3 = (x+14)/8 Rs/---------(4) ----------------(3)

Now he left with [(x-10)/4] [(x+14)/8] = (x-34)/8 Rs/Finally he had 1 Rupee in his purse. i .e; Actual amount Expended amount = 1 :. from (1),(2),(3) and (4) we have x-{ [ (x+2)/2 ] + [ (x+6)/4 ] + [ (x+14)/8 ] } = 1 =>x-34=8 => x=42 :. The original amount in his purse at the beginning = Rs 42/-

A PAIR OF PALLINDROMES Multiply 21978 by 4.Comment about the result? Solution: 21978*4=87912. If we clearly observe the two numbers 21978 and 87912, the resultant number ie; 87912 is the reverse number of the number 219780. There fore these two numbers are a pair of paindromes.

A COMPUTING PROBLEM Compute: Solution: [5-2/(4-5)]pow(2) [5-2/(4-5)]pow(2).

file:///E|/work/books/placement/09_Aptitude/puzzles1.html[1/28/2012 12:45:31 AM]

=[5-2/(-1)]pow(2) =[5+2]pow(2)=49.

CONTINUE THE SERIES 1,3,6,10.Name the next three numbers in the series. Solution: The series is +2,+3,+4,----------------There fore next three numbers are: 10+5,10+5+6,10+5+6+7 = 15,21,28.

NAME FIVE TERMS OF ANOTHER SERIES These are the numbers that are the first five terms of a series that add upto 150.Can you name five terms of another series without fractons that add upto 153? [ex: 10,20,30,40,50. sum=150.] Solution: Each term in this series is a factorial, in other words,the product of a the numbers from 1 to that particular term considered. The first five terms of the series are,there fore 1,2,6,24,120. Their sum is 153 and are factorials of 1,2,3,4,5 respectively.

FIND OUT THE TIME What does 1408 hours mean? Solution: 1408 hours is actually 8 minutes past 2 PM.

This is the system of twenty-four-hour cock.Writing the hours and minutes this way is a sensible means of avoiding confusion between AM and PM.

FIND OUT TTHE PATTERN What do you think the pattern is? Solution: The series is 1.2.3, 2.3.4, 3.4.5, 4.5.6, 5.6.7, 6.7.8,---------8.9.10,------6,24,60,120,210,336,

The next numbers would be 7.8.9, (or)

504, 720,-----------------------

THE TRAIN AND THE CYCLIST A railway track runs parallel to a road until a bend brings the road to a level crossing. A cyclist rides along to work along the road every day at a constant speed of 12 miles per hour. He normally meets a train that travels in the same direction at the crossing

file:///E|/work/books/placement/09_Aptitude/puzzles1.html[1/28/2012 12:45:31 AM]

One day he was late by 25 minutes and met the train 6 miles ahead of the level crossing. Can you figure out the speed of the train? Solution: Suppose that the train and the cyclist meet everyday at the crossing at 8:00A.M. i. e; starts at 7:00A.M Since the cyclist is late by 25 minutes, he starts at 7:25A.M As his speed is 12 miles per hour, he reaches the crossing at 7:25A.M + 1 Hour = 8:25A.M By 8:30A.M the train is 6 miles ahead of the cyclist The difference between their timings = 8:30A.M 8:25A.M = 5 Minutes The difference between their distances = 6 Miles Therefore,the train travels 6 miles in 5 minutes In 1 minute it travels ---------------------? = [(1 * 6) /5] * 60 = 72 Miles/hour

HEIGHT OF THE PALM TREE A palm tree was 90 cm high, when it was planted. It grows by an equal number of cm each year, and at the end of the seventh year it was one ninth taller than at the end of the sixth year. Can you tell how tall was the tree at the end of the twelfth year? Solution: Suppose that the tree grows x cm each year Height of the tree at the end of the sixth year = (90 + 6x) cm Growth in seventh year is, X = 1/9(90 + 6x) cm x = 10 + 2x/3 x = 30 Therefore the height of the tree at the end of the twelfth year=(90+12*30)=450cm

PROBLEM OF AGE Recently I attended a cocktail party. There was a beautiful young lady, who seemed very vitty and intelligent. She was posed a question, how old are you? . She answered , my age 3 years hence munltiply by 3 and from that subtracted 3 times my age 3 years ago will give you my exact age? How od is the lady? Solution: Let the age be x Age after 3 years wi be (x + 3) Age before 3 years = (x 3) Hence 3(x + 3) 3(x 3) = x x = 3x + 9 3x + 9 x = 18 Therefore the age of the lady = 18 years

file:///E|/work/books/placement/09_Aptitude/puzzles1.html[1/28/2012 12:45:31 AM]

CONSECUTIVE NATURASL NUMBERS There are two consecutive natural numbers whose product is equal to the product of three consecutive natural numbers,for example x(x+1) = y(y+1)(y+2).What are the two numbers? Solution: 14 * 15 = 5 * 6 * 7

SOME GLUTTON A man sitting beside me at a hotel ate idlis one after the other by ordering plate by plate. He said to me after drinking some water the last one I ate was my hundredth idli in last five days. Each day I ate 6 more than the previous day. Can you tell me how many he ate yesterday? Soluton: First day the number of idlis he ate be x Second day the count is (x+6) Third day ------------------------- (x+12) Fourth day ------------------------(x+18) Fifth day ---------------------------(x+24) Total is 5x + 6(1+2+3+4) =100 5x + 60 =100 x = 8 Day 1 2 3 4 5 Idlis 8 14 20 26 32

So, on fourth day the number of idlis the man ate were 26.

BACK

file:///E|/work/books/placement/09_Aptitude/puzzles1.html[1/28/2012 12:45:31 AM]

APTITUDE
Numbers H.C.F and L.C.M Decimal Fractions Simplification Square and Cube roots Average Problems on Numbers Problems on Ages Surds and Indices Percentage Profit and Loss Ratio And Proportions Partnership Chain Rule Time and Work Pipes and Cisterns Time and Distance Trains Boats and Streams Alligation or Mixture Simple Interest Compound Interest Logorithms Areas Volume and Surface area Races and Games of Skill Calendar Clocks Stocks ans Shares True Discount Bankers Discount Oddmanout and Series Data Interpretation probability Permutations and Combinations Puzzles BACK

RACES AND GAMES OF SKILL


RACES AND GAMES OF SKILL Races :- A contest of speed in running ,riding,driving,sailing or rowing is called a Race

Race Course :-The ground or path on which contests are made is called a race course

STARTING POINT :-The point from which a race begins is called starting point.

Winning point or goal:-The point set to bound a race is called a winning point.

Dead Heat Race:-If all the persons contesting a race reach the goal exactly at the same time,then the race is called a dead heat race.

Start:-suppose A and B are two contestants in a race .If before the start of the race,A is at the satrtint point and B is ahead of A by 12 metres. Then we say that "A gives B a start 12 metres.

->To cover a race of 100metres in this case,A will have to cover 100m while B will have to cover 88m=(100-12)

->In a100m race 'A can give B 12m' or 'A can give B a start of 12m' or 'A beats B by 12m'means that while A runs 100m B runs 88m. GAMES:- A game of 100m,means that the person among the contestants who scores 100 points first is the winner.

If A scores 100 points while B scores only 80 points then we say that 'A can give B 20 points'. BACK

file:///E|/work/books/placement/09_Aptitude/raceconcept.html[1/28/2012 12:45:32 AM]

APTITUDE
Numbers H.C.F and L.C.M Decimal Fractions Simplification Square and Cube roots Average Problems on Numbers Problems on Ages Surds and Indices Percentage Profit and Loss Ratio And Proportions Partnership Chain Rule Time and Work Pipes and Cisterns Time and Distance Trains Boats and Streams Alligation or Mixture Simple Interest Compound Interest Logorithms Areas Volume and Surface area Races and Games of Skill Calendar Clocks Stocks ans Shares True Discount Bankers Discount Oddmanout and Series Data Interpretation probability Permutations and Combinations Puzzles BACK

RACES AND GAMES OF SKILL


PROBLEMS :- 1) In a 1 km race,A beats B by 28 m or 7sec.Find A's time over the course? Sol: B covers 28 m in 7sec so,B's time over the course = 7/28 *1000 =250 sec A's time over the course =250-7 =243 sec = 4min ,3 sec. 2) A runs 1 3/4 times as fast as B.If A gives B a start of 84 m,how far must be winning post be so that A and B might reach it at the same time ? Sol: Ratio of rates of A:B =7/4 :1 =7 :4 In a game of 7 m A gains 3m over B. 3m are gained by A in a race of 7m 84 m are gained by A in a race of 7*84/3 =196 m Winning post must be 196m away from the starting point. 3)A can run 1km in 3 min ,10sec and B can cover same distance in 3 min 20 sec. By what distance A beat B? Sol: clearly A beats B by 10 sec. Distance covered by B in 10 sec =1000/200 *10 =50 m A beats B by 50 metres. 4) In a 100m race,A runs at 8km per hour.If A gives b a start of 4 m and still beats him by 15 sec,what is the speed of B? Sol: 8000 m -------60*60 sec 100m ------- 60*60*1000/8000 =45 sec. Time taken by A to cover 100m =45 sec. B covers 100-4 m =96 min 45 sec =60 sec B's speed =96 *60*60/60*1000 =5.76 km/hr 5) A and B take part in 100m race .A runs at 5km per hour. A gives B a start of 8 m and still beats him by 8 sec. What is the speed of B? Sol : A'speed = 5km/hr =5*5/18 =25/18 m/s Time taken by A to cover 100m =100*18/25 =7.2 sec Time taken by A to cover 92m = 72+8 =80 sec B's speed =92*18/80*5 = 4.14 kmph. 6) A runs 1 2/3 times as fast as B.If A gives B a start of 80 m, how far must the winning post be so that A and B might reach if at the same time? Sol: Ratio of the speed of A and B =5/3 :1 Thus in a race of 5m ,A gains 2m over B 2m are gained by in a race of 5m 80 m will be gained by A in a race of 5/2* 80 =200 m 7) A,B and C are three contestans in a kmrace .If A can give B a start of 40 m and A can give C a start of 64 m,how amny metres start can b give C? Sol: A covers 100m,B covers (1000-40) =960 m C covers 1000-64 m or 936m when B covers 960 m,C covers 936 m when B covers 1000 m,C covers 936*1000/960 m =975 m B can give C a start of 1000-975 or 25 m. 8) In a 100m race,A covers the distance in 36 sec and B in 45 sec.In this race A beats B by? Sol: Distance covered by B in 9secs =100*9/45 =20 m A beats B by 20m 9) In a 200 m race A beats B by 35m or 7 sec.What is the A's time over the course? Sol: B runs 35 m in 7sec. B covers 200m in =7*200/35 = 40 sec. B's time over the course =40 sec A's time over the course =40-7 =33 sec. 10) In a 300 m race A beats B by 22.5 m or 6 sec.What is the B's time over the course? Sol: B runs 22.5 m in 6sec. B runs 300m in =6*300*2/45 =80 sec. B's time over the course =80 sec. 11) A can run 22.5 m while B runs 25 m.In a kilometre race B beats A by? Sol: B runs 25 m ,A runs 45/2 m B runs 1000 A runs = 1000*45/2*25 =900m B beats A by 100m 12)In a 500 m race, the ratio of the speeds of two contestants A and B is 3:4.A has a start of 140 m.Then ,A win by B? Sol: The speeds of A and B =3:4 To reach the winning post A will have to cover a distance of 500-140 m i.e 360 m while A covers 3m ,b covers 4m A covers 360m B covers 4/3*360=480m Thus when A reaches the winning post, b covers 480m and therefore remains 20m behind. A wins by 20m.

file:///E|/work/books/placement/09_Aptitude/raceproblems.html[1/28/2012 12:45:33 AM]

13) In a 100m race,A can beat B by 25 m and B can beat c by 4m.In the same race A can beat C by/ Sol: If A:B =100 :75 B :C=100 :96 then A :C =A/B*B/C=100/75* 100/96 = 100/72 A beats C by 100 -72 m=28 m 14) In a 100 race,A can give B 10 mand C 28 m.In the same race B can give C? Sol: A:B =100 :90 A :C=100 :72 B:C =B/A *A/C =90/100*100/72 =90/72 When B runs 90 C runs 72 when B runs 100 C runs =72*100/90 B beats C by 20m 15) In a 100m race ,A beats B by 10 m and C by 13 m.In the race of 180m. B will beat c by? Sol :A : B =100 :90 A/B =100/90 A/C =100/87 B/C =B/A *A/C =90 /87 When B runs 90m C runs 87 When B runs 180 m then C runs =87 *180/90 B beats C by 180-174= 6m 16) In a race of 200 m, A can beat B by 31 m and C by 18m.In arace of 350 m, C will beat B by? Sol : In a race of 200 m A :B =200:169 A :C =200 :182 C/B =C/A*A/B =182/200*200/169 When C runs 182 m B runs 169 when C runs 350 m B runs =350*169/182 =325m 17) In agame of 100 points A can give B 20 points and C 28 points then B can give C? In a game of 100 points. A :B =100 :80 A :C =100 :72 B/C=B/A*A/C =80/100*100/72 = 80/72 when B runs 80m C runs 72 when B runs 100m C runs =100*72/80 =90 B can give C 10 points in agame of 100. 18)At a game of billiards,A can give B 15 points in 60 and A can give C 20 points in60.How amny points can B give C in a game of 90? Sol: A:B =60:45 A:C =60:40 B/C =B/A*A/C =45/60*60/40 =90/80 B can give C 10 points in agame of 90. 19) in agame of 80 points,A can give B 5 points and C 15 points.then how many points B can give C in agame of 60? Sol: A :B =80 :75 A :C =80:65 B/C=B/A*A/C = 75/80*80/65 =15/13 B:C =60:52 BACK

file:///E|/work/books/placement/09_Aptitude/raceproblems.html[1/28/2012 12:45:33 AM]

APTITUDE
Numbers H.C.F and L.C.M Decimal Fractions Simplification Square and Cube roots Average Problems on Numbers Problems on Ages Surds and Indices Percentage Profit and Loss Ratio And Proportions Partnership Chain Rule Time and Work Pipes and Cisterns Time and Distance Trains Boats and Streams Alligation or Mixture Simple Interest Compound Interest Logorithms Areas Volume and Surface area Races and Games of Skill Calendar Clocks Stocks ans Shares True Discount Bankers Discount Oddmanout and Series Data Interpretation probability Permutations and Combinations Puzzles

RACES AND GAMES OF SKILL


CONCEPTS PROBLEMS

file:///E|/work/books/placement/09_Aptitude/races.html[1/28/2012 12:45:33 AM]

Races and Games of Skill


Important Facts and Formulae: Races: A contest of speed in running ,riding,driving,sailing or rowing is called a Race Race Course: The ground or path on which contests are made is called a race course Starting Point: The point from which a race begins is called starting point. Winning point or goal: The point set to bound a race is called a winning point. Dead Heat Race: If all the persons contesting a race reach the goal exactly at the same time,then the race is called a dead heat race. Start: Suppose A and B are two contestants in a race .If before the start of the race,A is at the satrtint point and B is ahead of A by 12 metres. Then we say that "A gives B a start 12 metres. ->To cover a race of 100metres in this case,A will have to cover 100m while B will have to cover 88m=(100-12) ->In a100m race 'A can give B 12m' or 'A can give B a start of 12m' or 'A beats B by 12m'means that while A runs 100m B runs 88m. Games: A game of 100m,means that the person among the contestants who scores 100 points first is the winner. If A scores 100 points while B scores only 80 points then we say that 'A can give B 20 points'.

Top
Problems: 1) In a 1 km race,A beats B by 28 m or 7sec.Find A's time over the course? Sol: B covers 28 m in 7sec so,B's time over the course = 7/28 *1000 =250 sec A's time over the course =250-7 =243 sec = 4min ,3 sec. 2) A runs 1 3/4 times as fast as B.If A gives B a start of 84 m,how far must be winning post be so that A and B might reach it at the same time ? Sol: Ratio of rates of A:B =7/4 :1 =7 :4 In a game of 7 m A gains 3m over B. 3m are gained by A in a race of 7m 84 m are gained by A in a race of 7*84/3 =196 m Winning post must be 196m away from the starting point. 3)A can run 1km in 3 min ,10sec and B can cover same distance in 3 min 20 sec. By what distance A beat B? Sol: clearly A beats B by 10 sec. Distance covered by B in 10 sec =1000/200 *10 =50 m A beats B by 50 metres. 4) In a 100m race,A runs at 8km per hour.If A gives b a start of 4 m and still beats him by 15 sec,what is the speed of B? Sol: 8000 m -------60*60 sec 100m ------- 60*60*1000/8000 =45 sec. Time taken by A to cover 100m =45 sec. B covers 100-4 m =96 min 45 sec =60 sec B's speed =96 *60*60/60*1000 =5.76 km/hr 5) A and B take part in 100m race .A runs at 5km per hour. A gives B a start of 8 m and still beats him by 8 sec. What is the speed of B? Sol : A'speed = 5km/hr =5*5/18 =25/18 m/s Time taken by A to cover 100m =100*18/25 =7.2 sec Time taken by A to cover 92m = 72+8 =80 sec B's speed =92*18/80*5 = 4.14 kmph. 6) A runs 1 2/3 times as fast as B.If A gives B a start of 80 m, how far must the winning post be so that A and B might reach if at the same time? Sol: Ratio of the speed of A and B =5/3 :1 Thus in a race of 5m ,A gains 2m over B 2m are gained by in a race of 5m 80 m will be gained by A in a race of 5/2* 80 =200 m 7) A,B and C are three contestans in a kmrace .If A can give B a

file:///E|/work/books/placement/09_Aptitude/racesandgamesofskill.html[1/28/2012 12:45:33 AM]

start of 40 m and A can give C a start of 64 m,how amny metres start can b give C? Sol: A covers 100m,B covers (1000-40) =960 m C covers 1000-64 m or 936m when B covers 960 m,C covers 936 m when B covers 1000 m, C covers 936*1000/960 m =975 m B can give C a start of 1000-975 or 25 m. 8) In a 100m race,A covers the distance in 36 sec and B in 45 sec.In this race A beats B by? Sol: Distance covered by B in 9secs =100*9/45 =20 m A beats B by 20m 9) In a 200 m race A beats B by 35m or 7 sec.What is the A's time over the course? Sol: B runs 35 m in 7sec. B covers 200m in =7*200/35 = 40 sec. B's time over the course =40 sec A's time over the course =40-7 =33 sec. 10) In a 300 m race A beats B by 22.5 m or 6 sec.What is the B's time over the course? Sol: B runs 22.5 m in 6sec. B runs 300m in =6*300*2/45 =80 sec. B's time over the course =80 sec.

Top
11) A can run 22.5 m while B runs 25 m.In a kilometre race B beats A by? Sol: B runs 25 m ,A runs 45/2 m B runs 1000 A runs = 1000*45/2*25 =900m B beats A by 100m 12)In a 500 m race, the ratio of the speeds of two contestants A and B is 3:4.A has a start of 140 m.Then ,A win by B? Sol: The speeds of A and B =3:4 To reach the winning post A will have to cover a distance of 500-140 m i.e 360 m while A covers 3m, b covers 4m A covers 360m B covers 4/3*360=480m Thus when A reaches the winning post, b covers 480m and therefore remains 20m behind. A wins by 20m. 13) In a 100m race,A can beat B by 25 m and B can beat c by 4m. In the same race A can beat C by/ Sol: If A:B =100 :75 B :C=100 :96 then A :C =A/B*B/C=100/75* 100/96 = 100/72 A beats C by 100 -72 m=28 m 14) In a 100 race,A can give B 10 mand C 28 m.In the same race B can give C? Sol: A:B =100 :90 A :C=100 :72 B:C =B/A *A/C =90/100*100/72 =90/72 When B runs 90 C runs 72 when B runs 100 C runs =72*100/90 B beats C by 20m 15) In a 100m race ,A beats B by 10 m and C by 13 m.In the race of 180m. B will beat c by? Sol :A : B =100 :90 A/B =100/90 A/C =100/87 B/C =B/A *A/C =90 /87 When B runs 90m C runs 87 When B runs 180 m then C runs =87 *180/90 B beats C by 180-174= 6m 16) In a race of 200 m, A can beat B by 31 m and C by 18m.In arace of 350 m, C will beat B by? Sol : In a race of 200 m A :B =200:169 A :C =200 :182 C/B =C/A*A/B =182/200*200/169 When C runs 182 m B runs 169 when C runs 350 m B runs =350*169/182 =325m 17) In agame of 100 points A can give B 20 points and C 28 points then B can give C? Sol: In a game of 100 points.A :B =100 :80 A :C =100 :72 B/C=B/A*A/C =80/100*100/72 = 80/72 when B runs 80m C runs 72 when B runs 100m C runs =100*72/80 =90 B can give C 10 points in agame of 100. 18)At a game of billiards,A can give B 15 points in 60 and A can give C 20 points in60.How amny points can B give C in a game of 90? Sol: A:B =60:45 A:C =60:40 B/C =B/A*A/C =45/60*60/40 =90/80 B can give C 10 points in agame of 90. 19) in agame of 80 points,A can give B 5 points and C 15 points. Then how many points B can give C in agame of 60? Sol: A :B =80 :75 A :C =80:65 B/C=B/A*A/C = 75/80*80/65 =15/13 B:C =60:52

Back Back To Main

Top

Contact: 040-23000700

file:///E|/work/books/placement/09_Aptitude/racesandgamesofskill.html[1/28/2012 12:45:33 AM]

file:///E|/work/books/placement/09_Aptitude/racesandgamesofskill.html[1/28/2012 12:45:33 AM]

APTITUDE
Numbers H.C.F and L.C.M Decimal Fractions Simplification Square and Cube roots Average Problems on Numbers Problems on Ages Surds and Indices Percentage Profit and Loss Ratio And Proportions Partnership Chain Rule Time and Work Pipes and Cisterns Time and Distance Trains Boats and Streams Alligation or Mixture Simple Interest Compound Interest Logorithms Areas Volume and Surface area Races and Games of Skill Calendar Clocks Stocks ans Shares True Discount Bankers Discount Oddmanout and Series Data Interpretation probability Permutations and Combinations Puzzles

RATIO AND PROPORTION


Concepts Simple Problems Difficult Problems

file:///E|/work/books/placement/09_Aptitude/ratioandproportion.html[1/28/2012 12:45:34 AM]

APTITUDE
Numbers H.C.F and L.C.M Decimal Fractions Simplification Square and Cube roots Average Problems on Numbers Problems on Ages Surds and Indices Percentage Profit and Loss Ratio And Proportions Partnership Chain Rule Time and Work Pipes and Cisterns Time and Distance Trains Boats and Streams Alligation or Mixture Simple Interest Compound Interest Logorithms Areas Volume and Surface area Races and Games of Skill Calendar Clocks Stocks ans Shares True Discount Bankers Discount Oddmanout and Series Data Interpretation probability Permutations and Combinations Puzzles BACK

RATIO AND PROPORTION


Important Facts:
1.Ratio a/b and term of Ex: The : The ratio of two qualities a and b in the same units, is the fraction we write it as a:b. In the ratio, a:b, we call a as the first antecedent and b, the second term consequent. ratio 5:9 represents 5/9 with antecedent=5 ,consequent=9

3Rule: The multiplication or division of each term of 9 ratio by the same nonzero number does not affect the ratio. 4.Proportion: The equality of two ratios is called proportion. If a:b=c:d, we write a:b::c:d and we say that a,b,c and d are in proportion. Here a and b are called extremes, while b and c are called mean terms. Product of means=product of extremes Thus, a:b::c:d => (b*c)=(a*d) 5.Fourth proportional: If a:b::c:d, then d is called the fourth proportional to a,b and c. 6.Third proportional: If a:b::b:c, then c is called third proportional to a and b. 7.Mean proportional: Mean proportional between a and b is SQRT(a*b).

COMPARISION OF RATIOS:
We say that (a:b)>(c:d) => (a/b)>(c/d) 8.Compounded ratio: The compounded ratio of the ratios (a:b), (c:d),(e:f) is (ace:bdf). 9.Duplicate Ratio: If (a:b) is (a2: b2 ) 10.Sub-duplicate ratio of (a:b) is (SQRT(a):SQRT(b)) 11.Triplicate ratio of (a:b) is (a3: b3 )

12.Sub-triplicate ratio of (a:b) is (a1/3: b1/3 ). 13.If a/b=c/d, then (a+b)/(a-b)=(c+d)/(c-d) (componend o and dividend o)

VARIATION:
14.we say that x is directly proportional to y, if x=ky for some constant k and we write. 15.We say that x is inversely proportional to y, if xy=k for some constant we write. 16. and

X is inversely proportional to y. If a/b=c/d=e/f=g/h=k then k=(a+c+e+g)/(b+d+f+h) If a1/b1,a2/b2, a3/b3..............an/bn are unequal fractions then the ratio. a1+a2+a3+..........an/(b1+b2+b3+...............bn) lies between the lowest & the highest of the three fractions.
BACK

file:///E|/work/books/placement/09_Aptitude/ratioconcepts.html[1/28/2012 12:45:34 AM]

APTITUDE
Numbers H.C.F and L.C.M Decimal Fractions Simplification Square and Cube roots Average Problems on Numbers Problems on Ages Surds and Indices Percentage Profit and Loss Ratio And Proportions Partnership Chain Rule Time and Work Pipes and Cisterns Time and Distance Trains Boats and Streams Alligation or Mixture Simple Interest Compound Interest Logorithms Areas Volume and Surface area Races and Games of Skill Calendar Clocks Stocks ans Shares True Discount Bankers Discount Oddmanout and Series Data Interpretation probability Permutations and Combinations Puzzles

BACK
RATIO AND PROPORTION
DIFFICULT PROBLEMS
1.Three containers A,B and C are having mixtures of milk and water in the ratio of 1:5 and 3:5 and 5:7 respectively. If the capacities of the containers are in the ratio of all the three containers are in the ratio 5:4:5, find the ratio of milk to water, if the mixtures of all the three containers are mixed together. Sol: Assume that there are 500,400 and 500 liters respectively in the 3 containers. Then ,we have, 83.33, 150 and 208.33 liters of milk in each of the three containers. Thus, the total milk is 441.66 liters. Hence, the amount of water in the mixture is 1400-441.66=958.33liters. Hence, the ratio of milk to water is 441.66:958.33 => 53:115(using division by .3333) The calculation thought process should be (441*2+2):(958*3+1)=1325:2875 Dividing by 25 => 53:115. 2.A certain number of one rupee,fifty parse and twenty five paise coins are in the ratio of 2:5:3:4, add up to Rs 210. How many 50 paise coins were there? Sol: the ratio of 2.5:3:4 can be written as 5:6:8 let us assume that there are 5 one rupee coins,6 fifty paise coins and 8 twenty-five paise coins in all. their value=(5*1)+(6*.50)+(8*.25)=5+3+2=Rs 10 If the total is Rs 10,number of 50 paise coins are 6. if the total is Rs 210, number of 50 paise coins would be 210*6/10=126. 3.The incomes of A and B are in the ratio of 4:3 and their expenditure are in the ratio of 2:1 . if each one saves Rs 1000,what are their incomes? Sol: Ratio of incomes of A and B=4:3 Ratio of expenditures of A and B=2:1 Amount of money saved by A=Amount of money saved by B=Rs 1000 let the incomes of A and B be 4x and 3x respectively let the expense of A and B be 2y and 1yrespectively Amount of money saved by A=(income-expenditure)=4x-2y=Rs 1000 Amount of money saved by B=3x-y=Rs 1000 this can be even written as 6x-2y=Rs 2000 now solve 1 and 3 to get x=Rs 500 therefore income of A=4x=4*500=Rs 2000 income of B=3x=3*500=Rs 1500 4.A sum of Rs 1162 is divided among A,B and C. Such that 4 times A's share share is equal to 5 times B's share and 7 times C's share . What is the share of C? Sol: 4 times of A's share =5 times of B's share=7 times of C's share=1 therefore , the ratio of their share =1/4:1/5:1/7 LCM of 4,5,7=140 therefore, :1/5:1/7=35:28:20 the ratio now can be written as 35:28:20 therefore C's share=(20/83)*1162=20*14=Rs 280. 5.The ratio of the present ages of saritha and her mother is 2:9, mother's age at the time of saritha's birth was 28 years , what is saritha's present age? Sol: ratio of ages of saritha and her mother =2:9 let the present age of saritha be 2x years. then the mother's present age would be 9x years Difference in their ages =28 years 9x-2x=28 years 7x=28=>x=4 therefore saritha's age =2*4=8 years

BACK

file:///E|/work/books/placement/09_Aptitude/ratiodifficult.html[1/28/2012 12:45:34 AM]

file:///E|/work/books/placement/09_Aptitude/ratiodifficult.html[1/28/2012 12:45:34 AM]

APTITUDE
Numbers H.C.F and L.C.M Decimal Fractions Simplification Square and Cube roots Average Problems on Numbers Problems on Ages Surds and Indices Percentage Profit and Loss Ratio And Proportions Partnership Chain Rule Time and Work Pipes and Cisterns Time and Distance Trains Boats and Streams Alligation or Mixture Simple Interest Compound Interest Logorithms Areas Volume and Surface area Races and Games of Skill Calendar Clocks Stocks ans Shares True Discount Bankers Discount Oddmanout and Series Data Interpretation probability Permutations and Combinations Puzzles

RATIO AND PROPORTION


Concepts Simple Problems Difficult Problems

file:///E|/work/books/placement/09_Aptitude/ratios.html[1/28/2012 12:45:35 AM]

Ratio and Proportions


Important Facts: Ratio: The ratio of two qualities a and b in the same units, is the fraction a/b and we write it as a:b. In the ratio, a:b, we call a as the first term of antecedent and b, the second term consequent. Ex: The ratio 5:9 represents 5/9 with antecedent=5 ,consequent=9 Rule: The multiplication or division of each term of 9 ratio by the same non-zero number does not affect the ratio. Proportion: The equality of two ratios is called proportion. If a:b=c:d, we write a:b::c:d and we say that a,b,c and d are in proportion. Here a and b are called extremes, while b and c are called mean terms. Product of means=product of extremes Thus, a:b::c:d => (b*c)=(a*d) Fourth proportional: If a:b::c:d, then d is called the fourth proportional to a,b and c. Third proportional: If a:b::b:c, then c is called third proportional to a and b. Mean proportional: Mean proportional between a and b is SQRT(a*b). Comparision of Ratios: We say that (a:b)>(c:d) => (a/b)>(c/d)

Compounded ratio: The compounded ratio of the ratios (a:b), (c:d),(e:f) is (ace:bdf). Duplicate Ratio: If (a:b) is (a2: b2 ) Sub-duplicate ratio of (a:b) is (SQRT(a):SQRT(b)) Triplicate ratio of (a:b) is (a3: b3 )

Sub-triplicate ratio of (a:b) is (a1/3: b1/3 ). If a/b=c/d, then (a+b)/(a-b)=(c+d)/(c-d) (componend o and dividend o) Variation: we say that x is directly proportional to y, if x=ky for some constant k and we write. We say that x is inversely proportional to y, if xy=k for some constant and we write. X is inversely proportional to y. If a/b=c/d=e/f=g/h=k then k=(a+c+e+g)/(b+d+f+h) If a1/b1,a2/b2, a3/b3..............an/bn are unequal fractions then the ratio. a1+a2+a3+..........an/(b1+b2+b3+...............bn) lies between the lowest & the highest of the three fractions.

Top
Simple Problems 1.If a:b =5:9 and b:c=4:7 Find a:b:c? Sol: a:b=5:9 and b:c=4:7=4*9/4:9*4/9=9:63/9 a:b:c=5:9:63/9=20:36:63

2.Find the fourth proportion to 4,9,12 Sol: d is the fourth proportion to a,b,c a:b=c:d 4:9=12:x 4x=9*12=>x=27

3.Find third proportion to 16,36 Sol: If a:b=b:c then c is the third proportion to a,b 16:36=36:x 16x=36*36 x=81 4.Find mean proportion between 0.08 and 0.18 Sol: 5.If mean proportion between a and b=square root of ab mean proportion =square root of 0.08*0.18=0.12 a:b=2:3 b:c=4:5, c:d=6:7 then a:b:c:d is

file:///E|/work/books/placement/09_Aptitude/ratiosandproportions.html[1/28/2012 12:45:35 AM]

Sol:

a:b=2:3 and b:c=4:5=4*3/4:5*3/4=3:15/4 c:d=6:7=6*15/24:7*15/24=15/4:35/8 a:b:c:d=2:3:15/4:35/8=16:24:30:35

6.2A=3B=4C then A:B:C? Sol: let 2A=3B=4C=k then A=k/2, B=k/3, C=k/4 A:B:C=k/2:k/3:k/4=6:4:3

7.15% of x=20% of y then x:y is Sol: (15/100)*x=(20/100)*y 3x=4y x:y=4:3

8.a/3=b/4=c/7 then (a+b+c)/c= Sol: let a/3=b/4=c/7=k (a+b+c)/c=(3k+4k+7k)/7k=2

9.Rs 3650 is divided among 4 engineers, 3 MBAs and 5 CAs such that 3 CAs get as much as 2 MBAs and 3 Engs as much as 2 CAs .Find the share of an MBA. Sol: 4E+3M+5C=3650 3C=2M, that is M=1.5C 3E=2C that is E=.66 C Then, (4*0.66C)+(3*1.5C)+5C=3650 C=3650/12.166 C=300 M=1.5 and C=450.

Top
Difficule Problems 1.Three containers A,B and C are having mixtures of milk and water in the ratio of 1:5 and 3:5 and 5:7 respectively. If the capacities of the containers are in the ratio of all the three containers are in the ratio 5:4:5, find the ratio of milk to water, if the mixtures of all the three containers are mixed together. Sol: Assume that there are 500,400 and 500 liters respectively in the 3 containers. Then ,we have, 83.33, 150 and 208.33 liters of milk in each of the three containers. Thus, the total milk is 441.66 liters. Hence, the amount of water in the mixture is 1400-441.66=958.33liters. Hence, the ratio of milk to water is 441.66:958.33 => 53:115(using division by .3333) The calculation thought process should be (441*2+2):(958*3+1)=1325:2875 Dividing by 25 => 53:115. 2.A certain number of one rupee,fifty parse and twenty five paise coins are in the ratio of 2:5:3:4, add up to Rs 210. How many 50 paise coins were there? Sol: the ratio of 2.5:3:4 can be written as 5:6:8 let us assume that there are 5 one rupee coins,6 fifty paise coins and 8 twenty-five paise coins in all. their value=(5*1)+(6*.50)+(8*.25)=5+3+2=Rs 10 If the total is Rs 10,number of 50 paise coins are 6. if the total is Rs 210, number of 50 paise coins would be 210*6/10=126. 3.The incomes of A and B are in the ratio of 4:3 and their expenditure are in the ratio of 2:1 . if each one saves Rs 1000,what are their incomes? Sol: Ratio of incomes of A and B=4:3 Ratio of expenditures of A and B=2:1 Amount of money saved by A=Amount of money saved by B=Rs 1000 let the incomes of A and B be 4x and 3x respectively let the expense of A and B be 2y and 1yrespectively Amount of money saved by A=(income-expenditure)=4x-2y=Rs 1000 Amount of money saved by B=3x-y=Rs 1000 this can be even written as 6x-2y=Rs 2000 now solve 1 and 3 to get x=Rs 500 therefore income of A=4x=4*500=Rs 2000 income of B=3x=3*500=Rs 1500 4.A sum of Rs 1162 is divided among A,B and C. Such that 4 times A's share share is equal to 5 times B's share and 7 times C's share . What is the share of C? Sol: 4 times of A's share =5 times of B's share=7 times of C's share=1 therefore , the ratio of their share =1/4:1/5:1/7 LCM of 4,5,7=140 therefore, :1/5:1/7=35:28:20 the ratio now can be written as 35:28:20 therefore C's share=(20/83)*1162=20*14=Rs 280. 5.The ratio of the present ages of saritha and her mother is 2:9, mother's age at the time of saritha's birth was 28 years, what is saritha's present age? Sol: ratio of ages of saritha and her mother =2:9 let the present age of saritha be 2x years. then the mother's present age would be 9x years Difference in their ages =28 years 9x-2x=28 years 7x=28=>x=4 therefore saritha's age =2*4=8 years

file:///E|/work/books/placement/09_Aptitude/ratiosandproportions.html[1/28/2012 12:45:35 AM]

Back Back To Main

Top

Contact: 040-23000700

file:///E|/work/books/placement/09_Aptitude/ratiosandproportions.html[1/28/2012 12:45:35 AM]

APTITUDE
Numbers H.C.F and L.C.M Decimal Fractions Simplification Square and Cube roots Average Problems on Numbers Problems on Ages Surds and Indices Percentage Profit and Loss Ratio And Proportions Partnership Chain Rule Time and Work Pipes and Cisterns Time and Distance Trains Boats and Streams Alligation or Mixture Simple Interest Compound Interest Logorithms Areas Volume and Surface area Races and Games of Skill Calendar Clocks Stocks ans Shares True Discount Bankers Discount Oddmanout and Series Data Interpretation probability Permutations and Combinations Puzzles BACK

RATIO AND PROPORTION


SIMPLE PROBLEMS
1.If a:b =5:9 and b:c=4:7 Find a:b:c? Sol: a:b=5:9 and b:c=4:7=4*9/4:9*4/9=9:63/9 a:b:c=5:9:63/9=20:36:63 2.Find the fourth proportion to 4,9,12 Sol: d is the fourth proportion to a,b,c a:b=c:d 4:9=12:x 4x=9*12=>x=27 3.Find third proportion to 16,36 Sol: if a:b=b:c then c is the third proportion to a,b 16:36=36:x 16x=36*36 x=81 4.Find mean proportion between 0.08 and 0.18 Sol: mean proportion between a and b=square root of ab mean proportion =square root of 0.08*0.18=0.12 5.If Sol: a:b=2:3 b:c=4:5, c:d=6:7 then a:b:c:d is a:b=2:3 and b:c=4:5=4*3/4:5*3/4=3:15/4 c:d=6:7=6*15/24:7*15/24=15/4:35/8 a:b:c:d=2:3:15/4:35/8=16:24:30:35

6.2A=3B=4C then A:B:C? Sol: let 2A=3B=4C=k then A=k/2, B=k/3, C=k/4 A:B:C=k/2:k/3:k/4=6:4:3 7.15% of x=20% of y then x:y is Sol: (15/100)*x=(20/100)*y 3x=4y x:y=4:3 8.a/3=b/4=c/7 then (a+b+c)/c= Sol: let a/3=b/4=c/7=k (a+b+c)/c=(3k+4k+7k)/7k=2 9.Rs 3650 is divided among 4 engineers, 3 MBAs and 5 CAs such that 3 CAs get as much as 2 MBAs and 3 Engs as much as 2 CAs .Find the share of an MBA. Sol: 4E+3M+5C=3650 3C=2M, that is M=1.5C 3E=2C that is E=.66 C Then, (4*0.66C)+(3*1.5C)+5C=3650 C=3650/12.166 C=300 M=1.5 and C=450.

BACK

file:///E|/work/books/placement/09_Aptitude/ratiosimple.html[1/28/2012 12:45:36 AM]

Reasoning
Reasoning CLASSIFICATION TYPE Read the following information carefully and answer the questions that follow: 1) There are six cities A,B,C,D,E and F A is not a hill station. B and E are not historical places. D is not an industrial city. A and D are not historical craj_konkepudiities. A and B are not alike. 1.Which two cities are industrial centres ? a)A,B b)E,F* c)C,D d)B,F 2.Which two cities are historical places ? a)A,C b)B,F c)C,F* d)B,E 3.Which two cities are hill stations ? a)A,B b)C,A c)B,D* d)A,F e)A,D e)A,D e)none

4.Which city is a hill station and an industrial centre but not ahistorical place ? a)E* b)F c)A d)B e)C 5.Which two cities are neither historical places nor industrial centres? a)A,B b)D,E c)F,C d)B,D* e)none Solution: A Historical Industrial Hillstation 2) Five friends Indu,Pinki,Sai,Srujan,Pavan travelled to five different cities of Chennai,Calcutta,Delhi,Bangalore and Hyderabad by five differnet modes of transport of Bus,Train,Aeroplane,Car and Boat from Mumbai. The person who travelled to Delhi did not travel by Boat. Sai went to Banalore by Car and Pinki went to Calcutta by Aeroplane. Srujan travelled by Boat whereas Pavan travelled by Train. Mumbai is not connected by bBus to Delhi and Chennai. 1.Which of the following combinations of person and mode is not correct ? a)Indu-Bus b)Pinki - Aeroplane c)Sai - Car d)Srujan - Boat e)Pavan - Aeroplane* 2.Which of the following combinations is true for Srujan ? a)Delhi-Bus b)Chennai - Bus c)Chennai - Boat* d)Data inadequate 3.Which of the following combinations of place and mode is not correct ? a)Delhi-Bus* b)Calcutta - Aeroplane c)Bangalore - Car d)Chennai - Boat 4.Person travelling to Delhi went by which of the following modes ? a)Bus b)Train* c)Aeroplane d)Car e)Boat 5.Who among the following traj_konkepudiravelled to Delhi ? a)Sai b)Srujan c)Pavan* d)Data inadequate e)none x y x B x x y C y y y D x x y E x y y F y y y

Solution: Sai travels by Car. Pinki travels by Aeropraj_konkepudilane. Srujan travels by Boat. Pavan travels by Train. Indu travels by Bus. Sai goes to Bangalore. Pinki goes to Calcutta. Bus facility is not there for Delhi or Chennai. Indu goes to Hyderabad by Bus. From given information it is clear that Srujan goes by Boat but not to Delhi. Hence Srujan goes to Chennai. Pavan goes to Delhi. PLACE Indu Pinki Sai Hyderabad Calcutta Bangalore MODE Bus Aeroplane Car

file:///E|/work/books/placement/09_Aptitude/reasoning.html[1/28/2012 12:45:36 AM]

Srujan Pavan

Chennai Delhi

Boat Train

3) Four youngmen Thirbhuvan,Thrishanth,trinath,Trived are friendly with four girls Indira,Madhuri,Swetha and Dimple.Indira and Swetha are friends.Trinath's girlfriend does not like Indira and Swetha. Madhuri does not care for Trinath.Trishanth's girlfriend is friendly with Indira.Indira does not like Thribhu. 1.Who is Thribhu's girlfriend ? a)Indira b)Madhuri* c)Swetha 2.With whom is Indira friendly ? a)Thribhuvan b)Thrishanth c)Trinath 3.who is Dimple's boyfriend ? a)Trived b)Trinath* c)Thrishanth d)Dimple d)Trived* d)Thribhuvan d)Trinath

4.Who does not like Indira and Swetha ? a)Dimple* b)Thribhuvan c)Trived

Solution: Given Indira and Swetha are friends. Thrishanth's girlfriend is friendly with Indira. Hence,Thrishanth's girlfriend is Swetha. Given,Trinath's girlfriend does not like Indira and Swetha. =>She might be Madhuri or Dimple. But Madhuri does not care for Trinath. =>Trinath's girlfriend is Dimple. Given,Indira does not like Thrinbhuvan. =>Thribhuvan's girlfriend is Madhuri. Clearly,Trived's girlfriend is Indira. BOY Thribhuvan Thrishanth Trinath Trived COMPARISION TYPE QUESTIONS 1) Clues will be given regarding comparisions among a ssset of persons or things with respest to one or more qualities.After analysing a proper ascending ,descending sequence is formed and then are supposed to answer. There are five friends - Sachin,Sourav,Rahul,Zaheer and Yuvi.Sachin is shorter than Sourav but taller than Yuvi.Rahul is the tallest. Zaheer is little shorter than Sourav and little taller than Sachin. 1.Who is the shortest ? a)Yuvi* b)Sachin c)Zaheer d)Sourav e)none GIRL Madhuri Swetha Dimple Indira

Top

2.If they stand inorder of their heights,who will be in the middle ? a)Sourav b)Yuvi c)Sachin d)Zaheer* e)none 3.If they stand inorder of their increasing heights,who will be the second? a)Zaheer b)Sachin* c)Yuvi d)Sourav e)none 4.Who is the second tallest ? a)Sachin b)Sourav* c)Zaheer d)Yuvi e)none e)none

5.Who is taller than Zaheer but shorter than Rahul ? a)Sourav* b)Yuvi c)Sachin d)datainadequate Solution: Sachin < Sourav Rahul is tallest Zaheer < Sourav Yuvi < Sachin < Sourav < Rahul Yuvi < Sachin < Zaheer 2) Sourav > Yuvi

Zaheer > Sachin and Sachin < Zaheer < Sourav < Sourav < Rahul

Among five boys Vineeth is taller than Manick,but not as tall as Ravi,Jacob is taller than Dilip but shorter than Manick.Who is the tallest in their group ? a)Ravi* b)Manick c)Vineeth d)can't say e)none

Solution: Manick < Vineeth ; Vineeth < Ravi ; Dilip < Jacob ; Jacob < Mainck =>Dilip < Jacob < Manick < Vineeth < Ravi 3) Sudhanshu is as much older than Kokila as he is younger than Praveen. Nitin is as old as Kokila.Which of the following is wrong ? a)Kokila is younger than Praveen. b)Nitin is younger than Praveen. c)Sudhanshu is older than Nitin. d)Praveen is not the oldest.* e)Kokila is younger than Sudhanshu. Solution: =>Kokila < Sudhanshu; Praveen > Sudhanshu; =>Nitin = Kokila < Sudhanshu < Praveen =>Praveen is the oldest. SELECTION BASED ON GIVEN CONDITIONS Nitin = Kokila

Top

file:///E|/work/books/placement/09_Aptitude/reasoning.html[1/28/2012 12:45:36 AM]

1) A few essential criteria for selection of group of items are given: From amongst five doctors A,B,C,D and E;four engineers G,H,K and L; six teachers M,N,O,P,Q and R some teams are to be selected.A,B,G,H,O,P and Q are females and the rest are males. Where ever there is a male doctor ,no female teacher. Where ever there is a male engineer, no female doctor. There shall not be more than two male teachers in any team. 1.If the team consists of 2 doctors,3 female teachers and 2 engineers, the members of the team are: a)A B O P Q G H* b)C D O P Q G H c)C D K L O P Q d)D E G H O P Q 2.If the team following are a)A B G M N O d)A B K N R P consists of 2 doctors,1 engineer and 4 teachers all the possible except P b)A B H M O P Q c)A B H M R P Q Q*

3.If the team consists of2 doctors,2 female teachers and two engineers all the following teams are possible except: a)A B G H O Q b)A B G H P Q c)A B K L P Q d)O P G H A B 4.If the team consists of 3 doctors,2 male engineers and 2 teachers, the members of the team could be: a)A B C K L M R b)B C D K L N R c)C D E K L M N d)C D E K L P R 5.if the team consists of two doctors,two engineers and two teachers, all the following teams are possible except: a)A B G H O P b)A B G H M N c)C E K L N R d)C D K L O P Explanation:Doctors A B C D E Engineers G H K L Teachers M N O P Q R Given Females A B G H O P Q Males C D E K L M N R Male doctor and female teacher male engineer and female doctor More than two teachers/team combination are not allowed. 1Q) Explanation: Doctors are A B C D E Female teachers are O P Q Engineers are G H K L Male doctor and no female teacher => A,B are Doctors. So the Team consists of A B O P Q G H. 2Q) Explanation: Teachers are M N O P Q R Four are needed Three are male teachers. Female teachers are also to be selected.Hence no male doctors C D E are selected. So Doctors => A B Both doctors are Females =>no male Engineer to be selected. Ans:Team =>A B K N R P Q. 3Q) Explanation: The doctors are A B C D E, Female Teachers are O P Q, Engineers are G H K L, Since 2 Female Teachers are to be selected.So male doctors i.e;C D E cannot be selected. So, 2 Doctors selected will be A,B. Both Doctors are Females. So, male Engineer K L cannot be selected. G H are chosen. =>A B K L P Q are selected. 4Q)Explanation: Doctors A,B,C,D,E Male Engineers K,L Teachers M,N,O,P,Q,R Male Engineers =>no female Doctors=>no A,B 3 Doctors to be selected are C,D,E who are all males.

Top
FAMILY BASED PROBLEMS 1) There is a group of six persons A,B,C,D,E and F from a family. They are psychologist,manager,lawyer,jeweler,doctor and engineer. The doctor is the grandfather of F who is a psychologist. The manager D is married to A. C,the jeweler is married to the lawyer. B is the mother of F and E. There are two married couples in the family. 1.What is the profession of E ? a)Doctor b)Jeweler c)Manager 2.How is A related to E ? a)Brother b)Uncle c)Father d)Psychologist e)None*

d)Grandfather*

3.How many male members are there in the family ? a)One b)Three c)Four d)Data insufficient* 4.What is the profession of A ? a)Doctor* b)Lawyer c)Jeweler d)Manager

file:///E|/work/books/placement/09_Aptitude/reasoning.html[1/28/2012 12:45:36 AM]

5.Which of the following is one of the pairs of couples in the family ? a)AB b)AC c)AD* d)cant say Solution: F is a Psychologist. B is the mother of F and E. =>E is brother or sister of F. There are two married couples in the family. Since D is married to A,C the jeweler is married to lawyer B. Manager D is married to A means A is doctor,grandfather of E and F. So no one else is engineer till this point.=>E is engineer. AD - - - >CB - - - - - - >EF 2) Prashanth Arora has three children - Sangeeta,Vimal and Ashish.Ashish married Monika,the eldest daughter of Mr.andMrs.Roy.The Roys married their youngest daughter to the eldest son of Mr.and Mrs.Sharma,and they had two children named Amit and Shashi.The Roys have two more children, Roshan and Vandana,both elder to Veena.Sameer and Ajay are sons of Ashish and Monika.Rashmi is the daughter of Amit. Prashanth Arora - - - - - - - > 1.Sangeeta 2.Vimal 3.Ashish(Monika) - - - - - - - - - - - - - - - - - - - - > 1.Sameer 2.Ajay Mr & Mrs.Sharma- - - - - - - - - - - - - - - - > 1.Amit(Veena)---------------->1.Rashmi 2.Shashi Mr & Mrs.Roy- - - - - - - - - - - - - - - - - - - - - > 1.Monika 2.Roshan 3.Vandana 4.Veena 1.What is the surname of Rashmi ? a)Sharma* b)Roy c)Arora

d)cant say

e)none e)none e)none

2.How is Sameer related to father of Monika ? a)Grandson* b)Son c)Cousin 3.What is the surname of Sameer ? a)Roy b)Sharma c)Arora* 4.How is Mrs.Roy related to Ashish ? a)aunt b)Mother - in - law* 3)

d)Son - in - law d)cant say

c)mother

d)sister - in - law

In a family of six persons,there are three generations.Each person has seperate profession and also they like different colours.There are two couples in the family.Rohan is a CA and his wife neither is a doctor nor likes green colour. Engineer likes red colour and his wife is a teacher.Mohini is mother - in - law of Sunita and she likes orange colour. Vinod is grandfather of Tanmay,Tanmay is principal likes black colour. Nanu is granddaughter of Mohini and she likes blue colour.Nanus mother likes white colour. 1.Who is an Engineer ? a)Nanu b)Mohini c)Sunita d)cant say d)cant say e)none* e)none

2.What is the profession of Sunita ? a)engineer b)doctor c)teacher* 3.Which of the a)Mohini- Vinod b)Vinod-Mohini c)Rohan-Sunita d)cant say following is the correct pair ? and Rohan - Sunita* and Rohan - Nanu and Tanmay - Nanu e)none

4.How many ladies are there in the family ? a)2 b)3* c)4 d)cant say e)none 5.Which colour is liked by CA ? a)green* b)white c)white/green d)cant say e)none

Solution: (Engineer) Vinod - Mohini(Teacher) - - - - - - - - - - - - - >(CA)Rohan - Sunita - - - - - - - - - >Nanu&Tanmay(principal) Vinod--- - - - - - - - - - Red Mohini--- - - - - - - - - Orange Rohan--- - - - - - - - - - - Green Sunita--- - - - - - - - - - - - White Nanu----- - - - - - - - - - - Black Tanmay--- - - - - - - - - Black Mohini is mother - in - law of Sunita and grandmother of Nanu. Vinod is grandfather of Tanmay. So,Nanu and Tanmay represent third generation. Mohini and vinod form a couple and represent first generation. Rohan and Sunita form the other couple and represdent second generation. Rohan is a CA. Since,engineer is married,Vinod is engineer. Given Vinod loves red colour. Mohini is ateacher and likes Orange colour. Nanu likes blue colour. Tanmay is principal and likes black colour. Sunita Nanus mother likes white colour.=>Rohan likes green colour.

Back

Top

file:///E|/work/books/placement/09_Aptitude/reasoning.html[1/28/2012 12:45:36 AM]

Back To Main

Contact: 040-23000700

file:///E|/work/books/placement/09_Aptitude/reasoning.html[1/28/2012 12:45:36 AM]

APTITUDE
Numbers H.C.F and L.C.M Decimal Fractions Simplification Square and Cube roots Average Problems on Numbers Problems on Ages Surds and Indices Percentage Profit and Loss Ratio And Proportions Partnership Chain Rule Time and Work Pipes and Cisterns Time and Distance Trains Boats and Streams Alligation or Mixture Simple Interest Compound Interest Logorithms Areas Volume and Surface area Races and Games of Skill Calendar Clocks Stocks ans Shares True Discount Bankers Discount Oddmanout and Series Data Interpretation probability Permutations and Combinations Puzzles

SIMPLE INTEREST
CONCEPT SIMPLE PROBLEMS MEDIUM PROBLEMS COMPLEX PROBLEMS

file:///E|/work/books/placement/09_Aptitude/simple.html[1/28/2012 12:45:37 AM]

APTITUDE
Numbers H.C.F and L.C.M Decimal Fractions Simplification Square and Cube roots Average Problems on Numbers Problems on Ages Surds and Indices Percentage Profit and Loss Ratio And Proportions Partnership Chain Rule Time and Work Pipes and Cisterns Time and Distance Trains Boats and Streams Alligation or Mixture Simple Interest Compound Interest Logorithms Areas Volume and Surface area Races and Games of Skill Calendar Clocks Stocks ans Shares True Discount Bankers Discount Oddmanout and Series Data Interpretation probability Permutations and Combinations Puzzles BACK

COMPLEX PROBLEMS
1. A certain sum of money amounts t 1680 in 3yrs & it becomes 1920 in 7 yrs .What is the sum. Sol:3 yrs - - - - - - - - - - - - - 1680 7 yrs - - - - - - - - - - - - - 1920 then, 4 yrs - - - - - - - - - - - - - 240 1 yr - - - - - - - - - - - - ? (1/4) * 240 = 60 S.I in 3 yrs = 3*60 = 18012 Sum = Amount - S.I = 1680 - 180 = 1500 we get the same amount if we take S.I in 7 yrs I.e., 7*60 =420 Sum = Amount - S.I = 1920 - 420 = 1500 2. A Person takes a loan of Rs 200 at 5% simple Interest. He returns Rs.100 at the end of 1 yr. In order to clear his dues at the end of 2yrs ,he would pay: Sol:Amount to be paid = Rs(100 + (200*5*1)/100 + (100*5*1)/100) = Rs 115 3. A Man borrowed Rs 24000 from two money lenders.For one loan, he paid 15% per annum and for other 18% per annum .At the end of one year,he paid Rs.4050.How much did he borrowed at each rate? Sol:Let the Sum at 15% be Rs.x & then at 18% be Rs (24000-x) P1 = x R1 = 15 P2 = (24000-x) R2 = 18 At the end of ine year T = 1 (P1*T*R1)/100 + (P2*T*R2)/100 = 4050 (x*1*15)/100 + ((24000-x)*1*18)/100 = 4050 15x + 432000 - 18x = 405000 x = 9000 Money borrowed at 15% = 9000 Money borrowed at 18% = (24000 - 9000) = 15000 4. What annual instalment will discharge a debt of Rs. 1092 due in 3 years at 12% Simple Interest ? Sol:Let each instalment be Rs x (x + (x * 12 * 1)/100) + (x + (x * 12 * 2)/100) + x = 1092 28x/25 + 31x/25 + x =1092 (28x +31x + 25x) = (1092 * 25) 84x = 1092 * 25 x = (1092*25)/84 = 325 Each instalement = 325 5. If x,y,z are three sums of money such that y is the simple interest on x,z is the simple interest on y for the same time and at the same rate of interest ,then we have: Sol:y is simple interest on x, means y = (x*R*T)/100 RT = 100y/x z is simple interest on y, z = (y*R*T)/100 RT = 100z/y 100y/x = 100z/y y * y = xz 6.A Sum of Rs.1550 was lent partly at 5% and partly at 5% and partly at 8% p.a Simple interest .The total interest received after 3 years was Rs.300.The ratio of the money lent at 5% to that lent at 8% is: Sol:Let the Sum at 5% be Rs x at 8% be Rs(1550-x)

file:///E|/work/books/placement/09_Aptitude/simplecomplex.html[1/28/2012 12:45:37 AM]

(x*5*3)/100 + ((1500-x)*8*3)/100 = 300 15x + 1500 * 24 - 24x = 30000 x = 800 Money at 5%/ Money at 8% = 800/(1550 - 800) = 800/750 = 16/15 7. A Man invests a certain sum of money at 6% p.a Simple interest and another sum at 7% p.a Simple interest. His income from interest after 2 years was Rs 354 .one fourth of the first sum is equal to one fifth of the second sum.The total sum invested was: Sol:Let the sums be x & y R1 = 6 R2 = 7 T = 2 (P1*R1*T)/100 + (P2*R2*T)/100 = 354 + (y * 7 * 2)/100 = 354 6x + 7y = 17700 (1) also one fourth of the first sum is equal to one fifth of the second sum x/4 = y/5 => 5x - 4y = 0 (2) By solving 1 & 2 we get, x = 1200 y = 1500 Total sum = 1200 +1500 = 2700 8. Rs 2189 are divided into three parts such that their amounts after 1,2& 3 years respectively may be equal ,the rate of S.I being 4% p.a in all cases. The Smallest part is: Sol:Let these parts be x,y and[2189-(x+y)] then, (x*1*4)/100 = (y*2*4)/100 = (2189-(x+y))*3*4/100 4x/100 = 8y/100 x = 2y By substituting values (2y*1*4)/100 = (2189-3y)*3*4/100 44y = 2189 *12 y = 597 Smallest Part = 597 9. A man invested 3/3 of his capital at 7% , 1/4 at 8% and the remainder at 10% If his annual income is Rs.561. The capital is: Sol:Let the capital be Rs.x Then, (x/3 * 7/100 * 1) + ( x/4 * 8/100 * 1) + (5x/12 * 10/100 * 1) = 561 7x/300 + x/50 + x/24 = 561 51x = 561 * 600 x = 6600
BACK

(x * 6 * 2)/100

file:///E|/work/books/placement/09_Aptitude/simplecomplex.html[1/28/2012 12:45:37 AM]

APTITUDE
Numbers H.C.F and L.C.M Decimal Fractions Simplification Square and Cube roots Average Problems on Numbers Problems on Ages Surds and Indices Percentage Profit and Loss Ratio And Proportions Partnership Chain Rule Time and Work Pipes and Cisterns Time and Distance Trains Boats and Streams Alligation or Mixture Simple Interest Compound Interest Logorithms Areas Volume and Surface area Races and Games of Skill Calendar Clocks Stocks ans Shares True Discount Bankers Discount Oddmanout and Series Data Interpretation probability Permutations and Combinations Puzzles BACK

CONCEPT
->Principal or Sum:- The money borrowed or lent out for a certain period is called Principal or the Sum. ->Interest:- Extra money paid for using others money is called Interest. ->Simple Interest:- If the interest on a sum borrowed for a certain period is reckoned uniformly,then it is called Simple Interest. ->Formulae:Principal = P Rate = R% per annum Time = T years. Then, (i)Simple Interest(S.I)= (P*T*R)/100

(ii) Principal(P) = (100*S.I)/(R*T) Rate(R) = (100*S.I)/(P*T) Time(T) = (100*S.I)/(P*R)


BACK

file:///E|/work/books/placement/09_Aptitude/simpleconcept.html[1/28/2012 12:45:37 AM]

Simple Intrest
Important Facts and Formulae: Principal or Sum:- The money borrowed or lent out for a certain period is called Principal or the Sum. Interest:- Extra money paid for using others money is called Interest. Simple Interest:- If the interest on a sum borrowed for a certain period is reckoned uniformly,then it is called Simple Interest. Formulae: Principal = P Rate = R% per annum Time = T years. Then, (i)Simple Interest(S.I)= (P*T*R)/100

(ii) Principal(P) = (100*S.I)/(R*T) Rate(R) = (100*S.I)/(P*T) Time(T) = (100*S.I)/(P*R) Simple Problems 1.Find S.I on Rs68000 at 16 2/3% per annum for 9months. Sol:P=68000 R=50/3% p.a T=9/12 years=4/3 years S.I=(P*R*T)/100 =(68000*(50/3)*(3/4)*(1/100)) =Rs 8500

Note:If months are given we have to converted into years by dividing 12 ie., no.of months/12=years 2.Find S.I on Rs3000 at 18% per annum for the period from 4th Feb to 18th April 1995 Sol:Time=(24+31+18)days =73 days =73/365=1/5 years P= Rs 3000 R= 18% p.a S.I = (P*R*T)/100 =(3000*18*1/5*1/100) =Rs 108 Remark:- The day on which money is deposited is not counted while the day on which money is withdrawn is counted. 3. In how many years will a sum of money becomes triple at 10% per annum. Sol:Let principal S.I S.I 2P T =P = 2P = (P*T*R)/100 = (P*T*10)/100 = 20 years

Note: (1) Total amount = Principal + S.I (2) If sum of money becomes double means Total amount or Sum = Principal + S.I = P + P = 2P

Top
Medium Problems 1.A sum at Simple interest at 13 1/2% per annum amounts to Rs 2502.50 after 4 years.Find the sum. Sol:Let Sum be x. then, S.I = (P*T*R)/100 = ((x*4*27)/(100*2)) = 27x/100 Amount = (x+(27x)/100) = 77x/50 77x/50 = 2502.50 x = (2502.50*50)/77 = 1625 Sum = 1625

2. A some of money becomes double of itself in 4 years in 12 years it will become how many times at the same rate. Sol:4 yrs - - - - - - - - P

file:///E|/work/books/placement/09_Aptitude/simpleintrest.html[1/28/2012 12:45:38 AM]

12 yrs - - - - - - - - ? (12/4)* P =3P Amount or Sum = P+3P = 4 times 3. A Sum was put at S.I at a certain rate for 3 years. Had it been put at 2% higher rate ,it would have fetched Rs 360 more .Find the Sum. Sol:Let Sum =P original rate = R T = 3 years If 2% is more than the original rate ,it would have fetched 360 more ie., R+2 (P*(R+2)*3/100) - (P*R*3)/100 = 360 3PR+ 6P-3PR = 36000 6P = 36000 P = 6000 Sum = 6000.

4.Rs 800 amounts to Rs 920 in 3yrs at S.I.If the interest rate is increased by 3%, it would amount to how much? Sol:S.I Rate New Rate Principal Time S.I New Amount = = = = = = = = 920 - 800 = 120 (100*120)/(800*3) = 5% 5 + 3 = 8% 800 3 yrs (800*8*3)/100 = 192 800 + 192 992

5. Prabhat took a certain amount as a loan from bank at the rate of 8% p.a S.I and gave the same amount to Ashish as a loan at the rate of 12% p.a . If at the end of 12 yrs, he made a profit of Rs. 320 in the deal,What was the original amount? Sol:Let the original amount be Rs x. T = 12 R1 = 8% R2 = 12% Profit = 320 P = x (P*T*R2)/100 - (P*T*R1)/100 =320 (x*12*12)/100 - (x*8*12)/100 = 320 x = 2000/3 x = Rs.666.67

6. Simple Interest on a certail sum at a certain rate is 9/16 of the sum . if the number representing rate percent and time in years be equal ,then the rate is. Sol:Let Sum = x .Then, S.I = 9x/16 Let time = n years & rate = n% n = 100 * 9x/16 * 1/x * 1/n n * n = 900/16 n = 30/4 = 7 1/2%

Complex Problems 1. A certain sum of money amounts t 1680 in 3yrs & it becomes 1920 in 7 yrs .What is the sum. Sol:then, - - - - - - - - - - - - - - - - - - - - - - - - - - - - - - - - 240 = 60 3*60 = 18012 Sum = Amount - S.I = 1680 - 180 = 1500 we get the same amount if we take S.I I.e., 7*60 =420 Sum = Amount - S.I = 1920 - 420 = 1500 - - - - - - - - (1/4) S.I in 3 yrs 3 7 4 1 yrs yrs yrs yr * = 1680 1920 240 ?

in 7 yrs

2. A Person takes a loan of Rs 200 at 5% simple Interest. He returns Rs.100 at the end of 1 yr. In order to clear his dues at the end of 2yrs ,he would pay: Sol:Amount to be paid = Rs(100 + (200*5*1)/100 + (100*5*1)/100) = Rs 115

3. A Man borrowed Rs 24000 from two money lenders.For one loan, he paid 15% per annum and for other 18% per annum. At the end of one year,he paid Rs.4050.How much did he borrowed at each rate? Sol:Let the Sum at 15% be Rs.x & then at 18% be Rs (24000-x) P1 = x R1 = 15 P2 = (24000-x) R2 = 18 At the end of ine year T = 1 (P1*T*R1)/100 + (P2*T*R2)/100 = 4050 (x*1*15)/100 + ((24000-x)*1*18)/100 = 4050 15x + 432000 - 18x = 405000 x = 9000 Money borrowed at 15% = 9000 Money borrowed at 18% = (24000 - 9000) = 15000

Top
4.What annual instalment will discharge a debt of Rs. 1092 due in 3 years at 12% Simple Interest ?

file:///E|/work/books/placement/09_Aptitude/simpleintrest.html[1/28/2012 12:45:38 AM]

Sol:Let each instalment be Rs x (x + (x * 12 * 1)/100) + (x + (x * 12 * 2)/100) + x = 1092 28x/25 + 31x/25 + x =1092 (28x +31x + 25x) = (1092 * 25) 84x = 1092 * 25 x = (1092*25)/84 = 325 Each instalement = 325 5.If x,y,z are three sums of money such that y is the simple interest on x,z is the simple interest on y for the same time and at the same rate of interest ,then we have: Sol:y z is simple interest on x, means y = (x*R*T)/100 RT = 100y/x is simple interest on y, z = (y*R*T)/100 RT = 100z/y 100y/x = 100z/y y * y = xz

6.A Sum of Rs.1550 was lent partly at 5% and partly at 5% and partly at 8% p.a Simple interest .The total interest received after 3 years was Rs.300.The ratio of the money lent at 5% to that lent at 8% is: Sol:Let the Sum at 5% be Rs x at 8% be Rs(1550-x) (x*5*3)/100 + ((1500-x)*8*3)/100 = 300 15x + 1500 * 24 - 24x = 30000 x = 800 Money at 5%/ Money at 8% = 800/(1550 - 800) = 800/750 = 16/15

7. A Man invests a certain sum of money at 6% p.a Simple interest and another sum at 7% p.a Simple interest. His income from interest after 2 years was Rs 354 .one fourth of the first sum is equal to one fifth of the second sum.The total sum invested was: Sol:Let the sums be x & y R1 = 6 R2 = 7 T = 2 (P1*R1*T)/100 + (P2*R2*T)/100 = 354 (x * 6 * 2)/100 + (y * 7 * 2)/100 = 354 6x + 7y = 17700 (1) also one fourth of the first sum is equal to one fifth of the second sum x/4 = y/5 => 5x - 4y = 0 (2) By solving 1 & 2 we get, x = 1200 y = 1500 Total sum = 1200 +1500 = 2700

8. Rs 2189 are divided into three parts such that their amounts after 1,2& 3 years respectively may be equal, the rate of S.I being 4% p.a in all cases. The Smallest part is: Sol:Let these parts be x,y and[2189-(x+y)] then, (x*1*4)/100 = (y*2*4)/100 = (2189-(x+y))*3*4/100 4x/100 = 8y/100 x = 2y By substituting values (2y*1*4)/100 = (2189-3y)*3*4/100 44y = 2189 *12 y = 597 Smallest Part = 597 9. A man invested 3/3 of his capital at 7% , 1/4 at 8% and the remainder at 10%.If his annual income is Rs.561. The capital is: Sol:Let the capital be Rs.x Then, (x/3 * 7/100 * 1) + ( x/4 * 8/100 * 1) + (5x/12 * 10/100 * 1) = 561 7x/300 + x/50 + x/24 = 561 51x = 561 * 600 x = 6600

Back Back To Main

Top

Contact: 040-23000700

file:///E|/work/books/placement/09_Aptitude/simpleintrest.html[1/28/2012 12:45:38 AM]

APTITUDE
Numbers H.C.F and L.C.M Decimal Fractions Simplification Square and Cube roots Average Problems on Numbers Problems on Ages Surds and Indices Percentage Profit and Loss Ratio And Proportions Partnership Chain Rule Time and Work Pipes and Cisterns Time and Distance Trains Boats and Streams Alligation or Mixture Simple Interest Compound Interest Logorithms Areas Volume and Surface area Races and Games of Skill Calendar Clocks Stocks ans Shares True Discount Bankers Discount Oddmanout and Series Data Interpretation probability Permutations and Combinations Puzzles BACK

MEDIUM PROBLEMS
1. A sum at Simple interest at 13 1/2% per annum amounts to Rs 2502.50 after 4 years.Find the sum. Sol:- Let Sum be x. then, S.I = (P*T*R)/100 = ((x*4*27)/(100*2)) = 27x/100 Amount = (x+(27x)/100) = 77x/50 77x/50 = 2502.50 x = (2502.50*50)/77 = 1625 Sum = 1625 2. A some of money becomes double of itself in 4 years in 12 years it will become how many times at the same rate. Sol:4 yrs - - - - - - - - - P 12 yrs - - - - - - - - ? (12/4)* P =3P Amount or Sum = P+3P = 4 times 3. A Sum was put at S.I at a certain rate for 3 years.Had it been put at 2% higher rate ,it would have fetched Rs 360 more .Find the Sum. Sol:Let Sum =P original rate = R T = 3 years If 2% is more than the original rate ,it would have fetched 360 more ie., R+2 (P*(R+2)*3/100) - (P*R*3)/100 = 360 3PR+ 6P-3PR = 36000 6P = 36000 P = 6000 Sum = 6000. 4.Rs 800 amounts to Rs 920 in 3yrs at S.I.If the interest rate is increased by 3%, it would amount to how much? Sol:S.I = 920 - 800 = 120 Rate = (100*120)/(800*3) = 5% New Rate = 5 + 3 = 8% , Principal = 800 , Time = 3 yrs S.I = (800*8*3)/100 = 192 New Amount = 800 + 192 = 992 5. Prabhat took a certain amount as a loan from bank at the rate of 8% p.a S.I and gave the same amount to Ashish as a loan at the rate of 12% p.a . If at the end of 12 yrs, he made a profit of Rs. 320 in the deal,What was the original amount? Sol:Let the original amount be Rs x. T = 12 R1 = 8% R2 = 12% Profit = 320 P = x (P*T*R2)/100 - (P*T*R1)/100 =320 (x*12*12)/100 - (x*8*12)/100 = 320 x = 2000/3 x = Rs.666.67 6. Simple Interest on a certail sum at a certain rate is 9/16 of the sum . if the number representing rate percent and time in years be equal ,then the rate is. Sol:Let Sum = x .Then, S.I = 9x/16 Let time = n years & rate = n%

file:///E|/work/books/placement/09_Aptitude/simplemedium.html[1/28/2012 12:45:38 AM]

n = 100 * 9x/16 * 1/x * 1/n n * n = 900/16 n = 30/4 = 7 1/2%


BACK

file:///E|/work/books/placement/09_Aptitude/simplemedium.html[1/28/2012 12:45:38 AM]

APTITUDE
Numbers H.C.F and L.C.M Decimal Fractions Simplification Square and Cube roots Average Problems on Numbers Problems on Ages Surds and Indices Percentage Profit and Loss Ratio And Proportions Partnership Chain Rule Time and Work Pipes and Cisterns Time and Distance Trains Boats and Streams Alligation or Mixture Simple Interest Compound Interest Logorithms Areas Volume and Surface area Races and Games of Skill Calendar Clocks Stocks ans Shares True Discount Bankers Discount Oddmanout and Series Data Interpretation probability Permutations and Combinations Puzzles

CALENDAR
BACK

Simple problems
Shortcuts : This shortcut must be applied only starting with 19 series. Example :What day of the week on 17th june , 1998? Solution : 5 -> the given month code(august) 17 -> the given date 98->(19 th century after years) 24-> ((47/4) = 11 i.e how many leap years -------total = 144 ((144/7) = 20 and the remainder is 4) therefore in the above week table the no 4 code represents wednesday so the required day is wednesday. Problem 1 : The first republic day of the India was celebrated on 26th January,1950. It was : Solution : 01 26 50 12 ---------total = 89 ((89/7) = 12 and the remainder is 5) therefore in the above week table represents the number 5 as thursday, so the required day was Thursday. Problem 2 : Find on which day 15th august1947 ? Solution : 03 15 47 11 ---------total = 76 Then (76)/7 = 6 odd days 6 indicates friday in the above week table. Therefore required day is friday. Problem 3 : Find on which day jan 26th 1956 ? Solution : 01 26 56 14 -1 (-1 indicates leapyear(i.e 1956),so 1 reduce from the total) --------total = 96 Then (96)/7 = 5 odd days 5 indicates thursday in the above week table Therefore our required day is Thursday. Problem 4 : Today is friday after 62 days,it will be : Solution : Each day of the week is repeated after 7 days. so, after 63 days,it will be friday. Hence ,after 62 days,it will be thursday.

file:///E|/work/books/placement/09_Aptitude/Simpleproblems.html[1/28/2012 12:45:38 AM]

Therefore the required day is thursday. Problem 5 : Find the day of the week on 25th december,1995? Solution : 06 25 95 23 --------total = 149 Then (149)/7=(23)=2 odd days Therefore the required day is "Monday".
BACK

file:///E|/work/books/placement/09_Aptitude/Simpleproblems.html[1/28/2012 12:45:38 AM]

APTITUDE
Numbers H.C.F and L.C.M Decimal Fractions Simplification Square and Cube roots Average Problems on Numbers Problems on Ages Surds and Indices Percentage Profit and Loss Ratio And Proportions Partnership Chain Rule Time and Work Pipes and Cisterns Time and Distance Trains Boats and Streams Alligation or Mixture Simple Interest Compound Interest Logorithms Areas Volume and Surface area Races and Games of Skill Calendar Clocks Stocks ans Shares True Discount Bankers Discount Oddmanout and Series Data Interpretation probability Permutations and Combinations Puzzles BACK

SIMPLE PROBLEMS
1.Find S.I on Rs68000 at 16 2/3% per annum for 9months. Sol:P=68000 R=50/3% p.a T=9/12 years=4/3 years S.I=(P*R*T)/100 =(68000*(50/3)*(3/4)*(1/100)) =Rs 8500 Note:- If months are given we have to converted into years by dividing 12 ie., no.of months/12=years (2)Find S.I on Rs3000 at 18% per annum for the period from 4th Feb to 18th April 1995 Sol:Time=(24+31+18)days =73 days =73/365=1/5 years P= Rs 3000 R= 18% p.a S.I = (P*R*T)/100 =(3000*18*1/5*1/100) =Rs 108 Remark:- The day on which money is deposited is not counted while the day on which money is withdrawn is counted. 3. In how many years will a sum of money becomes triple at 10% per annum. Sol:- Let principal =P S.I = 2P S.I = (P*T*R)/100 2P = (P*T*10)/100 T = 20 years Note:-(I) Total amount = Principal + S.I (2) If sum of money becomes double means Total amount or Sum = Principal + S.I = P + P = 2P
BACK

file:///E|/work/books/placement/09_Aptitude/simplesimple.html[1/28/2012 12:45:39 AM]

Simplifications
Introduction: 'BODMAS' rule: This rule depicts the correct sequence in which the operations are to be executed, so as to find out the value of a given expression. Here B stands for Bracket, O for Of, D for Division, M for Multiplication, A for Addition and S for Subtraction. First of all the brackets must be removed, strictly in the order () , {} , []. After removing the brackets, we want use the following operations: 1.Of 2. Division 3. Multiplication 4. Addition 5. Subtraction Modulus of a real number: Modulus of a real number is a defined as |a| = a, if a>0 or -a, if a < 0; Problems: 1.(5004 /139) 6= ? Sol: Expression = 5004/ 139 6 = 36 6 = 30; 2.What mathematical operations should come at the place of ? in the equation : (2 ? 6 12 / 4 + 2 = 11) ? Sol: 2 ? 6 = 11 + 12 / 4 2 = 11 + 3 2 = 12 2 * 6 = 12 3.( 8 / 88) * 8888088 = ? Sol : (1/11) * 8888088 = 808008 4.How many 1/8's are there in 371/2 ? Sol: (371/2) /(1/8)= (75/2) /(1/8) = 300 5.Find the values of 1/2*3 +1/3*4 +1/4*5+ .................+1/9*10 ? Sol: 1/2*3 +1/3*4+1/4*5+ ..................+1/9*10 = [ -1/3] +[ 1/3 ] + [- 1/5] +...............+[1/9-1/10] = [ 1/10] = 4/15 = 2/5

Top
6.The value of 999 of 995/999* 999 is: Sol: [1000- 4/1000]*999 = 999000-4 = 998996 7.Along a yard 225m long, 26 trees are planted at equal distance, one tree being at each end of the yard. what is the distance between two consecutive trees ? Sol: 26 trees have 25 gaps between them. Hence , required distance = 225/ 25 m= 9m 8.In a garden , there are 10 rows and 12 columns of mango trees. the distance between the two trees is 2 m and a distance of one meter is left from all sides of the boundary of the length of the garden is : Sol: Each row contains 12 plants. leaving 2 corner plants, 10 plants in between have 10 * 2 meters and 1 meter on each side is left. length = (20 + 2) m = 22m 9.Eight people are planning to share equally the cost of a rental car, if one person with draws from the arrangement and the others share equally the entire cost of the car, then the share of each of the remaining persons increased by? Sol: Original share of one person = 1/8 new share of one person = 1/7 increase = 1/7 1/8 = 1/56 required fractions = (1/56)/(1/8) = 1/7 10.A piece of cloth cost Rs 35. if the length of the piece would

file:///E|/work/books/placement/09_Aptitude/simplification.html[1/28/2012 12:45:39 AM]

have been 4m longer and each meter cost Re 1 less , the cost would have remained unchanged. how long is the piece? Sol: Left the length of the piece be x m. then, cost of 1m of piece = Rs [35 / x] 35/ x 35 /x+4 = 1 x + 4 x = x(x+ 4)/35 x2 + 4x 140 = 0 x= 10 11.A man divides Rs 8600 among 5sons, 4 daughters and 2 nephews. If each daughter receives four times as much as each nephew, and each son receives five as much as each nephew. how much does each daughter receive ? Sol: Let the share of each nephew be Rs x. then, share of each daughter Rs 4x. share of each son = 5x Rs so, 5 *5x+ 4 * 4x + 2x =8600 2x + 16x + 25x= 8600 43x = 8600 x = 200 share of each daughter = 4 * 200 = Rs 800 12.A man spends 2/5 of his salary on house rent, 3/10 of his salary on food, and 1/8 of his salary on conveyance. if he has Rs 1400 left with him, find his expenditure on food and conveyance? Sol: Part of the salary left = 1-[2/5 +3/10+1/9] = 1- 33/40 =7/40 Let the monthly salary be rs x then, 7/40 of x = 1400 x= [1400*40]/7 x= 8000 Expenditure on food = 3/10*8000 =Rs 2400 Expenditure on conveyance= 1/8*8000 =Rs 1000

Back Back To Main

Top

Contact: 040-23000700

file:///E|/work/books/placement/09_Aptitude/simplification.html[1/28/2012 12:45:39 AM]

APTITUDE
Numbers H.C.F and L.C.M Decimal Fractions Simplification Square and Cube roots Average Problems on Numbers Problems on Ages Surds and Indices Percentage Profit and Loss Ratio And Proportions Partnership Chain Rule Time and Work Pipes and Cisterns Time and Distance Trains Boats and Streams Alligation or Mixture Simple Interest Compound Interest Logorithms Areas Volume and Surface area Races and Games of Skill Calendar Clocks Stocks ans Shares True Discount Bankers Discount Oddmanout and Series Data Interpretation probability Permutations and Combinations Puzzles BACK

SIMPLIFICATIONS
Important Concepts: i)1. 'BODMAS' rule: This rule depicts the correct sequence in which the operations are to be executed, so as to find out the value of a given expression. 2. Here B stands for Bracket, O for Of, D for Division, M for Multiplication, A for Addition and S for Subtraction. 3. First of all the brackets must be removed, strictly in the order () , {} , []. 4.After removing the brackets, we want use the following operations: 1.Of 2. Division 3. Multiplication 4. Addition 5. Subtraction ii)Modulus of a real number: Modulus of a real number is a defined as |a| = a, if a>0 or -a, if a < 0; BACK

file:///E|/work/books/placement/09_Aptitude/simplificationconcept.html[1/28/2012 12:45:40 AM]

APTITUDE
Numbers H.C.F and L.C.M Decimal Fractions Simplification Square and Cube roots Average Problems on Numbers Problems on Ages Surds and Indices Percentage Profit and Loss Ratio And Proportions Partnership Chain Rule Time and Work Pipes and Cisterns Time and Distance Trains Boats and Streams Alligation or Mixture Simple Interest Compound Interest Logorithms Areas Volume and Surface area Races and Games of Skill Calendar Clocks Stocks ans Shares True Discount Bankers Discount Oddmanout and Series Data Interpretation probability Permutations and Combinations Puzzles BACK

SIMPLIFICATIONS
Problems: 1.(5004 /139) 6= ? Sol: Expression = 5004/ 139 6 = 36 6 = 30; Ans : 30 2.What mathematical operations should come at the place of ? in the equation : (2 ? 6 12 / 4 + 2 = 11) ? Sol: 2 ? 6 = 11 + 12 / 4 2 = 11 + 3 2 = 12 2 * 6 = 12 Ans : * 3.( 8 / 88) * 8888088 = ? Sol : (1/11) * 8888088 = 808008 Ans: 808008 4.How many 1/8's are there in 371/2 ? Sol: (371/2) /(1/8)= (75/2) /(1/8) = 300 Ans: 300 5.Find the values of 1/2*3 +1/3*4 +1/4*5+ .................+1/9*10 ? Sol: 1/2*3 +1/3*4+1/4*5+ ..................+1/9*10 = [ -1/3] +[ 1/3 ] + [- 1/5] +...............+[1/9-1/10] = [ 1/10] = 4/15 = 2/5 Ans : 2/5 6.The value of 999 of 995/999* 999 is: Sol: [1000- 4/1000]*999 = 999000-4 = 998996 7.Along a yard 225m long, 26 trees are planted at equal distance, one tree being at each end of the yard. what is the distance between two consecutive trees ? Sol: 26 trees have 25 gaps between them. Hence , required distance = 225/ 25 m= 9m Ans: 9m 8.In a garden , there are 10 rows and 12 columns of mango trees. the distance between the two trees is 2 m and a distance of one meter is left from all sides of the boundary of the length of the garden is : Sol: Each row contains 12 plants. leaving 2 corner plants, 10 plants in between have 10 * 2 meters and 1 meter on each side is left.

file:///E|/work/books/placement/09_Aptitude/simplificationproblem.html[1/28/2012 12:45:40 AM]

length = (20 + 2) m = 22m 9.Eight people are planning to share equally the cost of a rental car , if one person with draws from the arrangement and the others share equally the entire cost of the car, then the share of each of the remaining persons increased by? Sol: Original share of one person = 1/8 new share of one person = 1/7 increase = 1/7 1/8 = 1/56 required fractions = (1/56)/(1/8) = 1/7 10.A piece of cloth cost Rs 35. if the length of the piece would have been 4m longer and each meter cost Re 1 less , the cost would have remained unchanged. how long is the piece? Sol: Left the length of the piece be x m. then, cost of 1m of piece = Rs [35 / x] 35/ x 35 /x+4 = 1 x + 4 x = x(x+ 4)/35 x2 + 4x 140 = 0 x= 10 Ans : 10 m 11.A man divides Rs 8600 among 5sons, 4 daughters and 2 nephews. if each daughter receives four times as much as each nephew, and each son receives five as much as each nephew. how much does each daughter receive ? Sol: Let the share of each nephew be Rs x. then, share of each daughter Rs 4x. share of each son = 5x Rs so, 5 *5x+ 4 * 4x + 2x =8600 2x + 16x + 25x= 8600 43x = 8600 x = 200 share of each daughter = 4 * 200 = Rs 800 12.A man spends 2/5 of his salary on house rent, 3/10 of his salary on food, and 1/8 of his salary on conveyance. if he has Rs 1400 left with him, find his expenditure on food and conveyance? Sol: Part of the salary left = 1-[2/5 +3/10+1/9] = 1- 33/40 =7/40 Let the monthly salary be rs x then, 7/40 of x = 1400 x= [1400*40]/7 x= 8000 Expenditure on food = 3/10*8000 =Rs 2400 Expenditure on conveyance= 1/8*8000 =Rs 1000 BACK

file:///E|/work/books/placement/09_Aptitude/simplificationproblem.html[1/28/2012 12:45:40 AM]

Square and Cube Roots


Formula: The Product of two same numbers in easiest way as follow. Example:let us calculate the product of 96*96 Solution: Here every number must be compare with the 100. See here the given number 96 which is 4 difference with the 100. so subtract 4 from the 96 we get 92 ,then the square of the number 4 it is 16 place the 16 beside the 92 we get answer as 9216. 9 6 4 -------------9 2 -------------4*4=16 9 2 1 6 therefore square of the two numbers 96*96=9216. Example: Calculate product for 98*98 Solution: Here the number 98 is having 2 difference when compare to 100 subtract 2 from the number then we get 96 square the number 2 it is 4 now place beside the 96 as 9604 9 8 2 ------------9 6 ------------2*2=4 9 6 0 4. so, we get the product of 98*98=9604. Example: Calculate product for 88*88 Solution: Here the number 88 is having 12 difference when compare to 100 subtract 12 from the 88 then we get 76 the square of the number 12 is 144 (which is three digit number but should place only two digit beside the 76) therefore in such case add one to 6 then it becomes 77 now place 44 beside the number 77 we will get 7744. 88 -12 -----------76 ----------12*12=144 76 + 144 -------------------7744 -------------------Example: Find the product of the numbers 46 *46? Solution:consider the number 50=100/2. Now again go comparision with the number which gets when division with 100.here consider the number 50 which is nearer to the number given. 46 when compared with the number 50 we get the difference of 4. Now subtract the number 4 from the 46, we get 42. As 50 got when 100 get divided by 2. so, divided the number by 2 after subtraction. 42/2=21 now square the the number 4 i.e, 4*4=16 just place the number 16 beside the number 21 we get 2116. 4 6 4 ---------------4 2 as 50 = 100/2 42/2=21 now place 4*4=16 beside 21 2 1 1 6 Example: Find the product of the numbers 37*37 Solution: consider the number 50=100/2 now again go comparision with the number which division with 100. here consider the number 50 which is nearer to 37 when compared with the number 50 we get the now subtract the number 13 from the 37, we get as 50 got when 100 get divided by 2. so, divided the number by 2 after subtraction. 24/2=12 now square the the number 13 i.e, 13*13=169 just place the number 169 beside the number 21 now as 169 is three digit number then add 1 to 1t as 13 then place 69 beside the 13

gets when the number given. difference of 13. 24.

2 we get

file:///E|/work/books/placement/09_Aptitude/squareandcube.html[1/28/2012 12:45:41 AM]

we get 1369. 3 7 1 3 ----------------2 4 as 24/2=12 square 13* 13=169 1 2 + 1 6 9 ----------------------1 3 6 9 -------------------------

50

100/2

Top
Example: Find the product of 106*106 Solution: now compare it with 100 , The given number is more then 100 then add the extra number to the given number. That is 106+6=112 then square the number 6 that is 6*6=36 just place beside the number 36 beside the 112,then we get 11236. 1 0 6 + 6 --------------------1 1 2 -------------------now 6* 6=36 place this beside the number 112, we get 1 1 2 3 6 Square root: If x2=y ,we say that the square root of y is x and we write ,y=x. Cube root: The cube root of a given number x is the number whose cube is x. we denote the cube root of x by x1/3 . Examples: 1.Evaluate 60841/2 by factorization method. Solution: Express the given number as the product of prime factors. Now, take the product of these prime factors choosing one out of every pair of the same primes. This product gives the square root of the given number. Thus resolving 6084 in the prime factors ,we get 6084 2 6024 2 3042 3 1521 3 507 13 169 13 6084=21/2 *31/2 *131/2 60841/2=2*3*13=78. Answer is 78. 2.what will come in place of question mark in each of the following questions? i)(32.4/?)1/2 = 2 ii)86.491/2 + (5+?1/2)2 =12.3 Solution: 1) (32.4/x)1/2=2 Squaring on both sides we get 32.4/x=4 =>4x=32.4 =>x=8.1 Answer is 8.1 ii)86.491/2 + (5+x2)=12.3 solutin:86.491/2 + (5+x1/2 )=12.3 9.3+ (5+x1/2 )=12.3 => (5+x1/2 ) =12.3-9.3 => (5+x1/2 )=3 Squaring on both sides we get (5+x1/2 )=9 x1/2 =9-5 x1/2 =4 x=2. Answer is 2. 3. 0.00004761 equals: Solution: (4761/108) 4761/ 108 . 69/10000 0.0069. Answer is 0.0069 4.If 18225=135,then the value of 182.25 + 1.8225 + 0.018225 + 0.00018225. Solution: (18225/100) +(18225/10000) + (18225/1000000) +(18225/100000000) =(18225)/10 + (18225)1/2/100 + (18225)/1000 + (18225)/10000 =135/10 + 135/100 + 135/1000 + 135/10000 =13.5+1.35+0.135+0.0135=14.9985. Answer is 14.9985. 5.what should come in place of both the question marks in the equation (?/ 1281/2= (162)1/2/?) ?

file:///E|/work/books/placement/09_Aptitude/squareandcube.html[1/28/2012 12:45:41 AM]

Solution: x/ 1281/2= (162)1/2/x =>x1/2= (128*162)1/2 => x1/2= (64*2*18*9)1/2 =>x2= (82*62*32) =>x2=8*6*3 =>x2=144 =>x=12. 6.If 0.13 / p1/2=13 then p equals Solution: 0.13/p2=13 =>p2=0.13/13 =1/100 p2=(1/100) =>p=1/10 therefore p=0.1 Answer is 0.1

Top
7.If 13691/2+(0.0615+x)1/2=37.25 then x is equals to: Solution 37+(0.0615+x)1/2=37.25(since 37*37=1369) =>(0.0615+x)1/2=0.25 Squaring on both sides (0.0615+x)=0.0625 x=0.001 x=10-3. Answer is 10-3. 8.If (x-1)(y+2)=7 x& y being positive whole numbers then values of x& y are? Solution: (x-1)(y+2)=7 Squaring on both sides we get (x-1)(y+2)=72 x-1=7 and y+2=7 therefore x=8 , y=5. Answer x=8 ,y=5. 9.If 3*51/2+1251/2=17.88.then what will be the value of 801/2+6*51/2? Solution: 3*51/2+1251/2=17.88 3*51/2+(25*5)1/2=17.88 3*51/2+5*51/2=17.88 8*51/2=17.88 51/2=2.235 therefore 801/2+6 51/2=(16*51/2)+6*1/25 =4 51/2+6 51/2 =10*2.235 =22.35 Answer is 22.35 10.If 3a=4b=6c and a+b+c=27*29 then Find c value is: Solution: 4b=6c =>b=3/2*c 3a=4b =>a=4/3b =>a=4/3(3/2c)=2c therefore a+b+c=27*291/2 2c+3/2c+c=27*291/2 =>4c+3c+2c/2=27*291/2 =>9/2c=27*291/2 c=27*291/2*2/9 c=6*291/2 11.If 2*3=131/2 and 3*4=5 then value of 5*12 is Solution: Here a*b=(a2+b2)1/2 therefore 5*12=(52+122)1/2 =(25+144)1/2 =1691/2 =13 Answer is 13. 12.The smallest number added to 680621 to make the sum a perfect square is Solution: Find the square root number which is nearest to this number 8 680621 824 64 162 406 324 1644 8221 6576 1645 therefore 824 is the number ,to get the nearest square root number take (825*825)-680621 therefore 680625-680621=4 hence 4 is the number added to 680621 to make it perfect square. 13.The greatest four digit perfect square number is Solution: The greatest four digit number is 9999. now find the square root of 9999. 9 9999 99 81 189 1819 1701 198 therefore 9999-198=9801 which is required number. Answer is 9801.

file:///E|/work/books/placement/09_Aptitude/squareandcube.html[1/28/2012 12:45:41 AM]

14.A man plants 15376 apples trees in his garden and arranges them so, that there are as many rows as there are apples trees in each row .The number of rows is. Solution: Here find the square root of 15376. 1 15376 124 1 22 53 44 244 976 976 0 therefore the number of rows are 124. 15.A group of students decided to collect as many paise from each member of the group as is the number of members. If the total collection amounts to Rs 59.29.The number of members in the group is: Solution: Here convert Money into paise. 59.29*100=5929 paise. To know the number of member ,calculate the square root of 5929. 7 5929 77 49 147 1029 1029 0 Therefore number of members are 77. 16.A general wishes to draw up his 36581 soldiers in the form of a solid square ,after arranging them ,he found that some of them are left over .How many are left? Solution: Here he asked about the left man ,So find the square root of given number the remainder will be the left man 1 36581 191 1 29 265 261 381 481 381 100(since remaining) Therefore the left men are 100. 17.By what least number 4320 be multiplied to obtain number which is a perfect cube? Solution: find l.c.m for 4320. 2 4320 2 2160 2 1080 2 540 2 270 3 135 3 45 3 15 5 4320=25 * 33 * 5 =23 * 33 * 22 *5 so make it a perfect cube ,it should be multiplied by 2*5*5=50 Answer is 50. 18.3(4*12/125)1/2=? Solution: 3(512/125)1/2 3(8*8*8)1/2/(5*5*5) 3(83)1/2/(53) ((83)/(53))1/3 =>8/5 or 1 3/5.

Back Back To Main

Top

Contact: 040-23000700

file:///E|/work/books/placement/09_Aptitude/squareandcube.html[1/28/2012 12:45:41 AM]

APTITUDE
SQUARE ROOTS AND CUBE ROOTS
Formula: The Product of two same numbers in easiest way as follow. Example: let us calculate the product of 96*96 Solution: Here every number must be compare with the 100. See here the given number 96 which is 4 difference with the 100. so subtract 4 from the 96 we get 92 , then the square of the number 4 it is 16. place the 16 beside the 92 we get answer as 9216. 9 6 4

-------------9 2

-------------4*4=16 9 2 1 6 therefore square of the two numbers 96*96=9216.

Example: Calculate

product for

98*98

Solution: Here the number 98 is having 2 difference when compare to 100 subtract 2 from the number then we get 96 square the number 2 it is 4 now place beside the 96 as 9604 9 8 2 ------------9 6 ------------2*2=4 9 6 0 4. so, we get the product of 98*98=9604. Example : Calculate product for 88*88 Solution: Here the number 88 is having 12 difference when compare to 100 subtract 12 from the 88 then we get 76 the square of the number 12 is 144 (which is three digit number but should place only two digit beside the 76) therefore in such case add one to 6 then it becomes 77 now place 44 beside the number 77 we will get 7744 . answer will be 7744.

88 -12 -----------76 ----------12*12=144 76

file:///E|/work/books/placement/09_Aptitude/squareandcuberoot1.html[1/28/2012 12:45:41 AM]

+ 144 -------------------7744 -------------------Example: Find the product of the numbers Solution: consider the number 50=100/2 now again go comparision with the number which gets when division with 100. here consider the number 50 which is nearer to the number given. 46 when compared with the number 50 we get the difference of 4. now subtract the number 4 from the 46, as 50 got when 100 get divided by 2. so, divided the number by 2 after subtraction. 42/2=21 now square the the number 4 i.e, 4*4=16 just place the number 16 beside the number 21 we get 2116. 4 6 4 ---------------4 2 42/2=21 now place 4*4=16 beside 21 2 1 1 6 Answer will be 2116. Example: Find the product of the numbers 37*37 Solution: consider the number 50=100/2 now again go comparision with the number which gets when division with 100. here consider the number 50 which is nearer to the number given. 37 when compared with the number 50 we get the difference of 13. now subtract the number 13 from the 37, we get 24. as 50 got when 100 get divided by 2. so, divided the number by 2 after subtraction. 24/2=12 now square the the number 13 i.e, 13*13=169 just place the number 169 beside the number 21 now as 169 is three digit number then add 1 to 2 we get 1t as 13 then place 69 beside the 13 we get 1369. 3 7 1 3 ----------------2 4 as 24/2=12 square 13* 13=169 1 2 + 1 6 9 ----------------------1 3 6 9 ------------------------Example: Find the product of 106*106 Solution: now compare it with 100 , The given number is more then 100 then add the extra number to the given number. That is 106+6=112 then square the number 6 that is 6*6=36 just place beside the number 36 beside the 112,then we get 11236. 1 0 6 + 6 --------------------1 1 2 -------------------now 6* 6=36 place this beside the number 112, we get 1 1 2 3 6 50 = 100/2 we get 42. 46 *46?

as

50

100/2

Square root: If x2=y ,we say that the square root of y is x and we write ,y 1/2=x.

file:///E|/work/books/placement/09_Aptitude/squareandcuberoot1.html[1/28/2012 12:45:41 AM]

Cube root : The cube root of a given number x is the number whose cube is x. we denote the cube root of x by x1/3 . Examples: 1.Evaluate 6084 by factorization method. Solution: Express the given number as the product of prime factors. Now, take the product of these prime factors choosing one out of every pair of the same primes. This product gives the square root of the given number. Thus resolving 6084 in the prime factors ,we get 6084 2 1521 3 13 13 6084=2*3*13=78. Answer is 78. 2.what will come in place of question mark in each of the following questions? i)(32.4/?)1/2 = 2 ii)86.49 1/2+ (5+?^2) 1/2=12.3 Solution: 1) (32.4/x) 1/2=2 Squaring on both sides we get 32.4/x=4 =>4x=32.4 =>x=8.1 Answer is 8.1 ii)86.49 1/2 + (5+x^2) 1/2=12.3 86.49 1/2 + (5+x^2 )1/2=12.3 9.3+ => => (5+x^2 )1/2=12.3 (5+x^2 ) 1/2=12.3-9.3 (5+x^2 )1/2=3 507 169 3042 6084=2^2*3^2*13^2 3 2 6024

Squaring on both sides we get (5+x^2 )=9 x^2 =9-5 x^2 =4 x=2. Answer is 2. 3.0.00004761 1/2 equals: Solution: (4761/10^8) 1/2 4761 1/2/ 10^8 1/2 . 69/10000 0.0069. Answer is 0.0069 4.If 18225 1/2=135,then the value of 182.251/2 + 1.82251/2 + 0.018225 1/2 + 0.00018225 1/2. Solution: (18225/100) 1/2+(18225/10000) 1/2 +(18225/1000000) 1/2+(18225/100000000) 1/2 (18225)/10 1/2 + (18225)/1001/2 + (18225)/1000 1/2+ (18225)/10000 1/2 135/10 + 135/100 + 13.5+1.35+0.135+0.0135=14.9985. Answer is 14.9985. 5.what should come in place of both the question marks in the equation ?/ 128 1/2= (162) 1/2/? Solution: x/ 128 1/2= (162) 1/2/x =>x^2= (128*162) 1/2 => x^2= (64*2*18*9)1/2 =>x^2= (82*62*32) 1/2 =>x^2=8*6*3 =>x^2=144 =>x=12. 6.If 0.13 / p^2=13 then p equals Solution: 0.13/p^2=13 =>p^2=0.13/13 =1/100 p^2=(1/100)1/2 =>p=1/10 therefore p=0.1 Answer is 0.1 135/1000 + 135/10000

file:///E|/work/books/placement/09_Aptitude/squareandcuberoot1.html[1/28/2012 12:45:41 AM]

7.If 1369 1/2+(0.0615+x)1/2=37.25 then x is equals to: Solution 37+(0.0615+x) 1/2=37.25(since 37*37=1369) =>(0.0615+x) 1/2=0.25 Squaring on both sides (0.0615+x)=0.0625 x=0.001 x=10^-3. Answer is 10^-3. 8.If (x-1)(y+2) 1/2=7 x& y being positive whole numbers then values of x& y are?

Solution: (x-1)(y+2) 1/2=7 Squaring on both sides we get (x-1)(y+2)=7^2 x-1=7 and y+2=7 therefore x=8 , y=5. Answer x=8 ,y=5. 9.If 3*5 1/2+12 1/25=17.88.then what will be the value of 80 1/2+6*5 1/2? Solution: 3*5 1/2+125 1/2=17.88 3*5 1/2+(25*5) 1/2=17.88 3*5 1/2+5*5 1/2=17.88 8*5 1/2=17.88 51/2=2.235 therefore 80 1/2+6 51/2=(16*5) 1/2+6*5 1/2 =4 51/2+6 1/25=10 51/2 10*2.235 =22.35 Answer is 22.35 10.If 3a=4b=6c and a+b+c=27*291/2 then Find c value is: Solution: 4b=6c =>b=3/2*c 3a=4b =>a=4/3b =>a=4/3(3/2c)=2c therefore a+b+c=27*291/2 2c+3/2c+c=27*29 1/2 =>4c+3c+2c/2=27*29 1/2 =>9/2c=27*29 1/2 c=27*29 1/2*2/9 c=6*291/2 11.If 2*3=131/2 and 3*4=5 Solution: then value of 5*12 is

Here a*b=(a^2+b^2) 1/2

therefore 5*12=(52+122) 1/2 =(25+144) 1/2 =169 1/2 =13 Answer is 13. 12.The smallest number added to 680621 to make the sum a perfect square is Solution: Find the square root number which is nearest to this number 8 162 1644 680621 824 64 406 324 8221 6576 1645

therefore 824 is the number ,to get the nearest square root number take (825*825)-680621 therefore 680625-680621=4 hence 4 is the number added to 680621 to make it perfect square. 13.The greatest four digit perfect square number is Solution: The greatest four digit number is 9999. now find the square root of 9999. 9 189 9999 81 1819 99

file:///E|/work/books/placement/09_Aptitude/squareandcuberoot1.html[1/28/2012 12:45:41 AM]

1701 198 therefore 9999-198=9801 Answer is 9801. which is required number.

Numbers H.C.F and L.C.M Decimal Fractions Simplification Square and Cube roots Average Problems on Numbers Problems on Ages Surds and Indices Percentage Profit and Loss Ratio And Proportions Partnership Chain Rule Time and Work Pipes and Cisterns Time and Distance Trains Boats and Streams Alligation or Mixture Simple Interest Compound Interest Logorithms Areas Volume and Surface area Races and Games of Skill Calendar Clocks Stocks ans Shares True Discount Bankers Discount Oddmanout and Series Data Interpretation probability Permutations and Combinations Puzzles

file:///E|/work/books/placement/09_Aptitude/squareandcuberoot1.html[1/28/2012 12:45:41 AM]

SURDS AND INDICES


Simple problems: 1. Laws of Indices: (i) (ii) (iii) (iv) (v) (vi) am * an am / an (am)n (ab)n (a/b)n a0 = = = = = = a(m+n) a(m-n) a(m*n) an * bn an / bn 1

2.Surds :Let 'a' be a rational number & 'n' be a positive integer such that a1/n = nth root a is irrational.Then nth root a is called 'a' surd of 'n'. Problems:(1) (i) (27)2/3 = (33)2/3 = 32 = 9. (ii) (1024)-4/5 = (45)-4/5 = (4)-4= 1/(4)4 = 1/256. (iii)(8/125)-4/3 =((2/5)3)-4/3 = (2/5)-4 = (5/2)4 = 625/16 (2) If 2(x-1)+ 2(x+1) = 1280 then find the value of x . Solution: 2x/2+2x.2 = 1280 2x(1+22) = 2*1280 2x = 2560/5 2x = 512 => 2x = 29 x = 9 (3) Find the value of [5[81/3+271/3]3]1/4 Solution: [5[(23)1/3+(33)1/3]3]1/4 [5[2+3]3]1/4 [54]1/4 => 5. (4) If (1/5)3y= 0.008 then find the value of (0.25)y Solution: (1/5)3y = 0.008 (1/5)3y =[0.2]3 (1/5)3y =(1/5)3 3y= 3 => y=1. (0.25)y = (0.25)1 =>

0.25 = 25/100 32n+1 / / (32)n 9n

= 1/4

(5) Find the value of (243)n/5 * Solution: (35)n/5 * 32n +1 33n+1 / 33n-1 3 33n+1 * 3-3n+1

* 3 n-1

* 3n-1 =>9.

=> 32

(6) Find the value of Solution:

(21/4-1)( 23/4 +21/2+21/4+1)

Let us say 21/4 = x (x-1)(x3+x2+x+1) (x-1)(x2(x+1)+(x+1)) (x-1) (x2+1) (x+1) [(x-1)(x+1) = (x2-1)] (x2+1) (x2-1) => (x4-1) ((21/4))4 - 1) = > (2-1) = > 1. , z = xc then find the value

(7)

If x= ya , y = zb of abc. Solution:

z= xc z= (ya)c [ z= (y)ac z= (zb)ac z= zabc abc = 1

x= ya ] [y= zb]

(8)Simplify (xa/xb)a2+ab+b2*(xb/xc)b2+bc+c2*(xc/xa)c2+ca+a2 Solution:[xa-b]a2+ab+b2 * [xb-c]b2+bc+c2 * [xc-a]c2+ca+a2 [ (a-b)(a2+ab+b2) = a3-b3]

from the above formula => xa3-b3 xb3-c3 xc3-a3 => xa3-b3+b3-c3+c3-a3 => x0 = 1

file:///E|/work/books/placement/09_Aptitude/surdsandindices.html[1/28/2012 12:45:42 AM]

(9) (1000)7 (a) 10

/1018 (b)

= ? 100 (c ) 1000 = > (10)21 / (d) 10000 (10)18 1000

Solution: (1000)7 / 1018 (103)7 / (10)18 => Ans :( c ) (10) The value of (10)21-18

=> (10)3 =>

(8-25-8-26)

is (d) None

(a) 7* 8-25

(b) 7*8-26

(c ) 8* 8-26

Solution: ( 8-25 - 8-26 ) => 8-26 (8-1 ) => 7* 8-26 Ans: (b) Top (11) 1 / (1+ an-m ) +1/ (1+am-n) (a) 0 (b) 1/2 (c ) 1 1/ => => => = ? (d) an+m

Solution:

(1+ an/am) + 1/ ( 1+ am/an) am / (am+ an ) + an /(am +an ) (am +an ) /(am + an) 1

Ans: ( c) (12) 1/(1+xb-a+xc-a)+1/(1+xa-b+xc-b)+1/(1+xb-c+xa-c)=? (a) 0 (b) 1 ( c ) xa-b-c (d) None of the above

Solution:

1/ (1+xb/xa+xc/xa) + 1/(1+xa/xb +xc/xb) + 1/(1+xb/xc +xa/xc) => xa /(xa +xb+xc) + xb/(xa +xb+xc) +xc/(xa +xb+xc) =>(xa +xb+xc) /(xa +xb+xc) =>1 Ans: (b) (13) If x=3+2 2 then the value of (x 1/ x) is [ =root] (a) 1 (b) 2 (c ) 22 ( d) 33

Solution:

(x-1/x)2 = x+ 1/x-2 3+22 + (1/3+22 )-2 3+22 + 3-22 -2 6-2 = 4 (x-1/x)2 = 4 =>(x-1/x)2 = 22 (x-1/x) = 2. Ans : (b) => => => (14) (xb/xc)b+c-a (xc/xa)c+a-b (xa/xb)b+a-c (a) xabc (b) 1 = ?

( c) xab+bc+ca (d) xa+b+c [xc-a]c+a-b [xa-b]a+b-c

Solution: [xb-c]b+c-a

=>x(b-c)(b+c-a) x(c-a)(c+a-b) x(a-b)(a+b-c) =>x(b2-c2-ab-ac) x(c2-a2-bc-ab) x(a2-b2-ac-bc) =>x(b2-c2-ab-ac+c2-a2-bc-ab+a2-b2-ac-bc) => x0 =>1 Ans: (b) (15) If 3x-y (a) Solution: 0 = 27 and (b) 2 3x+y = 243 then x is equal to (d) 6

(c ) 4

3x-y = 27 => 3x-y = 33 x-y= 3 3x+y = 243 => 3x+y = 35 x+y = 5 From above two equations x = 4 , y=1

Ans: (c ) (16) If ax = by = cz and b2 = ac then yequals

(a)xz/x+z (b)xz/2(x-z) (c)xz/2(z-x) (d)2xz/x+z Solution: Let us say ax = by = cz = k ax =k => [ax]1/x = k1/x => a = k1/x Simillarly b = k1/y c = k1/z b2 = ac [k1/y]2=k1/xk1/z =>k2/y = k1/x+1/z => 2/y = 1/x+1/y =>y= 2xz/x+z

Ans: (17)

(d) ax = b,by = c ,cz = a then the value of xyz is is

(a) 0

(b) 1

(c ) 1/abc

(d) abc

file:///E|/work/books/placement/09_Aptitude/surdsandindices.html[1/28/2012 12:45:42 AM]

Solution:

ax = (cz)x by)xz =>xyz

b = b = b =1

[cz = a] [by = c]

Ans: (b) (18) If 2x = 4y =8z and (1/2x +1/4y +1/6z) =24/7 then the value of 'z' is (a) 7/16 (b) 7 / 32 (c ) 7/48 (d) 7/64

Solution:

2x = 4y=8z 2x = 22y = 23z x= 2y = 3z Multiply above equation with 2 2x = 4y= 6z (1/2x+1/4y+1/6z) = 24/7 =>(1/6z+1/6z+1/6z) = 24/7 => 3 / 6z = 24/7 => z= 7/48

Ans: ( c)

Back Back To Main

Top

Contact: 040-23000700

file:///E|/work/books/placement/09_Aptitude/surdsandindices.html[1/28/2012 12:45:42 AM]

Time and Distance


Formulae: I)Speed = Distance/Time II)Time = Distance/speed III) Distance = speed*time IV) 1km/hr = 5/18 m/s V)1 m/s = 18/5 Km/hr VI)If the ratio of the speed of A and B is a:b,then the ratio of the time taken by them to cover the same distance is 1/a : 1/b or b:a VII) suppose a man covers a distance at x kmph and an equal distance at y kmph.then the average speed during the whole journey is (2xy/x+y)kmph Problems 1)A person covers a certain distance at 7kmph .How many meters does he cover in 2 minutes. Solution:: speed=72kmph=72*5/18 = 20m/s distance covered in 2min =20*2*60 = 2400m 2)If a man runs at 3m/s. How many km does he run in 1hr 40min Solution:: speed of the man = 3*18/5 kmph = 54/5kmph Distance covered in 5/3 hrs=54/5*5/3 = 18km 3)Walking at the rate of 4knph a man covers certain distance in 2hr 45 min. Running at a speed of 16.5 kmph the man will cover the same distance in. Solution:: Distance=Speed* time 4*11/4=11km New speed =16.5kmph therefore Time=D/S=11/16.5 = 40min

Top
Complex Problems 1)A train covers a distance in 50 min ,if it runs at a speed of 48kmph on an average.The speed at which the train must run to reduce the time of journey to 40min will be. Solution:: Time=50/60 hr=5/6hr Speed=48mph distance=S*T=48*5/6=40km time=40/60hr=2/3hr New speed = 40* 3/2 kmph= 60kmph 2)Vikas can cover a distance in 1hr 24min by covering 2/3 of the distance at 4 kmph and the rest at 5kmph.the total distance is? Solution:: Let total distance be S total time=1hr24min A to T :: speed=4kmph diistance=2/3S T to S :: speed=5km distance=1-2/3S=1/3S 21/15 hr=2/3 S/4 + 1/3s /5 84=14/3S*3 S=84*3/14*3 = 6km 3)walking at of his usual speed ,a man is late by 2 hr. the usual time is. Solution:: Usual speed = S Usual time = T Distance = D New Speed is S New time is 4/3 T 4/3 T T = 5/2 T=15/2 = 7

file:///E|/work/books/placement/09_Aptitude/timeanddistance.html[1/28/2012 12:45:42 AM]

4)A man covers a distance on scooter .had he moved 3kmph faster he would have taken 40 min less. If he had moved 2kmph slower he would have taken 40min more.the distance is. Solution:: Let distance = x m Usual rate = y kmph x/y x/y+3 = 40/60 hr 2y(y+3) = 9x --------------1 x/y-2 x/y = 40/60 hr y(y-2) = 3x -----------------2 divide 1 & 2 equations by solving we get x = 40 5)Excluding stoppages,the speed of the bus is 54kmph and including stoppages,it is 45kmph.for how many min does the bus stop per hr. Solution:: Due to stoppages,it covers 9km less. time taken to cover 9 km is [9/54 *60] min = 10min 6)Two boys starting from the same place walk at a rate of 5kmph and 5.5kmph respectively.wht time will they take to be 8.5km apart, if they walk in the same direction Solution:: The relative speed of the boys = 5.5kmph 5kmph = 0.5 kmph Distance between them is 8.5 km Time= 8.5km / 0.5 kmph = 17 hrs 7)2 trains starting at the same time from 2 stations 200km apart and going in opposite direction cross each other ata distance of 110km from one of the stations.what is the ratio of their speeds. Solution:: In same time ,they cover 110km & 90 km respectively so ratio of their speed =110:90 = 11:9 8)Two trains start from A & B and travel towards each other at speed of 50kmph and 60kmph resp. At the time of the meeting the second train has traveled 120km more than the first.the distance between them. Solution:: Let the distance traveled by the first train be x km then distance covered by the second train is x + 120km x/50 = x+120 / 60 x= 600 so the distance between A & B is x + x + 120 = 1320 km 9)A thief steals a ca r at 2.30pm and drives it at 60kmph.the theft is discovered at 3pm and the owner sets off in another car at 75kmph when will he overtake the thief Solution:: Let the thief is overtaken x hrs after 2.30pm distance covered by the thief in x hrs = distance covered by the owner in x-1/2 hr 60x = 75 ( x- ) x= 5/2 hr thief is overtaken at 2.30 pm + 2 hr = 5 pm 10)In covering distance,the speed of A & B are in the ratio of 3:4.A takes 30min more than B to reach the destion.The time taken by A to reach the destinstion is. Solution:: Ratio of speed = 3:4 Ratio of time = 4:3 let A takes 4x hrs,B takes 3x hrs then 4x-3x = 30/60 hr x = hr Time taken by A to reach the destination is 4x = 4 * = 2 hr 11)A motorist covers a distance of 39km in 45min by moving at a speed of xkmph for the first 15min.then moving at double the speed for the next 20 min and then again moving at his original speed for the rest of the journey .then x=? Solution:: Total distance = 39 km Total time = 45 min D = S*T x * 15/60 + 2x * 20/60 + x * 10/60 = 39 km x = 36 kmph 12)A & B are two towns.Mr.Fara covers the distance from A t0 B on cycle at 17kmph and returns to A by a tonga running at a uniform speed of 8kmph.his average speed during the whole journey is. Solution:: When same distance is covered with different speeds,then the average speed = 2xy / x+y =10.88kmph 13)A car covers 4 successive 3km stretches at speed of 10kmph,20kmph,30kmph&:60kmph resp. Its average speed is. Solution:: Average speed = total distance / total time total distance = 4 * 3 = 12 km total time = 3/10 + 3/20 + 3/30 + 3/60 = 36/60 hr speed =12/36 * 60 = 20 kmph

file:///E|/work/books/placement/09_Aptitude/timeanddistance.html[1/28/2012 12:45:42 AM]

Top
14)A person walks at 5kmph for 6hr and at 4kmph for 12hr. The average speed is. Solution:: avg speed = total distance/total time = 5*6 + 4*12 / 18 =4 1/3 mph 15)A bullock cart has to cover a distance of 80km in 10hrs. If it covers half of the journeyin 3/5th time.wht should be its speed to cover the remaining distance in the time left. Solution:: Time left = 10 - 3/5*10 = 4 hr speed =40 km /4 hr =10 kmph 16)The ratio between the speeds of the A& B is 2:3 an therefore A takes 10 min more than the time taken by B to reach the destination.If A had walked at double the speed ,he would have covered the distance in ? Solution:: Ratio of speed = 2:3 Ratio of time = 3:2 A takes 10 min more 3x-2x = 10 min A's time=30 min --->A covers the distance in 30 min ,if its speed is x -> He will cover the same distance in 15 min,if its speed doubles (i.e 2x) 17)A is twice as fast as B and B is thrice as fast as C is. The journey covered by B in? Solution:: speed's ratio a : b = 2: 1 b : c = 3:1 Time's ratio b : c = 1:3 b : c = 18:54 (if c covers in 54 min i..e twice to 18 min ) 18)A man performed 3/5 of the total journey by ratio 17/20 by bus and the remaining 65km on foot.wht is his total journey. Solution:: Let total distance is x x-(3/5x + 17/20 x) =6.5 x- 19x/20 = 6.5 x=20 * 6.5 =130 km 19)A train M leaves Meerat at 5 am and reaches Delhi at 9am . Another train N leaves Delhi at 7am and reaches Meerut at 1030am At what time do the 2 trains cross one another Solution:: Let the distance between Meerut & Delhi be x they meet after y hr after 7am M covers x in 4hr N covers x in 3 i.e 7/2 hr speed of M =x/4 speed of N = 2x/7 Distance covered by M in y+2 hr + Distance covered by N in y hr is x x/4 (y+2) +2x/7(y)=x y=14/15hr or 56 min 20)A man takes 5hr 45min in walking to certain place and riding back. He would have gained 2hrs by riding both ways.The time he would take to walk both ways is? Solution:: Let x be the speed of walked Let y be the speed of ride Let D be the distance Then D/x + D/y = 23/4 hr -------1 D/y + D/y = 23/4 2 hr D/y = 15/8 --------2 substitute 2 in 1 D/x + 15/8 = 23/4 D/x = 23/4 -15/8 =46-15/8 =31/8 Time taken for walk one way is 31/8 hr time taken to walk to and fro is 2*31/8 = 31/4 hr =7 hr 45 min

Back Back To Main

Top

Contact: 040-23000700

file:///E|/work/books/placement/09_Aptitude/timeanddistance.html[1/28/2012 12:45:42 AM]

file:///E|/work/books/placement/09_Aptitude/timeanddistance.html[1/28/2012 12:45:42 AM]

Time and Work


Important Facts: 1.If A can do a piece of work in n days, then A's 1 day work=1/n 2.If A's 1 day's work=1/n, then A can finish the work in n days. Ex: If A can do a piece of work in 4 days,then A's 1 day's work=1/4. If A's 1 days work=1/5, then A can finish the work in 5 days 3.If A is thrice as good workman as B,then: Ratio of work done by A and B =3:1. Ratio of time taken by A and B to finish a work=1:3 4.Definition of Variation: The change in two different variables follow some definite rule. It said that the two variables vary directly or inversely.Its notation is X/Y=k, where k is called constant. This variation is called direct variation. XY=k. This variation is called inverse variation. 5.Some Pairs of Variables: i)Number of workers and their wages. If the number of workers increases, their total wages increase. If the number of days reduced, there will be less work. If the number of days is increased, there will be more work. Therefore, here we have direct proportion or direct variation. ii)Number workers and days required to do a certain work is an example of inverse variation. If more men are employed, they will require fewer days and if there are less number of workers, more days are required. iii)There is an inverse proportion between the daily hours of a work and the days required. If the number of hours is increased, less number of days are required and if the number of hours is reduced, more days are required. 6.Some Important Tips: More Men -Less More Men -More More Days-More Number of days days work. Days and Work and Work and required Conversely More Conversely More Conversely More to complete the Day-Less Men. Work-More Men. Work-More Days. given work=Total work/One

Since the total work is assumed to be one(unit), the number of days required to complete the given work would be the reciprocal of one days work. Sometimes, the problems on time and work can be solved using the proportional rule ((man*days*hours)/work) in another situation. 7.If men is fixed,work is proportional to time. If work is fixed, then time is inversely proportional to men therefore, (M1*T1/W1)=(M2*T2/W2) Problems 1)If 9 men working 6 hours a day can do a work in 88 days. Then 6 men working 8 hours a day can do it in how many days? Sol: From the above formula i.e (m1*t1/w1)=(m2*t2/w2) so (9*6*88/1)=(6*8*d/1) on solving, d=99 days. 2)If 34 men completed 2/5th of a work in 8 days working 9 hours a day. How many more man should be engaged to finish the rest of the work in 6 days working 9 hours a day ? Sol: From the above formula i.e (m1*t1/w1)=(m2*t2/w2) so, (34*8*9/(2/5))=(x*6*9/(3/5)) so x=136 men number of men to be added to finish the work=136-34=102 men

3)If 5 women or 8 girls can do a work in 84 days. In how many days can 10 women and 5 girls can do the same work ? Sol: Given that 5 women is equal to 8 girls to complete a work so, 10 women=16 girls. Therefore 10women +5girls=16girls+5girls=21girls. 8 girls can do a work in 84 days then 21 girls -------------- -? answer= (8*84/21)=32days. Therefore 10 women and 5 girls can a work in 32days

4)Worker A takes 8 hours to do a job. Worker B takes 10hours to do the same job. How long it take both A & B, working together but independently, to do the same job ? Sol: A's one hour work=1/8. B's one hour work=1/10 (A+B)'s one hour work=1/8+1/10 =9/40 Both A & B can finish the work in 40/9 days

5)A can finish a work in 18 days and B can do the same work in half the time taken by A. Then, working together, what part of the same work they can finish in a day ? Sol: Given that B alone can complete the same work in days=half the time taken by A=9days A's one day work=1/18 B's one day work=1/9 (A+B)'s one day work=1/18+1/9=1/6

6)A is twice as good a workman as B and together they finish a piece of

file:///E|/work/books/placement/09_Aptitude/timeandwork.html[1/28/2012 12:45:43 AM]

work in 18 days.In how many days will A alone finish the work. Sol: if A takes x days to do a work then B takes 2x days to do the same work =>1/x+1/2x=1/18 =>3/2x=1/18 =>x=27 days. Hence, A alone can finish the work in 27 days.

7)A can do a certain work in 12 days. B is 60% more efficient than A. How many days does B alone take to do the same job? Sol: Ratio of time taken by A&B=160:100 =8:5 Suppose B alone takes x days to do the job. Then, 8:5::12:x => 8x=5*12 => x=15/2 days.

8)A can do a piece of work n 7 days of 9 hours each and B alone can do it in 6 days of 7 hours each. How long will they take to do it working together 8 2/5 hours a day ? Sol: A can complete the work in (7*9)=63 days B can complete the work in (6*7)=42 days => A's one hour's work=1/63 and B's one hour work=1/42 (A+B)'s one hour work=1/63+1/42=5/126 Therefore, Both can finish the work in 126/5 hours. Number of days of 8 2/5 hours each=(126*5/(5*42))=3days

9)A takes twice as much time as B or thrice as much time to finish a piece of work. Working together they can finish the work in 2 days. B can do the work alone in ? Sol: Suppose A,B and C take x,x/2 and x/3 hours respectively finish the work then 1/x+2/x+3/x=1/2 => 6/x=1/2 =>x=12 So, B takes 6 hours to finish the work.

10)X can do of a work in 10 days, Y can do 40% of work in 40 days and Z can do 1/3 of work in 13 days. Who will complete the work first? Sol: Whole work will be done by X Whole work will be done by Y Whole work will be done by Z Therefore,Z will complete the in 10*4=40 days. in (40*100/40)=100 days. in (13*3)=39 days work first.

Top
Complex Problems 1)A and B undertake to do a piece of workfor Rs 600.A alone can do it in 6 days while B alone can do it in 8 days. With the help of C, they can finish it in 3 days, Find the share of each ? Sol: C's one day's work=(1/3) - (1/6+1/8)=1/24 Therefore, A:B:C= Ratio of their one days work=1/6:1/8:1/24=4:3:1 A's share=Rs (600*4/8)=300 B's share= Rs (600*3/8)=225 C's share=Rs[600 - (300+225)]=Rs 75

2)A can do a piece of work in 80 days. He works at it for 10 days & then B alone finishes the remaining work in 42 days. In how much time will A and B, working together, finish the work ? Sol: Work done by A in 10 days=10/80=1/8 Remaining work=(1 - (1/8))=7/8 Now, work will be done by B in 42 days. Whole work will be done by B in (42*8/7)=48 days Therefore, A's one day's work=1/80 Bs one day's work=1/48 (A+B)'s one day's work=1/80+1/48=8/240=1/30 Hence, both will finish the work in 30 days.

3)P,Q and R are three typists who working simultaneously can type 216 pages in 4 hours In one hour , R can type as many pages more than Q as Q can type more than P. During a period of five hours, R can type as many pages as P can during seven hours. How many pages does each of them type per hour? Sol:Let the number of pages typed in one hour by P, Q and R be x,y and z respectively Then x+y+z=216/4=54 ---------------1 z-y=y-x => 2y=x+z -----------2 5z=7x => x=5x/7 ---------------3 Solving 1,2 and 3 we get x=15,y=18, and z=21 4)Ronald and Elan are working on an assignment. Ronald takes 6 hours to type 32 pages on a computer, while Elan takes 5 hours to type 40 pages. How much time will they take, working together on two different computers to type an assignment of 110 pages ? Sol: Number of pages typed Number of pages typed Number of pages typed Time taken by both to by Ronald in one hour=32/6=16/3 by Elan in one hour=40/5=8 by both in one hour=((16/3)+8)=40/3 type 110 pages=110*3/40=8 hours.

5)Two workers A and B are engaged to do a work. A working alone takes 8 hours more to complete the job than if both working together. If B worked alone, he would need 4 1/2 hours more to compete the job than they both working together. What time would they take to do the work together. Sol: (1/(x+8))+(1/(x+(9/2)))=1/x =>(1/(x+8))+(2/(2x+9))=1/x => x(4x+25)=(x+8)(2x+9) => 2x2 =72 => x2 = 36 => x=6 Therefore, A and B together can do the work in 6 days.

6)A and B can do a work in12 days, B and C in 15 days, C and A in 20 days. If A,B and C work together, they will complete the work in how many days? Sol: (A+B)'s one day's work=1/12; (B+C)'s one day's work=1/15; (A+C)'s one day's work=1/20; Adding we get 2(A+B+C)'s one day's work=1/12+1/15+1/20=12/60=1/5 (A+B+C)'s one day work=1/10 So, A,B,and C together can complete the work in 10 days.

7)A and B can do a work in 8 days, B and C can do the same wor in 12 days. A,B and C together can finish it in 6 days. A and C together will do it in

file:///E|/work/books/placement/09_Aptitude/timeandwork.html[1/28/2012 12:45:43 AM]

how many days ? Sol: (A+B+C)'s one day's work=1/6; (A+B)'s one day's work=1/8; (B+C)'s one day's work=1/12; (A+C)'s one day's work=2(A+B+C)'s one day's work-((A+B)'s one day work+(B+C)'s one day work) = (2/6) -(1/8+1/12) =(1/3) - (5/24) =3/24 =1/8 So, A and C together will do the work in 8 days.

8)A can do a certain work in the same time in which B and C together can do it. If A and B together could do it in 10 days and C alone in 50 days, then B alone could do it in how many days ? Sol: (A+B)'s one day's work=1/10; C's one day's work=1/50 (A+B+C)'s one day's work=(1/10+1/50)=6/50=3/25 Also, A's one day's work=(B+C)s one day's work From i and ii ,we get :2*(A's one day's work)=3/25 => A's one day's work=3/50 B's one days work=(1/10 - 3/50) =2/50 =1/25 B alone could complete the work in 25 days.

9) A is thrice as good a workman as B and therefore is able to finish a job in 60 days less than B. Working together, they can do it in: Sol: Ratio of times taken by A and B=1:3. If difference of time is 2 days , B takes 3 days If difference of time is 60 days, B takes (3*60/2)=90 days So, A takes 30 days to do the work=1/90 A's one day's work=1/30; B's one day's work=1/90; (A+B)'s one day's work=1/30+1/90=4/90=2/45 Therefore, A&B together can do the work in 45/2days

Top
10) A can do a piece of work in 80 days. He works at it for 10 days and then B alone finishes the remaining work in 42 days. In how much time will A&B, working together, finish the work ? Sol: Work Done by A n 10 days =10/80=1/8 Remaining work =1 - 1/8=7/8 Now 7/8 work is done by B in 42 days Whole work will be done by B in 42*8/7= 48 days => A's one day's work =1/80 and B's one day's work =1/48 (A+B)'s one day's work = 1/80+1/48 = 8/240 = 1/30 Hence both will finish the work in 30 days.

11) 45 men can complete a work in 16 days. Six days after they started working, so more men joined them. How many days will they now take to complete the remaining work ? Sol: M1*D1/W1=M2*D2/W2 =>45*6/(6/16)=75*x/(1 - (6/16)) => x=6 days

12)A is 50% as efficient as B. C does half the work done by A&B together. If C alone does the work n 40 days, then A,B and C together can do the work in: Sol: A's one day's work:B's one days work=150:100 =3:2 Let A's &B's one day's work be 3x and 2x days respectively. Then C's one day's work=5x/2 => 5x/2=1/40 => x=((1/40)*(2/5))=1/100 A's one day's work=3/100 B's one day's work=1/50 C's one day's work=1/40 So, A,B and C can do the work in 13 1/3 days.

13)A can finish a work in 18 days and B can do the same work in 15 days. B worked for 10 days and left the job. In how many days A alone can finish the remaining work ? Sol: B's 10 day's work=10/15=2/3 Remaining work=(1 - (2/3))=1/3 Now, 1/18 work is done by A in 1 day. Therefore 1/3 work is done by A in 18*(1/3)=6 days.

14)A can finish a work in 24 days, B n 9 days and C in 12 days. B&C start the work but are forced to leave after 3 days. The remaining work done by A in: Sol: (B+C)'s one day's work=1/9+1/12=7/36 Work done by B & C in 3 days=3*7/36=7/12 Remaining work=1 - (7/12)=5/12 Now , 1/24 work is done by A in 1 day. So, 5/12 work is done by A in 24*5/12=10 days

15)X and Y can do a piece of work n 20 days and 12 days respectively. X started the work alone and then after 4 days Y joined him till the completion of work. How long did the work last ? Sol: work done by X in 4 days =4/20 =1/5 Remaining work= 1 - 1/5 =4/5 (X+Y)'s one day's work =1/20+1/12 =8/60=2/15 Now, 2/15 work is done by X and Y in one day. So, 4/5 work will be done by X and Y in 15/2*4/5=6 days Hence Total time taken =(6+4) days = 10 days

16)A does 4/5 of work in 20 days. He then calls in B and they together finish the remaining work in 3 days. How long B alone would take to do the whole work? Sol: Whole work is done by A in 20*5/4=25 days Now, (1 - (4/5)) i.e 1/5 work is done by A& B in days. Whole work will be done by A& B in 3*5=15 days =>B's one day's work= 1/15 - 1/25=4/150=2/75 So, B alone would do the work in 75/2= 37 days.

17) A and B can do a piece of work in 45 days and 40 days respectively. They began to do the work together but A leaves after some days and then B completed the remaining work n 23 days. The number of days after which A left the work was Sol: (A+B)'s one day's work=1/45+1/40=17/360 Work done by B in 23 days=23/40 Remaining work=1 - (23/40)=17/40 Now, 17/360 work was done by (A+B) in 1 day.

file:///E|/work/books/placement/09_Aptitude/timeandwork.html[1/28/2012 12:45:43 AM]

17/40 work was done by (A+B) in (1*(360/17)*(17/40))= 9 days So, A left after 9 days. 18)A can do a piece of work in 10 days, B in 15 days. They work for 5 days. The rest of work finished by C in 2 days. If they get Rs 1500 for the whole work, the daily wages of B and C are Sol: Part of work done by A= 5/10=1/2 Part of work done by B=1/3 Part of work done by C=(1 - (1/2+1/3))=1/6 A's share: B's share: C's share=1/2:1/3:1/6= 3:2:1 A's share=(3/6)*1500=750 B's share=(2/6)*1500=500 C's share=(1/6)*1500=250 A's daily wages=750/5=150/ B's daily wages=500/5=100/ C's daily wages=250/2=125/ Daily wages of B&C = 100+125=225/-

19)A alone can complete a work in 16 days and B alone can complete the same in 12 days. Starting with A, they work on alternate days. The total work will be completed in how many days ? (a) 12 days (b) 13 days (c) 13 5/7 days (d)13 days Sol: (A+B)'s 2 days work = 1/16 + 1/12 =7/48 work done in 6 pairs of days =(7/48) * 6 = 7/8 remaining work = 1 - 7/8 = 1/8 work done by A on 13th day = 1/16 remaining work = 1/8 1/16 = 1/16 on 14th day, it is Bs turn 1/12 work is done by B in 1 day. 1/16 work is done by B in day. Total time taken= 13 days. So, Answer is: D

Top
20)A,B and C can do a piece of work in 20,30 and 60 days respectively. In how many days can A do the work if he is assisted by B and C on every third day? Sol: A's two day's work=2/20=1/10 (A+B+C)'s one day's work=1/20+1/30+1/60=6/60=1/10 Work done in 3 days=(1/10+1/10)=1/5 Now, 1/5 work is done in 3 days Therefore, Whole work will be done in (3*5)=15 days.

21)Seven men can complete a work in 12 days. They started the work and after 5 days, two men left. In how many days will the work be completed by the remaining men ? (A) 5 (B) 6 (C ) 7 (D) 8 (E) none Sol: 7*12 men complete the work in 1 day. Therefore, 1 man's 1 day's work=1/84 7 men's 5 days work = 5/12 =>remaining work = 1 - 5/12 = 7/12 5 men's 1 day's work = 5/84 5/84 work is don by them in 1 day 7/12 work is done by them in ((84/5) * (7/12)) = 49/5 days = 9 4/5 days. Ans: E

22).12 men complete a work in 9 days. After they have worked for 6 days, 6 more men joined them. How many days will they take to complete the remaining work? (a) 2 days (b) 3 days (c) 4 days (d) 5days Sol : 1 man's 1 day work = 1/108 12 men's 6 days work = 6/9 = 2/3 remaining work = 1 2/3 = 1/3 18 men's 1 days work = 18/108 = 1/6 1/6 work is done by them in 1 day therefore, 1/3 work is done by them in 6/3 = 2 days. Ans : A

23).A man, a woman and a boy can complete a job in 3,4 and 12 days respectively. How many boys must assist 1 man and 1 woman to complete the job in of a day? (a). 1 (b). 4 (c). 19 (d). 41 Sol : (1 man + 1 woman)'s 1 days work = 1/3+1/4=7/12 Work done by 1 man and 1 women n 1/4 day=((7/12)*(1/4))=7/48 Remaining work= 1 - 7/48= 41/48 Work done by 1 boy in day= ((1/12)*(1/4)) =1/48 Therefore, Number of boys required= ((41/48)*48)= 41 days So,Answer: D

24)12 men can complete a piece of work in 4 days, while 15 women can complete the same work in 4 days. 6 men start working on the job and after working for 2 days, all of them stopped working. How many women should be put on the job to complete the remaining work, if it is to be completed in 3 days. (A) 15 (B) 18 (C) 22 (D) data inadequate Sol: one man's one day's work= 1/48 one woman's one day's work=1/60 6 men's 2 day's work=((6/48)*2)= Remaining work=3/4 Now, 1/60 work s done in 1 day by 1 woman. So, work will be done in 3 days by (60*(3/4)*(1/3))= 15 woman. So, Answer: A

25)Twelve children take sixteen days to complete a work which can be completed by 8 adults in 12 days. Sixteen adults left and four children joined them. How many days will they take to complete the remaining work? (A) 3 (B) 4 ( C) 6 (D) 8 Sol: one child's one day work= 1/192; one adult's one day's work= 1/96; work done in 3 days=((1/96)*16*3)= 1/2 Remaining work= 1 =1/2 (6 adults+ 4 children)'s 1 day's work= 6/96+4/192= 1/12 1/12 work is done by them in 1 day. work is done by them 12*(1/2)= 6 days So, Answer= C

26)Sixteen men can complete a work in twelve days. Twenty four children can complete the same work in 18 days. 12 men and 8 children started working and after eight days three more children joined them. How many days will they now take to complete the remaining work ?

file:///E|/work/books/placement/09_Aptitude/timeandwork.html[1/28/2012 12:45:43 AM]

(A) 2 days (B) 4 days ( C) 6 days (D) 8 days ol: one man's one day's work= 1/192 one child's one day's work= 1/432 Work done in 8 days=8*(12/192+ 8/432)=8*(1/16+1/54) =35/54 Remaining work= 1 - 35/54= 19/54 (12 men+11 children)'s 1 day's work= 12/192 + 11/432 = 19/216 Now, 19/216 work is done by them in 1 day. Therefore, 19/54 work will be done by them in ((216/19)*(19/54))= 4 So,Answer: B 27)Twenty-four men can complete a work in 16 days. Thirty- two women can complete the same work in twenty - four days. Sixteen men and sixteen women started working and worked for 12 days. How many more men are to be added to complete the remaining work in 2 days ? (A) 16 men (B) 24 men ( C) 36 men (D) 48 men Sol: one man's one day's work= 1/384 one woman's one day's work=1/768 Work done in 12 days= 12*( 16/384 + 16/768) = 12*(3/48)=3/4 Remaining work=1 =1/4 (16 men+16 women)'s two day's work =12*( 16/384+16/768)=2/16=1/8 Remaining work = 1/4 - 1/8 =1/8 1/384 work is done n 1 day by 1 man. Therefore, 1/8 work will be done in 2 days in 384*(1/8)*(1/2)=24men

days

28)4 men and 6 women can complete a work in 8 days, while 3 men and 7 women can complete it in 10 days. In how many days will 10 women complete it? (A) 35 days (B) 40 days ( C) 45 days (D) 50 days Sol: Let 1 man's 1 day's work =x days and 1 woman's 1 day's work=y Then, 4x+6y=1/8 and 3x+7y=1/10. Solving these two equations, we get: x=11/400 and y= 1/400 Therefore, 1 woman's 1 day's work=1/400 => 10 women will complete the work in 40 days. Answer: B

29)One man,3 women and 4 boys can do a piece of work in 96hrs, 2 men and 8 boys can do it in 80 hrs, 2 men & 3 women can do it in 120hr. 5Men & 12 boys can do it in? (A) 39 1/11 hrs (B) 42 7/11 hrs ( C) 43 7/11 days (D) 44hrs Sol: Let 1 man's 1 hour's work=x 1 woman's 1 hour's work=y 1 boy's 1 hour's work=z Then, x+3y+4z=1/96 - - - - - - - - ---(1) 2x+8z= 1/80 ----------(2) adding (2) & (3) and subtracting (1) 3x+4z=1/96 - - - - - - - - - (4) From (2) and (4), we get x=1/480 Substituting, we get : y=1/720 and z= 1/960 (5 men+ 12 boy)'s 1 hour's work=5/480+12/960 =1/96 + 1/80=11/480 Therefore, 5 men and 12 boys can do the work in 480/11 or 43 So,Answer: C

7/11hours.

Back Back To Main

Top

Contact: 040-23000700

file:///E|/work/books/placement/09_Aptitude/timeandwork.html[1/28/2012 12:45:43 AM]

APTITUDE
Numbers H.C.F and L.C.M Decimal Fractions Simplification Square and Cube roots Average Problems on Numbers Problems on Ages Surds and Indices Percentage Profit and Loss Ratio And Proportions Partnership Chain Rule Time and Work Pipes and Cisterns Time and Distance Trains Boats and Streams Alligation or Mixture Simple Interest Compound Interest Logorithms Areas Volume and Surface area Races and Games of Skill Calendar Clocks Stocks ans Shares True Discount Bankers Discount Oddmanout and Series Data Interpretation probability Permutations and Combinations Puzzles BACK

COMPLEX PROBLEMS
1)A and B undertake to do a piece of workfor Rs 600.A alone can do it in 6 days while B alone can do it in 8 days. With the help of C, they can finish it in 3 days, Find the share of each? Sol: C's one day's work=(1/3)-(1/6+1/8)=1/24 Therefore, A:B:C= Ratio of their one days work=1/6:1/8:1/24=4:3:1 A's share=Rs (600*4/8)=300 B's share= Rs (600*3/8)=225 C's share=Rs[600-(300+225)]=Rs 75

2)A can do a piece of work in 80 days. He works at it for 10 days & then B alone finishes the remaining work in 42 days. In how much time will A and B, working together, finish the work? Sol: Work done by A in 10 days=10/80=1/8 Remaining work=(1-(1/8))=7/8 Now, work will be done by B in 42 days. Whole work will be done by B in (42*8/7)=48 days Therefore, A's one day's work=1/80 Bs one day's work=1/48 (A+B)'s one day's work=1/80+1/48=8/240=1/30 Hence, both will finish the work in 30 days.

3)P,Q and R are three typists who working simultaneously can type 216 pages in 4 hours In one hour , R can type as many pages more than Q as Q can type more than P. During a period of five hours, R can type as many pages as P can during seven hours. How many pages does each of them type per hour? Sol: Let the number of pages typed in one hour by P, Q and R be x,y and z respectively Then x+y+z=216/4=54 ---------------1 z-y=y-x => 2y=x+z -----------2 5z=7x => x=5x/7 ---------------3 Solving 1,2 and 3 we get x=15,y=18, and z=21 4)Ronald and Elan are working on an assignment. Ronald takes 6 hours to type 32 pages on a computer, while Elan takes 5 hours to type 40 pages. How much time will they take, working together on two different computers to type an assignment of 110 pages? Sol: Number of pages typed Number of pages typed Number of pages typed Time taken by both to by Ronald in one hour=32/6=16/3 by Elan in one hour=40/5=8 by both in one hour=((16/3)+8)=40/3 type 110 pages=110*3/40=8 hours.

5)Two workers A and B are engaged to do a work. A working alone takes 8 hours more to complete the job than if both working together. If B worked alone, he would need 4 1/2 hours more to compete the job than they both working together. What time would they take to do the work together. Sol: (1/(x+8))+(1/(x+(9/2)))=1/x =>(1/(x+8))+(2/(2x+9))=1/x => x(4x+25)=(x+8)(2x+9) => 2x2 =72 => x2 = 36 => x=6 Therefore, A and B together can do the work in 6 days.

6)A and B can do a work in12 days, B and C in 15 days, C and A in 20 days. If A,B and C work together, they will complete the work in how many days? Sol: (A+B)'s one day's work=1/12; (B+C)'s one day's work=1/15; (A+C)'s one day's work=1/20; Adding we get 2(A+B+C)'s one day's work=1/12+1/15+1/20=12/60=1/5 (A+B+C)'s one day work=1/10 So, A,B,and C together can complete the work in 10 days.

7)A and B can do a work in 8 days, B and C can do the same wor in 12 days.

file:///E|/work/books/placement/09_Aptitude/timeandworkcomplex.html[1/28/2012 12:45:44 AM]

A,B and C together can finish it in 6 days. A and C together will do it in how many days? Sol: (A+B+C)'s one day's work=1/6; (A+B)'s one day's work=1/8; (B+C)'s one day's work=1/12; (A+C)'s one day's work=2(A+B+C)'s one day's work-((A+B)'s one day work+(B+C)'s one day work) = (2/6)-(1/8+1/12) =(1/3)- (5/24) =3/24 =1/8 So, A and C together will do the work in 8 days.

8)A can do a certain work in the same time in which B and C together can do it. If A and B together could do it in 10 days and C alone in 50 days, then B alone could do it in how many days? Sol: (A+B)'s one day's work=1/10; C's one day's work=1/50 (A+B+C)'s one day's work=(1/10+1/50)=6/50=3/25 Also, A's one day's work=(B+C)s one day's work From i and ii ,we get :2*(A's one day's work)=3/25 => A's one day's work=3/50 B's one days work=(1/10-3/50) =2/50 =1/25 B alone could complete the work in 25 days.

9) A is thrice as good a workman as B and therefore is able to finish a job in 60 days less than B. Working together, they can do it in: Sol: Ratio of times taken by A and B=1:3. If difference of time is 2 days , B takes 3 days If difference of time is 60 days, B takes (3*60/2)=90 days So, A takes 30 days to do the work=1/90 A's one day's work=1/30; B's one day's work=1/90; (A+B)'s one day's work=1/30+1/90=4/90=2/45 Therefore, A&B together can do the work in 45/2days

10) A can do a piece of work in 80 days. He works at it for 10 days and then B alone finishes the remaining work in 42 days. In how much time will A&B, working together, finish the work? Sol: Work Done by A n 10 days =10/80=1/8 Remaining work =1-1/8=7/8 Now 7/8 work is done by B in 42 days Whole work will be done by B in 42*8/7= 48 days => A's one day's work =1/80 and B's one day's work =1/48 (A+B)'s one day's work = 1/80+1/48 = 8/240 = 1/30 Hence both will finish the work in 30 days.

11) 45 men can complete a work in 16 days. Six days after they started working, so more men joined them. How many days will they now take to complete the remaining work? Sol: M1*D1/W1=M2*D2/W2 =>45*6/(6/16)=75*x/(1-(6/16)) => x=6 days

12)A is 50% as efficient as B. C does half the work done by A&B together. If C alone does the work n 40 days, then A,B and C together can do the work in: Sol: A's one day's work:B's one days work=150:100 =3:2 Let A's &B's one day's work be 3x and 2x days respectively. Then C's one day's work=5x/2 => 5x/2=1/40 => x=((1/40)*(2/5))=1/100 A's one day's work=3/100 B's one day's work=1/50 C's one day's work=1/40 So, A,B and C can do the work in 13 1/3 days.

13)A can finish a work in 18 days and B can do the same work in 15 days. B worked for 10 days and left the job. In how many days A alone can finish the remaining work? Sol: B's 10 day's work=10/15=2/3 Remaining work=(1-(2/3))=1/3 Now, 1/18 work is done by A in 1 day.

file:///E|/work/books/placement/09_Aptitude/timeandworkcomplex.html[1/28/2012 12:45:44 AM]

Therefore 1/3 work is done by A in 18*(1/3)=6 days. 14)A can finish a work in 24 days, B n 9 days and C in 12 days. B&C start the work but are forced to leave after 3 days. The remaining work done by A in: Sol: (B+C)'s one day's work=1/9+1/12=7/36 Work done by B & C in 3 days=3*7/36=7/12 Remaining work=1-(7/12)=5/12 Now , 1/24 work is done by A in 1 day. So, 5/12 work is done by A in 24*5/12=10 days

15)X and Y can do a piece of work n 20 days and 12 days respectively. X started the work alone and then after 4 days Y joined him till the completion of work. How long did the work last? Sol: work done by X in 4 days =4/20 =1/5 Remaining work= 1-1/5 =4/5 (X+Y)'s one day's work =1/20+1/12 =8/60=2/15 Now, 2/15 work is done by X and Y in one day. So, 4/5 work will be done by X and Y in 15/2*4/5=6 days Hence Total time taken =(6+4) days = 10 days

16)A does 4/5 of work in 20 days. He then calls in B and they together finish the remaining work in 3 days. How long B alone would take to do the whole work? Sol: Whole work is done by A in 20*5/4=25 days Now, (1-(4/5)) i.e 1/5 work is done by A& B in days. Whole work will be done by A& B in 3*5=15 days =>B's one day's work= 1/15-1/25=4/150=2/75 So, B alone would do the work in 75/2= 37 days.

17) A and B can do a piece of work in 45 days and 40 days respectively. They began to do the work together but A leaves after some days and then B completed the remaining work n 23 days. The number of days after which A left the work was Sol: (A+B)'s one day's work=1/45+1/40=17/360 Work done by B in 23 days=23/40 Remaining work=1-(23/40)=17/40 Now, 17/360 work was done by (A+B) in 1 day. 17/40 work was done by (A+B) in (1*(360/17)*(17/40))= 9 days So, A left after 9 days.

18)A can do a piece of work in 10 days, B in 15 days. They work for 5 days. The rest of work finished by C in 2 days. If they get Rs 1500 for the whole work, the daily wages of B and C are Sol: Part of work done by A= 5/10=1/2 Part of work done by B=1/3 Part of work done by C=(1-(1/2+1/3))=1/6 A's share: B's share: C's share=1/2:1/3:1/6= 3:2:1 A's share=(3/6)*1500=750 B's share=(2/6)*1500=500 C's share=(1/6)*1500=250 A's daily wages=750/5=150/B's daily wages=500/5=100/C's daily wages=250/2=125/Daily wages of B&C = 100+125=225/-

19)A alone can complete a work in 16 days and B alone can complete the same in 12 days. Starting with A, they work on alternate days. The total work will be completed in how many days? (a) 12 days (b) 13 days (c) 13 5/7 days (d)13 days Sol: (A+B)'s 2 days work = 1/16 + 1/12 =7/48 work done in 6 pairs of days =(7/48) * 6 = 7/8 remaining work = 1- 7/8 = 1/8 work done by A on 13th day = 1/16 remaining work = 1/8 1/16 = 1/16 on 14th day, it is Bs turn 1/12 work is done by B in 1 day. 1/16 work is done by B in day. Total time taken= 13 days. So, Answer is: D

20)A,B and C can do a piece of work in 20,30 and 60 days respectively. In how many days can A do the work if he is assisted by B and C on every third day? Sol: A's two day's work=2/20=1/10 (A+B+C)'s one day's work=1/20+1/30+1/60=6/60=1/10 Work done in 3 days=(1/10+1/10)=1/5

file:///E|/work/books/placement/09_Aptitude/timeandworkcomplex.html[1/28/2012 12:45:44 AM]

Now, 1/5 work is done in 3 days Therefore, Whole work will be done in (3*5)=15 days. 21)Seven men can complete a work in 12 days. They started the work and after 5 days, two men left. In how many days will the work be completed by the remaining men? (A) 5 (B) 6 (C ) 7 (D) 8 (E) none Sol: 7*12 men complete the work in 1 day. Therefore, 1 man's 1 day's work=1/84 7 men's 5 days work = 5/12 =>remaining work = 1-5/12 = 7/12 5 men's 1 day's work = 5/84 5/84 work is don by them in 1 day 7/12 work is done by them in ((84/5) * (7/12)) = 49/5 days = 9 4/5 Ans: E 22).12 men complete a work in 9 days. After they have worked for 6 days, 6 more men joined them. How many days will they take to complete the remaining work? (a) 2 days (b) 3 days (c) 4 days (d) 5days Sol : 1 man's 1 day work = 1/108 12 men's 6 days work = 6/9 = 2/3 remaining work = 1 2/3 = 1/3 18 men's 1 days work = 18/108 = 1/6 1/6 work is done by them in 1 day therefore, 1/3 work is done by them in 6/3 = 2 days. Ans : A

days.

23).A man, a woman and a boy can complete a job in 3,4 and 12 days respectively. How many boys must assist 1 man and 1 woman to complete the job in of a day? (a). 1 (b). 4 (c). 19 (d). 41 Sol : (1 man + 1 woman)'s 1 days work = 1/3+1/4=7/12 Work done by 1 man and 1 women n 1/4 day=((7/12)*(1/4))=7/48 Remaining work= 1- 7/48= 41/48 Work done by 1 boy in day= ((1/12)*(1/4)) =1/48 Therefore, Number of boys required= ((41/48)*48)= 41 days So,Answer: D

24)12 men can complete a piece of work in 4 days, while 15 women can complete the same work in 4 days. 6 men start working on the job and after working for 2 days, all of them stopped working. How many women should be put on the job to complete the remaining work, if it is to be completed in 3 days. (A) 15 (B) 18 (C) 22 (D) data inadequate Sol: one man's one day's work= 1/48 one woman's one day's work=1/60 6 men's 2 day's work=((6/48)*2)= Remaining work=3/4 Now, 1/60 work s done in 1 day by 1 woman. So, work will be done in 3 days by (60*(3/4)*(1/3))= 15 woman. So, Answer: A

25)Twelve children take sixteen days to complete a work which can be completed by 8 adults in 12 days. Sixteen adults left and four children joined them. How many days will they take to complete the remaining work? (A) 3 (B) 4 ( C) 6 (D) 8 Sol: one child's one day work= 1/192; one adult's one day's work= 1/96; work done in 3 days=((1/96)*16*3)= 1/2 Remaining work= 1 =1/2 (6 adults+ 4 children)'s 1 day's work= 6/96+4/192= 1/12 1/12 work is done by them in 1 day. work is done by them 12*(1/2)= 6 days So, Answer= C

26)Sixteen men can complete a work in twelve days. Twenty four children can complete the same work in 18 days. 12 men and 8 children started working and after eight days three more children joined them. How many days will they now take to complete the remaining work? (A) 2 days (B) 4 days ( C) 6 days (D) 8 days ol: one man's one day's work= 1/192

file:///E|/work/books/placement/09_Aptitude/timeandworkcomplex.html[1/28/2012 12:45:44 AM]

one child's one day's work= 1/432 Work done in 8 days=8*(12/192+ 8/432)=8*(1/16+1/54) =35/54 Remaining work= 1 -35/54= 19/54 (12 men+11 children)'s 1 day's work= 12/192 + 11/432 = 19/216 Now, 19/216 work is done by them in 1 day. Therefore, 19/54 work will be done by them in ((216/19)*(19/54))= 4 days So,Answer: B 27)Twenty-four men can complete a work in 16 days. Thirty- two women can complete the same work in twenty-four days. Sixteen men and sixteen women started working and worked for 12 days. How many more men are to be added to complete the remaining work in 2 days? (A) 16 men (B) 24 men ( C) 36 men (D) 48 men Sol: one man's one day's work= 1/384 one woman's one day's work=1/768 Work done in 12 days= 12*( 16/384 + 16/768) = 12*(3/48)=3/4 Remaining work=1 =1/4 (16 men+16 women)'s two day's work =12*( 16/384+16/768)=2/16=1/8 Remaining work = 1/4-1/8 =1/8 1/384 work is done n 1 day by 1 man. Therefore, 1/8 work will be done in 2 days in 384*(1/8)*(1/2)=24men 28)4 men and 6 women can complete a work in 8 days, while 3 men and 7 women can complete it in 10 days. In how many days will 10 women complete it? (A) 35 days (B) 40 days ( C) 45 days (D) 50 days Sol: Let 1 man's 1 day's work =x days and 1 woman's 1 day's work=y Then, 4x+6y=1/8 and 3x+7y=1/10. Solving these two equations, we get: x=11/400 and y= 1/400 Therefore, 1 woman's 1 day's work=1/400 => 10 women will complete the work in 40 days. Answer: B

29)One man,3 women and 4 boys can do a piece of work in 96hrs, 2 men and 8 boys can do it in 80 hrs, 2 men & 3 women can do it in 120hr. 5Men & 12 boys can do it in? (A) 39 1/11 hrs (B) 42 7/11 hrs ( C) 43 7/11 days (D) 44hrs Sol: Let 1 man's 1 hour's work=x 1 woman's 1 hour's work=y 1 boy's 1 hour's work=z Then, x+3y+4z=1/96 -----------(1) 2x+8z= 1/80 ----------(2) adding (2) & (3) and subtracting (1) 3x+4z=1/96 ---------(4) From (2) and (4), we get x=1/480 Substituting, we get : y=1/720 and z= 1/960 (5 men+ 12 boy)'s 1 hour's work=5/480+12/960 =1/96 + 1/80=11/480 Therefore, 5 men and 12 boys can do the work in 480/11 or 43 So,Answer: C
BACK

7/11hours.

file:///E|/work/books/placement/09_Aptitude/timeandworkcomplex.html[1/28/2012 12:45:44 AM]

APTITUDE
Numbers H.C.F and L.C.M Decimal Fractions Simplification Square and Cube roots Average Problems on Numbers Problems on Ages Surds and Indices Percentage Profit and Loss Ratio And Proportions Partnership Chain Rule Time and Work Pipes and Cisterns Time and Distance Trains Boats and Streams Alligation or Mixture Simple Interest Compound Interest Logorithms Areas Volume and Surface area Races and Games of Skill Calendar Clocks Stocks ans Shares True Discount Bankers Discount Oddmanout and Series Data Interpretation probability Permutations and Combinations Puzzles BACK

TIME AND WORK


1.If A can do a piece of work in n days, then A's 1 day work=1/n 2.If A's 1 day's work=1/n, then A can finish the work in n days. Ex: If A can do a piece of work in 4 days,then A's 1 day's work=1/4. If A's 1 days work=1/5, then A can finish the work in 5 days 3.If A is thrice as good workman as B,then: Ratio of work done by A and B =3:1 Ratio of time taken by A and B to finish a work=1:3 4.DEFINATION OF VARIATION: The change in two different variables follow some definite rule. It said that the two variables vary directly or inversely. Its notation is X/Y=k, where k is called constant. This variation is called direct variation. XY=k. This variation is called inverse variation. 5.SOME PAIRS OF VARIABLES: i)Number of workers and their wages. If the number of workers increases, their total wages increase. If the number of days reduced, there will be less work. If the number of days is increased, there will be more work. Therefore, here we have direct proportion or direct variation. ii)Number workers and days required to do a certain work is an example of inverse variation. If more men are employed, they will require fewer days and if there are less number of workers, more days are required. iii)There is an inverse proportion between the daily hours of a work and the days required. If the number of hours is increased, less number of days are required and if the number of hours is reduced, more days are required. 6.SOME IMPORTANT TIPS: More Men -Less Days and More Men -More Work and More Days-More Work and Number of days required Conversely More Conversely More Conversely More to complete the Day-Less Men. Work-More Men. Work-More Days. given work=Total work/One days work.

Since the total work is assumed to be one(unit), the number of days required to complete the given work would be the reciprocal of one days work. Sometimes, the problems on time and work can be solved using the proportional rule ((man*days*hours)/work) in another situation. 7.If men is fixed,work is proportional to time. If work is fixed, then time is inversely proportional to men therefore, (M1*T1/W1)=(M2*T2/W2)
BACK

file:///E|/work/books/placement/09_Aptitude/timeandworkconcept.html[1/28/2012 12:45:44 AM]

APTITUDE
Numbers H.C.F and L.C.M Decimal Fractions Simplification Square and Cube roots Average Problems on Numbers Problems on Ages Surds and Indices Percentage Profit and Loss Ratio And Proportions Partnership Chain Rule Time and Work Pipes and Cisterns Time and Distance Trains Boats and Streams Alligation or Mixture Simple Interest Compound Interest Logorithms Areas Volume and Surface area Races and Games of Skill Calendar Clocks Stocks ans Shares True Discount Bankers Discount Oddmanout and Series Data Interpretation probability Permutations and Combinations Puzzles BACK

SIMPLE PROBLEMS
1)If 9 men working 6 hours a day can do a work in 88 days. Then 6 men working 8 hours a day can do it in how many days? Sol: From the above formula i.e (m1*t1/w1)=(m2*t2/w2) so (9*6*88/1)=(6*8*d/1) on solving, d=99 days. 2)If 34 men completed 2/5th of a work in 8 days working 9 hours a day.How many more man should be engaged to finish the rest of the work in 6 days working 9 hours a day? Sol: From the above formula i.e (m1*t1/w1)=(m2*t2/w2) so, (34*8*9/(2/5))=(x*6*9/(3/5)) so x=136 men number of men to be added to finish the work=136-34=102 men 3)If 5 women or 8 girls can do a work in 84 days. In how many days can 10 women and 5 girls can do the same work? Sol: Given that 5 women is equal to 8 girls to complete a work so, 10 women=16 girls. Therefore 10women +5girls=16girls+5girls=21girls. 8 girls can do a work in 84 days then 21 girls ---------------? answer= (8*84/21)=32days. Therefore 10 women and 5 girls can a work in 32days

4)Worker A takes 8 hours to do a job. Worker B takes 10hours to do the same job. How long it take both A & B, working together but independently, to do the same job? Sol: A's one hour work=1/8. B's one hour work=1/10 (A+B)'s one hour work=1/8+1/10 =9/40 Both A & B can finish the work in 40/9 days

5)A can finish a work in 18 days and B can do the same work in half the time taken by A. Then, working together, what part of the same work they can finish in a day? Sol: taken Given that B alone can complete the same work in days=half the time by A=9days A's one day work=1/18 B's one day work=1/9 (A+B)'s one day work=1/18+1/9=1/6 6)A is twice as good a workman as B and together they finish a piece of work in 18 days.In how many days will A alone finish the work. Sol: if A takes x days to do a work then B takes 2x days to do the same work =>1/x+1/2x=1/18 =>3/2x=1/18 =>x=27 days. Hence, A alone can finish the work in 27 days.

7)A can do a certain work in 12 days. B is 60% more efficient than A. How many days does B alone take to do the same job? Sol: Ratio of time taken by A&B=160:100 =8:5 Suppose B alone takes x days to do the job. Then, 8:5::12:x => 8x=5*12 => x=15/2 days.

8)A can do a piece of work n 7 days of 9 hours each and B alone can do it in 6 days of 7 hours each. How long will they take to do it working together 8 2/5 hours a day? Sol: A can complete the work in (7*9)=63 days B can complete the work in (6*7)=42 days

file:///E|/work/books/placement/09_Aptitude/timeandworksimple.html[1/28/2012 12:45:45 AM]

=> A's one hour's work=1/63 and B's one hour work=1/42 (A+B)'s one hour work=1/63+1/42=5/126 Therefore, Both can finish the work in 126/5 hours. Number of days of 8 2/5 hours each=(126*5/(5*42))=3days 9)A takes twice as much time as B or thrice as much time to finish a piece of work. Working together they can finish the work in 2 days. B can do the work alone in ? Sol: Suppose A,B and C take x,x/2 and x/3 hours respectively finish the work then 1/x+2/x+3/x=1/2 => 6/x=1/2 =>x=12 So, B takes 6 hours to finish the work.

10)X can do of a work in 10 days, Y can do 40% of work in 40 days and Z can do 1/3 of work in 13 days. Who will complete the work first? Sol: Whole work will be done by X in 10*4=40 days. Whole work will be done by Y in (40*100/40)=100 days. Whole work will be done by Z in (13*3)=39 days Therefore,Z will complete the work first.

BACK

file:///E|/work/books/placement/09_Aptitude/timeandworksimple.html[1/28/2012 12:45:45 AM]

APTITUDE
Numbers H.C.F and L.C.M Decimal Fractions Simplification Square and Cube roots Average Problems on Numbers Problems on Ages Surds and Indices Percentage Profit and Loss Ratio And Proportions Partnership Chain Rule Time and Work Pipes and Cisterns Time and Distance Trains Boats and Streams Alligation or Mixture Simple Interest Compound Interest Logorithms Areas Volume and Surface area Races and Games of Skill Calendar Clocks Stocks ans Shares True Discount Bankers Discount Oddmanout and Series Data Interpretation probability Permutations and Combinations Puzzles BACK

TRAINS
General Concept:(1) Time taken by a train x mt long in passing a signal post or a pole or a standing man = time taken by the train to cover x mt (2) Time taken by a train x mt long in passing a stationary object of length y mt = time taken by the train to cover x+y mt (3) Suppose two trains or two bodies are moving in the same direction at u kmph and v kmph such that u > v then their relative speed is u-v kmph (4)If two trains of length x km and y km are moving in opposite diredtions at u kmph and vmph,then time taken by the train to cross each other = (x+y)/(u+v) hr (5) Suppose two trains or two bdies are moving in opposite direction at u kmph and v kmph then,their relative speed = (u+v) kmph (6)If two train start at the same time from 2 points A & B towards each other and after crossing they take a & b hours in reaching B & A respectively then A's speed : B's speed = (b^1/2 : a^1/2 ) BACK

file:///E|/work/books/placement/09_Aptitude/trainconcept.html[1/28/2012 12:45:45 AM]

APTITUDE
Numbers H.C.F and L.C.M Decimal Fractions Simplification Square and Cube roots Average Problems on Numbers Problems on Ages Surds and Indices Percentage Profit and Loss Ratio And Proportions Partnership Chain Rule Time and Work Pipes and Cisterns Time and Distance Trains Boats and Streams Alligation or Mixture Simple Interest Compound Interest Logorithms Areas Volume and Surface area Races and Games of Skill Calendar Clocks Stocks ans Shares True Discount Bankers Discount Oddmanout and Series Data Interpretation probability Permutations and Combinations Puzzles BACK

TRAINS
Problems:(1)Find the time taken by a train 180m long,running at 72kmph in crossing an electric pole Solution :: Speed of the train =72*5/18m/s =20 m/s Distance move din passing the pole = 180m Requiredtime = 180/20 = 9 seconds

(2)A train 140 m long running at 60kmph.In how much time will it pass a platform 260m long. Solution :: Distance travelled =140 + 260 m =400 m, speed = 60 * time=400*3 5/18 = 50//3 m / 50 = 24 Seconds

(3)A man is standing on a railway bridge which is 180 m.He finds that a train crosses the bridge in 20 seconds but himself in 8 sec. Find the length of the train and its sppeed Solution :: i)D=180+x T = 20 seconds S= 180+x / 20 ii)D=x T=8 seconds D=ST x=8S Substitute 2 in 1 S=180 + 8 S S=15 m/s Length of the train,x is 8 *15 = 120 m / 20 ------------2 ------------ 1

file:///E|/work/books/placement/09_Aptitude/trainproblem.html[1/28/2012 12:45:45 AM]

(4)A train 150m long is running with a speed of 68 mphIn wht time will it pass a man who is running at a speed of 8kmph in the same direction in which the train is going Solution :: Relative Speed = 68-8=60kmph*5/18 = 50/3 m/s time= 150 * 3 /50 =9sec

(5)A train 220m long is running with a speed

of 59 k mph /..In what

time will it pass a man who is running at 7 kmph in the direction opposite to that in which train is going. Solution :: Relative Speed = 59+7=66kmph*5/18 = 55/3 m/s time= 220/55 * 3 =12sec (6)Two trains 137m and 163m in length are running towards each other on parallel lines,one at the rate of 42kmph & another at 48 mph.In wht time will they be clear of each other from the moment they meet. Solution :: Relative speed =42+48 = 90 *5/18 = 25m/s time taken by the train to pass each other = time taken to cover (137+163)m at 25 m/s = 300 /25 s =12 s

(7)A train running at 54 kmph takes 20 sec to pass a platform .Next it takes 12 sec to pass a man walking at 6kmph in the same direction in which the train is going.Find length of the train and length of platform Solution :: Relative speed w.r.t man = 54-6=48kmph the length of the train is 48 * 5/18 * 12 =160m time taken to pass platform =20 sec

Speed of the train = 54 * 5/18 =15m/s 160+x =20 *15 x=140m length of the platform is 140m (8)A man sitting in a train which is travelling at 50mph observes that a goods train travelling in opposite irection takes 9 sec to pass him .If the goos train Solution :: is 150m long fin its speed

file:///E|/work/books/placement/09_Aptitude/trainproblem.html[1/28/2012 12:45:45 AM]

Relative speed =150/9 m/s =60 mph speed of the train = 60-50 =10kmph

(9)Two trains are moving in the sam e direction at 65kmph and 45kmph.the faster train crosses a man in slower train in18sec.the length of the faster train is Solution :: Relative speed =65-45 kmph = 50/9 m/s Distancce covered in18 s =50/9 * 18 = 100m the length of the train is 100m

(10)Atrain overtakes two persons who are walking in the same direction in which the train is going at the rate of 2kmph an 4kmph and passes them completely in 9 sec an 10 sec respectively.the length of train is Solution :: 2kmph = 5/9 m/s 4 mph =10/9 m/s Let the length 10 9y-5 =x 9y-x=5 and and 10(9y-10)=9x 90y-9x=100 of the trainbe x meters and its speed is y m/s = 9 and x / (y- 10/9) =

then x / (y- 5/9)

on solving we get x=50,lenght of trains

(11) Two stations A & B are 110 km from A at 7am and travels towards B

apart on

a straight line.One train starts

at 20kmph.Another train starts

from B at 8am an travels toward A at a speed of 25kmph.At what time will they meet Solution :: Suppose the train meet x hr after 7am Distance covered by A in x hr=20x km 20x+25(x-1) = 110 45x=135 x=3 So they meet at 10 am (12)A traintravelling at 48kmph completely crosses another train having half its length an travelling inopposite direction at 42kmph in12 sec.It also passes a railway platform in 45sec.the length of platform is Solution :: Let the length of the first train be x mt then,the length of second train is x/2 mt

file:///E|/work/books/placement/09_Aptitude/trainproblem.html[1/28/2012 12:45:45 AM]

relative speed = 48+42 kmph =90 * 5/18 m/s = 25m/s (x+ x/2)/25 =12 x=200 Length of the train is 200m Let the length of the platform be y mt speed f the first train = 48*5/18 m/s = 40/3 m/s 200+y * 3/40 = 45 y=400m

(13)The length of a running trsain in 30% more than the length of anpther train B runnng in the opposite direction.To find out the speed of trtain B,which of the following information given in the statements P & Q is sufficient P : The speed of train A is 80 kmph Q : They too 90 sec to cross each other (a) Either P & Q is sufficient (b)Both P & Q are not sufficient (c)only Q is sufficient (d)Both P & Q are neeed Ans: B Solution :: Let the length of th e train A be x mt Length of the train B = 130/100 x mt =13x/10 mt Let the speed of B be y mph,speed of the train A=80mph relative speed= y+80 * 5/18 m/s time taken by the trains t cross each other is gven by 90 = (x + 13x/10)/ (5y+400 / 18) to find y,clearly xis also needed so,both P & Q are not sufficient (14)The speed of a train A,100m long is 40% more than then the speedd of another train B,180m long running in opposite direction.To fin out the speed of B,which of the information given in statements P & Q is sufficient P :The two trains crossed each other in 6 seconds Q : The difference between the spee of the trains is 26kmph (a)Only P is sufficient (b)Only Q is sufficient (c)Both P & Q are needed (d)Both P & Q are not sufficient Ans : A Solution :: Let speed of B be x kmph then,speed of A =140x/100 kmph =7x/5 mph relative speed = x + 7x/5 =2x/3 m/s time taken to cross each other = (100+180)*3/2x s =420/x s now,420/x = 6 x=70 mph thus,only P is sufficient (15)The train running at certain speed crosses astationary enginein 20 seconds.to find out the sped of the train,which of the following information is necessary (a)Only the length of the train (b)only the length of the engine (c)Either the length of the train or length of engine (d)Both the length of the train or length of engine Ans : D Solution :: Since the sum of lengths of the tran and the engine is needed, so both the length must be known BACK

file:///E|/work/books/placement/09_Aptitude/trainproblem.html[1/28/2012 12:45:45 AM]

Trains
General Concept: (1) Time taken by a train x mt long in passing a signal post or a pole or a standing man = time taken by the train to cover x mt (2) Time taken by a train x mt long in passing a stationary object of length y mt = time taken by the train to cover x+y mt (3) Suppose two trains or two bodies are moving in the same direction at u kmph and v kmph such that u > v then their relative speed is u-v kmph (4)If two trains of length x km and y km are moving in opposite diredtions at u kmph and vmph,then time taken by the train to cross each other = (x+y)/(u+v) hr (5) Suppose two trains or two bdies are moving in opposite direction at u kmph and v kmph then,their relative speed = (u+v) kmph (6)If two train start at the same time from 2 points A & B towards each other and after crossing they take a & b hours in reaching B & A respectively then A's speed : B's speed = (b^1/2 : a^1/2 ) Problems (1)Find the time taken by a train 180m long,running at 72kmph in crossing an electric pole Solution: Speed of the train =72*5/18m/s =20 m/s Distance move din passing the pole = 180m Requiredtime = 180/20 = 9 seconds (2)A train 140 m long running at 60kmph.In how much time will it pass a platform 260m long. Solution: Distance travelled =140 + 260 m =400 m, speed = 60 * 5/18 = 50//3 m time=400*3 / 50 = 24 Seconds (3)A man is standing on a railway bridge which is 180 m.He finds that a train crosses the bridge in 20 seconds but himself in 8 sec. Find the length of the train and its sppeed Solution: i)D=180+x T = 20 seconds S= 180+x / 20 ------------ 1 ii)D=x T=8 seconds D=ST x=8S ------------- 2 Substitute 2 in 1 S=180 + 8 S / 20 S=15 m/s Length of the train,x is 8 *15 = 120 m (4)A train 150m long is running with a speed of 68 mphIn wht time will it pass a man who is running at a speed of 8kmph in the same direction in which the train is going Solution: Relative Speed = 68-8=60kmph*5/18 = 50/3 m/s time= 150 * 3 /50 =9sec 5)A train 220m long is running with a speed of 59 k mph /..In what time will it pass a man who is running at 7 kmph in the direction opposite to that in which train is going. Solution: Relative Speed = 59+7=66kmph*5/18 = 55/3 m/s time= 220/55 * 3 =12sec

Top
(6)Two trains 137m and 163m in length are running towards each other on parallel lines,one at the rate of 42kmph & another at 48 mph.In wht time will they be clear of each other from the moment they meet. Solution: Relative speed =42+48 = 90 *5/18 = 25m/s time taken by the train to pass each other = time taken to cover (137+163)m at 25 m/s = 300 /25 s =12 s

file:///E|/work/books/placement/09_Aptitude/trains.html[1/28/2012 12:45:46 AM]

(7)A train running at 54 kmph takes 20 sec to pass a platform. Next it takes 12 sec to pass a man walking at 6kmph in the same direction in which the train is going.Find length of the train and length of platform Solution: Relative speed w.r.t man = 54-6=48kmph the length of the train is 48 * 5/18 * 12 =160m time taken to pass platform =20 sec Speed of the train = 54 * 5/18 =15m/s 160+x =20 *15 x=140m length of the platform is 140m (8)A man sitting in a train which is travelling at 50mph observes that a goods train travelling in opposite irection takes 9 sec to pass him .If the goos train is 150m long fin its speed Solution: Relative speed =150/9 m/s =60 mph speed of the train = 60-50 =10kmph (9)Two trains are moving in the sam e direction at 65kmph and 45kmph. The faster train crosses a man in slower train in18sec.the length of the faster train is Solution: Relative speed =65-45 kmph = 50/9 m/s Distancce covered in18 s =50/9 * 18 = 100m the length of the train is 100m (10)Atrain overtakes two persons who are walking in the same direction in which the train is going at the rate of 2kmph an 4kmph and passes them completely in 9 sec an 10 sec respectively. The length of train is Solution: 2kmph = 5/9 m/s 4 mph =10/9 m/s Let the length of the trainbe x meters and its speed is y m/s then x / (y- 5/9) = 9 and x / (y- 10/9) = 10 9y-5 =x and 10(9y-10)=9x 9y-x=5 and 90y-9x=100 on solving we get x=50,lenght of trains (11) Two stations A & B are 110 km apart on a straight line. One train starts from A at 7am and travels towards B at 20kmph. Another train starts from B at 8am an travels toward A at a speed of 25kmph.At what time will they meet Solution: Suppose the train meet x hr after 7am Distance covered by A in x hr=20x km 20x+25(x-1) = 110 45x=135 x=3 So they meet at 10 am (12)A traintravelling at 48kmph completely crosses another train having half its length an travelling inopposite direction at 42kmph in12 sec.It also passes a railway platform in 45sec.the length of platform is Solution: Let the length of the first train be x mt then,the length of second train is x/2 mt relative speed = 48+42 kmph =90 * 5/18 m/s = 25m/s (x+ x/2)/25 =12 x=200 Length of the train is 200m Let the length of the platform be y mt speed f the first train = 48*5/18 m/s = 40/3 m/s 200+y * 3/40 = 45 y=400m

Top
(13)The length of a running trsain in 30% more than the length of another train B runnng in the opposite direction.To find out the speed of trtain B,which of the following information given in the statements P & Q is sufficient P : The speed of train A is 80 kmph Q : They too 90 sec to cross each other (a) Either P & Q is sufficient (b)Both P & Q are not sufficient (c)only Q is sufficient (d)Both P & Q are neeed Ans: B Solution: Let the length of th e train A be x mt Length of the train B = 130/100 x mt =13x/10 mt Let the speed of B be y mph,speed of the train A=80mph relative speed= y+80 * 5/18 m/s time taken by the trains t cross each other is gven by 90 = (x + 13x/10)/ (5y+400 / 18) to find y,clearly xis also needed so,both P & Q are not sufficient (14)The speed of a train A,100m long is of another train B,180m long running in the speed of B,which of the information sufficient P :The two trains crossed each other Q : The difference between the spee (a)Only P is sufficient (b)Only Q is sufficient (c)Both P & Q are needed 40% more than then the speed opposite direction.To fin out given in statements P & Q is in 6 seconds of the trains is 26kmph

file:///E|/work/books/placement/09_Aptitude/trains.html[1/28/2012 12:45:46 AM]

(d)Both P & Q are not sufficient Ans : A Solution: Let speed of B be x kmph then,speed of A =140x/100 kmph =7x/5 mph relative speed = x + 7x/5 =2x/3 m/s time taken to cross each other = (100+180)*3/2x s =420/x s now,420/x = 6 x=70 mph thus,only P is sufficient (15)The train running at certain speed crosses astationary enginein 20 seconds.to find out the sped of the train,which of the following information is necessary (a)Only the length of the train (b)only the length of the engine (c)Either the length of the train or length of engine (d)Both the length of the train or length of engine Ans : D Solution: Since the sum of lengths of the tran and the engine is needed, so both the length must be known

Back Back To Main

Top

Contact: 040-23000700

file:///E|/work/books/placement/09_Aptitude/trains.html[1/28/2012 12:45:46 AM]

APTITUDE
Numbers H.C.F and L.C.M Decimal Fractions Simplification Square and Cube roots Average Problems on Numbers Problems on Ages Surds and Indices Percentage Profit and Loss Ratio And Proportions Partnership Chain Rule Time and Work Pipes and Cisterns Time and Distance Trains Boats and Streams Alligation or Mixture Simple Interest Compound Interest Logorithms Areas Volume and Surface area Races and Games of Skill Calendar Clocks Stocks ans Shares True Discount Bankers Discount Oddmanout and Series Data Interpretation probability Permutations and Combinations Puzzles

TRUE DISCOUNT
CONCEPT SIMPLE PROBLEMS MEDIUM PROBLEMS COMPLEX PROBLEMS

file:///E|/work/books/placement/09_Aptitude/true.html[1/28/2012 12:45:46 AM]

APTITUDE
Numbers H.C.F and L.C.M Decimal Fractions Simplification Square and Cube roots Average Problems on Numbers Problems on Ages Surds and Indices Percentage Profit and Loss Ratio And Proportions Partnership Chain Rule Time and Work Pipes and Cisterns Time and Distance Trains Boats and Streams Alligation or Mixture Simple Interest Compound Interest Logorithms Areas Volume and Surface area Races and Games of Skill Calendar Clocks Stocks ans Shares True Discount Bankers Discount Oddmanout and Series Data Interpretation probability Permutations and Combinations Puzzles BACK

COMPLEX PROBLEMS
1.If Rs.10 be allowed as truedscount on a bill of Rs.110 at the end of a certaimn time ,then discount allowed on the same sum due at the end of double the time.. sol: Amount=Rs110 TrueDiscount=Rs10 Present worth=Amount-TrueDiscount =Rs110-10 =Rs.100 SI on Rs.100 for a certain time =Rs.10 SI on Rs.100 for doube the time =Rs.20 TrueDiscount on (100+20)=120-100 =Rs20 TrueDiscount on Rs.110 =(110*20)/120 =Rs18.33

2.A man wants to se hs scooter .There are two offers one at Rs12000 cash and other at acredit of Rs12880 to be paid after 8 months ,money being at 18% per annum which is better offer? Sol: offer1=Rs12000 offer2 Present worth= (Amount*100) /(100+(R*T)) =Rs.12880*100/(100+(18*(8/12)) =Rs11,500 The first offer is better as if he gains 500/if he sells at Rs.12000 is better

3.Goods were bought for Rs.600 and sold the same day for Rs.688.80 at a credit of 9 months and thus gaining 2% rate of interest per annum is sol: MethodI: Amount=Rs.688.50 Time=9.12 Gaining 2%----------->Presentworth=102%of 600=(102*600)/100 =Rs.612 Presentworth=(Amount*100)/(100+(T*R)) 612=(688.80*100)/(100+((9/2)*R)) =>R=162/3% MethodII: TrueDiscount=688.50-612 =Rs.76.50 Rate=TrueDiscount*100/(P.W*T) =(76.50*100)/(612*(9/12)) =162/3%

file:///E|/work/books/placement/09_Aptitude/truecomplex.html[1/28/2012 12:45:47 AM]

4.The present worth of Rs.1404 due in 2 equal halfyearly instalments at 8% per annu8m S.I is SOl: Presentworth=(Amount*100)/(100+(T*R)) PresentWorth=Presentworth of Rs.702 6 months hence + Presentworth of Rs.702 1yr hence = (702*100)/(100+(1/2*8)) + (702t*100)/(100+(1*8)) =Rs.1325

BACK

file:///E|/work/books/placement/09_Aptitude/truecomplex.html[1/28/2012 12:45:47 AM]

APTITUDE
Numbers H.C.F and L.C.M Decimal Fractions Simplification Square and Cube roots Average Problems on Numbers Problems on Ages Surds and Indices Percentage Profit and Loss Ratio And Proportions Partnership Chain Rule Time and Work Pipes and Cisterns Time and Distance Trains Boats and Streams Alligation or Mixture Simple Interest Compound Interest Logorithms Areas Volume and Surface area Races and Games of Skill Calendar Clocks Stocks ans Shares True Discount Bankers Discount Oddmanout and Series Data Interpretation probability Permutations and Combinations Puzzles BACK

CONCEPT
Let rate=R%per annum, Time= T years 1.Present worth (PW) = (100*Amount)/(100+(R*T)) = (100* TrueDiscount)/(R*T) 2.TrueDiscount (TD) = (P.W*R*T)/100 = (Amount*R*T)/(100+(R*T)) 3.Sum =(SimpeInterest*TrueDiscount)/(SimpleInterest-TrueDiscount) 4.SimpleInterest-TrueDiscount=SimpeInterest on TrueDiscount 5.When the sum is put at CompoundInterest,then PresentWorth=Amount/(1+(R/100))^T GeneralConcept: Suppose a man has to pay Rs.156 after 4 years and the rate of interest is 14%per annum Clearly ,Rs.100 at 14% will amount to Rs156 in 4 years So,the payment of Rs.100 now wil cear off the debt of Rs156 due 4 years hence We say that Sum due = Rs156 due 4 years hence Present Worth =Rs100 True Discount=Rs.156-Rs100=Rs56 =Sumdue-PW We define TrueDiscount= Interest on Present Worth(PW) Amount = PresentWorth+TrueDiscount
BACK

file:///E|/work/books/placement/09_Aptitude/trueconcept.html[1/28/2012 12:45:47 AM]

True Discount
Formulae: Let rate=R%per annum, Time= T years 1.Present worth (PW) = (100*Amount)/(100+(R*T)) = (100* TrueDiscount)/(R*T) 2.TrueDiscount (TD) = (P.W*R*T)/100 = (Amount*R*T)/(100+(R*T)) 3.Sum =(SimpeInterest*TrueDiscount)/(SimpleInterest-TrueDiscount) 4.SimpleInterest-TrueDiscount=SimpeInterest on TrueDiscount 5.When the sum is put at CompoundInterest,then PresentWorth=Amount/(1+(R/100))^T GeneralConcept: Suppose a man has to pay Rs.156 after 4 years and the rate of interest is 14%per annum Clearly ,Rs.100 at 14% will amount to Rs156 in 4 years So,the payment of Rs.100 now wil cear off the debt of Rs156 due 4 years hence We say that Sum due = Rs156 due 4 years hence Present Worth =Rs100 True Discount=Rs.156-Rs100=Rs56 =Sumdue-PW We define TrueDiscount= Interest on Present Worth(PW) Amount = PresentWorth+TrueDiscount 1.Find the present worth of Rs.930 due 3 years hence at 8% per annum.Aso find the discount? Sol: Amount=RS.930,Time=3years,Rate=8% TrueDiscount = (Presentworth*Time*Rate)/100 Presentworth = (Amount*100) /(100+(R*T)) Presentworth = (930*100)/(100+(8*3) =Rs.750 TrueDiscount = (930*3*8)/(100+(8*3)) =Rs.180 (or) TrueDiscount = Amount-Presentworth =Rs.930-Rs.750 =Rs.180 2.The truediscount on a bill due 9 months hence at 12% per annum is Rs540.Find the amount of the bill and its presentworth? Sol: Time=9months=9/12years=3/4years Rate=12% TrueDiscount=Rs540 TrueDiscount=(Amount*100)/(100+(R*T)) =>Amount=(TrueDiscount*(100+(R*T))) /100 =(540*(100+12*(3/4))/100 =Rs.6540 Presentworth=Amount-TrueDiscount =Rs.6540-Rs540 =Rs.6000 3.The TrueDiscount on a certain sum of money due 3 years hence is Rs.250 and SimpeInterest on the same sum for same time and same rate is Rs375 find sum and rate%? Sol: Time=3 years Truediscount=Rs.250 SimpeInterest=Rs375 Sum = (SimpeInterest*TrueDiscount)/(SimpeInterest-TrueDiscount) = (375*250)/(375-250) =Rs.750 SimpeInterest=(Principle*Time*Rate)/100 375 = ( 750*3R)/100 R=50/3% => Rate=162/3%

Top
4.The difference between SimpeInterest and TrueDiscount on a certain sum of money for 6 months at 121/2% per annum is Rs25. Find the sum. Sol: Let amount be Rs.x SimpleInterest=(Amount*T*R)/100 TrueDiscount=(Amount*T*R)/(100+(R*T)) SI-T.D=Rs25 =>((x*6/12*25/2)/100)-((x*6/12*25/2)/(100+(6/12*25/2))=25 =>x=Rs.6800 5.The Present worth of Rs.2310 due 21/2yrs henceThe interest

file:///E|/work/books/placement/09_Aptitude/truediscount.html[1/28/2012 12:45:47 AM]

being 15% per annum is sol: Amount=Rs2310,Time=21/2yrs,Rate=15% Presentworth = (Amount*100) /(100+(R*T)) =2310*100/(100+15*5/2) =Rs1680. Medium Problems 1.A bill falls due in 1 year.The creditor agrees to accept immediate payment of the half and to defer the payment of the other half for 2 years .By this arrangement he gains Rs.40.what is the amount of bill,if the money be worth 121/2%? sol: Let the amount be Rs.x If the bill falls due for 1 yr, Truediscount=(Amount*100)/(100+R*T) =(x*100)/(100+(25/2*1) If Accepting immediate payment of the half and to defer the payment of the other half for 2 years, Truediscount=(Amount*100)/(100+R*T) =[x/2+((x/2*100)/(100+(25/2*2))] x/2 for immediate payment, ((x/2*100)/(100+(25/2*2)) for paying after 2 yrs He gains Rs40 =>[x/2+((x/2*100)/(100+(25/2*2))] (x*100)/(100+(25/2*1) = Rs40 => x/2+2x/5-8x/9=40 =>x=Rs.3600 Amount of the bill=Rs.3600 2.If the truediscount on a sum due 2yrs hence at 14% per annum be Rs168.The sum due is? Sol: Time=2yrs, Rate=14% Truediscount=Rs168,Amount=x TrueDiscount=(Amount*R*T)/(100+R*T) 168 =(x*14*2)/(100+(14*2)) =>x=Rs.768 3.The truediscount on Rs.2562 due 4 months hence is Rs.122.The rate % is? Sol: Amount=Rs.2562 Time=4/12yrs TrueDiscount=Rs122 Rate=X% TrueDiscount=(Amount*R*T)/(100+R*T) =>122=(2562*x*(4/12))/(100+(x*(4/12)) =>36600+122x=2562x =>Rate=15% 4.The Truediscount on Rs1760 due after a certain time at 12% per annum is Rs160The time after which it is due is: Sol: Amount=Rs.1760 Rate=12% Truediscount=Rs160 Time=x TrueDiscount=(Amount*R*T)/(100+R*T) =>x*11*12=100+12x =>x=5/6yr Time=10months 5.The interest on Rs.760 for 2yrs is the same as the Truediscount on Rs960 due 2yrs hence .If the rate of interest is same in both cases,it is sol: Principal=Rs.760 Time=2yrs amount=Rs960 P*T*R/100=(Amount*R*T)/(100+R*T) =>(750*2*r)/100=(960*r*2)/(100+2r) =>150r=2100 rate=14% 6.The SimpleInterest & TrueDiscount on a certain sum of money for a given time & at a given rate are Rs85 & Rs.80.The sum is............. sol: S.I= Rs.85 TrueDiscount=Rs80 Sum=(S.I.*TD)/(SI-T.D) =Rs.1360

Top
7.A trader owes a maerchant Rs10028 due 1yr hence The trader wants to settle the account after 3 months .If the rate of interest is 12%per annum How much cash should he pay? Sol: Time=1yr but he settles account after 3months so time = 9months Cash to pay=(Amount*R*T)/(100+R*T) =(10028*100)/(100+12*(9/12)) =Rs.9200 8.A man buys a watch for Rs1950 in cash and sells it for Rs2200 at a credit of 1yrThe rate of interest is 10% per annum Then he gains / loose---------------amount? Sol:

file:///E|/work/books/placement/09_Aptitude/truediscount.html[1/28/2012 12:45:47 AM]

If he sells it for Rs2200 at a credit of 1yr then the present worth of that amount is Presentworth = (Amount*100) /(100+(R*T)) =(2200*100)/(100+10*1) =2000/So he gains 2000-1950=Rs.50 9.A owes Rs.1573 payable 11/2yrs hence ,Aso B owes A Rs.144450 payabe 6months hence If they want to settle the accunt forth with ,keeping 14% as the rate of interest ,then who should pay & how much? Sol: B->A-----------Amount=Rs.1573 Rate=14% Time=3/2 Presentworth = (Amount*100) /(100+(R*T)) =(1573*100)/(100+(3/2)*14) =Rs1300 A->B------------Amount=Rs.1444.50 Rate=14% Time=1/2 Presentworth = (Amount*100) /(100+(R*T)) =(1444.50*100)/(100+(1/2)*14) =Rs1350 so B should pay Rs.1350 10.If Rs20 is the TrueDiscount on Rs260due after a certain time. What wil be trueDiscount on same sum due after of former time, the rate of interest beng the same? Sol: Simple Interest on (260-20) for a gven time=Rs20 Simple Interest on (260-20) for a half time=Rs20*(1/2) =Rs10 True Discount on Rs.250=Rs10 True Discount on Rs.260=Rs10*260/250 =Rs10.40 11.A has to pay Rs.220 to B after 1yr.B asks A to pay Rs.110 in cash and defer the payment of Rs,.110 for 2 yrs.A agrees to it.If the rate of interest be 10% per annum in this mode of payment sol: A has to pay =Present worth of Rs220 due 1yr hence =Rs.(220*100)/(100+(10*1)) =Rs.200 A actually pays=Rs.110+Presentworth of Rs110 due 2 yrs hence =[110+ (Amount*100) /(100+(R*T))] =[110+((110*100)/(100+(10*2)))] =Rs192.66 So A gains Rs(200-192.66)=Rs.7.34

Top
Complex Problems 1.If Rs.10 be allowed as truedscount on a bill of Rs.110 at the end of a certaimn time ,then discount allowed on the same sum due at the end of double the time.. sol: Amount=Rs110 TrueDiscount=Rs10 Present worth=Amount-TrueDiscount =Rs110-10 =Rs.100 SI on Rs.100 for a certain time =Rs.10 SI on Rs.100 for doube the time =Rs.20 TrueDiscount on (100+20)=120-100 =Rs20 TrueDiscount on Rs.110 =(110*20)/120 =Rs18.33 2.A man wants to se hs scooter .There are two offers one at Rs12000 cash and other at acredit of Rs12880 to be paid after 8 months ,money being at 18% per annum which is better offer? Sol: offer1=Rs12000 offer2 Present worth= (Amount*100) /(100+(R*T)) =Rs.12880*100/(100+(18*(8/12)) =Rs11,500 The first offer is better as if he gains 500/if he sells at Rs.12000 is better 3.Goods were bought for Rs.600 and sold the same day for Rs.688.80 at a credit of 9 months and thus gaining 2% rate of interest per annum is sol: MethodI: Amount=Rs.688.50 Time=9.12 Gaining 2%----------->Presentworth=102%of 600=(102*600)/100 =Rs.612 Presentworth=(Amount*100)/(100+(T*R)) 612=(688.80*100)/(100+((9/2)*R)) =>R=162/3% MethodII: TrueDiscount=688.50-612 =Rs.76.50 Rate=TrueDiscount*100/(P.W*T) =(76.50*100)/(612*(9/12))

file:///E|/work/books/placement/09_Aptitude/truediscount.html[1/28/2012 12:45:47 AM]

=162/3% 4.The present worth of Rs.1404 due in 2 equal halfyearly instalments at 8% per annu8m S.I is SOl: Presentworth=(Amount*100)/(100+(T*R)) PresentWorth=Presentworth of Rs.702 6 months hence + Presentworth of Rs.702 1yr hence = (702*100)/(100+(1/2*8)) + (702t*100)/(100+(1*8)) =Rs.1325

Back Back To Main

Top

Contact: 040-23000700

file:///E|/work/books/placement/09_Aptitude/truediscount.html[1/28/2012 12:45:47 AM]

APTITUDE
Numbers H.C.F and L.C.M Decimal Fractions Simplification Square and Cube roots Average Problems on Numbers Problems on Ages Surds and Indices Percentage Profit and Loss Ratio And Proportions Partnership Chain Rule Time and Work Pipes and Cisterns Time and Distance Trains Boats and Streams Alligation or Mixture Simple Interest Compound Interest Logorithms Areas Volume and Surface area Races and Games of Skill Calendar Clocks Stocks ans Shares True Discount Bankers Discount Oddmanout and Series Data Interpretation probability Permutations and Combinations Puzzles BACK

MEDIUM PROBLEMS
1.A bill falls due in 1 year.The creditor agrees to accept immediate payment of the half and to defer the payment of the other half for 2 years .By this arrangement he gains Rs.40.what is the amount of bill,if the money be worth 121/2%? sol: Let the amount be Rs.x If the bill falls due for 1 yr, Truediscount=(Amount*100)/(100+R*T) =(x*100)/(100+(25/2*1) If Accepting immediate payment of the half and to defer the payment of the other half for 2 years, Truediscount=(Amount*100)/(100+R*T) =[x/2+((x/2*100)/(100+(25/2*2))] x/2 for immediate payment, ((x/2*100)/(100+(25/2*2)) for paying after 2 yrs He gains Rs40 =>[x/2+((x/2*100)/(100+(25/2*2))] (x*100)/(100+(25/2*1) = Rs40 => x/2+2x/5-8x/9=40 =>x=Rs.3600 Amount of the bill=Rs.3600

2.If the truediscount on a sum due 2yrs hence at 14% per annum be Rs168.The sum due is? Sol: Time=2yrs, Rate=14% Truediscount=Rs168,Amount=x TrueDiscount=(Amount*R*T)/(100+R*T) 168 =(x*14*2)/(100+(14*2)) =>x=Rs.768

3.The truediscount on Rs.2562 due 4 months hence is Rs.122.The rate % is? Sol: Amount=Rs.2562 Time=4/12yrs TrueDiscount=Rs122 Rate=X% TrueDiscount=(Amount*R*T)/(100+R*T) =>122=(2562*x*(4/12))/(100+(x*(4/12)) =>36600+122x=2562x =>Rate=15%

4.The Truediscount on Rs1760 due after a certain time at 12% per annum is Rs160The time after which it is due is: Sol: Amount=Rs.1760 Rate=12% Truediscount=Rs160

file:///E|/work/books/placement/09_Aptitude/truemedium.html[1/28/2012 12:45:48 AM]

Time=x TrueDiscount=(Amount*R*T)/(100+R*T) =>x*11*12=100+12x =>x=5/6yr Time=10months

5.The interest on Rs.760 for 2yrs is the same as the Truediscount on Rs960 due 2yrs hence .If the rate of interest is same in both cases,it is sol: Principal=Rs.760 Time=2yrs amount=Rs960 P*T*R/100=(Amount*R*T)/(100+R*T) =>(750*2*r)/100=(960*r*2)/(100+2r) =>150r=2100 rate=14%

6.The SimpleInterest & TrueDiscount on a certain sum of money for a given time & at a given rate are Rs85 & Rs.80.The sum is............. sol: S.I= Rs.85 TrueDiscount=Rs80 Sum=(S.I.*TD)/(SI-T.D) =Rs.1360 7.A trader owes a maerchant Rs10028 due 1yr hence The trader wants to settle the account after 3 months .If the rate of interest is 12%per annum How much cash should he pay? Sol: Time=1yr but he settles account after 3months so time = 9months Cash to pay=(Amount*R*T)/(100+R*T) =(10028*100)/(100+12*(9/12)) =Rs.9200

8.A man buys a watch for Rs1950 in cash and sells it for Rs2200 at a credit of 1yrThe rate of interest is 10% per annum Then he gains / loose---------------amount? Sol: If he sells it for Rs2200 at a credit of 1yr then the present worth of that amount is Presentworth = (Amount*100) /(100+(R*T)) =(2200*100)/(100+10*1) =2000/So he gains 2000-1950=Rs.50

9.A owes Rs.1573 payable 11/2yrs hence ,Aso B owes A Rs.144450 payabe 6months hence If they want to settle the accunt forth with ,keeping 14% as the rate of interest ,then who should pay & how much? Sol: B->A-----------Amount=Rs.1573 Rate=14%

file:///E|/work/books/placement/09_Aptitude/truemedium.html[1/28/2012 12:45:48 AM]

Time=3/2 Presentworth = (Amount*100) /(100+(R*T)) =(1573*100)/(100+(3/2)*14) =Rs1300 A->B------------Amount=Rs.1444.50 Rate=14% Time=1/2 Presentworth = (Amount*100) /(100+(R*T)) =(1444.50*100)/(100+(1/2)*14) =Rs1350 so B should pay Rs.1350

10.If Rs20 is the TrueDiscount on Rs260due after a certain time. What wil be trueDiscount on same sum due after of former time, the rate of interest beng the same? Sol: Simple Interest on (260-20) for a gven time=Rs20 Simple Interest on (260-20) for a half time=Rs20*(1/2) =Rs10 True Discount on Rs.250=Rs10 True Discount on Rs.260=Rs10*260/250 =Rs10.40

11.A has to pay Rs.220 to B after 1yr.B asks A to pay Rs.110 in cash and defer the payment of Rs,.110 for 2 yrs.A agrees to it.If the rate of interest be 10% per annum in this mode of payment:... ............ sol: A has to pay =Present worth of Rs220 due 1yr hence =Rs.(220*100)/(100+(10*1)) =Rs.200 A actually pays=Rs.110+Presentworth of Rs110 due 2 yrs hence =[110+ (Amount*100) /(100+(R*T))] =[110+((110*100)/(100+(10*2)))] =Rs192.66 So A gains Rs(200-192.66)=Rs.7.34
BACK

file:///E|/work/books/placement/09_Aptitude/truemedium.html[1/28/2012 12:45:48 AM]

APTITUDE
Numbers H.C.F and L.C.M Decimal Fractions Simplification Square and Cube roots Average Problems on Numbers Problems on Ages Surds and Indices Percentage Profit and Loss Ratio And Proportions Partnership Chain Rule Time and Work Pipes and Cisterns Time and Distance Trains Boats and Streams Alligation or Mixture Simple Interest Compound Interest Logorithms Areas Volume and Surface area Races and Games of Skill Calendar Clocks Stocks ans Shares True Discount Bankers Discount Oddmanout and Series Data Interpretation probability Permutations and Combinations Puzzles BACK

SIMPLE PROBLEMS

1.Find the present worth of Rs.930 due 3 years hence at 8% per annum.Aso find the discount? Sol: Amount=RS.930,Time=3years,Rate=8% TrueDiscount = (Presentworth*Time*Rate)/100 Presentworth = (Amount*100) /(100+(R*T)) Presentworth = (930*100)/(100+(8*3) =Rs.750 TrueDiscount = (930*3*8)/(100+(8*3)) =Rs.180 (or) TrueDiscount = Amount-Presentworth =Rs.930-Rs.750 =Rs.180 2.The truediscount on a bill due 9 months hence at 12% per annum is Rs540.Find the amount of the bill and its presentworth? Sol: Time=9months=9/12years=3/4years Rate=12% TrueDiscount=Rs540 TrueDiscount=(Amount*100)/(100+(R*T)) =>Amount=(TrueDiscount*(100+(R*T))) /100 =(540*(100+12*(3/4))/100 =Rs.6540 Presentworth=Amount-TrueDiscount =Rs.6540-Rs540 =Rs.6000 3.The TrueDiscount on a certain sum of money due 3 years hence is Rs.250

file:///E|/work/books/placement/09_Aptitude/truesimple.html[1/28/2012 12:45:48 AM]

and SimpeInterest on the same sum for same time and same rate is Rs375 find sum and rate%? Sol: Time=3 years Truediscount=Rs.250 SimpeInterest=Rs375 Sum = (SimpeInterest*TrueDiscount)/(SimpeInterest-TrueDiscount) = (375*250)/(375-250) =Rs.750 SimpeInterest=(Principle*Time*Rate)/100 375 = ( 750*3R)/100 R=50/3% => Rate=162/3%

4.The difference between SimpeInterest and TrueDiscount on a certain sum of money for 6 months at 121/2% per annum is Rs25.find the sum. Sol: Let amount be Rs.x SimpleInterest=(Amount*T*R)/100 TrueDiscount=(Amount*T*R)/(100+(R*T)) SI-T.D=Rs25 =>((x*6/12*25/2)/100)-((x*6/12*25/2)/(100+(6/12*25/2))=25 =>x=Rs.6800

5.The Present worth of Rs.2310 due 21/2yrs henceThe interest being 15% per annum is sol: Amount=Rs2310,Time=21/2yrs,Rate=15% Presentworth = (Amount*100) /(100+(R*T)) =2310*100/(100+15*5/2) =Rs1680.

BACK

file:///E|/work/books/placement/09_Aptitude/truesimple.html[1/28/2012 12:45:48 AM]

APTITUDE
Numbers H.C.F and L.C.M Decimal Fractions Simplification Square and Cube roots Average Problems on Numbers Problems on Ages Surds and Indices Percentage Profit and Loss Ratio And Proportions Partnership Chain Rule Time and Work Pipes and Cisterns Time and Distance Trains Boats and Streams Alligation or Mixture Simple Interest Compound Interest Logorithms Areas Volume and Surface area Races and Games of Skill Calendar Clocks Stocks ans Shares True Discount Bankers Discount Oddmanout and Series Data Interpretation probability Permutations and Combinations Puzzles

TRUE DISCOUNT
CONCEPT SIMPLE PROBLEMS MEDIUM PROBLEMS COMPLEX PROBLEMS

file:///E|/work/books/placement/09_Aptitude/truesquareandcuberoot.html[1/28/2012 12:45:49 AM]

True Discount
General Concepts:

Back Back To Main

Top

Contact: 040-23000700

file:///E|/work/books/placement/09_Aptitude/turediscount.html[1/28/2012 12:45:54 AM]

APTITUDE
Numbers H.C.F and L.C.M Decimal Fractions Simplification Square and Cube roots Average Problems on Numbers Problems on Ages Surds and Indices Percentage Profit and Loss Ratio And Proportions Partnership Chain Rule Time and Work Pipes and Cisterns Time and Distance Trains Boats and Streams Alligation or Mixture Simple Interest Compound Interest Logorithms Areas Volume and Surface area Races and Games of Skill Calendar Clocks Stocks ans Shares True Discount Bankers Discount Oddmanout and Series Data Interpretation probability Permutations and Combinations Puzzles

CALENDAR
BACK

Typical problems
Problem 1 : on what dates of october,1994 did monday fall ? Solution : 01 01 94 23 ------total = 119 Then (119)/7 = (17) = 0 odd days so the day is saturday Therefore in october first the day is saturday.so, the monday fell on 3rd october 1994.During october 1994, monday fell on 3rd ,10th,17th and 24th. Problem 2 : How many days are there from 2nd january 1995 o 15 th march,1995 ? Solution : Jan + Feb + March 30 + 28 + 15 = 73 days

Problem 3 : The year next to 1996 having the same calendar as that of 1996 is ? Solution : Starting with 1996 , we go on countig the number of odd days till the sum is divisible by 7. Year 1996 1997 1998 1999 2000 odd days 2 1 1 1 2

2 + 1 + 1 + 1 + 2 = 7 odd days i.e odd day. Therefore calendar for 2001 will be the same as that of 1995. Problem 4 : The calendar for 1990 is same as for : Solution : count the number of days 1990 onwards to get 0 odd day. Year 1990 1991 1992 1993 1994 1995 odd days 1 1 2 1 1 1

1 + 1 + 2 + 1 + 1 + 1 = 7 or 0 odd days Therefore calendar for 1990 is the same as for the year 1996. Problem 5 : The day on 5th march of year is the same day on what date of the same year? Solution : In the given monthly code table represents the march code and november code both are same.that means any date in march is the same day of week as the corresponding date in november of that year, so the same day falls on 5th november.

BACK

file:///E|/work/books/placement/09_Aptitude/typicalproblems.html[1/28/2012 12:45:54 AM]

You might also like